aafp

Ace your homework & exams now with Quizwiz!

A 47-year-old postmenopausal female falls while carrying groceries into her house and sustains a right distal radial fracture. A chemistry panel reveals a calcium level of 11.2 mg/dL (N 8.6-10.6) and further evaluation leads to a diagnosis of primary hyperparathyroidism. Which one of the following is the best course of treatment for this patient? (check one) A. Estrogen replacement therapy B. Long-term bisphosphonate therapy C. Daily furosemide treatment with increased oral fluids D. Elimination of calcium and vitamin D from the diet E. Referral to a surgeon for consideration of parathyroidectomy

E. parathyroidectomy! Studies that localize the glands, such as a technetium scan or ultrasonography, help surgeons who are familiar with this condition achieve a cure rate of 95%-98%, with an estimated complication rate of 1%-3%.

A 40-year-old female sees you for a health maintenance visit. She has no complaints and other than being overweight she has an unremarkable examination. Laboratory results are also unremarkable except for her lipid profile. She has a total cholesterol level of 251 mg/dL, an HDL-cholesterol level of 31 mg/dL, and a triglyceride level of 1250 mg/dL. The LDL-cholesterol level could not be calculated and measured 145 mg/dL. In addition to lifestyle changes, this patient would most likely benefit from (check one) A. niacin B. omega-3 fatty acid supplementation C. atorvastatin (Lipitor) D. ezetimibe (Zetia) E. fenofibrate (Tricor)

hypertriglyceridemia mild: 150-199 mg/dL moderate: 200-999 mg/dL severe: 1000-1999 mg/dL very severe: >2000 mg/dL mild to moderate range: may be at risk for cardiovascular disease severe or very severe range: risk of pancreatitis. - avoid alcohol - fibrates are recommended as the initial treatment for pancreatitis Not statin: for moderate range treatment only Not omega 3: may increase LDL-cholesterol levels; does not decrease total mortality or cardiovascular events Not Niacin: lower LDL & raise HDL, does not prevent cardiovascular disease.

Which one of the following medications is most likely to cause hypokalemia? (check one) A. Albuterol (Proventil, Ventolin) B. Doxazosin (Cardura) C. Erythromycin D. Felodipine (Plendil) E. Lisinopril (Prinivil, Zestril)

A. Albuterol (Proventil, Ventolin) β-Agonists activate potassium uptake by the cells. This includes bronchodilators and tocolytic agents. Other agents that can induce hypokalemia include pseudoephedrine and insulin. Diuretics, particularly thiazides, can also cause hypokalemia as a result of the renal loss of potassium.

Which one of the following conditions presents an unacceptable health risk for combined oral contraceptive use? (check one) A. Migraine with aura B. Endometrial hyperplasia C. Breastfeeding 1-6 months post partum D. Chronic hepatitis E. Previous laparoscopic banding weight-loss surgery

A. Migraine with aura A history of migraine with aura is classified as category 4 for oral contraceptives. Women with a history of migraines are 2-4 times as likely to have a stroke compared to women without migraines, and women who have an aura associated with their migraines are at even higher risk.

A 45-year-old female has ultrasonography of her kidneys as part of an evaluation for uncontrolled hypertension. The report notes an incidental finding of stones in the gallbladder, confirmed on right upper quadrant ultrasonography. She has no symptoms you can relate to the gallstones. Other than hypertension she has no chronic medical problems. Which one of the following should you recommend to her at this time regarding the gallstones? (check one) A. Expectant management B. Oral dissolution therapy C. Extracorporeal lithotripsy D. Endoscopic retrograde cholangiopancreatography (ERCP) E. Laparoscopic cholecystectomy

A. Expectant management

A 48-year-old male who weighs 159 kg (351 lb) is admitted to the hospital with a left leg deep vein thrombosis and pulmonary embolism. Treatment is begun with enoxaparin (Lovenox). Which one of the following would be most appropriate for monitoring the adequacy of anticoagulation in this patient? (check one) A. Anti-factor Xa levels B. Activated partial thromboplastin time (aPTT) C. Daily INRs D. Daily factor VIII levelsA

A. Anti-factor Xa - In severely obese patients (>330 lb) and those with renal failure, low molecular weight heparin therapy should be monitored with anti-factor Xa levels obtained 4 hours after injection. Not INR - for warfarin Not activated partial thromboplastin time (aPTT) - for unfractionated heparin. Not Factor VIII - not used for any therapy

Which one of the following is a frequent cause of cross-reactive food-allergy symptoms in latex-allergic individuals? (check one) A. Avocadoes B. Goat's milk C. Pecans D. Pastrami E. Peppermint

A. Avocadoes Latex allergy has IgE antibodies that cross-react with some proteins in plant-derived foods.

A 56-year-old African-American male with long-standing hypertension and a 30-pack-year smoking history has a 2-day history of dyspnea on exertion. A physical examination is unremarkable except for rare crackles at the bases of the lungs. Which one of the following serologic tests would be most helpful for detecting left ventricular dysfunction? (check one) A. B-type natriuretic peptide B. Troponin T C. C-reactive protein (CRP D. D-dime E. Cardiac interleukin-2

A. B-type natriuretic peptide

Which one of the following treatments for childhood nocturnal enuresis has both the highest cure rates and the lowest relapse rates? (check one) A. Bed-wetting alarms B. Positive reinforcement C. Responsibility training D. Desmopressin (DDAVP) E. Imipramine (Tofranil)

A. Bed-wetting alarms the bed-wetting alarm has a higher success rate (75%) and a lower relapse rate (41%)

56-year-old female with well-controlled diabetes mellitus and hypertension presents with an 18-hour history of progressive left lower quadrant abdominal pain, low-grade fever, and nausea. She has not been able to tolerate oral intake over the last 6 hours. An abdominal examination reveals significant tenderness in the left lower quadrant with slight guarding but no rebound tenderness. Bowel sounds are hypoactive. Rectal and pelvic examinations are unremarkable. Which one of the following is recommended as the initial diagnostic procedure in this situation? (check one) A. CT of the abdomen and pelvis B. Abdominal and pelvic ultrasonography C. A barium enema D. Colonoscopy E. Laparoscopy

A. CT of the abdomen and pelvis - acute diverticulitis. - Ultrasonography may be helpful in suggesting other diagnoses, but it is not as specific or as sensitive for diverticulitis as CT.

About a month after returning from the Middle East, an American soldier develops a papule on his forearm that subsequently ulcerates to form a shallow annular lesion with a raised margin. The lesion shows no signs of healing 3 months after it first appeared. He has no systemic symptoms. The most likely diagnosis is: (check one) A. leishmaniasis B. schistosomiasis C. malaria D. trypanosomiasis E. syphilis

A. leishmaniasis The indolent course of the sore described favors the diagnosis of cutaneous leishmaniasis. Neither malaria nor schistosomiasis produces these sores. The chancres of syphilis and trypanosomiasis are more fleeting in duration.

A previously alert, otherwise healthy 74-year-old African-American male has a history of slowly developing progressive memory loss and dementia associated with urinary incontinence and gait disturbance resembling ataxia. This presentation is most consistent with: (check one) A. normal pressure hydrocephalus B. Alzheimer's disease C. subacute sclerosing panencephalitis D. multiple sclerosis

A. normal pressure hydrocephalus

Which one of the following is known to cause hyperthyroidism? (check one) A. Propranolol (Inderal) B. Amiodarone (Cordarone) C. Methimazole (Tapazole) D. Propylthiouracil E. Methotrexate (Rheumatrex, Trexall)

Amiodarone - Amiodarone is 37% iodine and is the most common source of iodine excess in the United States.

Which one of the following is most appropriate for the treatment of fibromyalgia syndrome? (check one) A. Metaxalone (Skelaxin) B. Hydrocodone C. Naproxen D. Tizanidine (Zanaflex) E. Amitriptyline

E. Amitriptyline short-term use of amitriptyline and duloxetine can be considered for the treatment of pain and sleep disturbance in patients with fibromyalgia.

In assessing the nutritional status of an infant it is useful to know that birth weight is expected to be regained within: (check one) A. 5 days B. 14 days C. 21 days D. 28 days

B. 14 days.

A 34-year-old white female sees you for a routine follow-up visit. She takes haloperidol, 2 mg after each meal, for schizophrenia, and you notice that she seems unable to sit still and is extremely anxious. The most likely cause of her restlessness is (check one) A. drug-induced parkinsonism B. akathisia C. tardive dyskinesia D. hysteria E. dystonia

B. akathisia

A 28-year-old male has had bright red blood in his semen with his last three ejaculations. He is sexually active. He considers himself in good health, takes no medications, has no other symptoms to suggest a coagulopathy, and has no other genitourinary symptoms. Examination of the testes shows no masses or tenderness. Findings on a digital rectal examination are normal. Which one of the following would be appropriate at this time? (check one) A. Coagulation studies including a platelet count and a prothrombin time B. A serum PSA level C. A urine probe for Neisseria gonorrhoeae and Chlamydia trachomatis D. CT of the pelvis E. Referral to a urologist

C. A urine probe for Neisseria gonorrhoeae and Chlamydia trachomatis

A 73-year-old female presents with complaints of dyspnea and decreasing exercise tolerance over the past few months. She says she has to prop herself up on two pillows in order to breathe better. She also complains of palpitations, even at rest. She has long-standing hypertension, but has not taken any antihypertensive medications for several years. She has no history of ischemic heart disease. On examination her blood pressure is 155/92 mm Hg, her pulse rate is 108 beats/min and irregular, and her lungs have bibasilar crackles. An EKG reveals atrial fibrillation, but no changes of acute ischemia. Which one of the following would be most useful for determining her initial treatment? (check one) A. A chest radiograph B. Cardiac catheterization C. Echocardiography D. A TSH level E. A D-dimer level

C. Echocardiography

A 64-year-old male presents with a 3-month history of difficulty sleeping. A history and physical examination, followed by appropriate ancillary testing, leads to a diagnosis of chronic primary insomnia. Which one of the following would be most appropriate for managing this patient's problem? (check one) A. An SSRI B. A small glass of wine 1 hour before bedtime C. Cognitive-behavioral therapy D. Watching television at bedtime, with the timer set to turn off in 60 minutes E. Reading in bed with a soft light

C. Cognitive-behavioral therapy

A 25-year-old male daycare worker presents with a 3-week history of bloating and foul-smelling stools. On examination the patient has mild, diffuse abdominal tenderness and increased bowel sounds. Which one of the following is the most likely cause of this patient's problem? (check one) A. Hepatitis A B. Clostridium difficile C. Enterotoxigenic Escherichia coli D. Giardia lamblia E. Campylobacter

D. Giardia lamblia Giardia: daycare - metronidazole C. diff: hospital - metronidazole E. coli (enterotoxigenic)/Norovirus: traveler's diarrhea - cipro Campylobacter: food + bloody diarrhea - cipro

53-year-old female without risk factors for colorectal cancer undergoes a screening colonoscopy. A high-quality examination reveals five 3- to 7-mm sessile polyps in the sigmoid and rectal areas. Biopsy results show that they are hyperplastic polyps. No other abnormalities are noted. When should this patient have her next colonoscopy? (check one) A. 1 year B. 3 years C. 5 years D. 10 years E. No further colonoscopies needed

D. 10 years Hyperplastic polyps <10 mm in size in the rectum and sigmoid colon carry a low risk for developing into colon cancer. If they are the only finding, colonoscopy may be repeated in 10 years.

In the United States, cow's milk is not recommended for children until the age of (check one) A. 4 months B. 6 months C. 9 months D. 12 months E. 15 months

D. 12 mos Whole cow's milk does not supply infants with enough vitamin E, iron, and essential fatty acids, and overburdens them with too much protein, sodium, and potassium. Skim and low-fat milk lead to the same problems as whole milk, and also fail to provide adequate calories for growth. For these reasons cow's milk is not recommended for children under 12 months of age. Human breast milk or iron-fortified formula, with introduction of certain solid foods and juices after 4-6 months of age if desired, is appropriate for the first year of life.

For a healthy 1-month-old, daily vitamin D intake should be: (check one) A. 50 IU B. 100 IU C. 200 IU D. 400 IU E. 800 IU

D. 400 IU

What is the recommended compression-to-breath ratio for basic life support with a single rescuer for a 2-year-old child? (check one) A. 10:2 B. 15:2 C. 20:2 D. 25:2 E. 30:2

Correct. For a single rescuer performing CPR on a 2-year-old, the ratio is 30 compressions to 2 ventilations. The compression rate should be approximately 100 beats/min, and the chest should be compressed one-third to one-half its depth with each compression.

A 4-year-old male is brought to your office by his parents who are concerned that he is increasingly "knock-kneed." His uncle required leg braces as a child, and the parents are worried about long-term gait abnormalities. On examination, the patient's knees touch when he stands and there is a 15° valgus angle at the knee. He walks with a stable gait. Which one of the following should you do now? (check one) A. Refer to orthopedics for therapeutic osteotomy B. Refer to physical therapy for customized bracing C. Prescribe quadriceps-strengthening exercises D. Provide reassurance to the patient and his family

D. . Provide reassurance to the patient and his family- genu valgus - Toddlers under 2 years of age typically have a varus angle at the knee (bowlegs). - This transitions to physiologic genu valgus, which gradually normalizes by around 6 years of age.

In addition to exercise, which one of the following vitamin supplements is recommended by the U.S. Preventive Services Task Force to help prevent falls in elderly patients living at home? (check one) A. A B. B complex C. C D. D E. E

D. D

Which one of the following medications is associated with a higher risk of death due to stroke or sudden cardiac death in patients with dementia? (check one) A. Diazepam (Valium) B. Fluoxetine (Prozac) C. Paroxetine (Paxil) D. Quetiapine (Seroquel) E. Venlafaxine

D. Quetiapine (Seroquel) In April 2005 the FDA issued a boxed warning for second-generation antipsychotics, including quetiapine related in part to sudden cardiac death and also to stroke.

Which one of the following is the most appropriate adjunct medication for treating patients with post-traumatic stress disorder? (check one) A. Alprazolam (Xanax) B. Haloperidol (Haldol) C. Methylphenidate (Ritalin) D. Sertraline (Zoloft) E. Temazepam (Restoril)

D. Sertraline (Zoloft) - medications such as SSRIs, tricyclic antidepressants, and monoamine oxidase inhibitors alleviate the symptoms of post-traumatic stress disorder (PTSD) and are associated with improvements in overall functioning. - SSRIs are a first-line medication because they are safer and better tolerated than other types of psychotropic medications. - Sertraline and paroxetine are the only agents that have been approved by the FDA

In the elderly, the risk of heat wave-related death is highest in those who: (check one) A. have COPD B. have diabetes and are insulin dependent C. have a functioning fan, but not air conditioning D. are homebound

D. are homebound

A 57-year-old male presents to the emergency department complaining of dyspnea, cough, and pleuritic chest pain. A chest radiograph shows a large left-sided pleural effusion. Thoracentesis shows a pleural fluid protein to serum protein ratio of 0.7 and a pleural fluid LDH to serum LDH ratio of 0.8. Which one of the following causes of pleural effusion would be most consistent with these findings? (check one) A. Cirrhosis B. Heart Failure C. Nephrotic syndrome D. Superior vena cava obstruction

D. superior vena cava obstruction i.e. pulmonary embolism The protein and lactate dehydrogenase (LDH) levels in pleural fluid can help differentiate between transudative and exudative effusions. Light's criteria: 1. pleural fluid protein to serum protein ratio >0.5, 2. pleural fluid LDH to serum LDH ratio >0.6, and/or 3. pleural LDH >0.67 times the upper limit of normal for serum LDH i.e. exudative effusions Of the listed pleural effusion etiologies, only pulmonary embolism is exudative. The remainder are all transudative.

The preferred method for diagnosing psychogenic nonepileptic seizures is: (check one) A. inducing seizures by suggestion B. postictal prolactin levels C. EEG monitoring D. video-electroencephalography (vEEG) monitoring E. brain MRI

D. video-electroencephalography (vEEG) monitoring

On examination a 2-year-old child is found to have otalgia, a temperature of 39.0°C (102.2°F), and a bulging, red tympanic membrane. She weighs 17 kg (35 lb). Which one of the following would be the appropriate dosage of amoxicillin (Amoxil) for this child? (check one) A. 375 mg/day B. 500 mg/day C. 750 mg/day D. 1000 mg/day E. 1500 mg/day

E. 1500 mg/day For treating acute otitis media in this patient, the current recommended dosage of amoxicillin is 80-90 mg/kg/day.

Which one of the following is the best diagnostic test for vitamin D deficiency? (check one) A. Ionized calcium B. Serum phosphorus C. 24-hour urine for calcium D. 1,25-hydroxyvitamin D E. 25-hydroxyvitamin D

E. 25-hydroxyvitamin D 25 <20 ng/mL (50 nmol/L).

Intravenous magnesium is used to correct which one of the following arrhythmias? (check one) A. Wenckebach second-degree heart block B. Complete heart block C. Idioventricular rhythm D. Reentrant supraventricular tachycardia E. Ventricular tachycardia of torsades de pointes

E. Ventricular tachycardia of torsades de pointes

hich one of the following has been shown to benefit from screening for asymptomatic bacteriuria? (check one) A. Women with diabetes mellitus B. Men with prostatic enlargement on examination C. All adults with newly diagnosed hypertension D. Nursing-home residents with an indwelling Foley catheter E. Women who are pregnant

E. Women who are pregnant

PCOS increases likelihood of what cancer?

Endometrial carcinoma (PCOS RFs: obesity, hyperinsulinemia, diabetes mellitus, anovulatory cycles, and high androgen levels)

You are asked to see a mentally challenged 45-year-old male from a nearby group home who has groin pain. On examination you notice that he has large ears, a prominent jaw, and large symmetric testicles. These findings are consistent with: (check one) A. a variant form of Down syndrome B. Asperger's syndrome C. Klinefelter's syndrome D. homocystinuria E. Fragile X syndrome

Fragile X syndrome - accounts for more cases of mental retardation in males than any other genetic disorder except Down syndrome - about one in 4000-6000 males is affected. Down syndrome Patients with Klinefelter's syndrome usually have small testicles.

Which one of the following metabolic abnormalities is most likely to be seen in patients with stage 4 kidney disease? (check one) A. Hyperaldosteronism B. Hyperparathyroidism C. Hypothyroidism D. Hypogonadism E. Type 2 diabetes mellitus

Hyperparathyroidism is present in more than half of patients who have a glomerular filtration rate <60 mL/min, and is independently associated with increased mortality and an increased prevalence of cardiovascular disease. In patients with stage 4 chronic kidney disease, current guidelines recommend monitoring of serum calcium and phosphate levels every 3-6 months and bone-specific alkaline phosphatase activity every 6-12 months with the goal of normalizing these values.

A 34-year-old sexually active female consults you about contraception options. She has late-stage kidney disease and her nephrologist has notified you that she will likely be recommended for kidney transplantation soon. Which one of the following would you recommend for safety and efficacy? (check one) A. Condoms B. Combined oral contraceptive pills C. Medroxyprogesterone acetate (Depo-Provera) injections D. An IUD E. An etonogestrel/ethinyl estradiol vaginal ring (NuvaRing)

Pregnancy should be avoided during the 12 months following transplantation because of the increased risk of preterm delivery and graft rejection. Female fertility typically increases post transplant. The use of an IUD avoids interactions with medications, does not increase the risk of infection, and is not affected by typical immunosuppressive therapies. The remaining options are incorrect because of their higher failure and discontinuation rates. The CDC cites failure rates with typical use of 9% for combined oral contraceptives and the etonogestrel/ethinyl estradiol vaginal ring, 6% for injectable progesterone, 0.2% for levonorgestrel IUDs, and 0.08% for the copper IUD. Barrier method failure rates exceed 18%. Combined oral contraceptives and the vaginal ring also have potential estrogen-related side effects, and injectable progesterone use increases the risk for osteoporosis.

You have prescribed oral iron replacement for a 46-year-old female with iron deficiency anemia related to heavy menses. She wants to be sure that the iron she takes will be absorbed well. Which one of the following would you suggest for improving iron absorption? (check one) A. Calcium B. Vitamin C C. Coffee D. Tea

Vitamin C and a meal high in protein increases absorption of iron.

A 65-year-old male with end-stage renal disease requires postoperative pain management. Which one of the following medications would be safest to use? (check one) A. Fentanyl B. Hydrocodone C. Hydromorphone (Dilaudid) D. Meperidine (Demerol) E. Morphine

A. Fentanyl - one of the preferred narcotics in patients with end-stage renal disease. - Fentanyl elimination is 99% hepatic and it has a long history of safe use in patients with renal failure. - these drugs require close monitoring for side effects and indications for dosage reduction because they have active metabolites that accumulate in patients with renal failure. Meperidine, codeine, and propoxyphene are all contraindicated in chronic kidney disease because of the accumulation of toxic metabolites.

Which one of the following is the only medication that has consistent evidence for decreasing depressive symptoms in children and adolescents? (check one) A. Fluoxetine (Prozac) B. Venlafaxine (Effexor XR) C. Nortriptyline (Pamelor) D. Aripiprazole (Abilify) E. Paroxetine (Paxil)

A. Fluoxetine - Prozac Paroxetine (Paxil) causes youth suicide risk

The Strength-of-Recommendation Taxonomy (SORT) is used to grade key recommendations in clinical review articles. Which one of the following grades indicates that a recommendation is based on consistent, good-quality, patient-oriented evidence? (check one) A. A B. B C. C D. X

A. A B - limited-quality patient-oriented evidence C - consensus, disease-oriented evidence, usual practice, expert opinion, or case series for studies of diagnosis, treatment, prevention, or screening.

A 19-year-old college student comes to your office with her mother. The mother reports that her daughter has frequently been observed engaging in binge eating followed by induced vomiting. She has also admitted to using laxatives to prevent weight gain. Which one of the following laboratory abnormalities is most likely to be found in this patient? (check one) A. Hypokalemia B. Hypoglycemia C. Hyponatremia D. Hypercalcemia E. Hypermagnesemia

A. Hypokalemia no explanation

Dizziness is most likely to have a serious etiology when it (check one) A. is associated with diplopia B. is associated with intense nausea and vomiting C. occurs when the patient rolls over in bed D. occurs when the patient first arises in the morning E. occurs after 2 minutes of hyperventilation

A. Diplopia - along with other neurologic symptoms such as weakness or difficulty with speech, suggests a central cause of vertigo and requires a complete workup - Dizziness on first arising, dizziness with rolling over in bed, and dizziness with nausea and vomiting are consistent with peripheral causes of vertigo, such as benign positional vertigo. Dizziness that occurs after a couple of minutes of hyperventilation suggests a psychogenic cause.

A 42-year-old male has symptoms of hypogonadism. Which one of the following should be ordered first? (check one) A. Early morning total serum testosterone B. Early morning total and free serum testosterone C. Early morning total and late afternoon total serum testosterone D. Early morning and late afternoon free serum testosterone E. Early morning and late afternoon total and free serum testosterone

A. Early morning total serum testosterone - Low concentrations of testosterone in serum should be confirmed by repeat measurement. - If abnormalities in concentrations of sex hormone-binding globulin are suspected, measurement of free or bioavailable testosterone is indicated. - Examples of conditions associated with altered sex hormone-binding globulin include liver disease, obesity, and diabetes mellitus

In a 27-year-old white female with irregular menstrual cycles and infertility, which one of the following would be more indicative of Cushing's syndrome rather than the more common polycystic ovarian syndrome? (check one) A. Easy bruising B. Acne C. Hirsutism D. Androgenic alopecia E. Acanthosis nigricans

A. Easy bruising - also moon facies, buffalo hump, abdominal striae, hypertension, and proximal myopathy suggest Cushing's syndrome. - very rare compared to polycystic ovarian syndrome, routine screening is not indicated in women with hypoandrogenic anovulation

The National Weight Control Registry includes individuals who have lost substantial weight without surgery, and have maintained the weight loss for an average of 5 years. Which one of the following behaviors is typical of these individuals? (check one) A. Eating breakfast every day B. Taking daily vitamin and mineral supplements C. Being physically active >2 hours a day D. Eating a low-protein diet

A. Eating breakfast every day Individuals on the National Weight Control Registry typically 1. eat a low-fat diet rich in complex carbohydrates 2. eat breakfast daily 3. weigh themselves at least once a week 4. are physically active for 60-90 minutes a day.

A 50-year-old male has a pre-employment chest radiograph showing a pulmonary nodule. There are no previous studies available. Which one of the following would raise the most suspicion that this is a malignant lesion if found on the radiograph? . (check one) A. The absence of calcification B. Location above the midline of the lung C. A diameter of 4 mm D. A solid appearance

A. The absence of calcification benign nodules include - diameter <5 mm - smooth border - solid appearance, concentric calcification, and a doubling time of less than 1 month or more than 1 year. Features of malignant nodules - a size >10 mm, an irregular border - "ground glass" appearance - no calcification or an eccentric calcification, and - a doubling time of 1 month to 1 year

When obtaining informed consent from a patient, which one of the following is NOT required for a patient to legally have decision-making capacity? (check one) A. The absence of mental illness B. The ability to express choice C. The ability to understand relevant information D. The ability to engage in reasoning E. The ability to appreciate the significance of information and its consequences

A. The absence of mental illness

A 34-year-old female with a history of bilateral tubal ligation consults you because of excessive body and facial hair. She has a normal body weight, no other signs of virilization, and regular menses. Which one of the following is the most appropriate treatment for her mild hirsutism? (check one) A. Spironolactone (Aldactone) B. Leuprolide (Lupron) C. Prednisone D. Metformin (Glucophage)

A. Spironolactone (Aldactone) Antiandrogens such as spironolactone, along with oral contraceptives, are recommended for treatment of hirsutism in premenopausal women (SOR C). Leuprolide, although better than placebo, has many side effects and is expensive.

A 50-year-old male comes to your office for a "doctor's excuse" for days of work he missed last week. He attended a picnic where he and other guests developed nausea and vomiting 2 hours after eating. Within 48 hours, the symptoms had resolved. The most likely etiology of the illness is which one of the following? (check one) A. Staphylococcus B. Clostridium botulinum C. Clostridium perfringens D. Clostridium difficile E. Actinomycosis

A. Staphylococcal food poisoning Not botulism - in home canned foods - symptoms begin 18-36 hours after ingestion. Not Clostridium perfringens - transmitted in feces and water, and symptoms begin 6-24 hours after ingestion Not Clostridium difficile - associated with antibiotic use. Not Actinomycosis - causes local abscesses, not gastroenteritis.

Which one of the following is true regarding death certificates? (check one) A. The immediate cause of death is the final or terminal cause of death, such as cardiac arrest B. A physician can certify a death from a natural cause but a coroner or medical examiner must certify a death due to any other cause C. In a case of unknown or probable cause of death, the manner of death is designated as "uncertain" D. Death certificates are part of the patient's medical record and, as such, are confidential and regulated by HIPAA laws E. In a case of death due to an accidental fall, the immediate attending physician must complete the death certificate

B. A physician can certify a death from a natural cause but a coroner or medical examiner must certify a death due to any other cause

Patient-centered medical home is a term used to describe which one of the following developments in medical care? (check one) A. A federally imposed restriction on family medicine's role in providing care B. A physician-led team of care providers taking responsibility for the quality and safety of an individual's health C. A "practice without walls" that provides primary care services in the homes of patients D. A small group of patients paying an annual fee to have a physician be available to them at all times E. Improving the dignity of care for nursing-home residents

B. A physician-led team of care providers taking responsibility for the quality and safety of an individual's health

Which one of the following is most likely to be seen with diastolic dysfunction? (check one) A. A dialated left ventricle B. A preserved ejection fraction C. Aortic insufficiency D. Pericardial effusion

B. A preserved ejection fraction Heart failure due to diastolic dysfunction occurs in the older population. The criteria for diastolic heart failure include symptoms and signs consistent with heart failure (including dyspnea), a nondilated left ventricle with a preserved ejection fraction (³50%), and evidence of structural heart disease such as diastolic dysfunction on echocardiography (SOR C).

A 50-year-old male sees you for a health maintenance visit. He has not been to a physician for 5 years because he feels very healthy and believed he was up-to-date on all preventive screenings. You review his medical record and notice he has never had an HIV screening test. On further questioning you confirm that he is at very low risk for contracting HIV. Based on recommendations from the U.S. Preventive Services Task Force, you tell him that you routinely conduct opt-out HIV screening for (check one) A. all patients age 5 to 75 B. all patients age 15 to 65 C. all patients younger than 50, and patients 50 or older who are at high risk D. only patients at high risk for HIV, regardless of age

B. All patients 15-65

Which one of the following antihypertensive drugs is most likely to cause ankle edema? (check one) A. Hydrochlorothiazide B. Amlodopine (Norvasc) C. Lisinopril (Prinivil, Zestril) D. Losartan (Cozar) E. Atenolol (Tenormin)

B. Amlodopine (Norvasc) - The most common side effects of calcium channel blockers, such as amlodipine, are due to vasodilation - One result of this may be peripheral edema, but it can also cause dizziness, nausea, hypotension, cough, and pulmonary edema. - These problems may decrease with time, with reductions in dosage, or with the addition of a diuretic or second calcium antagonist - Other classes of drugs are not associated with these problems.

23-year-old healthy male is sexually active with other men and does not use condoms. He is interested in reducing his risk of contracting HIV by using a daily oral antiretroviral medication. Which one of the following laboratory tests should be done no more than 7 days before initially prescribing pre-exposure prophylaxis with emtricitabine/tenofovir disoproxil (Truvada)? (check one) A. A CD4 cell count B. Antibody testing for HIV C. Hemoglobin concentration D. A platelet count E. An ALT level

B. Antibody testing for HIV - has to do it b/f pre-exposure prophylaxis (PrEP) (no more than 7 days prior) - via serum or fingerstick - emtricitabine/tenofovir is not good enough for HIV infection & cause resistance - other testing: Cr clearance, HBV antibody, STD screening, pregnancy test

A 50-year-old male is brought to the emergency department with shortness of breath, chest tightness, tremulousness, and diaphoresis. Aside from tachypnea, the physical examination is normal. Arterial blood gases on room air show a pO2 of 98 mm Hg (N 80-100), a pCO2 of 24 mm Hg (N 35-45), and a pH of 7.57 (N 7.38-7.44). The most likely cause of the patient's blood gas abnormalities is: (check one) A. carbon monoxide poisoning B. anxiety disorder with hyperventilation C. an acute exacerbation of asthma D. pulmonary embolus E. pneumothorax

B. Anxiety with hyperventilation - resp alkalosis Pulmonary embolism - pO2 and pCO2 are decreased, while the pH is elevated (acute resp alkalosis) Carbon monoxide poisoning - vitals will be normal Asthma exacerbation - prominent cough and wheezing, and possibly other abnormalities. Tension pneumothorax - significant physical findings including tachycardia, hypotension, and decreased mental activity.

Which one of the following sonographic measurements is most accurate for estimating gestational age? (check one) A. Amniotic sac size at 5 weeks of pregnancy B. Crown-rump length at 10 weeks of pregnancy C. Femur length at 16 weeks of pregnancy D. Biparietal diameter at 20 weeks of pregnancy E. Abdominal circumference at 24 weeks of pregnancy

B. Crown-rump length at 10 weeks of pregnancy - Estimation of gestational age by ultrasound is most accurate early in the first trimester and begins to *decline by 22 weeks gestation* - before *13 weeks* Crown-rump length - After *11 weeks* gestation, combinations of biparietal diameter, femur length, head circumference, and abdominal circumference are used to estimate the gestational age.

Which one of the following is a common cause of prerenal acute kidney injury? (check one) A. Acute tubular necrosis B. Diuretic overuse C. Glomerulonephritis D. Neurogenic bladder E. Prostate hypertrophy

B. Diuretic overuse

Pallidotomy is a surgical therapy for: (check one) A. Alzheimer's disease B. Parkinson's disease C. Huntington's chorea D. Vascular dementia E. Temporal lobe epilepsy

B. Parkinson's disease Thalamotomy and pallidotomy, contralateral to the side of the body that is most affected, are most effective for the treatment of disabling unilateral tremor and dyskinesia from Parkinson's disease.

A 15-year-old male is brought to the office for a well child visit. His parents report that he has had a nighttime cough and wheezing for the past several months. He is otherwise healthy and up-to-date on all of his immunizations. You suspect that he has asthma. Which one of the following would be most appropriate at this point? (check one) A. Treat empirically with a short-acting $-agonist B. Perform spirometry C. Order radiologic testing D. Start an inhaled corticosteroid E. Start a leukotriene inhibitor

B. Perform spirometry

A mother brings her 7-year-old son in for a well child check and you find that their main concern is bedwetting. He has never achieved consistent nighttime continence. He currently wets the bed about 4 nights per week but has no difficulty maintaining continence during the day and reports no symptoms such as dysuria or urinary frequency. The parents have tried limiting his evening fluid intake but this has not helped. He is otherwise healthy. The patient wants to stop wearing nighttime diapers. Which one of the following interventions has the best evidence of long-term success in addressing this condition? (check one) A. A reward system for achieving dry nights B. Use of a bed alarm C. Desmopressin (DDAVP) D. Imipramine (Tofranil) E. Oxybutynin

B. Use of a bed alarm

In a patient with hyperuricemia who has experienced an attack of gout, which one of the following is LEAST likely to precipitate another gout attack? (check one) A. Red meat B. Milk C. Seafood D. Nuts E. Beans

B. milk - milk, unlike other dairy product, decrease gout attack - nuts and beans are high in purines & worsens gout

A 12-year-old white male asthmatic has an acute episode of wheezing. You diagnose an acute asthma attack and prescribe an inhaled β2-adrenergic agonist. After 2 hours of treatment, he continues to experience wheezing and shortness of breath. Which one of the following is the most appropriate addition to acute outpatient management? (check one) A. Oral theophylline (Theo-Dur) B. Oral corticosteroids C. An oral β-adrenergic agonist D. Inhaled cromolyn (Intal) E. Inhaled corticosteroids

B. oral corticosteroids - 1st: β2-adrenergic agonist such as albuterol, terbutaline, or pirbuterol. - if ineffective: systemic corticosteroids. Theophylline - useless in acute attacks - less potent bronchodilator - adverse effects such as nausea and central nervous system stimulation in patients Cromolyn - can decrease airway hyperreactivity - no bronchodilating activity - only useful as prophylaxis. Inhaled corticosteroids - should be used to suppress the symptoms of chronic persistent asthma. Oral β2-selective agonists - less effective and have a slower onset of action than the same drugs given by inhalation.

Which one of the following Papanicolaou (Pap) test results is most likely to indicate a cancerous lesion? (check one) A. Atypical squamous cells of undetermined significance (ASC-US) B. Atypical squamous cells cannot exclude high-grade intraepithelial lesion (ASC-H) C. Atypical glandular cells not otherwise specified (AGC-NOS) D. Low-grade squamous intraepithelial lesion (LSIL) E. High-grade squamous intraepithelial lesion (HSIL)

C. Atypical glandular cells not otherwise specified (AGC-NOS) - associated with a 17% rate of cancer (8% carcinoma in situ and 9% invasive carcinoma). - High-grade squamous intraepithelial lesion (HSIL), which would seem worse intuitively, has only a 3% associated cancer rate.

A 30-year-old female comes to your office because she is concerned about irregular menses (fewer than 9/year), acne, and hirsutism. Her BMI is 36.0 kg/m2. She has no other medical problems and would like to have a baby. Her fasting blood glucose level is 140 mg/dL. Which one of the following would be the most appropriate treatment for this patient's condition and concerns? (check one) A. Lifestyle modification only B. Lifestyle modification and pioglitazone (Actos) C. Lifestyle modification and metformin (Glucophage) D. Lifestyle modification and an oral contraceptive E. Lifestyle modification and oral testosterone

C. lifestyle modification + Metformin This patient has classic features of polycystic ovary syndrome (PCOS). The diagnosis is based on the presence of two of the following: - oligomenorrhea or amenorrhea - clinical or biochemical hyperandrogenism - polycystic ovaries visible on ultrasonography.

A U.S. Preventive Services Task Force "D" recommendation indicates (check one) A. high certainty that the net benefit is substantial B. high certainty that the net benefit is moderate C. moderate or high certainty that the service has no net benefit or that the harms outweigh the benefits D. that the decision to provide the service should be based on professional judgment and patient preferences E. that current evidence is insufficient to assess the balance of benefits and harms of the service

C. moderate or high certainty that the service has no net benefit or that the harms outweigh the benefits

The most common source of chest pain in children is (check one) A. pulmonary B. cardiac C. musculoskeletal D. gastroesophageal E. psychogenic

C. musculoskeletal

Which one of the following nutritional management strategies is associated with better outcomes in patients with mild acute pancreatitis whose pain and nausea have resolved? (check one) A. Waiting until lipase has normalized before beginning oral intake B. Early initiation of a clear liquid diet C. Early initiation of a low-fat diet D. Early initiation of tube feeding E. Early initiation of total parenteral nutrition 70

D. Early low-fat diet - bowel rest is associated with intestinal mucosal atrophy and increased infectious complications because of bacterial translocation from the gut - patients who are provided oral feeding early in the course of acute pancreatitis have a shorter hospital stay, decreased infectious complications, decreased morbidity, and decreased mortality.

A previously healthy 24-year-old female presents with a 10-day history of facial pain and fever. On examination she has tenderness over the maxillary sinus on the left. Which one of the following would be most appropriate for treatment of this patient's condition? (check one) A. Intranasal saline flushes B. Intranasal antihistamines C. Oral antihistamines D. Oral antibiotics E. Reassurance only

D. Oral antibiotics - acute bacterial sinusitis (ABS) - symptoms of at least 10 days without improvement should be treated with antibiotics - Signs and symptoms may include nasal drainage and congestion, facial pressure and/or pain, sinus tenderness, and headache - can be amoxicillin alone; or going directly to amoxicillin/clavulanate - alternatives: "respiratory" quinolone or combination of a third-generation cephalosporin and clindamycin n pt w/ PCN allergy. - Due to the increasing emergence of resistant Strept and H. flu species, neither trimethoprim/sulfamethoxazole nor macrolides are now recommended for empiric treatment of ABS.

The most common cause of proteinuria in children is: (check one) A. Acute postinfectious glomerulonephritis B. Lupus glomerulonephritis C. Hydronephrosis D. Orthostatic proteinuria E. Reflux nephropathy

D. Orthostatic proteinuria accounts for up to 60% of all cases of asymptomatic proteinuria reported in children, with an even higher incidence in adolescents.

A 67-year-old male presents with a persistent, intermittent cough. He says that his exercise tolerance has decreased, noting that he becomes short of breath more easily while playing tennis. He smoked briefly while in college but has not smoked for over 45 years, and reports no history of known pulmonary disease. You obtain pulmonary function testing in the office to help you diagnose and manage his respiratory symptoms. His FVC and FEV1/FVC are both less than the lower limit of normal as defined by the Third National Health and Nutrition Examination Survey. Repeat testing following administration of a bronchodilator does not correct these values. Which one of the following would be most appropriate at this time? (check one) A. A methacholine challenge test B. A mannitol inhalation challenge test C. Exercise pulmonary function testing D. Testing for diffusing capacity of the lung for carbon monoxide (DLCO)

D. Testing for diffusing capacity of the lung for carbon monoxide (DLCO)

An otherwise healthy 40-year-old male comes to your office for follow-up of elevated liver enzymes on an insurance examination. He is 173 cm (68 in) tall and weighs 113 kg (250 lb) (BMI 37.7 kg/m2). He says he drinks about two beers per week. Findings are normal on a physical examination, except for a slightly enlarged liver. AST and ALT levels are twice the upper limits of normal. Which one of the following would be the most appropriate next step? (check one) A. A liver biopsy B. Ultrasonography of the liver C. Colonoscopy D. Testing for viral hepatitis E. Repeat AST and ALT levels in 3 months

D. Testing for viral hepatitis - hep B & hep C should be ruled out; even if Nonalcoholic fatty liver disease is the most likely diagnosis - pt's alcohol consumption of less than two drinks per week makes alcoholic fatty liver disease unlikely - A liver biopsy would not be appropriate at this time - Liver ultrasonography should be considered after hepatitis B and C are ruled out. - The patient is younger than the recommended screening age for colonoscopy.

A health-care worker has a negative tuberculin skin test (Mantoux method). A second test 10 days later is positive. This result indicates: (check one) A. Previous vaccination with BCG B. A false-positive skin test C. Recent conversion D. Long-standing, latent infection E. Probable immunodeficiency

D. long-standing, latent infection (no explanation)

Hantavirus pulmonary syndrome results from exposure to the excreta of: (check one) A. migratory fowl B. bats C. parrots D. mice E. turtles

D. mice Hantavirus pulmonary syndrome results from exposure to rodent droppings, mainly the deer mouse in the southwestern U.S

Which one of the following is the recommended duration of dual antiplatelet therapy after placement of a drug-eluting coronary artery stent? (check one) A. 1 week B. 1 month C. 2 months D. 3 months E. 1 year

E. 1 year - dual antiplatelet therapy: aspirin, 162-325 mg, and clopidogrel, 75 mg, or prasugrel, 10 mg - Ticlopidine is an option for patients who do not tolerate clopidogrel or prasugrel - The minimum recommended duration of dual antiplatelet therapy is: 1 month with bare-metal stents, 3 months with sirolimus-eluting stents, and 6 months with other drug-eluting stents. Dual antiplatelet therapy should extend beyond 1 year for patients with acute coronary syndrome who are not considered at high risk of bleeding, especially those with risk factors associated with high ischemic risk such as diabetes mellitus, peripheral artery disease, left main stenting, or a history of a cardiovascular event. For dual antiplatelet therapy that continues beyond a year, either ticagrelor, 60 mg twice daily, or clopidogrel, 75 mg daily, is recommended in addition to aspirin. The patient's bleeding and ischemic risk should be reevaluated at least annually.

Surgical management for an acute midshaft clavicle fracture would be appropriate in which one of the following? (check one) A. An 11-year-old male with a comminuted fracture B. A 15-year-old female with a ½-cm displaced fracture C. A 30-year-old male with a ½-cm displaced fracture D. A 40-year-old male with a nondisplaced fracture E. A 50-year-old female with a comminuted fracture

E. A 50-year-old female with a comminuted fracture - Midshaft clavicle fractures are usually treated nonoperatively, but have a higher risk of nonunion. - Risk factors for nonunion 1. female gender 2. fracture comminution or displacement (comminute = 2 or more pieces of bones) 3. clavicle shortening 4. advanced age 5. greater extent of initial trauma - children heal extremely well, even if displaced or comminuted, because of periosteal regenerative potential.

A 51-year-old immigrant from Vietnam presents with a 3-week history of nocturnal fever, sweats, cough, and weight loss. A chest radiograph reveals a right upper lobe cavitary infiltrate. A PPD produces 17 mm of induration, and acid-fast bacilli are present on a smear of induced sputum. While awaiting formal laboratory identification of the bacterium, which one of the following would be most appropriate? (check one) A. Observation only B. INH only C. INH and ethambutol (Myambutol) D. INH, ethambutol, and pyrazinamide E. INH, ethambutol, rifampin (Rifadin), and pyrazinamide

E. INH, ethambutol, rifampin (Rifadin), and pyrazinamide Treatment regimens can be modified once culture results are available.

Screening for chronic hepatitis B infection is NOT recommended for which one of the following? (check one) A. Patients on chronic immunosuppressive therapy B. Patients with end-stage renal disease who are on hemodialysis C. Household contacts of individuals with chronic hepatitis B D. Pregnant women with no risk factors for hepatitis B E. All newborns

E. All newborns Screen: - Hemodialysis pts - Household contacts of those with chronic hepatitis B - Pts on immunosuppressive therapy -All pregnant women - IV drug users - Pts from areas where HBsAg prevalence >2%

A 75-year-old otherwise healthy white female states that she has passed out three times in the last month while walking briskly during her daily walk with the local senior citizens mall walkers' club. This history would suggest which one of the following as the etiology of her syncope? (check one) A. Vasovagal syncope B. Transient ischemic attack C. Orthostatic hypotension D. Atrial myxoma E. Aortic stenosis

E. Aortic stenosis - Syncope with exercise is a manifestation of organic heart disease in which cardiac output is fixed and does not rise (or even fall) with exertion. - Syncope, commonly on exertion, is reported in up to 42% of patients with severe aortic stenosis. Not Vasovagal syncope - is associated with unpleasant stimuli or physiologic conditions - e.g. sights, sounds, smells, sudden pain, sustained upright posture, heat, hunger, and acute blood loss. Not Transient ischemic attacks - not related to exertion Not Orthostatic hypotension - associated with changing from a sitting or lying position to an upright position Not Atrial myxoma - associated with syncope related to changes in position e.g. bending, lying down from a seated position, or turning over in bed.

Mild cognitive impairment is characterized by which one of the following? (check one) A. Localized motor dysfunction B. Impairment in at least one activity of daily living C. Impairment in at least one instrumental activity of daily living D. The presence of the APO E4 allele E. Objective evidence of memory decline

E. Objective evidence of memory decline - intermediate stage between normal cognitive function and dementia - Motor function remains normal - pts have normal functional activities - pts knows of cognitive decline Not APO E4 allele - risk factor but is not necessary for a diagnosis.

Examination of a 2-day-old infant reveals flesh-colored papules with an erythematous base located on the face and trunk, containing eosinophils. Which one of the following would be most appropriate at this time? (check one) A. An allergy evaluation B. Low-dose antihistamines C. Hydrocortisone cream 0.5% D. A sepsis workup E. Observation only

E. Observation only - erythema toxicum neonatorum

A 36-year-old female presents with a several-week history of polyuria and intense thirst. She currently takes no medications. On examination her blood pressure and pulse rate are normal, and she is clinically euvolemic. Laboratory tests, including serum electrolyte levels, renal function tests, and plasma glucose, are all normal. A urinalysis is significant only for low specific gravity. Her 24-hour urine output is >5 L with low urine osmolality. The most likely cause of this patient's condition is a deficiency of: (check one) A. angiotensin II B. aldosterone C. renin D. insulin E. arginine vasopressin

E. Arginine vasopressin - diabetes insipidus - caused by a deficiency in the secretion or renal action of arginine vasopressin (AVP). - AVP, also known as antidiuretic hormone, is produced in the posterior pituitary gland and the route of secretion - its chief action is the concentration of urine in the distal tubules of the kidney.

A 42-year-old asymptomatic female presents for a routine evaluation. On examination her uterus is irregularly enlarged to the size seen at approximately 8 weeks gestation. Pelvic ultrasonography shows several uterine fibroid tumors measuring <5 cm. The patient does not desire future fertility. Which one of the following would be the most appropriate treatment option? (check one) A. Laparoscopic myomectomy B. Hysterectomy C. A gonadotropin-releasing hormone (GnRH) agonist D. An oral contraceptive E. Observation

E. Observe - uterine fibroids - asymptomatic fibroids - treatment options not really studied - Symptoms: include menorrhagia, pelvic pain, obstructive symptoms, infertility, or pregnancy loss - many fibroids are asymptomatic and are discovered incidentally, with expectant management being the treatment of choice in this situation (SOR B) - risk of malignant leiomyosarcoma is exceedingly small .

A 33-year-old female comes to your office for follow-up of irritable bowel syndrome. You ruled out other causes of her abdominal bloating, abdominal pain, and diarrhea at earlier visits. She has no change in symptoms, such as constipation or blood in her stool. She has resisted treatment in the past, but her symptoms are becoming more frequent and she would now like to consider treatment. Evidence shows that which one of the following would most likely be beneficial for this patient? (check one) A. Acupuncture B. Increased insoluble fiber in her diet C. Fluoxetine (Prozac), 20 mg daily D. Neomycin, 1000 mg every 6 hours for 7 days E. Polyethylene glycol (MiraLAX), 17 g daily

This patient has diarrhea-predominant irritable bowel syndrome (IBS). There are many treatments available, with varying degrees of evidence. SSRIs, along with tricyclic antidepressants, have been shown to decrease abdominal pain and improve global assessment scores in those with IBS. Polyethylene glycol is a treatment for constipation and would not help this patient. Acupuncture has not been shown to be superior to sham acupuncture in improving IBS symptoms. Neomycin has been shown to improve symptoms in constipation-predominant IBS but would not be helpful in diarrhea-predominant IBS. Soluble fiber such as psyllium improves symptoms and decreases abdominal pain scores in patients with IBS. Insoluble fiber has not been shown to improve any IBS outcomes.

A 56-year-old white male reports lower leg claudication that occurs when he walks approximately one block and is relieved by standing still or sitting. He has a history of diabetes mellitus and hyperlipidemia. His most recent hemoglobin A1c was 5.9% and his LDL-cholesterol level at that time was 95 mg/dL. Current medications include glyburide (DiaBeta), metformin (Glucophage), simvastatin (Zocor), and daily aspirin. He stopped smoking 1 month ago and began a walking program. A physical examination is normal except for barely palpable dorsalis pedis and posterior tibial pulses. Femoral and popliteal pulses are normal. Noninvasive vascular studies of his legs show an ankle-brachial index of 0.7 bilaterally and decreased flow. Which one of the following would be most appropriate for addressing this patient's symptoms? (check one) A. Fish oil B. Warfarin (Coumadin) C. Cilostazol (Pletal) D. Dipyridamole (Persantine) E. Clopidogrel (Plavix)

- The patient described has symptomatic arterial vascular disease manifested by intermittent claudication. He has already initiated the two most important changes: he has stopped smoking and started a walking program. His LDL-cholesterol is at target levels; further lowering is not likely to improve his symptoms. In the presence of diffuse disease, interventional treatments such as angioplasty or surgery may not be helpful; in addition, these interventions should be reserved as a last resort. -Cilostazol has been shown to help with intermittent claudication.

A 14-year-old female with a history of asthma is having daytime symptoms about once a week and symptoms that awaken her at night about once a month. Her asthma does not interfere with normal activity, and her FEV1 is >80% of predicted. Which one of the following is the most appropriate treatment plan for this patient? (check one) A. A short-acting inhaled β-agonist as needed B. Low-dose inhaled corticosteroids daily C. A leukotriene receptor antagonist daily D. Medium-dose inhaled corticosteroids daily E. Low-dose inhaled corticosteroids plus a long-acting inhaled β-agonist daily

A. A short-acting inhaled β-agonist as needed - intermittent asthma.

You would recommend pneumococcal vaccine for which one of the following? (check one) A. A 20-year-old male who smokes 1 pack of cigarettes daily B. A 52-year-old male with type 2 diabetes mellitus who received pneumococcal vaccine 6 years ago C. A 60-year-old male who is a long-term resident of a nursing home because of a previous stroke, and who received pneumococcal vaccine at age 54 D. A 62-year-old male with chronic renal failure who received pneumococcal vaccine at age 50 and age 55 E. A 71-year-old male with no medical problems who received pneumococcal vaccine at age 65

A. A 20 yr male 1 ppd smoker - cigarette smoking = high-risk conditions that are indications for PCV23 (23 valent pneumococcal polysaccharide vaccine) - All persons between the ages of 19 and 64 who smoke should receive this vaccine. - One-time revaccination after 5 years is recommended only if persons with - chronic renal failure - asplenia (functional or anatomic) - other immunocompromising conditions. - The patient with chronic renal failure in this question has already received two immunizations. - - diabetic patient and the nursing-home resident have both received one immunization and should not receive a second dose until age 65 - The 71-year-old has already been immunized after age 65, 2nd one not needed

A 60-year-old Chinese female asks you about being tested for osteoporosis. She is postmenopausal and has never used hormone therapy. She does not consume dairy products because she has lactose intolerance. She is on no medications, is otherwise healthy, and has no history of falls or fractures. Her mother had osteoporosis and vertebral compression fractures. Her BMI is 20 kg/m2 . Which one of the following tests would be best to determine whether this patient has osteoporosis? (check one) A. A central DXA scan of the lumbar spine and hips B. A forearm DXA scan C. Quantitative CT of the lumbar spine D. Quantitative calcaneal ultrasonography E. Measurement of biochemical markers of bone turnover in the urine

A. A central DXA scan of the lumbar spine and hips risk factors - Asian ethnicity, low body weight, positive family history, postmenopausal status with no history of hormone replacement, and low calcium intake - The best diagnostic test: central DXA scan of the hip, femoral neck, and lumbar spine - Quantitative CT is accurate, but cost and radiation exposure are issues. - Peripheral DXA and calcaneal sonography results do not correlate well with central DXA. - Measurement of biochemical markers is not recommended for the diagnosis of osteoporosis.

Which one of the following is an indication for a second dose of pneumococcal polysaccharide vaccine in children? (check one) A. A cerebrospinal fluid leak B. Cyanotic congenital heart disease C. Type 1 diabetes mellitus D. Sickle cell disease E. Chronic bronchopulmonary dysplasia

A. A cerebrospinal fluid leak Individuals with sickle cell disease, those with anatomic or functional asplenia, immunocompromised persons with renal failure or leukemia, and HIV-infected persons should receive polysaccharide vaccine on this same schedule and should also be revaccinated at least 3 years after the first dose.

A 45-year-old male with Down syndrome is brought to your office because of complaints of increased aggression toward the staff and peers at his group home. He is usually pleasant and compliant but he has been acting out for the last 2½ weeks. He is not considered to be a danger to himself or others at this point. He is minimally verbal and unable to give a history for himself. Staff members report no change in appetite or urination, and no signs of outward illness. His vital signs in your office are within normal limits. Which one of the following would be most appropriate at this point? (check one) A. A complete history, physical examination, and basic laboratory tests B. CT of the head C. Risperidone (Risperdal) D. Sertraline (Zoloft) E. Valproic acid (Depakene)

A. A complete history, physical examination, and basic laboratory tests

Which one of the following is diagnostic for type 2 diabetes mellitus? (check one) A. A fasting plasma glucose level ≥126 mg/dL on two separate occasions B. An oral glucose tolerance test (75-g load) with a 2-hour glucose level ≥160 mg/dL C. A random blood glucose level ≥200 mg/dL on two occasions in an asymptomatic person D. A hemoglobin A 1c ≥6.0% on two separate occasions

A. A fasting plasma glucose level ≥126 mg/dL on two separate occasions - hemoglobin A1c≥6.5% - fasting plasma glucose level ≥126 mg/dL - 2-hour plasma glucose level ≥200 mg/dL - in a symptomatic patient, a random blood glucose level 200 mg/dL

A 30-year-old female with abnormal uterine bleeding asks about treatment options. An examination is normal and blood testing is negative. She is unmarried and is undecided about having children. Which one of the following would be the most appropriate treatment for this patient? (check one) A. A levonorgestrel-releasing intrauterine device B. Endometrial ablation C. Hysterectomy D. Oral progestin during the luteal phase

A. A levonorgestrel-releasing intrauterine device - Progestin is effective when used on a 21-day cycle, but not if used only during the luteal phase. - In a young woman unsure about having children, the levonorgestrel releasing IUD is most effective and preserves fertility.

A 71-year-old female comes in for follow-up of hypertension. She is worried about her heart and says that some of her friends have had stress tests and she would like to get one as well just to be on the safe side. She has no chest pain, shortness of breath, or exercise intolerance, and a complete review of systems is negative. The patient's current medications include lisinopril (Prinivil, Zestril), 20 mg daily; metoprolol succinate (Toprol-XL), 25 mg daily; and omeprazole (Prilosec), 20 mg daily. Her past medical history includes hypertension, obesity, and gastroesophageal reflux disease. A physical examination reveals a blood pressure of 130/70 mm Hg, a heart rate of 90/min, and a BMI of 31.2 kg/m2. An EKG 2 years ago was normal. Which one of the following should be ordered to assess this patient's cardiovascular risk? (check one) A. A lipid profile B. A coronary artery calcification score C. A C-reactive protein level D. An EKG E. An exercise stress test

A. A lipid profile The U.S. Preventive Services Task Force does not recommend resting or stress EKG testing for asymptomatic low-risk patients (D recommendation); also has false positive . Asymptomatic patients should be risk stratified to assess the risk of chronic heart disease, and this patient should have a lipid profile for risk stratification. Low-risk patients do not benefit from nontraditional risk assessments, including high-sensitivity C-reactive protein or coronary artery calcium assessment.

A 55-year-old white male comes to your office with weakness and a headache. He also describes an annoying pruritus that occurs frequently after he takes a hot shower. The physical examination is remarkable for the presence of an enlarged spleen. He has a hemoglobin level of 21 g/dL (N 12-16) and a hematocrit of 63% (N 36-48). To confirm your clinical diagnosis, you obtain additional studies. Which one of the following would be most consistent with the most likely diagnosis in this patient? (check one) A. A low serum erythropoietin level B. A low platelet count C. A low arterial oxygen concentration D. An elevated carboxyhemoglobin level

A. A low serum erythropoietin level - polycythemia vera. - Pruritus after a hot shower (aquagenic pruritus) and splenomegaly distinguish polycythemia vera from other causes of erythrocytosis (hematocrit >55%). - Specific criteria: elevated red cell mass, a normal arterial oxygen saturation (>92%) - a low serum erythropoietin level - thrombocytosis (platelet count >400,000/mm3 ), leukocytosis (WBC>12,000/mm3 ) - high leukocyte alkaline phosphatase score - High carboxyhemoglobin levels are associated with secondary polycythemia.

A 62-year-old African-American female undergoes a workup for pruritus. Laboratory findings include a hematocrit of 55.0% (N 36.0-46.0) and a hemoglobin level of 18.5 g/dL (N 12.0-16.0). Which one of the following additional findings would help establish the diagnosis of polycythemia vera? (check one) A. A platelet count >400,000/mm3 B. An O2 saturation <90% C. A WBC count <4500/mm (N 4300-10,800)3 D. An elevated uric acid level

A. A platelet count >400,000/mm3 Major criteria - increased red cell mass, a normal O2 saturation, splenomegaly Minor criteria -high vitamin B 12, elevated leukocyte alkaline phosphatase, a platelet count >400,000/mm3 and a WBC count >12,000/mm3 . - Patients with polycythemia vera may present with gout and an elevated uric acid level, but neither is considered a criterion for the diagnosis.

When compared to a figure-of-eight dressing, which one of the following modalities of treatment has been shown to have similar fracture-healing outcomes and increased patient satisfaction for nondisplaced mid-shaft clavicular fractures? (check one) A. A shoulder sling B. A short arm cast C. A long arm cast D. Operative fixation

A. A shoulder sling a sling has been shown to have similar fracture healing rates in patients with a nondisplaced midshaft clavicular fracture. In addition, a figure-of-eight dressing is uncomfortable and difficult to adjust, and patients have reported increased satisfaction when treated with a sling. Long and short arm casts are not appropriate options to manage a patient with a clavicular fracture. Operative treatment is an option to treat displaced midshaft fractures (SOR B)

A 62-year-old male on hemodialysis develops a pruritic rash on his arms and chest, with erythematous, thickened plaques and edema. He had brain imaging with a gadolinium-enhanced MRI for neurologic symptoms 10 days ago. Which one of the following is true regarding this problem? (check one) A. A skin biopsy is diagnostic B. The problem is limited to the skin C. Immediate treatment is critical D. The disease is more common in males E. Death from the disease is unusual

A. A skin biopsy is diagnostic - gadolinium-associated nephrogenic systemic fibrosis - after use of gadolinium-based contrast material in patients with severe renal dysfunction, often on dialysis. - Associated proinflammatory states e.g. surgery, malignancy, and ischemia, are often present as well - Dermatologic manifestations are usually seen, but multiple organ systems may be involved. - There is no effective treatment, and mortality is approximately 30%. A deep biopsy of the affected skin is diagnostic.

A 30-year-old female is being evaluated for chronic pain, fatigue, muscle aches, and sleep disturbance. Which one of the following would be best for making a diagnosis of fibromyalgia? (check one) A. A structured symptom history B. Examination for tender points C. Laboratory testing D. A muscle biopsy E. Electromyography

A. A structured symptom history

Typically, a high-grade squamous intraepithelial lesion (HSIL) of the cervix is treated with ablation or excision. In which one of the following can treatment be deferred? (check one) A. Adolescents B. Patients attempting to conceive C. Patients with a history of three previous normal Papanicolaou smears D. Patients with a negative DNA test for HPV E. Patients over the age of 70

A. Adolescents Treat patients attempting to concieve - takes care of it first because treating it during pregnancy will be dangerous Treat patients with previous normal pap - it is no longer normal now Treat pt negative DNA test - A negative test for HPV can be used to assess the risk of patients with atypical squamous cells of undetermined significance (ASC-US) or a low-grade squamous intraepithelial lesion (LSIL) - it does not change the management of patients with a high-grade intraepithelial lesion (HSIL). Treat pt over 70 y/o

A 78-year-old white male presents to your office with his daughter for a follow-up visit for his diabetes. He has a history of peripheral neuropathy and mild Alzheimer's dementia. He continues to be socially active in his community. He is on several medications, including insulin glargine (Lantus), amitriptyline, donepezil (Aricept), and clonazepam (Klonopin). His daughter asks whether he should continue to drive his car. Which one of the following would be most appropriate in the context of this office visit with regard to evaluating his driving safety? (check one) A. A thorough history focused on the patient's driving, from both him and his daughter B. A written driving test C. A road test to observe his driving D. A letter to the local agency in charge of drivers' licenses advising license removal

A. A thorough history focused on the patient's driving, from both him and his daughter - further testing such as a Snellen eye test, audiometry, timed gait, range of motion, muscle strength, clock drawing, and a Mini-Mental State Examination may also be indicated, but a thorough driving history should be obtained first. If a concern arises about unsafe driving, referral to a rehabilitative driving center or the appropriate government agency for further written/road testing may be warranted.

In the secondary prevention of ischemic cardiac events, which one of the following is most likely to be beneficial in a 68-year-old female with known coronary artery disease and preserved left ventricular function? (check one) A. ACE inhibitors B. Hormone therapy C. Calcium channel blockers D. Vitamin E E. Oral glycoprotein IIb/IIIa receptor inhibitors

A. ACE inhibitors - in patients at high risk after myocardial infarction; angiotensin II receptor blockers in those with coronary artery disease - Effective treatments also include aspirin, β-blockers after myocardial infarction - Amiodarone in patients who have had a myocardial infarction and have a high risk of death from cardiac arrhythmias. - Oral glycoprotein II/IIIa receptor inhibitors increase mortality compared to ASA - Calcium Channel Blockers increase mortality

A patient is admitted to the hospital for acute deep vein thrombosis of the lower extremity and started on anticoagulation therapy. The nursing staff asks for an activity order. Which one of the following should be ordered? A. Activity as tolerated B. Bed rest until the patient has been hospitalized for 24 hours C. Bed rest with bathroom privileges until the patient has been hospitalized for 24 hours D. Bed rest until discharged E. Bed rest with bathroom privileges until discharged

A. Activity as tolerated

A 22-year-old male presents to your office for evaluation of fatigue, poor appetite, and nausea. He states that when he stands too long he often gets dizzy but this is relieved by sitting. His symptoms have been gradually getting worse over the past year. His vital signs are normal but he is found to be orthostatic. A physical examination is unremarkable except for hyperpigmentation in his palmar creases and around his nipples. A basic metabolic panel is notable for a sodium level of 131 mEq/L (N 135-145) and a potassium level of 5.1 mEq/L (N 3.5-5.0). Which one of the following is the most likely cause of this patient's symptoms? (check one) A. Addison's disease B. Cushing syndrome C. Neurally mediated hypotension D. Postural orthostatic hypotension and tachycardia syndrome E. Hypothyroidism

A. Addison's disease - autoimmune adrenalitis in which the adrenal cortex is destroyed - Causes loss of mineralocorticoid, glucocorticoid, and adrenal androgen hormone production. - anorexia, weakness, fatigue, gastrointestinal symptoms, hypotension, salt cravings, postural dizziness, vitiligo, muscle pain, and joint pain - Hyperpigmentation seen in the palmar creases, at the vermillion border of the lips, on the buccal mucosa, around the nipples, and around scars Low serum cortisol measured at 8 a.m. suggests adrenal insufficiency. Hyponatremia may also be seen, due to cortisol and mineralocorticoid deficiencies, and hyperkalemia may occur as a result of the lack of mineralocorticoids. If cortisol is low, a cosyntropin stimulation test is the first-line test for diagnosing adrenal insufficiency. People with Addison's disease require lifelong hormone therapy with glucocorticoids and mineralocorticoids. They also require stress-dose glucocorticoids for illnesses and before surgical procedures because they are unable to mount an adequate response to stress. Generally, the treatment will be prednisone or hydrocortisone along with fludrocortisone.

A 28-year-old gravida 1 para 0 at 39 weeks gestation presents for routine outpatient obstetric care and is found to have a blood pressure of 145/95 mm Hg. A complete review of systems is notable only for chronic low back pain causing poor sleep. The physical examination is normal, including a nontender, gravid uterus and a fetal heart rate of 150 beats/min. The cervical examination reveals firm consistency, 1 cm dilation, 50% effacement, and -3 station. The patient's blood pressure is checked 5 hours later and is 142/94 mm Hg. Based on the 2013 ACOG guidelines for management of hypertension in pregnancy, which one of the following should be the next step in management? (check one) A. Admit the patient for induction of labor B. Measure 24-hour urine protein, with induction of labor if the level exceeds 300 mg C. Begin oral nifedipine (Procardia) and recheck her blood pressure in 24-48 hours D. Place the patient on strict bed rest and check her blood pressure twice weekly E. Begin twice-weekly office visits with assessment for preeclampsia

A. Admit the patient for induction of labor - ACOG recommends induction of labor for gestational hypertension after 37 weeks - elevated urine protein is not required for this decision, as gestational hypertension and preeclampsia without severe features are managed in the same way at 39 weeks gestation

A 3-week-old infant is brought to your office with a fever. He has a rectal temperature of 38.3°C (101.0°F), but does not appear toxic. The remainder of the examination is within normal limits. Which one of the following would be the most appropriate management for this patient? (check one) A. Admit to the hospital and obtain urine, blood, and CSF cultures, then start intravenous antibiotics B. Admit to the hospital and treat for herpes simplex virus infection C. Follow up in the office in 24 hours and admit to the hospital if not improved D. Order a CBC and a urinalysis with culture, and send the patient home if the results are normal

A. Admit to the hospital and obtain urine, blood, and CSF cultures, then start intravenous antibiotics = Any child younger than 29 days old with a fever and any child who appears toxic, regardless of age, should undergo a complete sepsis workup and be admitted to the hospital for observation until culture results are known or the source of the fever is found and treated (SOR C). Observation only, with close follow-up: for nontoxic infants 3-36 months of age with a temperature <39.0°C (102.2°F) (SOR C). Sent home w/ 24h f/u: Children 29-90 days old who appear to be nontoxic and have negative screening laboratory studies, including a CBC and urinalysis (SOR B). Testing for neonatal herpes simplex virus infection: only in patients with risk factors, including maternal infection at the time of delivery, use of fetal scalp electrodes, vaginal delivery, CSF pleocytosis, or herpetic lesions; or pt not responding to abx.

A 40-year-old male is admitted to the hospital with a generalized rash consistent with Stevens-Johnson syndrome. His previous medical problems include obesity, gout, hypertension, type 2 diabetes mellitus, and depression. His medications include lisinopril (Prinivil, Zestril), allopurinol (Zyloprim), colchicine (Colcrys), metoprolol succinate (Toprol-XL), metformin (Glucophage), and venlafaxine (Effexor XR). Which one of these medications is most likely to be the cause of his Stevens-Johnson syndrome? (check one) A. Allopurinol B. Colchicine C. Lisinopril D. Metformin E. Venlafaxine

A. Allopurinol Other drugs commonly cited as causes include antibiotics, antiepileptics, and NSAIDs. Of these, antibiotics are the most common alleged cause of Stevens-Johnson syndrome.

A 25-year-old female has an annular rash on the dorsal surface of both hands. The rash does not respond to initial treatment with an antifungal medication, and a biopsy reveals granuloma annulare. Which one of the following would be the most appropriate advice for this patient? (check one) A. Allow the rash to resolve without further treatment B. Cover the rash because it is contagious C. Treat the rash with systemic corticosteroids D. Treat the rash with a stronger antifungal medication

A. Allow the rash to resolve without further treatment - Granuloma annulaire, usu self-limiting Not corticosteroid PO - injected or topical corticosteroids - need derm referal: many of the potential treatments can have serious side effects.

A 34-year-old female with newly diagnosed diarrhea-predominant irritable bowel syndrome (IBS) presents with worsening abdominal discomfort. Her abdominal discomfort is not severe but it is constant. She has tried dicyclomine (Bentyl) without relief and is interested in trying a different approach. The patient has had negative testing for inflammatory bowel disease and celiac disease, along with normal blood tests. She asks about specific dietary modifications or medications that may be helpful for her abdominal discomfort. Which one of the following interventions would you recommend? (check one) A. Amitriptyline B. Clarithromycin (Biaxin) C. Loperamide (Imodium) D. Increased intake of insoluble dietary fiber

A. Amitriptyline ricyclic antidepressants (TCAs) such as amitriptyline have shown benefit in patients with irritable bowel syndrome (IBS), as have SSRIs. Because of the anticholinergic properties of TCAs it is thought that TCAs may be more beneficial than SSRIs in patients with diarrhea-predominant IBS

A 3-year-old female is brought to your office with coughing and a tactile fever. Her only other symptom is mild rhinorrhea. She has a temperature of 38.2°C (100.8°F) and is mildly tachypneic. Her vital signs are otherwise normal and she appears to be well and in no respiratory distress. Her examination is unremarkable except for decreased breath sounds and crackles in the right lower lung field. She has no allergies to medications. Which one of the following would be the most appropriate treatment? (check one) A. Amoxicillin B. Azithromycin (Zithromax) C. Cefdinir D. Moxifloxacin (Avelox) E. Ceftriaxone (Rocephin)

A. Amoxicillin Amoxicillin is the recommended first-line treatment for previously healthy infants and school-age children with mild to moderate community-acquired pneumonia (CAP) (strong recommendation; moderate-quality evidence). The most prominent bacterial pathogen in CAP in this age group is Streptococcus pneumoniae, and amoxicillin provides coverage against this organism. Azithromycin would be an appropriate choice in an older child because Mycoplasma pneumoniae would be more common. Moxifloxacin should not be used in children. Ceftriaxone and cefdinir can both be used to treat CAP, but they are broader spectrum antibiotics and would not be a first-line choice in this age group.

A 52-year-old male presents with a swollen and tender area anterior to the left ear and extending to below the left angle of the mandible. One week ago he had a Nissen fundoplication for intractable GERD. This was complicated by difficulty swallowing and drinking. On examination his tympanic temperature is 37.7°C (99.9°F), his blood pressure is 110/70 mm Hg, and his pulse rate is 95 beats/min and regular. His left parotid gland is diffusely enlarged and tender. Purulent material is noted coming from the left parotid duct orifice. Which one of the following would be most appropriate at this point? (check one) A. Amoxicillin/clavulanate (Augmentin) B. Penicillin C. CT of the parotid gland D. Incision and drainage of the parotid gland E. Excision of the parotid gland

A. Amoxicillin/clavulanate (Augmentin) - acute parotitis - commonly caused by dehydration and can be diagnosed clinically - Empiric treatment is directed toward gram-positive and anaerobic organisms - most common pathogen being Staph, often penicillin resistant so a β-lactamase inhibitor is the agent of choice; followed by culture - sialagogues such as lemon drops may be helpful, as well as parotid gland massage. Not CT or MRI: not necessary Not Incision and drainage - only for an abscess, Not surgical removal: not indicated.

A 45-year-old white male presents with severe intermittent right flank pain that radiates into his right groin area. You suspect a ureteral stone. Which one of the following would most reliably confirm your suspected diagnosis? (check one) A. A helical CT scan of the abdomen and pelvis without contrast B. Intravenous pyelography C. Abdominal ultrasonography D. A KUB plain film of the abdomen E. A urinalysis

A. An unenhanced helical CT scan of the abdomen and pelvis - A CT scan may also reveal other pathology, such as appendicitis, diverticulitis, or abdominal aortic aneurysm. Not abdominal ultrasonography - sensitivity is much lower; thus, its use is usually confined to pregnant patients with a suspected stone. Not X-ray -only show with radiopaque stones

A healthy 24-year-old male presents with a sore throat of 2 days' duration. He reports mild congestion and a dry cough. On examination, his temperature is 37.2°C (99.0°F). His pharynx is red without exudates, and there are no anterior cervical nodes. His tympanic membranes are normal, and his chest is clear. You would do which one of the following? (check one) A. Treat with analgesics and supportive care B. Treat with azithromycin (Zithromax) C. Perform a throat culture and begin treatment with penicillin D. Perform a rapid strep test

A. Analgesics and supportive care streptococcal pharyngitis w/ Centor criteria. 1. tonsillar exudates 2. tender anterior cervical lymphadenopathy 3. absence of cough 4. history of fever. - three or four of these criteria has a positive predictive value of 40%-60% - absence of three or four of these criteria has a negative predictive value of 80%. 4 positive criteria - abx 3 positive criteria - should be tested and treated if positive 0-1 positive criteria - analgesics and supportive care only

A 16-year-old high-school football player plants his left foot to make a cut and feels his left leg give way. He feels a pop in the knee, followed by acute pain. He is evaluated on the field, and examination with the knee flexed 20° reveals that the tibia can be displaced farther anteriorly than with the uninvolved knee. Which one of the following conditions is most likely? (check one) A. Patellar tendon rupture B. Posterior cruciate ligament tear C. Anterior cruciate ligament tear D. Tibial plateau fracture E. Patellar dislocation

A. Anterior cruciate ligament (ACL) tears - most common ligament injury requiring surgery. - - Females have a significantly higher rate of ACL tears - majority of tears in both men and women occurring without physical contact - significant increase in premature osteoarthritis of the knee - Approximately 50% of patients with this injury develop osteoarthritis in 10-20 years. - Findings that help make the diagnosis of ACL tear 1. noncontact mechanism of injury 2. an audible popping sound 3. early swelling of the joint 4. the inability to participate in the game after the injury - Many patients can walk normally and can perform such straight-plane activities as climbing stairs, biking, or jogging.

Which one of the following potential bioterrorism agents requires treatment with 60 days of continuous antibiotics? (check one) A. Anthrax B. Botulism C. Pneumonic plague D. Smallpox E. Tularemia

A. Anthrax - only anthrax needs 60 days tx (60 days incubation) - oral fluoroquinolones and doxycycline. Smallpox (variola virus) - smallpox vaccine and two compounds currently in development. Pneumonic (Yersinia pestis) - 10-day course of an aminoglycoside or doxycycline. Inhalational botulism - antitoxin (equine-derived heptavalent antitoxin). Tularemia (intentional release of Francisella tularensis) - a 10-day course of an aminoglycoside, ciprofloxacin, or doxy

Which one of the following tests is most specific for diagnosing rheumatoid arthritis? (check one) A. Anti-cyclic citrullinated peptide (anti-CCP) antibody B. Antinuclear antibody C. Erythrocyte sedimentation rate D. Serum complement levels E. Anti-Sm antibody

A. Anti-cyclic citrullinated peptide (anti-CCP) antibody - recommended by rheumatologists to improve the specificity of testing for rheumatoid arthritis. - Anti-CCP is more specific than rheumatoid factor, and may predict erosive disease more accurately. Not Anti-Sm antibody - systemic lupus erythematosus. Not complement levels are seen in many - many rheumatologic disorders Not erythrocyte sedimentation - monitoring disease activity and the course of rheumatoid arthritis, but is not specific.

A 52-year-old female with a 60-pack-year history of cigarette smoking and known COPD presents with a 1-week history of increasing purulent sputum production and shortness of breath on exertion. Which one of the following is true regarding the management of this problem? (check one) A. Antibiotics should be prescribed B. Intravenous corticosteroids are superior to oral corticosteroids C. Inhaled corticosteroids should be started or the dosage increased D. Levalbuterol (Xopenex) is superior to albuterol E. Acetylcysteine should be given if the patient is hospitalized

A. Antibiotics should be prescribed - Brief courses of systemic corticosteroids shorten hospital stays and decrease treatment failures - Studies have not shown a difference between oral and intravenous corticosteroids - Inhaled corticosteroids are not helpful - Levalbuterol and albuterol have similar benefits and adverse effect - Acetylcysteine, a mucolytic agent, not helpful

A 24-year-old female has a history of mood swings over the past several months, which have created marital and financial problems, in addition to jeopardizing her career as a television news reporter. You have made a diagnosis of bipolar disorder, and she has finally accepted the need for treatment. However, she insists that you choose a drug that "won't make me fat." Which one of the following would be best for addressing her concerns? (check one) A. Aripiprazole (Abilify) B. Olanzapine (Zyprexa) C. Quetiapine (Seroquel) D. Risperidone (Risperdal)

A. Aripiprazole - least weight gain generally less than 1 kilogram

A 55-year-old male who had a recent episode of atrial fibrillation that converted in the emergency department is asymptomatic and currently in sinus rhythm. He is in good health otherwise and has no history of hypertension, diabetes mellitus, heart failure, transient ischemic attack, or stroke. Which one of the following would be best for preventing a stroke in this patient? (check one) A. Aspirin B. Clopidogrel (Plavix), 75 mg daily C. Warfarin (Coumadin), with a goal INR of 1.5-2.5 D. Warfarin, with a goal INR of 2.0-3.0 E. Warfarin, with a goal INR of 2.5-3.5

A. Aspirin - CHAD score: C (congestive heart failure), H (hypertension), A (age 75), D (diabetes mellitus), S (secondary prevention for prior ischemic stroke or transient attack—most experts include patients with a systemic embolic event). - Each of these clinical parameters is assigned one point, except for secondary prevention, which is assigned 2 points. - Patients are considered to be at low risk with a score of 0, at intermediate risk with a score of 1 or 2, and at high risk with a score 3. - Experts typically prefer treatment with aspirin rather than warfarin when the risk of stroke is low. -- The patient in this question has a CHADS score of 0, which is low risk. Treatment with aspirin is therefore appropriate.

Which one of the following drugs inhibits platelet function for the life of the platelet? (check one) A. Aspirin B. Ibuprofen C. Dipyridamole (Persantine) D. Ticlopidine (Ticlid) E. Warfarin (Coumadin)

A. Aspirin - permanently acetylating the platelet enzyme cyclooxygenase, thus inhibiting prostaglandin synthesis - The effect of a single aspirin on bleeding times can persist for up to 5 days. - Other NSAIDs (i.e., indomethacin, sulfinpyrazone) also inhibit platelet activity, but their effect on prostaglandin synthesis is reversible Dipyridamole -mechanism not known Warfarin -block prothrombin and vitamin K-dependent coagulation factors - no significant effect on platelet activity.

A 34-year-old G2P0101 at 11 weeks gestation comes to your office to establish care for her pregnancy. In reviewing her history you find that her first pregnancy was complicated by preeclampsia and she required induction of labor at 33 weeks. She also has chronic hypertension treated with chlorthalidone. Her blood pressure today is 128/78 mm Hg. Which one of the following medications, if started today, will lower her risk of preeclampsia in this pregnancy? (check one) A. Aspirin B. Calcium C. Labetalol D. Nifedipine (Procardia) E. Vitamin E

A. Aspirin 60-80 mg daily - reduce pre-eclampsia

A 43-year-old female smoker has type 2 diabetes mellitus, morbid obesity, and a recent diagnosis of symptomatic peripheral arterial disease. You have started her on atorvastatin (Lipitor), offered a supervised exercise program, and discussed smoking cessation and interventions. Which one of the following should be recommended to prevent cardiovascular events in this patient? (check one) A. Aspirin B. Cilostazol (Pletal) C. Enoxaparin (Lovenox) D. Pentoxifylline E. Warfarin (Coumadin)

A. Aspirin Patients with symptomatic peripheral arterial disease should be started on a daily dose of either aspirin or clopidogrel to prevent cardiovascular events such as acute myocardial infarction or stroke (SOR B). Not: Cilostazol - phosphodiesterase inhibitor with both antiplatelet and arterial vasodilatory activity; for claudication Not pentoxifylline - for iclaudication symptoms but is less effective than cilostazol and is reserved as a second-line agent. Neither agent has been shown to decrease cardiovascular events in patients with symptomatic peripheral artery disease. Not enoxaparin nor warfarin - not for symptomatic peripheral artery disease.

A 50-year-old male presents with a 1-day history of fever and chest pain. The chest pain is worse when he is in a supine position and with deep inspiration, and improves when he leans forward. He has no shortness of breath and has never had this problem before. His vital signs are normal except for a temperature of 37.8°C (100.0°F). He has no other medical problems or allergies, and takes no medications. An EKG reveals widespread ST-segment elevation, upright T waves, and PR-segment depression. His troponin level is normal. An echocardiogram is pending. Which one of the following would be the most appropriate treatment for this patient? (check one) A. Aspirin B. Prednisone C. Heparin D. Enoxaparin (Lovenox)

A. Aspirin - acute pericarditis. - Although the EKG findings appear specific for the early stages of pericarditis, myocardial infarction would also be included in the differential diagnosis - A friction rub can be heard in up to 85% of patients with acute pericarditis. - An echocardiogram is often performed to determine the type and amount of effusion. - Conventional therapy for acute pericarditis includes NSAIDs, such as aspirin and ibuprofen. - Recent studies demonstrate that adding *colchicine* to aspirin may be beneficial in reducing the persistence and recurrence of symptoms.

A 70-year-old male presents to your office for a follow-up visit for hypertension. He was started on lisinopril (Prinivil, Zestril), 20 mg daily, 1 month ago. Laboratory tests from his last visit, including a CBC and a complete metabolic panel, were normal except for a serum creatinine level of 1.5 mg/dL (N 0.6-1.5). A follow-up renal panel obtained yesterday shows a creatinine level of 3.2 mg/dL and a BUN of 34 mg/dL (N 8-25). Which one of the following is the most likely cause of this patient's increased creatinine level? (check one) A. Bilateral renal artery stenosis B. Coarctation of the aorta C. Essential hypertension D. Hyperaldosteronism E. Pheochromocytoma

A. Bilateral renal artery stenosis Classic clinical clues: 1. onset of stage 2 hypertension (blood pressure >160/100 mm Hg) after 50 years of age or no family history of hypertension 2. hypertension associated with renal insufficiency, especially if renal function worsens after the administration of an agent that blocks the renin-angiotensin-aldosterone system 3. hypertension with repeated hospital admissions for heart failure 4. drug-resistant hypertension (defined as blood pressure above the goal despite treatment with three drugs of different classes at optimal doses). - The other conditions mentioned do not cause a significant rise in serum creatinine after treatment with an ACE inhibitor.

A 26-year-old female presents with symptoms of anhedonia and anxiousness. Your evaluation leads to a diagnosis of major depressive disorder. The patient consents to medical treatment and counseling, but she is engaged to be married in 2 months and is concerned that antidepressants may lower her libido even further. Which one of the following would be best for reducing the likelihood of sexual dysfunction? (check one) A. Bupropion (Wellbutrin) B. Paroxetine (Paxil) C. Fluoxetine (Prozac) D. Sertraline (Zoloft)

A. Bupropion (Wellbutrin) Worst: Paroxetine (Paxil)

A 3-year-old male is carried into the office by his mother. Yesterday evening he began complaining of pain around his right hip. Today he has a temperature of 37.6°C (99.7°F), cries when bearing weight on his right leg, and will not allow the leg to be moved in any direction. A radiograph of the hip is normal. Which one of the following would be most appropriate at this time? (check one) A. A CBC and an erythrocyte sedimentation rate B. A serum antinuclear antibody level C. Ultrasonography of the hip D. MRI of the hip E. In-office aspiration of the hip

A. CBC & ESR - transient synovitis or septic arthritis of the hip. - septic arthritis: 1. fever over 38.7°C (101.7°F) 2. refuses to bear weight on the leg 3. WBC count >12,000 cells/hpf 4. ESR >40 mm/hr. - If several or all of these conditions exist, aspiration 3 of the hip guided by ultrasonography or fluoroscopy Not MRI - may be helpful in cases that are unclear based on standard data, or if other etiologies need to be excluded.

A 72-year-old male presents with unintentional weight loss of 25 lb over the last 6 months. His history, including a nutritional assessment, is unremarkable, as is his physical examination. His current medications include mirtazapine (Remeron) for depression and hydrochlorothiazide for hypertension.Which one of the following would be the most appropriate next step? (check one) A. Order a CBC, chemistry panel, stool for occult blood, and TSH B. Refer for immediate colonoscopy and esophagogastroduodenoscopy C. Schedule CT of the chest, abdomen, and pelvis D. Start megestrol (Megace) to promote weight gain E. Discontinue mirtazapine

A. CBC, CMP, stool for occult blood, TSH - a rational approach to evaluating weight loss in an elderly patient. - w/u should be directed by findings in the history and physical examination, with special emphasis given to neurologic and psychosocial aspects. - unless history indicates otherwise, standard test should be ordered first

A 55-year-old obese male with hypertension and daytime somnolence is found to have severe obstructive sleep apnea, with an apnea-hypopnea index of 32 on an overnight polysomnogram. Which one of the following is considered to be first-line therapy for this patient's condition? (check one) A. Continuous positive airway pressure (CPAP) B. An oral dental appliance C. Uvulopalatopharyngoplasty D. Sleep positioning therapy E. Tracheostomy

A. CPAP Patients with severe sleep apnea (apnea-hypopnea index >29) and concomitant cardiovascular disease benefit the most from treatment for obstructive sleep apnea. - continuous positive airway pressure (CPAP) is considered first-line therapy for severe apnea.

A 65-year-old male who has been in good health presents to your office with a 2-day history of a sensation of pressure and hearing loss in his left ear. A physical examination and a thorough neurologic examination are both unremarkable. Both tympanic membranes are normal. An audiogram shows a 30-decibel hearing loss at three consecutive frequencies in the left ear, with normal hearing on the right. Placing a vibrating tuning fork in the midline of the forehead reveals sound lateralizing to the right ear. Which one of the following would be most appropriate at this point? (check one) A. CT B. A CBC, metabolic profile, and thyroid studies C. Nifedipine (Procardia) D. Acyclovir (Zovirax) E. Oral corticosteroids

A. CT - Sudden sensorineural hearing loss - diagnosed by audiometry demonstrating a 30-decibel hearing loss at three consecutive frequencies, with no other cause indicated from the physical examination. - Evaluation for retrocochlear pathology may include auditory brainstem response, MRI, or follow-up audiometry. - Guidelines also strongly recommend against routine laboratory tests or CT of the head as part of the initial evaluation.

A 58-year-old male presents with a several-day history of shortness of breath with exertion, along with pleuritic chest pain. His symptoms started soon after he returned from a vacation in South America. He has a history of deep-vein thrombosis (DVT) in his right leg after surgery several years ago, and also has a previous history of prostate cancer. You suspect pulmonary embolism (PE.). Which one of the following is true regarding the evaluation of this patient? (check one) A. CT angiography would reliably either confirm or rule out PE B. Compression ultrasonography of the lower extremities will reveal a DVT in the majority of patients with PE C. No further testing is needed if a ventilation-perfusion lung scan shows a low probability of PE D. No further testing is needed if a D-dimer level is normal E. An elevated D-dimer level would confirm the diagnosis of PE

A. CT angiography would reliably either confirm or rule out PE

You respond to a code blue in the obstetrics department. The patient is a 19-year-old primigravida at 35 weeks gestation, hospitalized with severe preeclampsia. A nurse anesthetist has placed an oral airway and is administering 100% oxygen to the apneic patient. She reports no difficulty ventilating the patient with a bag and valve, and no gagging with oral airway insertion. The patient's blood pressure is 100/60 mm Hg and her pulse rate is 70 beats/min and regular. Her pupils are equal and sluggishly reactive, and she is flaccid and areflexic. The patient had been treated with a magnesium sulfate infusion and a recent bolus of labetalol. Which one of the following medications should you administer initially? (check one) A. Calcium gluconate B. Fosphenytoin C. Labetalol D. Lorazepam (Ativan) E. Dopamine

A. Calcium gluconate - magnesium toxicity. - Calcium chloride can be used if a central line has been established - Calcium gluconate would be safer with a peripheral intravenous site

Which one of the following is true regarding the risk of inducing cancer with CT scanning? (check one) A. CT of the chest is associated with a greater risk than CT of the head B. The risk increases with age at the time of the scan C. Males have a greater risk of ultimately developing CT-induced lung cancer than females D. Current techniques with rapid scanners make the risk comparable to that associated with standard radiographs of the same area E. The risk in neonates is markedly reduced because of the efficiency of DNA repair processes at this age

A. CT of the chest is associated with a greater risk than CT of the head - CT of the chest or abdomen leads to significantly more radiation exposure and cancer risk than CT of the brain. - CT imaging carries substantially more risk than plain radiographs of the same area. - Women are at greater risk for developing lung cancer after a chest CT than men, and CT also increases their risk of developing breast cancer.

A healthy 48-year-old bookkeeper who works in a medical office has a positive PPD on routine yearly screening. Which one of the following would be most appropriate at this point? (check one) A. A chest radiograph B. A repeat PPD C. Treatment with isoniazid and one other antituberculous drug for 12 months D. Anergy testing

A. CXR - Clinical evaluation and a chest radiograph are recommended in asymptomatic patients with a positive PPD (SOR C). - Asymptomatic patients with a positive PPD and an abnormal chest film should have a sputum culture for TB. - Persons with a PPD conversion should be encouraged to take INH for 9 months with proper medical supervision. A two-step PPD - for those at high risk whose initial test is negative Anergy testing - Patients with a negative PPD who are still at high risk for TB, especially HIV-positive patients

A 54-year-old white male presents with drooping of his right eyelid for 3 weeks. On examination, he has ptosis of the right upper lid, miosis of the right pupil, and decreased sweating on the right side of his face. Extraocular muscle movements are intact. In addition to a complete history and physical examination, which one of the following would be most appropriate at this point? (check one) A. A chest radiograph B. MRI of the brain and orbits C. 131I thyroid scanning D. A fasting blood glucose level E. An acetylcholine receptor antibody level

A. CXR - Horner's syndrome - ipsilateral ptosis, miosis, and decreased facial sweating - suggests decreased sympathetic innervation due to involvement of the stellate ganglion, a complication of Pancoast's superior sulcus tumors of the lung - Radiographs or MRI of the pulmonary apices and paracervical area is indicated. - Horner's syndrome may accompany intracranial pathology, such as the lateral medullary syndrome (Wallenbergs syndrome), but is associated with multiple other neurologic symptoms, so MRI of the brain is not indicated at this point.

A 28-year-old female just delivered a male infant over an intact perineum. She has had polyhydramnios during this pregnancy, but her prenatal course has otherwise been normal. Her only significant chronic medical problem is asthma, treated with a long-acting β-agonist/corticosteroid combination inhaler. Vital signs were stable throughout her labor. After delivery of the placenta, bleeding becomes brisk and you note a soft, boggy, uterus. Which one of the following medications is contraindicated in this patient? (check one) A. Carboprost (Hemabate) B. Methylergonovine C. Misoprostol (Cytotec) D. Oxytocin (Pitocin)

A. Carboprost (Hemabate): contraindicated for asthma. Methylergonovine: contraindicated in hypertension. All 4 are for uterine atony

A 76-year-old white male with heart failure is admitted to the hospital for the third time in a year. He responds to treatment with intravenous furosemide (Lasix), oxygen, and morphine. When he is discharged, his medications include carvedilol (Coreg), 25 mg twice daily; furosemide, 40 mg daily; and lisinopril (Prinivil, Zestril), 40 mg daily. He is also placed on a low-salt diet. Which one of the following is most likely to help prevent future admissions and decrease overall medical costs for this patient during the next year? (check one) A. Case management by a heart failure specialist nurse B. Nursing home admission C. Adding amiodarone (Cordarone) D. Increasing the dosage of lisinopril

A. Case management by a heart failure specialist nurse

A 72-year-old white male has new-onset hypertension with a current blood pressure of 190/110 mm Hg. Which one of the following agents can be used as part of a test for diagnosing renovascular hypertension, but would also increase the risk for azotemia if used for treatment? (check one) A. Captopril (Capoten) B. Metoprolol (Lopressor) C. Clonidine (Catapres) D. Furosemide (Lasix) E. Amlodipine (Norvasc)

A. Catopril (Capoten) - ACE inhibitors can significantly worsen renal failure in patients with hypertension caused by renovascular disease - Captopril renography is a useful diagnostic screening test. - Hyperkalemia is an associated problem - other agents are safe to lower BP but raise creatinine level

An asymptomatic 32-year-old male requests screening for sexually transmitted diseases. A nucleic acid amplification test is performed on a urine sample, and the results are positive for gonorrhea and negative for Chlamydia. The patient has no known drug allergies. Which one of the following is the recommended treatment for this patient? (check one) A. Ceftriaxone (Rocephin), 125 mg intramuscularly B. Ceftriaxone, 250 mg intramuscularly C. Ceftriaxone, 250 mg intramuscularly, plus azithromycin (Zithromax), 1 g orally D. Ceftriaxone, 125 mg intramuscularly, plus doxycycline, 100 mg orally twice daily for 7 days E. Ciprofloxacin (Cipro), 500 mg orally

A. Ceftriaxone (Rocephin) 250 mg intramuscularly - also, azithromycin, 1 g orally, because of the high incidence of coinfection with Chlamydia, even if testing is negative, and to decrease the risk for cephalosporin resistance. - 125-mg regimen is no longer recommended because of treatment failures and limited effectiveness in pharyngeal infections.

You are the medical director of a long-term-care facility that has 60 residents. Several patients experience fever, cough, and upper respiratory symptoms. Two of these patients test positive for influenza A (H1N1) virus. Which one of the following is recommended by the Centers for Disease Control and Prevention (CDC) for this situation? A. Chemoprophylaxis with appropriate medications for all residents B. Treatment initiated on an individual basis once testing confirms that a resident has influenza C. Prophylaxis only for staff who have had direct patient contact with a resident with laboratory-confirmed infection D. No chemoprophylaxis for staff or residents who have been appropriately vaccinated

A. Chemoprophylaxis with appropriate medications for all residents = two or more laboratory-confirmed cases of influenza A = outbreak in a long-term care facility. The CDC has specific recommendations for managing an outbreak, which include chemoprophylaxis with an appropriate medication for all residents who are asymptomatic and treatment for all residents who are symptomatic, regardless of laboratory confirmation of infection or vaccination status. - All staff should be considered for chemoprophylaxis regardless of whether they have had direct patient contact with an infected resident or have received the vaccine. - Requesting restriction of visitation is recommended; however, it cannot be strictly enforced due to residents' rights.

A 2-month-old female is brought to your office with tachypnea and a staccato cough. She is afebrile. A chest radiograph shows hyperinflation and bilateral infiltrates, and a CBC reveals eosinophilia. Which one of the following is the most likely etiologic agent? (check one) A. Chlamydia trachomatis B. Listeria pneumoniae C. Streptococcus pneumoniae D. A gram-negative bacteria E. Respiratory syncytial virus

A. Chlamydia trachomatis - 1-3 months after birth - tachypnea, a staccato cough, and no fever (SOR A). - Radiographs often show hyperinflation and infiltrates, and a CBC will reveal eosinophilia.

A 50-year-old female presents with a 3-week history of a moderately pruritic rash, characterized by flat-topped violaceous papules 3-4 mm in size. The lesions are located primarily on the volar wrists and forearms, lower legs, and dorsa of both feet. Ten days after the rash first appeared she went to the emergency department and was treated for "possible scabies," but the treatment has made little or no difference. Which one of the following treatments is indicated at this time? (check one) A. Clobetasol (Cormax, Temovate) 0.05% ointment B. Permethrin 5% cream C. Dipyridamole (Persantine) D. Triamcinolone 0.1% cream

A. Clobetasol (Cormax, Temovate) 0.05% ointment - lichen planus - pruritic, symmetrically distributed papular lesions. - flat-topped papules, usually 3-6 mm in size, are distinct and so characteristic in appearance that a biopsy is usually not necessary to make the diagnosis. - First-line treatment: high-potency topical corticosteroids such as clobetasol (mid-potency topical agents such as triamcinolone are ineffective) - Topical calcineurin inhibitors, including tacrolimus, can be used in cases not responding to topical corticosteroids.

A 63-year-old female with type 2 diabetes mellitus presents to the emergency department with unstable angina. Her blood pressure is 150/90 mm Hg, her pulse rate is 70 beats/min, and her lungs are clear to auscultation. The patient expresses a preference for conservative (i.e., noninvasive) therapy. In addition to aspirin, which one of the following agents should be administered at this time? (check one) A. Clopidogrel (Plavix) orally B. Indomethacin (Indocin) orally C. Nifedipine (Procardia) immediate-release, orally D. Abciximab (ReoPro) intravenously E. Enalaprilat intravenously

A. Clopidogrel (plavix) PO - should be administered as soon as possible in patients with unstable angina/NSTEMI who are to be treated conservatively - The standard dosage should then be prescribed, to be taken daily for at least 1 month along with aspirin (SOR B). Not calcium channel antagonists - e.g. nifedipine are not indicated - If β-blockers are contraindicated, verapamil or diltiazem would be the preferred agents Not Intravenous ACE inhibitors - may induce shock and should be avoided in the first 24 hours. Not Abciximab - is used for patients who will undergo rapid catheterization with a significant chance of acute coronary intervention Not NSAIDs - contraindicated - weaken areas of damaged myocardium and increase the risk of rupture - also increase the risk of infarction or extension. - used in the past for treatment of associated pericarditis, which most frequently develops a few days after presentation, but are now avoided.

Which one of the following is recommended for the treatment of patients with obsessive compulsive disorder? (check one) A. Cognitive-behavioral therapy B. Psychoanalytic therapy C. Family therapy D. Psychodynamic psychotherapy E. Motivational interviewing

A. Cognitive-behavioral therapy Not Psychoanalytic therapy - not useful Not Family therapy - help reduce family tensions only Not Psychodynamic psychotherapy and motivational interviewing - help patients overcome their resistance to treatment.

Which one of the following is true regarding the treatment of generalized anxiety disorder? (check one) A. Cognitive-behavioral therapy has been shown to be at least as effective as pharmacologic therapy B. Buspirone (BuSpar) is as effective as SSRI therapy for patients with comorbid depression C. Benzodiazepines are no more effective than placebo D. Duloxetine (Cymbalta) is no more effective than placebo E. Escitalopram (Lexapro) is no more effective than placebo

A. Cognitive-behavioral therapy has been shown to be at least as effective as pharmacologic therapy

A 52-year-old male requests "everything you've got" to help him stop smoking. You review common barriers to quitting and the benefits of cessation with him, and develop a plan that includes follow-up. He chooses to start varenicline (Chantix) to assist with his efforts, and asks about also using nicotine replacement. Which one of the following would be accurate advice? (check one) A. Combining these medications has not proven to be beneficial B. The addition of transdermal nicotine, but not nicotine gum, has proven benefits C. The combination is highly efficacious D. Nicotine replacement doses need to be doubled in a patient taking varenicline E. The combination of nicotine and varenicline is potentially lethal

A. Combining these medications are not proven beneficial - Varenicline works by binding to nicotine receptors in the brain, providing much lower stimulation than nicotine itself would. - Varenicline reduces the reinforcement and reward that smoking provides to the brain - However, this medication also blocks the benefit a patient would receive from nicotine replacement products. - Studies have shown that using nicotine replacement products concurrently with varenicline leads to an increase in nausea, headaches, dizziness, and fatigue.

A 68-year-old white male with diabetes mellitus is hospitalized after suffering a right middle cerebral artery stroke. A nurse in the intensive-care unit calls to advise you that his blood pressure is 200/110 mm Hg. You should: (check one) A. continue monitoring the patient B. administer labetalol (Trandate) C. administer nicardipine (Cardene) D. administer nitroprusside (Nitropress) E. administer nitroglycerin

A. Continue monitoring No additional treatment for patients with a systolic blood pressure <220 mm Hg or a diastolic blood pressure <120 mm Hg. The elevated blood pressure is thought to be a protective mechanism that increases cerebral perfusion, and lowering the blood pressure may increase morbidity.

The mother of an 8-year-old female is concerned about purple "warts" on her daughter's hands. The mother explains that the lesions started a few months ago on the right hand along the top of most of the knuckles and interphalangeal joints, and she has recently noticed them on the left hand. The child has no other complaints and the mother denies any unusual behaviors. A physical examination is unremarkable except for the slightly violaceous, flat-topped lesions the mother described. What is the most likely cause for this patient's finger lesions? (check one) A. Dermatomyositis B. Aggressive warts C. Rubbing/wringing of the hands D. Bulimia nervosa E. Child abuse

A. Dermatomyositis - Gottron's papules - flat-topped, sometimes violaceous papules that often occur on most, if not all, of the knuckles and interphalangeal join

A 77-year-old white male complains of urinary incontinence of more than one year's duration. The incontinence occurs with sudden urgency. No association with coughing or positional change has been noted, and there is no history of fever or dysuria. He underwent transurethral resection of the prostate (TURP) for benign prostatic hypertrophy a year ago, and he says his urinary stream has improved. A rectal examination reveals a smoothly enlarged prostate without nodularity, and normal sphincter tone. No residual urine is found with post-void catheterization. Which one of the following is the most likely cause of this patient's incontinence? (check one) A. Detrusor instability B. Urinary tract infection C. Overflow D. Fecal impaction E. Recurrent bladder outlet obstruction

A. Detrusor instability most common cause of urinary incontinence in both men and women. Incontinence may actually become worse after surgical relief of obstructive prostatic hypertrophy. Not UTI - no fever Not overflow - no residual urine Not outlet obstruction - urine stream ok, no residual urine

Which one of the following has been shown to reduce the croup score in children and lead to shorter hospital stays? (check one) A. Dexamethasone (Decadron), 0.6 mg/kg in a single oral dose B. Amoxicillin, 45 mg/kg/day divided into two doses, for 10 days C. Azithromycin (Zithromax), 10 mg/kg the first day, then 5 mg/kg daily for 4 days D. Albuterol (Ventolin), 0.63 mg by aerosol every 4 hours E. Ceftriaxone (Rocephin), 50 mg/kg intramuscularly in a single dose

A. Dexamethasone (Decadron), 0.6 mg/kg in a single oral dose - Oral or intramuscular dexamethasone, 0.6 mg/kg as a single dose, and nebulized budesonide have been shown to reduce croup scores and shorten hospital stays. Not Racemic epinephrine - may be used acutely, but rebound can occur. Not Albuterol - useless

A patient who takes fluoxetine (Prozac), 40 mg twice daily, develops shivering, tremors, and diarrhea after taking an over-the-counter cough and cold medication. On examination he has dilated pupils and a heart rate of 110 beats/min. His temperature is normal. Which one of the following medications in combination with fluoxetine could contribute to this patient's symptoms? (check one) A. Dextromethorphan B. Pseudoephedrine C. Phenylephrine D. Guaifenesin E. Diphenhydramine (Benadryl)

A. Dextromethorphan - SSRI (no other explanation)

Which one of the following is the most common secondary cause of nephrotic syndrome in adults? (check one) A. Diabetes mellitus B. Systemic lupus erythematosus C. Hepatitis D. NSAIDs E. Multiple myeloma

A. Diabetes mellitus

You see a 30-year-old white male for the first time for a routine evaluation. He says that he has been bothered by multiple skin lesions on the neck and axillae. On examination you note numerous skin tags. The presence of these lesions indicates an increased risk for: (check one) A. diabetes mellitus B. squamous cell skin cancer C. melanoma D. glioblastoma multiforme E. AIDS

A. Diabetes mellitus Skin tags, or acrochordons, are associated with diabetes mellitus and obesity. The onset often occurs in early adulthood, and the most common locations are the neck and axillae. These skin lesions are not associated with any significant cancer risk, and have not been associated with HIV infection.

A 74-year-old female presents with a several-month history of gradually increasing dyspnea on exertion, swelling in her feet and lower legs, and having to sleep sitting up due to increased shortness of breath while lying flat. She has been healthy otherwise, with no known heart disease or hypertension, and she has no significant family history of heart disease. An echocardiogram shows an ejection fraction of 20% and a thin-walled, diffusely enlarged left ventricle. Which one of the following is the most likely diagnosis? (check one) A. Dilated cardiomyopathy B. Hypertrophic cardiomyopathy C. Restrictive cardiomyopathy D. Arrhythmogenic right ventricular cardiomyopathy E. Athlete's heart

A. Dilated cardiomyopathy

A 62-year-old male comes to your office as a new patient. He has a past history of a myocardial infarction and is currently in stage C heart failure according to the American Heart Association classification. His ejection fraction is 30%. Which one of the following medications that the patient is currently taking is potentially harmful and should be discontinued if possible? (check one) A. Diltiazem (Cardizem) B. Lisinopril (Prinivil, Zestril) C. Carvedilol (Coreg) D. Atorvastatin (Lipitor)

A. Diltiazem (Cardizem) Nondihydropyridine calcium channel blockers with negative inotropic effects (verapamil and diltiazem) may be harmful in patients with low left ventricular ejection fractions. ACE inhibitors or angiotensin receptor blockers: reduced ejection fraction. Aldosterone receptor antagonists are indicated in patients who have a left ventricular ejection fraction 35%. Statin therapy is recommended in all patients with a history of myocardial infarction. Evidence-based beta-blockers (carvedilol or metoprolol succinate) should be used in all patients with a history of myocardial infarction.

Which one of the following is most associated with falls in older adults? (check one) A. Diphenhydramine (Benadryl) B. Atorvastatin (Lipitor) C. Metformin (Glucophage) D. Memantine (Namenda) E. Theophylline (Theo-24)

A. Diphenhydramine - Certain classes of medications causes falls: benzos, antidepressants, antipsychotics, antiepileptics, anticholinergics, sedative hypnotics, muscle relaxants, and cardiovascular medications. - diphenhydramine is one of the anticholinergic medications associated with falls in older adults. - The other drugs listed are not in the higher-risk groups of medications.

A 20-year-old female presents with a sudden onset of fever, chills, and headache of 2 days duration, and now has a pink blanching rash. The rash covers most of her body, including the palms of her hands and the soles of her feet, but not including her face. She recently returned from a camping trip, but has had no recent contact with anyone who has been ill. Which one of the following would be the most appropriate treatment for this patient's symptoms? (check one) A. Doxycycline, 100 mg twice daily for 10 days B. Azithromycin (Zithromax), 500 mg daily for 3 days C. Cephalexin (Keflex), 500 mg twice daily for 10 days D. Penicillin VK, 500 mg twice daily for 10 days E. Reassurance

A. Doxycycline, 100 mg bid - rickettsial illness (in the United States this would most likely be Rocky Mountain Spotted Fever): - outdoor activity, the sudden onset of fever, chills, and rash on the palms of the hands and the soles of the feet.

A 66-year-old male has hypertension that has become difficult to manage after several years of good control on a stable medical regimen. On evaluation, his BUN level is 40 mg/dL (N 8-25) and his serum creatinine level is 2.1 mg/dL (N 0.6-1.5). Which one of the following tests would be best to evaluate this patient for renovascular hypertension? (check one) A. Duplex Doppler ultrasonography B. CT angiography C. Aortography D. Captopril (Capoten) renography

A. Duplex Doppler ultrasonography - Tests involving intravenous radiographic contrast material may cause deterioration in renal function. - Captopril renography: not reliable in the setting of poor renal function. - Magnetic resonance angiography also could be considered: gadolinium contrast agents and nephrogenic systemic fibrosis in patients with renal dysfunction is a concern

A 30-year-old male presents with a 3-week history of severe, burning pain in his right shoulder. He recalls no mechanism of injury. An examination reveals weakness to resistance of the biceps and triceps, and with external rotation of the shoulder. Full range of motion of the neck and shoulder does not worsen the pain. Which one of the following would be most likely to identify the cause of this patient's problem? (check one) A. Electromyography and nerve conduction studies B. MRI of the neck C. MR arthrography (MRA) of the shoulder D. CT of the brain E. Ultrasonography of the upper extremity

A. Electromyography and nerve conduction studies - difficult to differentiate from cervical radiculopathy, shoulder pathology, and cerebrovascular accident - pain preceded the weakness, no trauma was involved, and the weakness is in a nondermatomal distribution, making brachial neuritis the most likely diagnosis. - Electromyography is most likely to show this lesion, but only after 3 weeks of symptoms. - MRI of the neck may show abnormalities, but not the cause

A 60-year-old male has moderate anemia, with a suggestion of hemolysis on a peripheral blood smear. Which one of the following patterns would be consistent with the presence of hemolysis? (check one) A. Elevated LDH, decreased haptoglobin, elevated indirect bilirubin B. Elevated LDH, elevated haptoglobin, decreased indirect bilirubin C. Decreased LDH, elevated haptoglobin, elevated indirect bilirubin D. Decreased LDH, decreased haptoglobin, elevated indirect bilirubin E. Decreased LDH, decreased haptoglobin, decreased indirect bilirubin

A. Elevated LDH, decreased haptoglobin, elevated indirect bilirubin Hemolytic anemia is established by reticulocytosis, increased unconjugated bilirubin, elevated lactate dehydrogenase (LDH), decreased haptoglobin, and peripheral blood smear findings.

A 7-year-old male presents with a fever of 38.5°C (101.3°F), a sore throat, tonsillar inflammation, and tender anterior cervical adenopathy. He does not have a cough or a runny nose. His younger sister was treated for streptococcal pharyngitis last week and his mother would like him to be treated for streptococcal infection. Which one of the following is true concerning this situation? (check one) A. Empiric antibiotic treatment for streptococcal pharyngitis is warranted. B. The chance of this patient having a positive rapid antigen detection test for Streptococcus is <50%. C. There is a generalized consensus among the various national guidelines for management of pharyngitis. D. The patient should have a tonsillectomy when he recovers from this infection. E. The family dog should be treated for streptococcal infection.

A. Empiric antibiotic treatment for streptococcal - scores 5 in modified centor criteria History of fever - 1 Tonsillar exudates - 1 Tender anterior cervical adenopathy - 1 Absence of cough - 1 The modified Centor criteria add the patient's age to the criteria: Age under 15 - 1 Age over 44 subtract 1 point

During rounds, you notice a new rash on a full-term 2-day-old white female. It consists of 1-mm pustules surrounded by a flat area of erythema, and is located on the face, trunk, and upper arms. An examination is otherwise normal, and she does not appear ill. Which one of the following is the most likely diagnosis? (check one) A. Erythema toxicum neonatorum B. Transient neonatal pustular melanosis C. Acne neonatorum D. Systemic herpes simplex E. Staphylococcus aureus sepsis

A. Erythema toxicum neonatorum Transient neonatal pustular melanosis is most common in African-American newborns, and the lesions lack the surrounding erythema typical of ETN. Acne neonatorum is associated with closed comedones, mostly on the face. As the infant described is not ill, infectious etiologies are unlikely.

You are covering the inpatient service and following up on a 67-year-old female admitted 3 days ago for severe pancreatitis. CT on admission showed edema and mild inflammation. Currently the patient is receiving intravenous fluids, daily laboratory evaluations, and pain medications. She is NPO and afebrile, with a blood pressure of 130/78 mm Hg and a pulse rate of 88 beats/min. Which one of the following therapies should be initiated to lower complication rates and shorten the patient's hospital stay? (check one) A. Enteral nutrition B. Parenteral nutrition C. Surgical debridement D. Prophylactic antibiotics

A. Enteral nutrition - Prophylactic antibiotics should only be used when there is significant necrosis (SOR C). - surgical debridement is indicated only if there is infected necrosis or persistent fluid collections (SOR C).

A 17-year-old male presents to the urgent care clinic 15 minutes after being stung by a wasp. He feels weak, his voice is hoarse, and he is beginning to have trouble breathing. Which one of the following should be administered first? (check one) A. Epinephrine IM B. An oral leukotriene-receptor antagonist C. Intranasal antihistamines D. Intranasal corticosteroids E. Oral antihistamine

A. Epinephrine IM

A 45-year-old female with rheumatoid arthritis has a hemoglobin level of 9.5 g/dL (N 11.5-16.0). Her arthritis is well controlled with methotrexate. Further evaluation reveals the following: Hematocrit............29.0% (N 35.0-47.0) Mean corpuscular volume............78 µm3 (N 80-98) Platelets............230,000/mm3 (N 150,000-400,000) WBCs............6900/mm3 (N 4000-11,000) Differential............normal Serum iron............15 µg/dL (N 50-170) Total iron binding capacity............150 µg/dL (N 45-70) Iron saturation............10% (N 15-50) Serum ferritin............7 ng/mL (N 12-150) Reticulocyte count............8 x 109/L (N 10-100) Stool guaiac............negative x 3 Which one of the following would be the most appropriate next step? (check one) A. Evaluation for a source of blood loss B. Hemoglobin electrophoresis to screen for thalassemia C. Stopping the methotrexate and beginning an alternative treatment for rheumatoid arthritis D. No further evaluation

A. Eval source of blood loss - Anemia of chronic disease - the underproduction of red cells - due to low serum iron caused by the uptake of iron by the reticuloendothelial system. - Total-body iron stores are increased but the iron in storage is not available for red cell production. - This anemia is normochromic and normocytic, and is associated with a *reduction in iron, transferrin, and transferrin saturation*. - Ferritin is either normal or increased, reflecting both the increased iron within the reticuloendothelial system and increases due to immune activation (acute phase reactant). Iron deficiency anemia - *total-body iron* levels are low, leading to hypochromia and microcytosis, low iron levels, increased transferrin levels, and reduced ferritin levels. - This patient's anemia is most likely multifactorial, with anemia of chronic disease and drug effects playing a role. However, she also has iron deficiency, and searching for a source of blood loss would be important. With thalassemia, marked microcytosis is seen, and with hemolysis, slight macrocytosis and an increased reticulocyte count would be expected

Information derived from which one of the following provides the best evidence when selecting a specific treatment plan for a patient? (check one) A. Meta-analyses B. Prospective cohort studies C. Expert opinion D. Consensus guidelines E. iliofemoral atherosclerosis

A. Meta-analysis

In the development of clinical guidelines, which one of the following is rated as the strongest and highest-quality evidence? (check one) A. Evidence from randomized, placebo-controlled studies B. Evidence from nonrandomized, double-blind, placebo-controlled studies C. Evidence from nonrandomized, double-blind, crossover, placebo-washout-controlled studies D. Evidence obtained from well-designed cohort or case-control analytical studies from more than one center or research group E. Evidence based on reports of expert committees or opinions of respected authorities in the appropriate specialty area

A. Evidence from randomized, placebo-controlled studies - yield strongest evidence (Level 1) 2a: Evidence obtained from nonrandomized, controlled studies is considered level 2b: well-designed case-control and cohort studies are considered level 3: expert committees or respected authorities are considered (weakest).

The most common presenting symptom of obstructive sleep apnea is: (check one) A. excessive daytime sleepiness B. snoring C. morning headache D. gastroesophageal reflux E. enuresis

A. Excessive daytime sleepiness; also - snoring - restless sleep - witnessed apneas & nocturnal choking - morning headache - nocturia, enuresis - GERD - reduced libido

A 39-year-old female presents with a 4-month history of gradually worsening left elbow pain. She does not recall an injury but frequently lifts and holds her 10-month-old son in her left arm. She has tenderness over the lateral epicondyle. Her elbow range of motion is normal but she has pain with supination and pronation. The remainder of the examination is normal. For long-term pain relief, the best evidence supports which one of the following? (check one) A. Expectant/conservative management B. Physical therapy C. Oral anti-inflammatory agents D. A corticosteroid injection

A. Expectant or Conservative management - Lateral epicondylitis - common condition characterized by degeneration of the extensor carpi radialis muscle tendon originating in the lateral epicondyle. - self-limited condition and usually resolves within 12-18 months without treatment (95%+ recovers) - not an inflammatory condition and anti-inflammatory agents does not help - Corticosteroid injections have been found to be associated with poor long-term outcomes, as well as high recurrence rates. - Neither physical therapy, bracing, nor splinting is proven to provide long-term pain relief.

Imiquimod (Aldara) is approved by the FDA for treatment of which one of the following conditions? (check one) A. External anogenital warts B. Plantar warts C. Flat warts D. Periungual warts E. Molluscum contagiosum

A. External anogenital warts Since its FDA approval, imiquimod has been used off-label to treat all of the conditions listed, but is approved only for treatment of external genital and perianal warts in patients 12 years of age and over

A 78-year-old male comes to your office with a 3-day history of pain in the right side of his chest. The pain is described as burning and intense. Two days ago he noted a rash at that site. Examination reveals groups of vesicles on an erythematous base in a T-5 dermatome distribution on the right. Which one of the following would be the most appropriate treatment to minimize the chance of post-herpetic neuralgia? (check one) A. Famciclovir (Famvir) B. Prednisone C. Capsaicin (Zostrix) D. Carbamazepine (Tegretol)

A. Famiciclovir (Famvir) postherpetic neuralgia is - persistent pain 3-6 months after an episode of herpes zoster. - Patients treated within 72 hours will benefit from antiviral therapy such as famciclovir to reduce the pain and decrease the risk of postherpetic neuralgia. - Treating zoster pain with tricyclic antidepressants in low dosage (10-25 mg amitriptyline) may also decrease risk.

At a routine visit, a 40-year-old female asks about beginning an exercise regimen. She has a family history of heart disease and hypertension. She currently has no medical problems, but she is sedentary. Which one of the following would be the most appropriate recommendation for this patient? (check one) A. Fast walking for 30 minutes on 5 or more days per week B. Jogging for 30 minutes every other day C. Weight training once weekly D. An exercise stress test prior to beginning exercise E. A baseline EKG and rhythm strip

A. Fast walking 30 mins 5 or more times/week Current recommendations are for healthy adults to engage in 30 minutes of accumulated moderate-intensity physical activity on 5 or more days per week.

Which one of the following surgical procedures is associated with the highest risk for perioperative myocardial ischemia? (check one) A. Femoropopliteal bypass B. Pulmonary lobectomy C. Hip arthroplasty D. Transurethral resection of the prostate E. Mastectomy

A. Femoropopliteal bypass; also - aortic and peripheral vascular sx - major operations esp. in pt 75 y/o or older intermediate risk - head and neck surgery - intraperitoneal, intrathoracic surgery - orthopedic surger - prostate surgery low risk - endoscopic - cataract - breast

A 46-year-old female presents to your office with a 2-week history of pain in her left shoulder. She does not recall any injury, and the pain is present when she is resting and at night. Her only chronic medical problem is type 2 diabetes mellitus. On examination, she has limited movement of the shoulder and almost complete loss of external rotation. Radiographs of the shoulder are normal, as is her erythrocyte sedimentation rate. Which one of the following is the most likely diagnosis? (check one) A. Frozen shoulder B. Torn rotator cuff C. Impingement syndrome D. Chronic posterior shoulder dislocation E. Osteoarthritis

A. Frozen shoulder - idiopathic condition ages of 40 and 60 - Diabetes mellitus is the most common risk factor - shoulder stiffness, loss of active and passive shoulder rotation, and severe pain, including night pain. - Laboratory tests and plain films are normal Chronic posterior shoulder dislocation or osteoarthritis - characteristic plain film findings Rotator cuff tear or Impingement syndrome - normal passive range of motion - impingement syndrome will have pain w/ elevation of the shoulder.

A 40-year-old male with acute pancreatitis has an alanine transaminase (ALT) level that is five times normal. Which one of the following is the most likely diagnosis? (check one) A. Gallstone pancreatitis B. Pancreatic necrosis C. Pancreatic pseudocyst D. Hepatitis C E. Alcohol-induced pancreatitis

A. Gallstone pancreatitis - a threefold or greater elevation of alanine transaminase has a positive predictive value of 95% for acute gallstone pancreatitis - High levels of C-reactive protein are associated with pancreatic necrosis - Hepatitis C is identified by antibody detection or polymerase chain reaction testing - Other markers are investigational.

Which one of the following is an effect of long-term treatment for narcotic addiction with methadone and buprenorphine? (check one) A. Greater success at producing minimal opiate use than detoxification programs B. Significant teratogenic effects C. Frequent diversion of opiates D. Decreased associated cocaine abuse

A. Greater success at producing minimal opiate use than detoxification programs

Which one of the following drugs is NOT effective for maintenance therapy in bipolar disorders? A. Haloperidol B. Lamotrigine (Lamictal) C. Lithium D. Quetiapine (Seroquel) E. Valproate sodium (Depacon)

A. Haloperidol Lithium, valproate, lamotrigine, and some antipsychotics (including quetiapine) are effective treatments for both acute depression and maintenance therapy of bipolar disorders. Haloperidol is an effective treatment for acute mania in bipolar disorders, but not for maintenance therapy or acute depression.

A 36-year-old female presents with the sudden onset of severe headache, nausea, and photophobia. Her level of consciousness is progressively diminishing. Which one of the following would be the most appropriate next step? (check one) A. Head CT without contrast B. Head CT with contrast C. Head MRI D. Lumbar puncture E. CT angiography

A. Head CT w/out contrast - subarachnoid hemorrhage - useful for identifying other sources for the headache, for predicting the site of the aneurysm, and for predicting cerebral vasospasm and poor outcome. - As blood is cleared from the affected area, CT sensitivity drops to 93% within 24 hours, and to 50% at 7 days. - Patients with a positive CT result for subarachnoid hemorrhage should proceed directly to angiography and treatment. - Patients with a suspected subarachnoid hemorrhage who have negative or equivocal results on head CT should have a lumbar puncture. MRI and CT with contrast are not used for the diagnosis of acute subarachnoid hemorrhage.

You see a 1-year-old male for a routine well child examination. Laboratory tests reveal a hemoglobin level of 10 g/dL (N 9-14), a hematocrit of 31% (N 28-42), a mean corpuscular volume of 68 :m3 (N 70-86), and a mean corpuscular hemoglobin concentration of 25 g/dL (N 30-36). A trial of iron therapy results in no improvement and a serum lead level is normal. Which one of the following would be the most appropriate test at this time? (check one) A. Hemoglobin electrophoresis B. Bone marrow examination C. Vitamin B12 and folate levels D. A TSH level

A. Hemoglobin electrophoresis In a child with a microcytic anemia who does not respond to iron therapy, hemoglobin electrophoresis is appropriate to diagnose thalassemia.

An 8-year-old female is brought to your office with a 3-day history of bilateral knee pain. She has had no associated upper respiratory symptoms. On examination she is afebrile. Her knees have full range of motion and no effusion, but she has a purpuric papular rash on both lower extremities. Which one of the following is the most likely cause of her symptoms? (check one) A. Henoch-Schönlein purpura B. Rocky Mountain spotted fever C. Juvenile rheumatoid arthritis D. Lyme disease E. Rheumatic fever

A. Henoch-Schönlein purpura - arthritis with a typical palpable purpuric rash - 2 to 8 years old - Arthritis is present in 2/3 of those affected - Gastrointestinal and renal involvement are also common Rocky Mountain spotted fever - rash, no arthralgia - fever & headache Juvenile rheumatoid arthritis - salmon-pink maculopapular rash, but not purpura Lyme disease Rheumatic fever - erythema marginatum, which is a pink, raised, macular rash with sharply demarcated borders.

A 4-year-old white male is brought to your office because he has had a low-grade fever and decreased oral intake over the past few days. On examination you note shallow oral ulcerations confined to the posterior pharynx. Which one of the following is the most likely diagnosis? (check one) A. Herpangina B. Herpes C. Mononucleosis D. Roseola infantum E. Rubella

A. Herpangina - febrile disease caused by coxsackieviruses and echoviruses. - Vesicles and subsequent ulcers develop in the posterior pharyngeal area (SOR C). Herpes infection - causes a gingivostomatitis that involves the anterior mouth. Mononucleosis - petechiae of the soft palate, but does not usually cause pharyngeal lesions Roseola - breaks out with fever; no mucosa Rubella may - soft palate (Forschheimer spots), not the pharynx.

A 25-year-old female presents with a maculopapular rash that has progressed to multiple areas and exhibits target lesions. A cold sore appeared on her upper lip 2 days before the rash appeared. She is not systemically ill and is on no medications. Which one of the following is true concerning this problem? (check one) A. Herpes simplex virus is a likely cause B. A skin biopsy will confirm the diagnosis C. The lesions usually disappear within 24 hours D. The palms of the hands and soles of the feet are not involved E. Scarring from the lesions is often seen after resolution

A. Herpes simplex virus is a likely cause - most common etiologic agent of erythema multiforme - also Mycoplasma pneumoniae, fungal, allergy or meds or vaccine Not biopsy - not specific Not 24 hrs - last at least 1 wk, looks like target Not palms & soles sparing Not scarring - no sequelae

A 3-week-old white male presents with a history of several days of projectile vomiting after feeding, and documented weight loss despite a good appetite. There is a questionable history of a paternal uncle having surgery for a similar problem when he was an infant. Which one of the following findings is a characteristic sign of this disease? (check one) A. Hypochloremic alkalosis B. Pneumonia C. Generalized abdominal distention D. Currant jelly stool E. Direct hyperbilirubinemia

A. Hypochloremic alkalosis - malnutrition, constipation, oliguria, and profound hypochloremic metabolic alkalosis. - sign of pyloric obstruction. - Pneumonia is not a common problem with pyloric stenosis, as it can be with congenital tracheoesophageal fistulae for example. - After feeding, there may be a visible peristaltic wave that progresses across the abdomen. - obstruction is proximal to the small and large intestines and affected infants lose weight - the abdomen is usually flat rather than distended, especially in the malnourished infant.

A 12-year-old male uses a short-acting bronchodilator three times per week to control his asthma. Lately he has been waking up about twice a week because of his symptoms. Which one of the following medications would be most appropriate? A. Inhaled medium-dose corticosteroids B. A scheduled short-acting bronchodilator C. A scheduled long-acting bronchodilator D. A leukotriene inhibitor E. Ordering a free T 4

A. Inhaled medium-dose corticosteroids - moderate persistent asthma. - although many parents are concerned about corticosteroid use in children with open growth plates, inhaled corticosteroids have not been proven to prematurely close growth plates and are the most effective treatment with the least side effects. - Scheduled use of a SABA or LABA has been shown to cause tachyphylaxis, and is not recommended. - Leukotriene use may be beneficial, but compared to those using inhaled corticosteroids, patients using leukotrienes are 65% more likely to have an exacerbation requiring systemic corticosteroids.

A 2-year-old female is brought to the emergency department with a 2-day history of fever and increasing redness on the left forearm. She is otherwise healthy. On examination her temperature is 39.9°C (103.8°F), pulse rate 140 beats/min, and respiratory rate 42/min. She is irritable, and the left forearm has a 4-cm erythematous, warm, tender area, with a fluctuant area centrally. Her WBC count is 21,000/mm3 (N 4300-1 3 0,800), with 14% immature bands. In addition to incision and drainage, which one of the following is the best initial treatment in this patient? (check one) A. Intravenous vancomycin B. Intravenous ampicillin/sulbactam (Unasyn) C. Intravenous nafcillin D. Intravenous clindamycin (Cleocin) E. No antibiotics

A. IV Vancomycin - systemic symptoms that suggest a severe underlying infection. - Community-acquired methicillin-resistant Staphylococcus aureus (CA-MRSA) should be considered the cause - CA-MRSA can cause aggressive infections in children, especially in the skin and soft tissue - Incision and drainage of the abscess is necessary for treatment.

A 34-year-old white male letter carrier has developed progressively worsening dysphagia for liquids and solids over the past 3 months. He says that he has lost about 30 lb during that time. On examination, you note that he is emaciated and appears ill. His pulse rate is 98 beats/min, temperature 37.8°C (100.2°F), respiratory rate 24/min, and blood pressure 95/60 mm Hg. His weight is 45 kg (99 lb) and his height is 170 cm (67 in). His dentition is poor, and there is evidence of oral thrush. His mucous membranes are dry. You palpate small posterior cervical and axillary nodes. The heart, lung, and abdominal examinations are normal. You promptly consult a gastroenterologist, who performs upper endoscopy, which reveals numerous small ulcers scattered throughout the esophagus with otherwise normal mucosa. As you continue to investigate, you take a more detailed history. Which one of the following is most likely to be related to the patient's problem? (check one) A. Intravenous drug use B. A family history of esophageal cancer C. Chest pain relieved by nitroglycerin D. Recent travel to Russia

A. IV drug use - young man with weight loss, oral thrush, lymphadenopathy, and ulcerative esophagitis is likely to have HIV infection. - Intravenous drug use is responsible for over a quarter of HIV infections in the United States. - Esophageal disease develops in more than half of all patients with advanced infection during the course of their illness. - The most common pathogens causing esophageal ulceration in HIV-positive patients include *Candida, herpes simplex virus, and cytomegalovirus* - Identifying the causative agent through culture or tissue sampling is important for providing prompt and specific therapy.

A 2-year-old female is brought to the emergency department with a 2-day history of fever and increasing redness on the left forearm. She is otherwise healthy. On examination her temperature is 39.9°C (103.8°F), pulse rate 140 beats/min, and respiratory rate 42/min. She is irritable, and the left forearm has a 4-cm erythematous, warm, tender area, with a fluctuant area centrally. Her WBC count is 21,000/mm3 (N 4300-1 3 0,800), with 14% immature bands. In addition to incision and drainage, which one of the following is the best initial treatment in this patient? (check one) A. Intravenous vancomycin B. Intravenous ampicillin/sulbactam (Unasyn) C. Intravenous nafcillin D. Intravenous clindamycin (Cleocin) E. No antibiotics

A. IV vancomycin - systemic symptoms that suggest a severe underlying infection - Community-acquired methicillin-resistant Staphylococcus aureus (CA-MRSA) should be considered the cause of this type of infection until definitive cultures are obtained. - CA-MRSA can cause aggressive infections in children, especially in the skin and soft tissue. Incision and drainage of the abscess is necessary for treatment. In a severe infection - vancomycin should be started initially until culture and sensitivities are available (SORB).

A school nurse discovers head lice on a fourth-grade student. When should the student be permitted to return to class? (check one) A. Immediately B. When there are no visible nits C. After a single treatment with a topical agent

A. Immediately Head lice are a common and easily treated inconvenience in school-aged children that, unlike body lice, are not associated with significant illnesses. Transmission generally requires head-to-head contact, as lice cannot survive when separated from their host for more than 24 hours and do not fly or hop.

You are treating an 18-year-old white male college freshman for allergic rhinitis. It is September, and he tells you that he has severe symptoms every autumn that impair his academic performance. He has a strongly positive family history of atopic dermatitis. Which one of the following medications is considered optimal treatment for this condition? (check one) A. Intranasal glucocorticoids B. Intranasal cromolyn sodium C. Intranasal decongestants D. Intranasal antihistamines

A. Intranasal glucocorticoids - best medications for the treatment of allergic rhinitis. Cromolyn sodium is also an effective topical agent for allergic rhinitis; however, it is more effecti00.-*lmlll\]=,p-;]8ve if started prior to the season of peak symptoms.

A 43-year-old female complains of a several-month history of unpleasant sensations in her legs and an urge to move her legs. These symptoms only occur at night and improve when she gets up and stretches. The sensations often awaken her, and she feels very tired. She has no other medical problems and takes no medication. Laboratory tests reveal a serum calcium level of 8.9 mg/dL (N 8.5-10.5), a serum potassium level of 4.1 mmol/L (N 3.5-5.0), a serum ferritin level of 15 ng/mL (N 10-200), and a serum magnesium level of 1.5 mEq/L (N 1.4-2.0). Which one of the following may improve her symptoms? (check one) A. Iron supplementation B. Magnesium supplementation C. Antihistamines D. Stopping calcium supplementation E. Amitriptyline

A. Iron supplementation - restless legs syndrome, - symptoms are relieved by movement. - treatment: lower-body resistance training and avoiding or changing medications that may exacerbate symptoms (e.g., antihistamines, caffeine, SSRIs, tricyclic antidepressants, etc.) - iron supplement: if serum ferritin level below 50 ng/mL take an iron supplement (SOR C).

Which one of the following is the leading cause of human death in the world as a whole? (check one) A. Ischemic heart disease B. Premature birth C. Diarrheal diseases D. HIV/AIDS E. Cancers of the lungs, bronchi, and trachea

A. Ischemic heart disease

Which one of the following treatment regimens is most appropriate for an HIV-positive 42-year old who has latent tuberculosis infection? (check one) A. Isoniazid daily for 9 months B. Rifampin (Rifadin) daily for 4 months C. Rifampin plus pyrazinamide daily for 2 months D. Combination therapy with isoniazid, rifampin, pyrazinamide, and ethambutol (Myambutol) for 2 months

A. Isoniazid daily for 9 months Latent tuberculosis infection carries a risk of progression to active disease, especially among patients who are immunosuppressed. Isoniazid monotherapy is the treatment of choice for most patients with latent tuberculosis infection. Rifampin is not recommended as monotherapy in patients with HIV infection because of increased rates of resistance and drug interactions with many antiretrovirals. Rifampin plus pyrazinamide is no longer recommended for treatment of latent tuberculosis infection because cases of significant hepatotoxicity have occurred with preventive therapy. Combination drug therapy is reserved for treatment of active tuberculosis in order to prevent drug resistance.

A 69-year-old female presents with postmenopausal bleeding. You consider whether to begin your evaluation with vaginal probe ultrasonography to assess the thickness of her endometrium. In evaluating the usefulness of this test to either support or exclude a diagnosis of endometrial cancer, which one of the following statistics is most useful? (check one) A. Likelihood ratio B. Number needed to treat C. Prevalence D. Incidence E. Relative risk

A. Likelihood ratio

Which one of the following is NOT considered a first-line treatment for head lice? (check one) A. Lindane 1% B. Malathion 0.5% (Ovide) C. Permethrin 1% (Nix) D. Pyrethrins 0.33%/pipernyl butoxide 4% (RID)

A. Lindane 1% *Lindane's* efficacy has waned over the years and it is inconsistently ovicidal. Because of its neurotoxicity, lindane carries a black box warning

A pregnant 32-year-old gravida 2 para 1 develops an acute deep-vein thrombosis in the left lower extremity during the third trimester. The patient had a cesarean delivery with her first pregnancy and wants to breastfeed. Which one of the following is the treatment of choice? (check one) A. Low molecular weight heparin B. Unfractionated heparin C. Warfarin (Coumadin) D. A vena cava filter

A. Low molecular weight heparin - current preferred anticoagulant is low molecular weight heparin. Unfractionated heparin requires more monitoring and may increase the risk of heparin-induced thrombocytopenia.

A 70-year-old male complains of lower-extremity pain. Increased pain with which one of the following would be most consistent with lumbar spinal stenosis? (check one) A. Lumbar spine extension B. Lumbar spine flexion C. Internal hip rotation D. Pressure against the lateral hip and trochanter E. Walking uphill

A. Lumbar spine extension Extension that increases lumbar lordosis decreases the cross-sectional area of the spinal canal; compressing the spinal cord further. - Walking downhill can cause this - Spinal flexion that decreases lordosis has the opposite effect, and will usually improve the pain, as will sitting. Hip Arthritis - Pain with internal hip rotation. felt in the groin Trochanteric bursitis - Lateral hip is more typical of trochanteric bursitis Claudication - Increased pain walking uphill

Which one of the following agents used for tocolysis has the unique adverse effect of respiratory depression? (check one) A. Magnesium sulfate B. Ritodrine (Yutopar) C. Terbutaline (Brethine, Bricanyl) D. Indomethacin (Indocin) E. Nifedipine (Adalat, Procardia)

A. Mag Sulfate Terbutaline and ritodrine - causes pulmonary edema.

A 50-year-old female sees you for follow-up of uncontrolled hypertension. Her recent blood pressure measurements average >175/105 mm Hg. The patient has diabetes mellitus and a BMI of 32.3 kg/m2. Physical findings are otherwise noncontributory. Recent laboratory studies include three different potassium levels <3.5 mEq/L (N 3.5-5.0) despite increasing dosages of oral potassium supplements, with the dosage now at 100 mEq daily. Which one of the following would be most appropriate at this point? (check one) A. Measurement of peripheral aldosterone concentration and peripheral renin activity B. CT of the abdomen C. Renal CT angiography D. An aldosterone suppression test

A. Measurement of peripheral aldosterone concentration and peripheral renin activity - Hyperaldosteronism, usually caused by a hyperaldosterone-secreting adrenal mass, has to be considered in a middle-aged patient with resistant hypertension and hypokalemia. - Peripheral aldosterone concentration (PAC) and peripheral renin activity (PRA), preferably after being upright for 2 hours, are the preferred screening tests for hyperaldosteronism. - PAC >15 ng/dL and a PAC/PRA ratio >20 suggest an adrenal cause. - Abdominal CT may miss adrenal hyperplasia or a microadenoma. Renal CT angiography is useful for detecting renal artery stenosis. If the PAC/PRA is abnormal, an aldosterone suppression test should be ordered.

A 15-year-old male presents to the emergency department after suffering a lateral dislocation of his patella. Which one of the following would be the best method for reducing this dislocation? (check one) A. Medially directed pressure on the patella while extending the leg B. Medially directed pressure on the patella while flexing the leg C. Rapid leg extension D. Lateral retinacular release

A. Medially directed pressure on the patella while extending the leg It is usually simple to reduce a lateral patellar dislocation, and these injuries rarely require acute surgical management. The proper technique is to have the patient sit or lie with the leg in a flexed position and then apply gentle medial pressure to the patella until the most lateral edge is over the femoral condyle. The leg should then be gently extended and the knee brought into full extension. This should cause the patella to slip back into place, and the knee should then be immobilized.

An overweight 11-year-old male with acanthosis nigricans is found to have a fasting plasma glucose level of 175 mg/dL on two occasions. Over the next 6 months, despite reasonable adherence to a diet and exercise regimen, he has preprandial and bedtime finger-stick blood glucose levels that average 180 mg/dL. His hemoglobin A1c is 9.0%. Which one of the following oral agents would be most appropriate at this time? (check one) A. Metformin (Glucophage) B. Glyburide (DiaBeta) C. Sitagliptin (Januvia) D. Pioglitazone (Actos) E. Acarbose (Precose)

A. Metformin - Metformin and insulin are the only agents approved for treatment of type 2 diabetes mellitus in children.

A 25-year-old female has been trying to conceive for over 1 year without success. Her menstrual periods occur approximately six times per year. Laboratory evaluation of her hormone status has been negative, and her husband has a normal semen analysis. Her only other medical problem is hirsutism, which has not responded to topical treatment. Pelvic ultrasonography of her uterus and ovaries is unremarkable. Of the following, which one would be the most appropriate treatment for her infertility? (check one) A. Metformin (Glucophage) B. Danazol C. Medroxyprogesterone (Provera) D. Spironolactone (Aldactone)

A. Metformin (Glucophage) - PCOS

A 40-year-old obese African-American male presents with a history of excessive daytime drowsiness. At home he falls asleep shortly after starting to read or watch television. He admits to nearly crashing his car twice in the past month because he briefly fell asleep behind the wheel. Most frightening to the patient have been episodes characterized by sudden loss of muscle tone, lasting about 1 minute, associated with laughing. An overnight sleep study shows decreased sleep latency and no evidence of obstructive sleep apnea. A. Methylphenidate (Ritalin) B. Zolpidem (Ambien) at bedtime C. Carbidopa/levodopa (Sinemet) D. Weight reduction E. Avoidance of daytime napping

A. Methylphenidate (Ritalin)

A 60-year-old right-handed white male arrives in the emergency department with symptoms and signs consistent with a stroke. His past medical history is significant for tobacco abuse and chronic treated hypertension. He is alert and afebrile. His pulse rate is 100 beats/min, respirations 20/min, and blood pressure 190/95 mm Hg. He has a moderate right-sided hemiparesis and is aphasic. There are no other significant physical findings. While appropriate tests are being ordered, immediate management in the emergency department should include which one of the following? (check one) A. Monitoring oxygenation status with pulse oximetry B. Prompt lowering of systolic blood pressure to <140 mm Hg C. Beginning an intravenous heparin infusion D. Restricting fluid intake to 75 cc/hr E. Giving parenteral corticosteroids

A. Monitoring oxygenation status with pulse oximetry - Maintaining adequate tissue oxygenation is important for stroke - Hypoxia leads to anaerobic metabolism and depletion of energy stores, increasing brain injury. - potential need for oxygen should be assessed using pulse oximetry or blood gas measurement. - Overzealous use of antihypertensive drugs is contraindicated, since this can reduce cerebral perfusion. - blood pressure goal: Mean Arterial Pressure is >130 mm Hg or systolic blood pressure is >220 mm Hg.

You are caring for a 70-year-old male with widespread metastatic prostate cancer. Surgery, radiation, and hormonal therapy have failed to stop the cancer, and the goal of his care is now symptom relief. He is being cared for through a local hospice. Over the past few days he has been experiencing respiratory distress. His oxygen saturation is 94% on room air and his lungs are clear to auscultation. His respiratory rate is 16/min. Which one of the following would be best at this point? (check one) A. Morphine B. Oxygen C. Albuterol (Proventil, Ventolin) D. Haloperidol

A. Morphine Dyspnea is a frequent and distressing symptom in terminally ill patients. In the absence of hypoxia, oxygen is not likely to be helpful. Opiates are the mainstay of symptomatic treatment.

Which one of the following is the most common cause of unintentional deaths in children? (check one) A. Motor vehicle accidents B. Drowning C. Poisoning D. Fires E. Falls

A. Motor vehicle accidents Drowning accounts for 10.9% of all unintentional deaths in children, poisoning for 7.7%, fires 5.7%, and falls 1.4%.

A 72-year-old male with a serum creatinine level of 1.8 mg/dL (N 0.6-1.5) requires a contrast dye study. Which one of the following is most effective when given prior to the administration of contrast to reduce the risk for contrast-induced renal failure? (check one) A. N-acetylcysteine B. Mannitol C. Furosemide (Lasix) D. Methylprednisolone sodium succinate (Solu-Medrol) E. Diphenhydramine (Benadryl) Correct.

A. N-acetylcysteine Current methods for reducing the risk of renal failure induced by contrast material include adequate hydration and the use of N-acetylcysteine. Neither mannitol nor furosemide has been shown to prevent contrast-induced renal failure. Corticosteroids and antihistamines are useful for the prevention of idiosyncratic reactions to contrast, but are not helpful in reducing the risk of renal failure.

A 45-year-old male asks about using nicotine replacement therapy (NRT) to help him quit smoking. You tell him that recent evidence shows that (check one) A. NRT usually doubles a smokers chance of quitting B. NRT must be tapered off C. NRT should be used for at least 6 months to be effective D. nicotine patches are the most effective form of NRT E. using combinations of NRT reduces the likelihood that a relapsed smoker will quit .

A. NRT douuble a smoker's chance of quitting A Cochrane meta-analysis of nicotine replacement - Treatment for 8 weeks was as effective as a longer course.

Which one of the following is the most likely cause of chronic unilateral nasal obstruction in an adult? (check one) A. Nasal septal deviation B. Foreign-body impaction C. Allergic rhinitis D. Adenoidal hypertrophy

A. Nasal septal deviation - most common cause of nasal obstruction in all age groups is the common cold aka mucosal disease (bilateral) - Anatomic abnormalities 1. septal deviation most common 2. adenoidal hypertrophy (most common nasal tumor) 3. nasal polyps - Foreign-body impaction infrequent cause of unilateral obstruction and purulent rhinorrhea..

Children of an elderly man who suffers from Alzheimer's disease are bothered by his wandering and pacing behaviors. You have started treatment with a cholinesterase inhibitor, but the behavior persists. They ask you to prescribe additional drug therapy. You would recommend which one of the following? (check one) A. No additional drug therapy B. Risperidone (Risperdal) C. Citalopram (Celexa) D. Lorazepam (Ativan) E. Valproic acid (Depakote)

A. No additional drug - Behavioral symptoms such as agitation and wandering become common as Alzheimer's disease progresses & not amendable w/ meds - Cholinesterase inhibitors may improve some of these symptoms. Not Atypical agents or typical agents (2nd line) - only for problematic delusions, hallucinations, severe psychomotor agitation, and combativeness. Not Mood-stabilizing drugs - only for severe agitated, repetitive, and combative behaviors. Not Benzodiazepines - for insomnia, anxiety and agitation.

A 26-year-old female presents with acute low back pain. She says it started a week ago after she lifted a sofa when helping a friend move. The patient's medical history is otherwise negative. The patient says the pain is limited to the lower back. The physical examination is normal, including the neurologic examination. Which one of the following would be the most appropriate choice for imaging at this time? (check one) A. No imaging B. A plain film of the lumbar spine C. MRI of the lumbar spine D. A DXA scan E. A PET scan

A. No imaging Plain film only if trauma or even a history of minor trauma in an elderly patient. MRI would if red flags such as bladder dysfunction, areflexia, saddle anesthesia, progressive motor weakness, a history of cancer, or the presence of fever, unexplained weight loss, or night sweats.

A healthy 18-year-old female sees you for a preparticipation evaluation and well care visit prior to soccer season. She has no significant previous medical history and no current problems. She says she is not sexually active. She has completed the HPV vaccine series. Which one of the following would be most appropriate for cervical cancer screening for this patient? (check one) A. No screening at this visit B. Annual Papanicolaou tests C. Papanicolaou testing alone every 3 years D. Erythema toxicum neonatorum E. Papanicolaou testing and HPV testing every 3 years

A. No screening at this visit The U.S. Preventive Services Task Force recommends against screening for cervical cancer for women younger than 21, for women over the age of 65 who have had adequate screening in the recent past and are not at high risk, and for women who have had a hysterectomy with removal of the cervix and no history of CIN 2 or 3 or cervical cancer (USPSTF D recommendation).

A 5-year-old male is scheduled for elective hernia repair at 11:00 a.m. Which one of the following would be the most appropriate recommendation? (check one) A. No solid food for 8 hours prior to surgery and clear liquids until 2 hours prior to surgery B. No solid food 4 hours prior to surgery and clear liquids until 2 hours prior to surgery C. No solid food after midnight and nothing by mouth 8 hours prior to surgery D. Nothing by mouth 2 hours prior to surgery E. Nothing by mouth 8 hours prior to surgery

A. No solid food for 8 hours prior to surgery and clear liquids until 2 hours prior to surgery - 8 hours for solid food, 6 hours for formula, 4 hours for breast milk, and 2 hours for clear liquids - resulted in decreased numbers of canceled cases and pediatric patients who are less irritable preoperatively and less dehydrated at the time of anesthesia induction.

A previously healthy 60-year-old male is diagnosed with multiple myeloma after a workup for an incidental finding on routine laboratory work. He has no identified organ or tissue damage and is asymptomatic. Which one of the following would be appropriate treatment of this patient's condition? (check one) A. No treatment B. Chemotherapy C. Autologous stem cell transplantation D. Radiation

A. No treatment - Pt has no organ or tissue damage related to this disease and has no symptoms - Early treatment of these patients does not improve mortality (SOR A) and may increase the likelihood of developing acute leukemia. - symptomatic patients under age 65 (or healthy over 65): autologous stem cell transplantation - Patients who are not candidates for autologous stem cell transplantation: melphalan and prednisolone with or without thalidomide - Radiotherapy can be used to relieve metastatic bone pain or spinal cord compression.

A 33-year-old white female presents with tremor and a history of weight loss. On examination she is found to have mild, regular tachycardia and exophthalmos. Laboratory tests confirm hyperthyroidism. Which one of the following treatments has been found to potentially worsen Grave's ophthalmopathy? (check one) A. Radioactive iodine B. Propylthiouracil C. Methimazole (Tapazole) D. Thyroid hormone replacement plus propylthiouracil E. Thyroidectomy

A. Radioactive iodine - ophthalmopathy of Grave's disease may initially flare and worsen when treated with radioactive iodine. - Antithyroid drugs, including propylthiouracil, and methimazole, are not associated with this problem.

A 24-year-old female presents for her annual examination. She is single and has had several male sexual partners during the past year. You include screening for chlamydial infection in your evaluation, and the test is reported as positive. She is asymptomatic. Which one of the following is true concerning this situation? (check one) A. Failure to treat this patient would place her at higher risk of later infertility B. Only sexual partners with whom she has been active during the last 2 weeks need to be treated C. She should avoid sexual intercourse for 1 month after treatment D. Use of barrier methods of contraception increases her risk for repeat infection Correct.

A. Not treating cause later infertility - sexually active women under the age of 25 years should be screened routinely for Chlamydia trachomatis. - *1-gram dose of oral azithromycin* is an appropriate treatment, including during pregnancy - avoid sexual intercourse for 7 days after initiation of treatment

During a well child examination of a healthy-appearing 4-week-old white male born at term, his mother questions you about a prominence in the left side of his scrotum, which she has noted since his baths were begun. Your physical examination reveals an oblong, nontender, nonreducible, light-transmitting mass closely adhered to or involving the testis. You should recommend which one of the following? (check one) A. Further observation B. Sterile aspiration of the mass C. Immediate surgery D. Surgery in 3-4 months

A. Observation - hydrocele of the tunica vaginalis testis - occurs frequently at birth but usually resolves in a few weeks or months. - No treatment is indicated during the first year of life unless there is a clinically evident hernia. - A simple scrotal hydrocele without communication with the peritoneal cavity and no associated hernia should be excised if it has not spontaneously resolved by the age of *12 months*.

During a comprehensive health evaluation a 65-year-old African-American male reports mild, very tolerable symptoms of benign prostatic hyperplasia, rated as a score of 7 on the American Urological Association Symptom Index. He has never smoked, and his medical history is otherwise unremarkable. Objective findings include an enlarged prostate that is firm and nontender, with no nodules. A urinalysis is normal and his prostate-specific antigen level is 1.8ng/mL. Based on current evidence, which one of the following treatment options is most appropriate at this time? (check one) A. Observation, with repeat evaluation in 1 year B. Saw palmetto C. An α-receptor antagonist D. A 5-α-reductase inhibitor

A. Observation, with repeat in 1 year - mild benign prostatic hyperplasia (BPH). 5 alpha-reductase inhibitors - PSA levels >2.0 ng/mL for men in their 60s (correlate with a prostatic volume >40 mL) Alpha antagonist - for severe symptoms

A 40-year-old nurse presents with a 1-year history of rhinitis, and a more recent onset of episodic wheezing and dyspnea. Her symptoms seem to improve when she is on vacation. She does not smoke, although she says that her husband does. Her FEV1 improves 20% with inhaled β-agonists. Which one of the following is the most likely diagnosis? (check one) A. Occupational asthma B. Sarcoidosis C. COPD D. Anxiety E. Vocal cord dysfunction

A. Occupational asthma - consider this with new adult asthma or recurrence of childhood asthma after a significant asymptomatic period (SOR C). - often preceded by the development of rhinitis in the workplace and should be considered in patients whose symptoms improve away from work Not COPD - reversed by β-agonist Not Cystic fibrosis - Pt is too old Not sarcoidosis - this is reversible airway obstruction & intermittent

A 34-year-old female with menorrhagia is found to have iron deficiency anemia. Which one of the following is true regarding the treatment of this problem with oral iron? (check one) A. An acidic environment enhances the absorption of iron from the gastrointestinal tract B. Iron is absorbed better if taken with food C. Diarrhea is a common complication D. Iron supplementation can be discontinued once the hemoglobin reaches a normal level E. Sustained-release formulations increase the total amount of iron available for absorption

A. Oral iron is absorbed better with an acidic gastric environment, which can be accomplished with the concomitant administration of vitamin C. - Agents that raise gastric pH, such as antacids, proton pump inhibitors, and H2 blockers, should be avoided if possible. - absorption is better if taken on an empty stomach, but this may not be well tolerated - Constipation also is common with oral iron therapy. - Iron therapy should be continued for several months after the hemoglobin reaches a normal level, in order to fully replenish iron stores.

A 24-year-old male presents with a fever of 38.9°C (102.0°F), generalized body aches, a sore throat, and a cough. His symptoms started 24 hours ago. He is otherwise healthy. You suspect novel influenza A H1N1 infection, as there have been numerous cases in your community recently. A rapid influenza diagnostic test is positive, and you recommend over-the-counter symptomatic treatment. You see him 2 days later after he is admitted to the hospital through the emergency department with dehydration and mild respiratory distress. A specimen is sent to the state laboratory for PCR testing. Which one of the following would be most appropriate at this point? (check one) A. Oseltamivir (Tamiflu) B. Zanamivir (Relenza) C. Amantadine (Symmetrel) D. Rimantadine (Flumadine) E. No antiviral treatment

A. Oseltamivir (Tamiflu) - H1N1 virus is almost always susceptible to neuraminidase inhibitors (oseltamivir and zanamivir) and resistant to the adamantanes (amantadine and rimantadine). - Zanamivir should not be used in patients with COPD, asthma, or respiratory distress. - Antiviral treatment of influenza is recommended for all persons with clinical deterioration requiring hospitalization, even if the illness started more than 48 hours before admission. - Antiviral treatment should be started as soon as possible. Waiting for laboratory confirmation is not recommended.

A 50-year-old female complains of a 6-month history of the insidious onset of right shoulder pain and decreased range of motion. She does not respond to consistent use of prescription strength anti-inflammatory medication. Radiographs are negative. Treatment of this patient's condition should include: (check one) A. physical therapy with home exercises B. early surgical referral C. a short course of oral methylprednisolone D. corticosteroid injection of the acromioclavicular joint

A. PT w/ home exercise - adhesive capsulitis or a degenerative rotator cuff tendinopathy. I - must rule out osteoarthritis with radiographs. - Treatment typically includes NSAIDs, subacromial cortisone injections, and physical therapy - These problems take months to treat and should not be referred quickly for surgical evaluation, unless the diagnosis is in question.

Which one of the following findings on examination of the head, oral cavity, and neck is associated with diabetes mellitus? (check one) A. Parotid enlargement B. Tooth erosion C. Diffuse melanin pigmentation D. Cobblestone oral mucosa E. Painful oral ulcers

A. Parotid enlargement- Sialadenosis, bilateral noninflammatory enlargement of the parotid gland, is associated with diabetes mellitus Also w/ Diabetes Periodontal bleeding and inflammation, candidiasis, and delayed wound healing Tooth erosion - GERD or bulimia Cobblestone oral mucosa - Crohn's disease Diffuse melanin pigmentation - is an oral finding of Addison's disease Painful oral ulcers - Behçet syndrome, aphthous ulcers, pemphigus, and pemphigoid.

A 52-year-old male has a skin lesion removed from his arm with appropriate sterile precautions. Which one of the following would be most appropriate to use on this surgical wound? (check one) A. Petrolatum B. Silver sulfadiazine (Silvadene) cream C. Mupirocin (Bactroban) ointment D. Polymyxin B/bacitracin ointment (Polysporin) E. Triple-antibiotic (neomycin/polymyxin B/bacitracin) ointment

A. Petrolatum - The American Academy of Dermatology recommends against the routine use of topical antibiotics for clean surgical wounds,

32-year-old white male undergoes an emergency splenectomy after a motor vehicle accident. Which one of the following should he receive after the surgery? (check one) A. Pneumococcal vaccine and meningococcal vaccine B. Pneumococcal vaccine alone C. Meningococcal vaccine alone D. No immunizations

A. Pneumococcal and meningococcal vaccines are - currently recommended for patients with asplenia.

A 61-year-old female is found to have a serum calcium level of 11.6 mg/dL (N 8.6-10.2) on routine laboratory screening. To confirm the hypercalcemia you order an ionized calcium level, which is 1.49 mmol/L (N 1.14-1.32). Additional testing reveals an intact parathyroid hormone level of 126 pg/mL (N 15-75) and a urine calcium excretion of 386 mg/24 hr (N 100-300). Which one of the following is the most likely cause of the patient's hypercalcemia? (check one) A. Primary hyperparathyroidism B. Malignancy C. Familial hypocalciuric hypercalcemia D. Hypoparathyroidism E. Hyperthyroidism

A. Primary hyperparathyroidism - Primary hyperparathyroidism and malignancy account for more than 90% of hypercalcemia cases - high PTH primary hyperparathyroidism: benign - low PTH Humoral hypercalcemia of malignancy - limited life expectancy (wks) Thyrotoxicosis-induced bone resorption - elevates serum calcium: also low PTH Familial hypocalciuric hypercalcemia (FHH) - moderate hypercalcemia but low urinary calcium excretion (24h urine collection) - PTH can be high

Which one of the following is the leading cause of death following bariatric surgery? (check one) A. Pulmonary embolism B. Adult respiratory distress syndrome C. Peritonitis secondary to an anastomotic leak D. Sepsis related to a wound infection E. Hemorrhage from an anastomotic ulcer

A. Pulmonary emboli; also - anastomotic leaks, and respiratory failure are responsible for 80% of deaths in the 30 days following bariatric surgery, with death from pulmonary embolism being the most frequent cause.

A 30-year-old female stepped off a curb earlier today and twisted her left ankle. She was able to bear weight immediately following the injury and tried to continue her normal routine, but the pain in her ankle and foot increased over the next few hours. She comes to your office and your examination reveals swelling of the ankle and bruising of the lateral foot. Tenderness to palpation is present over the distal aspect of the fibula and lateral malleolus and to a lesser degree over the proximal fifth metatarsal. No bony tenderness is present along the medial aspect of the ankle or foot. According to the Ottawa Ankle Rules, which one of the following would be most appropriate at this point? (check one) A. Radiographs of the ankle and foot B. Radiographs of the foot only C. Radiographs of the ankle only D. No radiographs

A. Radiographs of the ankle and foot The Ottawa Ankle Rules: adults age 19 or older Patients who were able to bear weight immediately following their injury and who can take 4 steps independently in a clinical setting require radiographic study only when the following criteria are met: 1. pain in the malleolar zone and bony tenderness of the posterior edge 2. pain in tip of either malleolus (ankle radiograph) 3. pain in the midfoot zone and bony tenderness of either the base of the fifth metatarsal 4. pain the navicular region is present.

A 29-year-old female presents with redness of her left eye. She has just returned from a summer beach vacation with her children and woke up with a red eye. Your examination reveals a watery discharge, a hyperemic conjunctiva, and a palpable preauricular lymph node. Her cornea is clear on fluorescein staining. Which one of the following is most appropriate for this patient? (check one) A. Reassurance only B. Culture-guided antibiotic therapy C. Quinolone eyedrops D. Corticosteroid/antibiotic eyedrops E. Urgent ophthalmologic referral

A. Reassurance - Viruses cause 80% of infectious conjunctivitis cases and viral conjunctivitis usually requires no treatment. - *adenoviruses*; also cause pharyngeal conjunctival fever and epidermic keratoconjunctivitis. - keratoconjunctivitis epidermic : watery discharge, hyperemia, and ipsilateral lymphadenopathy in >50% of cases. - Pharyngeal conjunctival fever: high fever, pharyngitis, and bilateral eye inflammation Bacterial conjunctivitis - associated with matte-ing and adherence of the eyelids. - Topical antibiotics reduce the duration of bacterial conjunctivitis but have no effect on viral conjunctivitis. Allergic conjunctivitis - will have itching

A 22-year-old female in her second trimester of pregnancy presents with a 48-hour history of a sore throat. She has also had coryza and a nonproductive cough. A physical examination reveals a temperature of 37.3°C (99.2°F) and a blood pressure of 110/70 mm Hg. A HEENT examination reveals tonsillar and pharyngeal erythema with no exudate. There is no adenopathy. Her chest is clear. Which one of the following would be most appropriate for this patient? (check one) A. Reassurance and symptomatic treatment only B. A routine throat culture C. A rapid antigen detection test for Streptococcus D. Azithromycin (Zithromax) for 5 days E. Penicillin V for 10 days

A. Reassurance and symptomatic treatment only pharyngitis - mostly viral

You see a 2-year-old African-American male for a well child check. He is a new patient and his examination is within normal limits except for an approximately 0.75-cm umbilical hernia that is easily reducible. The father states that the hernia has been present since birth, although he thinks it has grown slightly over the last year. The child does not seem to be bothered by the hernia and the father does not think it has ever become incarcerated. Which one of the following should you do now? (check one) A. Reassure and observe B. Advise daily application of pressure dressings C. Order an ultrasound examination D. Refer for surgical repair

A. Reassure and observe Surgical repair of pediatric umbilical hernias is indicated if the hernia has not resolved by 3-5 years of age or for incarcerated hernias at any age.

At a routine visit in October, a 17-year-old primigravida at 10 weeks gestation asks whether she should get influenza vaccine. Her mother recommended it, but she is concerned about the needle stick and potential harm to the fetus. Which one of the following would you do? (check one) A. Recommend intramuscular vaccine and tell her that evidence indicates some protection for the baby up to 6 months of age B. Recommend nasal vaccine because the patient is under age 50 and needle-averse C. Recommend vaccine only if the patient has a coexistent chronic illness D. Recommend that vaccination be delayed until the second trimester to reduce fetal risk E. Recommend immunization of household contacts to reduce maternal risk, but no immunization of the patient

A. Recommend intramuscular vaccine and tell her that evidence indicates some protection for the baby up to 6 months of age - Women who will be pregnant during the influenza season should receive the inactivated vaccine (SOR C). - The live nasal vaccine is not approved for use in pregnancy. The vaccine can be given in any trimester. - Coexistent illness is not required for this indication. There appears to be some protective effect for the infant up to the age of 6 months. - Immunization of family members is sometimes recommended for immunocompromised patients. In the absence of other indications, however, it has not been recommended for family members of pregnant patients

A 59-year-old white female has a blood pressure consistently at or above 140/90 mm Hg. Her only other significant medical problem is diabetes mellitus, which is controlled by diet. Which one of the following is the most clearly established advantage of angiotensin receptor blockers (ARBs) when compared with ACE inhibitors in patients such as this? (check one) A. Reduced risk of persistent cough B. Reduced risk of headache C. Reduced risk of heart failure D. Improved control of blood pressure E. Improved lipid profile

A. Reduced risk of persistent cough

A 50-year-old female presents with a 2-day history of four vesicles on her upper eyelid, but no pain or swelling. She has not experienced any eye trauma, has had no vision changes, and has no other skin changes. Which one of the following would be the most appropriate next step in treating this patient? (check one) A. Referral to an ophthalmologist B. A methylprednisolone (Medrol) dose pack C. A topical corticosteroid D. Topical mupirocin (Bactroban) E. Topical metronidazole (MetroGel)

A. Referral to an ophthalmologist - herpes zoster ophthalmicus. - In addition to treatment with a systemic antiviral agent, it is important that the patient see an ophthalmologist to be evaluated for corneal disease and iritis, as vision can be lost

A 75-year-old male develops a mild Clostridium difficile infection and is treated with 10 days of metronidazole (Flagyl), 500 mg orally 3 times daily. The diarrhea recurs 10 days after he completes the course of treatment. Which one of the following would be most appropriate? (check one) A. Repeat the course of metronidazole B. Repeat the course of metronidazole and add vancomycin C. Administer vancomycin intravenously D. Prescribe loperamide (Imodium), 4 mg twice daily as needed E. Prescribe a probiotic

A. Repeat the course of metronidazole For mild recurrent C. diff, repeating the course of the original agent is appropriate (SOR B). Multiple recurrences or severe disease warrants the use of both agents.

The "Get Up and Go Test" evaluates for which one of the following? (check one) A. Risk of falling B. Effects of peripheral neuropathy C. Kinetic tremor D. Neurocardiogenic syncope E. Central causes of vertigo

A. Risk of falling

An 80-year-old female is seen for progressive weakness over the past 8 weeks. She says she now has difficulty with normal activities such as getting out of a chair and brushing her teeth. Her medical problems include hypertension, diabetes mellitus, and hyperlipidemia. Her medications include glipizide (Glucotrol), simvastatin (Zocor), and lisinopril (Prinivil, Zestril). Findings on examination are within normal limits except for diffuse proximal muscle weakness and normal deep tendon reflexes. A CBC, urinalysis, erythrocyte sedimentation rate, TSH level, and serum electrolyte levels are normal. Her blood glucose level is 155 mg/dL, and her creatine kinase level is 1200 U/L (N 40-150). Which one of the following is the most likely diagnosis? (check one) A. Statin-induced myopathy B. Polymyalgia rheumatica C. Guillain-Barré syndrome D. Diabetic ketoacidosis

A. Statin induced myopathy - with elevated creatine kinase Not Polymyalgia rheumatica - high ESR Not Guillain-Barré syndrome - depressed deep tendon reflexes Not ketoacidosis -no clinical or lab features

An 80-year-old female is seen for progressive weakness over the past 8 weeks. She says she now has difficulty with normal activities such as getting out of a chair and brushing her teeth. Her medical problems include hypertension, diabetes mellitus, and hyperlipidemia. Her medications include glipizide (Glucotrol), simvastatin (Zocor), and lisinopril (Prinivil, Zestril). Findings on examination are within normal limits except for diffuse proximal muscle weakness and normal deep tendon reflexes. A CBC, urinalysis, erythrocyte sedimentation rate, TSH level, and serum electrolyte levels are normal. Her blood glucose level is 155 mg/dL and her creatine kinase level is 1200 U/L (N 40-150). Which one of the following is the most likely diagnosis? (check one) A. Statin-induced myopathy B. Polymyalgia rheumatica C. Guillain-Barre syndrome D. Diabetic ketoacidosis

A. Statin-induced myopathy This patient is most likely suffering from a drug-induced myopathy caused by simvastatin, which is associated with elevated creatine kinase. Polymyalgia rheumatica is usually associated with an elevated erythrocyte sedimentation rate. Guillain-Barré syndrome is associated with depressed deep tendon reflexes. This case has no clinical features or laboratory findings that suggest ketoacidosis.

A 68-year-old male was seen in a local urgent-care clinic 6 days ago for upper respiratory symptoms and was started on cefuroxime (Ceftin). He presents to your office with a 2-day history of 4-5 watery stools per day with no blood or mucus. He is afebrile and has a normal abdominal and rectal examination. A stool guaiac test is negative, and a stool sample is sent for further testing. What is the best initial management for this patient? (check one) A. Stop the cefuroxime B. Start ciprofloxacin (Cipro) C. Start metronidazole (Flagyl) D. Start loperamide (Imodium) E. Recommend probiotics until he completes the course of cefuroxime

A. Stop the cefuroxime - initial management is to stop the antibiotics. - treatment should not be initiated unless the stool is positive for toxins A and B.

A 3-year-old female is brought to your office for a health maintenance examination, and her father expresses concern about her vision. Her visual acuity is 20/20 bilaterally on a tumbling E visual acuity chart. With both eyes uncovered during a cover/uncover test, the corneal light reflex in the right eye is medial to the pupil when focused on a fixed point, but the light reflex in the left eye is almost centered in the pupil. When the left eye is covered, the right eye moves quickly inward to focus on the fixed point, and the corneal light reflex is centered in the pupil. When the left eye is uncovered, the right eye returns to its original position. When you cover the right eye, no left eye movement is noted. Which one of the following is the most likely diagnosis? (check one) A. Strabismus B. Amblyopia C. Cataract D. Esotropia E. Heterophoria

A. Strabismus - ocular misalignment that can be diagnosed on a cover/uncover test when the corneal light reflex is deviated from its normal position slightly nasal to mid-pupil. - The misaligned eye then moves to fixate on a held object when the opposite eye is covered - The eye drifts back to its original position when the opposite eye is uncovered. Amblyopia - cortical visual impairment from abnormal eye development -most often as a result of strabismus.

A 37-year-old recreational skier is unable to lift his right arm after falling on his right side with his arm elevated. Radiographs of the right shoulder are negative, but diagnostic ultrasonography shows a complete rotator cuff tear. Which one of the following is most accurate with regard to treatment? (check one) A. Surgery is most likely to be beneficial if performed less than 6 weeks after the injury B. Treatment with NSAIDs for 3 months is recommended before further intervention C. Subacromial corticosteroid injections will provide functional and symptomatic relief in the majority of patients D. Surgical repair of rotator cuff tears to restore function is necessary only in geriatric patients E. Therapeutic ultrasound of the shoulder will make the condition tolerable during spontaneous healing

A. Surgery is most likely to be beneficial if performed less than 6 weeks after the injury - most beneficial in young, active patients - in cases of acute, traumatic, complete rotator cuff tears, repair is recommended in less than 6 weeks, as muscle atrophy is associated with reduced surgical benefit (SOR B). - Advanced age and limited strength are also associated with reduced surgical benefit.

An 85-year-old male nursing home resident with a past history of a stroke has developed a pressure ulcer over his right greater trochanter. The ulcer is 2 cm in size and is noted to be shallow with a reddish-pink wound base. There is no evidence of secondary infection. Which one of the following would be best for cleansing the wound? (check one) A. Tap water B. Aluminum acetate (Burow's solution) C. Hydrogen peroxide D. Povidone/iodine solution (Betadine) E. Sodium hypochlorite (Dakin's solution)

A. Tap water - Stage 2 ulcer - No cleaning with povidone/iodine, Dakin's solution, hydrogen peroxide, wet-to-dry dressings, or anything that impedes granulation tissue formation. - Clean with ONLY saline or tap water, covered with hydrocolloid, foam, or other non-adherent dressing that promotes a moist environment.

A 24-year-old female has noted excessive hair loss over the past 2 months, with a marked increase in hairs removed when she brushes her hair. She delivered a healthy baby 5 months ago. She is on no medications, and is otherwise healthy. Examination of her scalp reveals diffuse hair thinning without scarring. An evaluation for thyroid dysfunction and iron deficiency is negative. Which one of the following is the most likely cause of her hair loss? (check one) A. Telogen effluvium B. Anagen effluvium C. Alopecia areata D. Female-pattern hair loss E. Discoid lupus erythematosus

A. Telogen effluvium - nonscarring, shedding hair loss - occurs when a stressful event, such as a severe illness, surgery, or pregnancy, triggers the shift of large numbers of anagen-phase hairs to the telogen phase - usu 3 mos after stressful trigger - 3 phase 1. anagen-phase: actively growing 2. catagen-phase: follicle shuts down 3. telogen-phase: hair shed (anagen: telogen = 90:10) Not Anagen effluvium - diffuse hair loss that occurs when chemo cause rapid destruction of anagen-phase hair Not Alopecia areata - round patches of hair loss Not Female-pattern hair loss - affects the central portion of the scalp - is not associated with an inciting trigger or shedding Discoid lupus erythematosus - scarring alopecia.

Which one of the following skin infections should initially be treated with oral antifungal therapy? (check one) A. Tinea capitis B. Tinea corporis C. Tinea cruris D. Erythrasma E. Mycosis fungoides

A. Tinea capitis - oral therapy is required for tinea capitis so that the drug will penetrate the hair shafts (SOR B). - Tinea corporis may require oral therapy in severe cases, but usually responds to topical therapy (SOR A). Not Erythrasma and mycosis fungoides: not fungal diseases

The results of a given study are reported as achieving significance at a p-value of <0.05 (the 5% level). True statements about this finding include which one of the following? (check one) A. There is a 5% likelihood of the results having occurred by chance alone B. If the study were replicated 100 times, 95 studies would repeat this finding and 5 would not C. The confidence interval is 0%-10% D. The null hypothesis has a 5% chance of being true E. The B or type II error is < 5%

A. There is a 5% likelihood of the results having occurred by chance alone p-value - level of statistical significance, and characterizes the likelihood of achieving the observed results of a study by chance alone, and in this case that likelihood is 5% confidence interval - measure of variance and is derived from the test data. The a or type I error - the error of false-positive assignment the B or type II error - to the false-negative rate - 1 - specificity, cannot be calculated from the information given

A 7-year-old male is brought to your office with a 2-day history of rash. He developed two itchy spots on his legs yesterday and today he has multiple purple, slightly painful lesions on his legs. A few days ago he was ill with cold-like symptoms, stomach pain, and a fever up to 101.2°F. He complained of leg pain at the time and his left ankle is now swollen. His fever resolved 2 days ago and he now feels fine but limps when he walks. On examination he is afebrile with a normal blood pressure and pulse rate. He is active in the examination room. His physical examination is normal except for purpuric lesions on his legs and buttocks and edema and mild pain of the left ankle. A urinalysis is negative. Which one of the following would be most appropriate in the management of this patient? (check one) A. Acetaminophen B. Amlodipine (Norvasc) C. Amoxicillin D. Cyclophosphamide E. Prednisone

A. This patient meets the clinical criteria for Henoch-Schönlein purpura (HSP), an immune-mediated vasculitis found commonly in children under the age of 10. The clinical triad of purpura, abdominal pain, and arthritis is classic. Almost 95% of children with HSP spontaneously improve, so supportive therapy is the main intervention. Acetaminophen or ibuprofen can be used for the arthritic pain. However, ibuprofen should be avoided in those with abdominal pain or known renal involvement. Prednisone has been found to help in those with renal involvement or other complications of the disease such as significant abdominal pain, scrotal swelling, or severe joint pains (SOR B). However, it is not effective for preventing renal disease or reducing the severity of renal involvement, as was once thought (SOR A). Immunosuppressants such as cyclophosphamide and cyclosporine have been suggested for treating patients with severe renal involvement, but there is insufficient evidence to support their use. Amoxicillin is appropriate for patients with a bacterial infection, such as streptococcal pharyngitis, which has led to HSP. Patients with renal involvement and resultant hypertension with HSP should be treated with calcium channel blockers such as amlodipine.

A 24-year-old female presents with pelvic pain. She says that the pain is present on most days, but is worse during her menses. Ibuprofen has helped in the past but is no longer effective. Her menses are normal and she has only one sexual partner. A physical examination is normal. Which one of the following should be the next step in the workup of this patient? (check one) A. Transvaginal ultrasonography B. CT of the abdomen and pelvis C. MRI of the pelvis D. A CA-125 level E. Colonoscopy

A. Transvaginal ultrasonography work up: a urinalysis and culture, cervical swabs for gonorrhea and Chlamydia, a CBC, an erythrocyte sedimentation rate, a β-hCG level, and pelvic ultrasonography

A 35-year-old right-handed softball player injures his left wrist when sliding into second base. When he sees you the next day his description of the injury indicates that he hyperextended his wrist while sliding, and the pain was later accompanied by swelling. Your examination is remarkable only for mild swelling and tenderness of the dorsal wrist, distal to the ulnar styloid. A radiograph of the wrist is shown in Figure 2. Figure 2 Which one of the following best describes this injury? (check one) A. Triquetral fracture B. Scaphoid (navicular) fracture C. Lunate fracture D. Lunate dislocation E. Wrist sprain

A. Triquetral fracture - hyperextension of the wrist - Dorsal avulsion fractures: more common than fractures of the body of the bone - Tenderness is characteristically noted on the dorsal wrist on the ulnar side distal to the ulnar styloid. - Xray: is a small bony avulsion visible on a lateral view of the wrist - Most studies indicate that this carpal bone has the second or third highest fracture rate after the navicular - Avulsion fractures respond well to 4 weeks of splinting and protection. Not navicular or scaphoid: normal Xray & tenderness to snuffbox Not lunate: not on Xray, diagnosis of exclusion and should not be considered too early.

Studies indicate that patients most frequently want physicians to ask about their spiritual beliefs in which one of the following situations? (check one) A. When being treated for a potentially fatal illness B. During the annual preventive visit C. During the initial office visit with the physician D. Only if specifically requested by the patient, a family member, their minister, or a chaplain E. When prayer is suggested by the patient or physician

A. When being treated for potentially illness inquiry should focus on understanding, compassion, and hope, and should be directed toward individuals who suffer from serious illness.

A 30-year-old white male visits your clinic after being in a bar fight. He describes hitting another man in the mouth with his closed fist. He reports a painful distal fifth metacarpal with a superficial abrasion. After assessing tetanus status and copiously irrigating the wound, you should do which one of the following? (check one) A. Obtain a radiograph and give prophylactic antibiotics B. Obtain a radiograph only C. Give prophylactic antibiotics only D. Probe the abrasion

A. Xray & prophylatic abx - a common injury called a "fight bite." - Radiographs are needed to determine if there is a distal metacarpal fracture so that it can be treated appropriately. - prophylactic antibiotics are recommended with any break in the skin

Which one of the following is true concerning the use of dexamethasone to treat acute laryngotracheitis (croup)? (check one) A. A single dose is adequate for treatment B. It commonly leads to a secondary bacterial infection due to immunosuppression C. It increases the need for hospitalization D. It is indicated only for patients with severe croup

A. a single dose is adequate - A single dose of dexamethasone, either orally or intramuscularly, is appropriate. - Prolonged courses of corticosteroids provide no additional benefit and may lead to secondary bacterial or fungal infections. - Secondary infections rarely occur with single-dose treatment. - Corticosteroid therapy shortens emergency department stays and decreases the need for return visits and hospitalizations. It is indicated for patients with croup of any severity.

Fibromyalgia is characterized by tender trigger points (check one) A. along the medial border of each scapula B. bilaterally at the anatomic snuffbox C. at the insertion of the Achilles tendon into the posterior heel D. at the second and third web spaces on the plantar surface of the foot

A. along the medial scapula borders; also - posterior neck - upper outer quadrants of the gluteal muscles - medial fat pads of the knees

The most appropriate initial pharmacologic treatment of panic disorder is: (check one) A. An SSRI B. A tricyclic antidepressant C. Valproic acid (Depakene) D. Lithium

A. an SSRI An SSRI is the treatment of choice for patients who have never had pharmacotherapy for panic disorder.

According to the U.S. Preventive Services Task Force, multivitamin supplements in the geriatric age group: (check one) A. are not recommended for prevention of any disorder B. should be prescribed to reduce elevated homocysteine levels C. decrease coronary atherosclerosis D. decrease the incidence of lung cancer E. decrease the incidence of colon cancer

A. are not recommended for prevention of any disorder The U.S. Preventive Services Task Force makes no specific recommendations for vitamins or antioxidants to prevent cancer or cardiovascular disease. Moreover, it makes no specific recommendations for vitamin supplements for any condition.

A 36-year-old female consults you because of concerns about "fatigue." After carefully reviewing her history and performing a physical examination, which one of the following would be LEAST valuable in assessing this patient? (check one) A. A baseline serum cortisol level B. An erythrocyte sedimentation rate C. A complete metabolic panel D. A TSH level E. A pregnancy test

A. baseline cortisol - only for Addison's disease (no other explanation)

The best available evidence supports which one of the following guidelines for discussing serious illnesses? (check one) A. Physicians should delay having a detailed discussion with the patient about the expected prognosis of cancer until staging is completed B. For patients who are ambivalent about knowing their prognosis, the discussion should focus on optimal potential outcomes and providing hope, even if this is unrealistic C. Physicians should delay discussions about palliative care until curative measures have failed D. Physicians should respect the family's wishes regarding how much information to share with the patient

A. best to discuss prognosis after accurate cancer staging, when specific details about survival rates will give a much clearer and more accurate picture. - After assessing the patient's readiness to receive prognostic information, the physician should focus on communicating an accurate prognosis without giving a false sense of hope. - Using simultaneous-care models, physicians can provide palliative and curative care at the same time. - Physicians should initiate a discussion about the availability of coordinated, symptom directed services such as palliative care early in the disease process; as the disease progresses, patients should transition from curative to palliative therapy. - How much information to share with the patient depends on the physician's assessment of the patient's level of understanding about the disease and how much patients themselves want to know.

A 24-year-old female who works at a day-care facility presents to your office to discuss ways to avoid getting "all the infections the kids get." She plans to enroll her child in the facility. She is specifically concerned about diarrheal illnesses, and a friend has suggested the use of probiotics. (check one) A. can lessen the severity and duration of infectious diarrhea B. are recommended only for patients who are immunocompromised C. have no known side effects D. often interact with common prescription medications E. are not appropriate for use in children

A. can lessen severity/duration of infectious diarrhea - Common side effects include flatulence and abdominal pain - Contraindications include short-gut syndromes and immunocompromised states. - There are no known drug interactions, and these agents appear safe for all ages (SOR A).

Which one of the following historical or audiographic findings in an elderly person would indicate that hearing loss is due to something other than presbycusis? (check one) A. Conductive hearing loss B. Bilateral hearing loss C. Symmetric hearing loss D. Gradual hearing loss E. High-frequency hearing loss

A. conductive hearing loss Presbycusis, the hearing loss associated with aging, is gradual in onset, bilateral, symmetric, and sensorineural

An 84-year-old African-American female is brought to your office by her daughter, who is concerned that the mother has memory problems and is neglecting to pay her monthly bills. The mother also is forgetting appointments and asks the same questions repeatedly. This problem has been steadily worsening over the last 1-2 years. The patient has very little insight into her problems, scores 24 out of a possible 30 points on the Mini-Mental State Examination, and has difficulty with short-term recall and visuospatial tasks. Her physical examination and a thorough laboratory workup are normal. A CT scan of the brain reveals diffuse atrophy. Which one of the following is the most likely etiology for this patient's memory problem? (check one) A. Alzheimer's disease B. Dementia resulting from depression C. Lewy body dementia D. Multi-infarct dementia E. Normal aging

A. early Alzheimer's disease - difficulties in at least two cognitive domains that are severe enough to influence daily living Not Lewy body dementia - physical findings of parkinsonism - visual hallucinations

When draining a felon, which one of the following incisions is recommended? (check one) A. A "fishmouth" bilateral incision B. A "hockey stick" J-shaped incision including the distal and lateral aspects of the digit C. A transverse volar incision D. A high lateral incision

A. high lateral incision When draining a felon, a volar longitudinal incision or a high lateral incision is recommended.

A 72-year-old female is admitted to the hospital after having surgery for a hip fracture. Her previous medical history is significant for hypertension and type 2 diabetes mellitus. Two days after admission the orthopedic surgeon consults with you because the patient has had several hours of fever to 39°C (102°F); tachycardia, with a pulse rate of 120 beats/min; and systolic blood pressures of 91-97 mm Hg (baseline 120-140 mm Hg with medication). When you examine the patient she says she feels weak and chilled but she is alert. Her oxygen saturation is excellent on room air, and a physical examination is normal except for the sinus tachycardia and low blood pressure. A urinary catheter is in place, but there has been little output over the last 4 hours. Her renal function was normal prior to her hospitalization. A chest radiograph is normal. Her electrolyte levels are normal, but laboratory tests reveal the following abnormal results: WBCs. . . .2500/mm3 (N 5000-10,000) BUN. . . . 50 mg/dL (N 10-15) Creatinine. . . 2.3 mg/dL (N 0.6-1.0) Bicarbonate. . . . 18 mmol/L (N 22-30) Urinalysis Specific gravity. . . >1.030 (N 1.003-1.040) WBCs. . . . . . >100/hpf RBCs.. . . . . 10-20/hpf Epithelial cells. . . 3-5/hpf Casts. . . . . . .few hyaline In addition to antibiotics, which one of the following would be the most appropriate management of this patient's problem? (check one) A. High-rate intravenous normal saline B. Intravenous furosemide, 40 mg every 6 hours C. Intravenous dopamine, 2-4 µg/kg/min D. Intravenous sodium bicarbonate E. Urgent nephrology consultation for dialysis

A. high rate IV fluid - sepsis syndrome due to urinary infection. - The renal failure that has resulted is almost certainly due to low perfusion of the kidneys (prerenal azotemia). - This condition requires aggressive intravenous fluids to halt and reverse the reduction in nephrologic function. - underperfusion can result in acute tubular necrosis (an intrinsic renal dysfunction) that may prevent excretion of any excess fluid, so the patient's fluid status should be monitored carefully. - Metabolic acidosis will likely reverse with appropriate hydration, and sodium bicarbonate should be reserved for severe acidosis (<10-15 mmol/L) or for those with chronic kidney disease. Not Low-dose dopamine - proven to be ineffective in acute renal failure, and this patient does not have an indication for dialysis. Not Intravenous furosemide - contraindicate

A 55-year-old white male notices a nodular thickening over the flexor tendons in his medial palm. He has no difficulty using his hand, and he is able to lay his palm flat on a tabletop. You suspect Dupuytren's disease. Which one of the following is true regarding this condition? (check one) A. There is a strong association with diabetes mellitus B. Surgical intervention is recommended at this point to prevent progression to contracture C. Once a contracture develops, it is irreversible and no treatment is indicated D. A single cortisone injection often leads to disease regression in mild to moderate cases E. A search for an occult malignancy is indicated

A. highly associated with Diabetes - up to 1/3 diabetic patients having evidence of the disease - also associated with alcohol use and smoking - Patients requiring surgery have an increased risk of dying from cancer, probably related to smoking, alcohol use, or diabetes mellitus, but a search for cancer at the time of diagnosis is not indicated. - shortening and thickening of the palmar fascia - initially asymptomatic (no treatment) - may progress and cause difficulty with function of the hand, and may eventually lead to contracture Not a single cortisone injection - serial injection Surgery - if metacarpal joint contracture reaches 30° - or with a proximal interphalangeal joint contracture of any degree. - If surgery is delayed, irreversible joint contracture may occur.

An 8-year-old white male presents with a 4-day history of erythematous cheeks, giving him a "slapped-cheek" appearance. Examination of the extremities reveals a mildly pruritic, reticulated, erythematous, maculopapular rash (see Figure 1). He is afebrile and no other constitutional symptoms are present. Figure 1 The most likely etiologic agent is (check one) A. human parvovirus B. adenovirus C. cytomegalovirus D. coxsackievirus

A. human parvovirus

In a patient with a severe anaphylactic reaction to peanuts, the most appropriate route for epinephrine is: (check one) A. intramuscular B. intravenous C. oral D. subcutaneous E. sublingual

A. intramuscular

A 42-year-old female presents to the emergency department with a 2-hour history of palpitations. Her physical examination is normal except for what seems to be a regular rhythm tachycardia and a blood pressure of 84/54 mm Hg. An EKG reveals a regular narrow-complex tachycardia at a rate of 180 beats/min without clear atrial activity. The optimal treatment for this patient is (check one) A. intravenous adenosine (Adenocard) B. intravenous amiodarone (Cordarone) C. intravenous diltiazem D. intravenous verapamil E. electrical cardioversion

A. intravenous adenosine (Adenocard) Vagal maneuvers and administration of adenosine are useful in the diagnosis and treatment of narrow-complex supraventricular tachycardias. *Adenosine*, a very short-acting endogenous nucleotide that *blocks atrioventricular nodal conduction*, *terminates* nearly *all atrioventricular nodal reentrant tachycardias* and *atrioventricular reciprocating tachycardias*, as well as up to 80% of *atrial tachycardias*. Although intravenous *verapamil* and *diltiazem*, which also block the atrioventricular node, have a potential diagnostic and therapeutic use in narrow-complex tachycardia, they may cause *hypotension* and thus are not a first choice in the emergency setting.

A 14-year-old female sees you for follow-up after hypercalcemia is found on a chemistry profile obtained during a 5-day episode of vomiting and diarrhea. She is now asymptomatic, but her serum calcium level at this visit is 11.0 mg/dL (N 8.5-10.5). Her aunt underwent unsuccessful parathyroid surgery for hypercalcemia a few years ago. Which one of the following laboratory findings would suggest a diagnosis other than primary hyperparathyroidism? (check one) A. Low 24-hour urine calcium B. Decreased serum phosphate C. High-normal to increased serum chloride D. Elevated alkaline phosphatase E. Elevated parathyroid hormone

A. low 24-hour urine calcium levels or a low urine calcium to urine creatinine ratio is not characteristic of hyperparathyroidism. - This finding should suggest familial hypocalciuric hypercalcemia (SOR C). - Awareness of this condition is important to avoid unnecessary surgery. - The parathyroid hormone level may be mildly elevated. Hyparathyroidism will have - high PTH - high serum chloride - high Alk phos - low serum phosphate

A 68-year-old female presents with a several-month history of weight loss, fatigue, decreased appetite, and vague abdominal pain. The most appropriate initial test to rule out adrenal insufficiency is: (check one) A. morning serum cortisol B. a cosyntropin (ACTH) stimulation test C. MRI D. an insulin tolerance test E. a metyrapone test

A. morning serum cortisol - >13µg/dL reliably excludes adrenal insufficiency. - If the morning cortisol level is lower than this, further evaluation with a 1µg ACTH stimulation test is necessary (difficult test) - It requires dilution of the ACTH prior to administration, and requires multiple blood draws. - The insulin tolerance test and metyrapone test, although historically considered to be "gold standards," are not widely available or commonly used in clinical practice.

A 70-year-old male with widespread metastatic prostate cancer is being cared for through a local hospice. Surgery, radiation, and hormonal therapy have failed to stop the cancer, and the goal of his care is now symptom relief. Over the past few days he has been experiencing respiratory distress. His oxygen saturation is 94% on room air and his lungs are clear to auscultation. His respiratory rate is 16/min. Which one of the following would be best at this point? A. Morphine B. Oxygen C. Albuterol (Proventil, Ventolin) D. Haloperidol

A. morphine Dyspnea is a frequent and distressing symptom in terminally ill patients. In the absence of hypoxia, oxygen is not likely to be helpful. Opiates are the mainstay of symptomatic treatment

A 45-year-old white male is admitted to the intensive-care unit after being pinned in a car wreck for 2 hours. He has sustained several broken bones and crush injuries to both thighs. On admission his urine is clear but the next morning it is burgundy colored. Some fresh urine is drawn from his Foley catheter and sent for analysis, with the following results: Specific gravity............1.020 pH............6.0 Protein............30 mg/dL (N 1-14) Glucose............negative Hemoglobin............4+ Urobilinogen............0.1 Ehrlich Units (N 0.1-1.0) Bile............negative RBCs............1-2/hpf WBCs............0-2/hpf Occasional hyaline casts You immediately order a CBC which shows his hematocrit to have dropped 4 percentage points overnight. Visual inspection of the serum shows it is light yellow. The color of his urine is most likely due to (check one) A. myoglobinuria B. hematuria from trauma to the urinary tract C. a transfusion reaction with hemolysis of RBCs and free hemoglobin into the urine D. hemoglobinuria resulting from reabsorption of hemoglobin from hematomas E. acute porphyria provoked by trauma

A. myoglobinemia - from broken down muscle - A positive dipstick for hemoglobin without any RBCs in the urine sediment = free hemoglobin or myoglobin in the urine - fresh sample so significant RBC hemolysis within the urine would not be expected Not transfusion reaction - haptoglobin binds enough hemoglobin to turn serum pink - Only when haptoglobin is saturated will the free hemoglobin be excreted in the urine. Not trauma - there would be many RBCs visible on microscopic examination of the urine. Not Free hemoglobin resorption - does not occur Not Porphyria - urine maybe be burgundy colored - not associated with a positive urine test for hemoglobin

A 17-year-old soccer player presents for a preparticipation examination. His family history is significant for the sudden death of his 12-year-old sister while playing basketball, and for his mother and maternal grandmother having recurrent syncopal episodes. His medical history and examination are completely normal. Prior to approving his participation in sports, which one of the following is recommended? (check one) A. A resting EKG B. A stress EKG C. An echocardiogram D. Pulmonary function testing E. No further evaluation

A. resting EKG - A family history of sudden death and recurrent syncope is highly suspicious for genetic long-QT syndrome. - a resting EKG that shows a QTc >460 msec in females and >440 msec in males - Management may include β-blockers, an implantable cardioverter-defibrillator, and no participation in competitive sports

You see a 5-year-old white female with in-toeing due to excessive femoral anteversion. She is otherwise normal and healthy, and her mobility is unimpaired. Her parents are greatly concerned with her appearance and possible future disability, and request that she be treated. You recommend which one of the following? (check one) A. Observation B. Medial shoe wedges C. Torque heels D. Sleeping in a Denis Browne splint for 6 months E. Derotational osteotomy of the femur

A. observation There is little evidence that femoral anteversion causes long-term functional problems. Studies have shown that shoe wedges, torque heels, and twister cable splints are not effective. Surgery should be reserved for children 8-10 years of age who still have cosmetically unacceptable, dysfunctional gaits.

Which one of the following is most likely to be associated with resistant hypertension in adults? (check one) A. Obstructive sleep apnea B. Primary aldosteronism C. Renal artery stenosis D. Renal parenchymal disease E. Thyroid disease

A. obstructive sleep apnea

A 7-year-old African-American male is brought to your office with a 1-day history of purulent, crusted eyelashes in the morning, and red eye. There is no history of visual change, foreign body, or injury. The child is otherwise in good health and has normal developmental milestones. No fever or respiratory distress is noted. A clinical diagnosis of bacterial conjunctivitis is made. The mother is anxious to keep the child in school. Which one of the following would be the most appropriate time for the child to return to school? (check one) A. Once treatment is started B. When there is no crusting or drainage in the morning C. After 1 week of treatment D. When the absence of fever for 24 hours is documented

A. once treatment has started Once therapy is initiated, children with bacterial conjunctivitis should be allowed to remain in school.

A 47-year-old male who lives at sea level attempts to climb Mt. Rainier. On the first day he ascends to 3400 m (11,000 ft). The next morning he complains of headache, nausea, dizziness, and fatigue, but as he continues the climb to the summit he becomes ataxic and confused. Which one of the following is the treatment of choice? (check one) A. Administration of oxygen and immediate descent B. Dexamethasone, 8 mg intramuscularly C. Acetazolamide (Diamox), 250 mg twice a day D. Nifedipine (Procardia), 10 mg immediately, followed by 30 mg in 12 hours E. Helicopter delivery of a portable hyperbaric chamber

A. oxygen and immediate descent - acute mountain sickness. - headache in an unacclimatized person who recently arrived at an elevation >2500 m (8200 ft), - plus the presence of one or more of the following: anorexia, nausea, vomiting, insomnia, dizziness, or fatigue. - pt already deteriorated to high-altitude cerebral edema i.e. ataxia and/or altered consciousness in someone - management of choice is a combination of descent and supplemental oxygen - Often, a descent of only 500-1000 m (1600-3300 ft) will lead to resolution of acute mountain sickness. - Simulated descent with a portable hyperbaric chamber also is effective, but descent should not be delayed while awaiting helicopter delivery. - if descent and/or administration of oxygen is not possible, medical therapy with *dexamethasone* and/or *acetazolamide* may reduce the severity of symptoms. - Nifedipine has also been shown to be helpful in cases of high-altitude pulmonary edema where descent and/or supplemental oxygen is unavailable.

A 32-year-old female experiences an episode of unresponsiveness associated with jerking movements of her arms and legs. Which one of the following presentations would make a diagnosis of true seizure more likely? (check one) A. Post-event confusion B. Eye closure during the event C. A history of fibromyalgia D. A history of chronic back pain E. A normal serum prolactin level after the event

A. post-event confusion - also tongue biting, aura, focal neurologic signs - Up to 20% of patients diagnosed with epilepsy actually have pseudoseizures Not Eye closure throughout the event - uncommon in true seizures Not fibromyalgia or chronic pain syndrome - predictive of pseudoseizures. Not serum prolactin - may be useful w/in 20 mins - not sensitive (60%) or specific (46%)

A 26-year-old gravida 1 para 0 presents for a prenatal examination. She has two cats and expresses concern about toxoplasmosis. Which one of the following would be most appropriate for this patient? (check one) A. Recommend that she avoid directly handling the cats' litter box B. Immunize the patient against toxoplasmosis C. Prophylactically treat the cats with antibiotics D. Screen the patient's urine for Toxoplasma antigens E. Screen the patient's serum for Toxoplasma antibody

A. recommend that she avoid direct handling of litter box - the American College of Obstetrics and Gynecology does not recommend routine screening except in patients who are known to be HIV positive - However, because the infection is thought to be passed primarily from undercooked meat or through infected animal feces, it is universally recommended that pregnant women avoid direct contact with cats' litter boxes.

An 85-year-old male is admitted to a nursing home due to weakness, debility, and limitation of activities of daily living (ADLs) after being hospitalized for acute community-acquired pneumonia. He previously lived with his wife independently and his goal is to return home when he is strong enough. He has a history of coronary artery disease, type 2 diabetes mellitus controlled with diet, hypertension, and chronic diastolic heart failure, but he has no symptoms related to these chronic problems. His appetite is poor and he has lost a significant amount of weight. His admission diet order from the hospital was a cardiac diet. Which one of the following would be the most appropriate diet for this patient? (check one) A. A regular diet B. An American Heart Association diet C. A diet with no added salt D. An 1800-calorie/day American Diabetes Association diet E. A diet with no concentrated sweets

A. regular diet Correct. - promote weight gain in nursing-home residents with unintentional weight loss. - Malnutrition and unintentional weight loss are significant problems in nursing-home residents and lead to multiple complications, including pressure ulcers and infections - equivalent glycemic control in nursing-home residents who ate a regular diet compared to those who ate a restricted American Diabetes Association diet (SOR C) - Low-salt and low-cholesterol diets are unpalatable and are often associated with protein-energy malnutrition and postural hypotension in older persons. Special diets should be avoided whenever possible in nursing-home patients.

An otherwise healthy 1-year-old white male has a screening hemoglobin level of 10.5 g/dL (N 11.3-14.1), a mean corpuscular volume of 68 fL (N 71-84), and an undetectable serum lead level. What should be your next step? (check one) A. A therapeutic trial of iron for 1 month B. A serum ferritin level C. An erythrocyte protoporphyrin level D. Hemoglobin electrophoresis E. Bone marrow examination

A. therapeutic iron x 1 mo - therapeutic trial of iron is the best approach to the treatment of iron deficiency in late infancy. - If the anemia fails to respond, investigating other causes of anemia is indicated.

A 35-year-old white female comes to your office with a 3-month history of the gradual onset of pain and tenderness in her wrists and hands. She also complains of 1 hour of morning stiffness. She denies rash, fever, or skin changes. On physical examination she has symmetric swelling of the proximal interphalangeal joints and metacarpophalangeal joints. Motion of these joints is painful. She has no rash or mouth ulcers. Radiographs of the hands and wrists are negative, and a chest film is unremarkable. A CBC is normal, but the erythrocyte sedimentation rate is elevated at 40 mm/hr. Latex fixation for rheumatoid factor is negative, and an antinuclear antibody (ANA) test is negative. The most likely diagnosis in this patient is (check one) A. rheumatoid arthritis B. systemic lupus erythematosus C. sarcoidosis D. Lyme disease

A. rheumatoid arthritis This patient has rheumatoid arthritis (RA) by symptoms and physical findings. A positive latex fixation test for rheumatoid factor is not necessary for the diagnosis. A negative rheumatoid factor does not exclude RA, and a positive rheumatoid factor is not specific. Rheumatoid factor is found in the serum of approximately 85% of adult patients with RA; in subjects without RA, the incidence of positive rheumatoid factor is 1%-5% and increases with age. The ANA test is positive in at least 95% of patients with systemic lupus erythematosus, but in only about 35% of patients with RA. Elevation of the erythrocyte sedimentation rate is seen in many patients with RA, and the degree of elevation roughly parallels disease activity. At a mean of 6 months after the onset of Lyme disease, 60% of patients in the United States have brief attacks of asymmetric, oligoarticular arthritis, primarily in the large joints and especially in the knee.

The Health Insurance Portability and Accountability Act (HIPAA) (check one) A. sets a federal minimum on the protection of privacy B. requires that privacy notices be acknowledged and signed at each office visit C. allows the patient to inspect and obtain a copy of his/her record without exception D. requires privacy notices prior to giving emergency care

A. sets a federal minimum on the protection of privacy

Which one of the following is most likely to induce withdrawal symptoms if discontinued abruptly? (check one) A. Venlafaxine (Effexor) B. Divalproex (Depakote) C. Fluoxetine (Prozac) D. Olanzapine (Zyprexa) E. Donepezil (Aricept)

A. venlafaxine (Effexor) - stop or a reduction in dosage, is - associated with withdrawal symptoms much more severe than those seen with other SSRIs such as fluoxetine - agitation, anorexia, confusion, impaired coordination, seizures, sweating, tremor, and vomiting. - To avoid this withdrawal symptom, dosage changes should be instituted gradually. - Abrupt discontinuation of mood stabilizers such as divalproex, and atypical antipsychotics such as olanzapine, can result in the return of psychiatric symptoms, but not severe physiologic dysfunction. - anticholinesterase inhibitors such as donepezil will not cause a withdrawal syndrome

Total parenteral nutrition is most appropriate for patients: (check one) A. With poorly functioning gastrointestinal tracts who cannot tolerate enteral feeding B. Who cannot swallow because of an esophageal motility problem C. Who refuse to eat D. In whom maintenance nutrition is desired for a short period following recovery from surgery

A. with poorly functioning gastrointestinal tracts who cannot tolerate other means of nutritional support Tube feeding - Patients who cannot swallow because of an esophageal motility problem and those who are resistant to feeding can be managed with tube feedings Peripheral alimentation - short term in patients recovering from surgery

A 25-year-old male presents with a 3-day history of cough, chills, and fever. The patient was previously healthy and has no chronic medical problems. He has no known drug allergies. On examination he is alert and oriented, and has a temperature of 38.4°C (101.1°F), a pulse rate of 88 beats/min, a blood pressure of 120/70 mm Hg, a respiratory rate of 16/min, and an oxygen saturation of 98%. Auscultation of the lungs reveals no wheezing and the presence of right basilar crackles. A chest radiograph shows a right lower lobe infiltrate. There is a low rate of macrolide-resistant pneumococcus in the community. Which one of the following is the most appropriate initial management of this patient? (check one) A. Outpatient treatment with azithromycin (Zithromax) B. Outpatient treatment with levofloxacin C. Inpatient treatment on the medical floor with ceftriaxone (Rocephin) and azithromycin D. Inpatient treatment on the medical floor with piperacillin/tazobactam (Zosyn) and levofloxacin E. Inpatient treatment in the intensive-care unit with ceftriaxone, levofloxacin, and vancomycin (Vancocin)

A: In patients with community-acquired pneumonia it is necessary to decide on both the antibiotic regimen and the treatment setting. The decision regarding site of care is based on the severity of illness, which can be assessed with tools such as the CURB-65 score, which take into account factors such as respiratory rate, blood pressure, uremia, confusion, and age. Patients who have only mild symptoms can be treated with azithromycin on an outpatient basis if there is a low level of macrolide resistance in the community. If there is a high level of resistance in the community, if the patient has comorbidities such as diabetes mellitus or COPD, or if there is a history of use of an immunosuppressing drug or recent use of an antibiotic, the patient can still be treated as an outpatient but should be treated with levofloxacin. Patients with more severe symptoms, such as an elevated pulse rate or respiratory rate, should be treated on an inpatient basis with ceftriaxone or azithromycin. Patients who have more severe symptoms along with bronchiectasis should be treated with piperacillin/tazobactam plus levofloxacin. Patients with the most severe symptoms, including hypotension, a more elevated pulse rate, low oxygen saturation, and confusion, should be treated in the intensive-care unit with levofloxacin and vancomycin.

A patient dying of cancer is suffering from pain in spite of his narcotic regimen. You increase his dosage of morphine, knowing it will probably hasten his death. Which ethical principle are you following? (check one) A. Distributive justice B. Double effect C. Death with dignity D. Futility E. Autonomy

B. "double effect" - dates back to the Middle Ages - justify medical treatment designed to relieve suffering when death is an unintended but foreseeable consequence Based on two basic presuppositions - first, that the doctor's motivation is to alleviate suffering - second, that the treatment is appropriate to the illness.

A healthy 68-year-old male is seen in December for a routine examination. A review of his immunizations indicates that he received a standard dose of inactivated influenza vaccine at the health clinic in September. He received 23-valent pneumococcal vaccine (Pneumovax 23) at age 65. He should now receive which one of the following? (check one) A. High-dose influenza vaccine B. 13-valent pneumococcal conjugate vaccine (Prevnar 13) C. 23-valent pneumococcal vaccine D. No vaccines at this time

B. 13-valent pneumococcal conjugate vaccine (Prevnar 13) The Advisory Committee on Immunization Practices advises that the 13-valent pneumococcal vaccine be given in addition to the 23-valent vaccine, preferably before the 23-valent vaccine.

A hemoglobin A1c of 7.0% would correspond to which one of the following mean (average) plasma glucose levels? (check one) A. 126 mg/dL B. 154 mg/dL C. 183 mg/dL D. 212 mg/dL E. 240 mg/dL

B. 154 mg/dL A rough guide for estimating average plasma glucose levels assumes that 6.0% equals an average glucose level of 120 mg/dL. Each percentage point increase in 1c is equivalent to a 30-mg/dL rise in average glucose. An HbA1cof 7.0% is therefore roughly equivalent to an average glucose level of 150 mg/dL, and an HbA1c of 8.0% translates to an average glucose level of 180 mg/dL.

The FDA recommends that over-the-counter cough and cold products not be used in children below the age of: (check one) A. 1 year B. 2 years C. 3 years D. 4 years E. 5 years

B. 2 years

An 80-year-old female is being started on warfarin (Coumadin) for atrial fibrillation. According to the American College of Chest Physicians guidelines, the initial dose in this patient should NOT exceed: (check one) A. 2.5 mg B. 5 mg C. 7.5 mg D. 10 mg E. 12.5 mg

B. 5 mg starting warfarin dosage of ≤5 mg/day in elderly patients, or in patients who have conditions such as heart failure, liver disease, or a history of recent surgery. The INR should be used to guide adjustments in the dosage.

A 55-year-old male consults you because he wants to begin an exercise program. He is asymptomatic, but because of his family history you determine that he should undergo a stress test with echocardiography. Which one of the following would be considered a normal ejection fraction in this patient? (check one) A. 48% B. 65% C. 76% D. 84% E. 92%

B. 65% The ejection fraction value is an important measure of left ventricular function, especially with regard to previous cardiac events, medications, exercise tolerance, and preoperative risk. The normal predicted value is 55%-75% when measured by echocardiography in a healthy asymptomatic patient.

A 73-year-old male is seen for follow-up of elevated blood pressure. He has no comorbidities. His blood pressure after several months of lifestyle modifications is 160/102 mm Hg. He is started on lisinopril (Prinivil, Zestril), 10 mg daily. According to the JNC 8 panel, the blood pressure goal for this patient is which one of the following? (check one) A. <160/100 mm Hg B. <150/90 mm Hg C. <140/90 mm Hg D. <130/85 mm Hg E. 120/80 mm Hg

B. <150/90 mm Hg

Screening for colon cancer would be recommended for which one of the following patients? (check one) A. A 35-year-old male whose mother was diagnosed with colon cancer at age 52 B. A 40-year-old female whose mother was diagnosed with colon cancer at age 54 C. A 44-year-old female whose father had a tubular adenoma <1 cm in size removed during colonoscopy at age 50 D. A 46-year-old male whose paternal uncle was diagnosed with colon cancer at age 51 E. A 48-year-old female whose father was diagnosed with colon cancer at age 74

B. A 40-year-old female whose mother was diagnosed with colon cancer at age 54 - A history of a first degree relative diagnosed with colon cancer before age 60 predicts a higher lifetime incidence of colorectal cancer (CRC) and a higher yield on colonoscopic screening. The overall colon cancer risk for these persons is three to four times that of the general population. - - Screening should consist of colonoscopy, beginning either at age 40 or 10 years before the age at diagnosis of the youngest affected relative, whichever comes first.

A 56-year-old male complains of daily early awakening and low energy for the past 3 weeks. Six weeks ago he had a myocardial infarction treated with a coronary artery stent. During that hospitalization his CBC, fasting glucose level, and thyroid function were normal. A recent phone note from the cardiac rehabilitation nurse indicates that he became apathetic and stopped attending his rehabilitation sessions. He admits to a feeling of hopelessness. He denies chest pain, dyspnea, orthopnea, and palpitations. His vital signs and physical examination are remarkable for a healing radial artery catheterization wound. In addition to resumption of cardiac rehabilitation, which one of the following would be most appropriate at this point? A. Reassurance and a follow-up appointment in 6 weeks B. A Patient Health Questionnaire 9 (PHQ-9) C. Polysomnography D. A BNP level E. An exercise thallium stress test

B. A Patient Health Questionnaire 9 (PHQ-9)

A 42-year-old female presents with a cough productive of blood-streaked sputum for the past 3 days. Her hemoptysis was preceded by several days of rhinorrhea, congestion, and subjective fever. She estimates the total amount of blood loss to be approximately 1 tablespoon. She is a nonsmoker and her past medical history is unremarkable. Vital signs are within normal limits, and other than an intermittent cough there are no abnormal findings on the physical examination. Which one of the following would be the most appropriate next step? (check one) A. Observation B. A chest radiograph C. Chest CT D. Bronchoscopy E. Antibiotics

B. A chest radiograph The first step in the evaluation of nonmassive hemoptysis is to obtain a chest radiograph. If this is normal and there is a high risk of malignancy (patient age 40 years or older with at least a 30-pack year smoking history), chest CT should be ordered. Bronchoscopy should also be considered in the workup of high-risk patients. If a chest radiograph shows an infiltrate, treatment with antibiotics is warranted. If the chest radiograph is normal the patient is at low risk for malignancy, and if the history does not suggest lower respiratory infection and hemoptysis does not recur, observation can be considered.

A 20-year-old male with a history of exercise-induced bronchoconstriction presents to your office with a complaint of cough and decreasing performance when he runs. He is training for a marathon and is currently running 30 miles/week, but has noted that his times have been worsening and that he is using his albuterol inhaler (Proventil, Ventolin) as needed for symptom relief 5 days a week. Which one of the following is the best regimen for treatment of this condition? (check one) A. Inhaled albuterol before he runs B. A daily low-dose inhaled corticosteroid C. A daily inhaled long-acting $2-agonist D. A daily low-dose oral corticosteroid E. Immunotherapy

B. A daily low-dose inhaled corticosteroid - daily use of short-acting beta 2-agonists can lead to overuse and tolerance. - Long-acting beta 2-agonists should not be used without the concomitant use of an inhaled corticosteroid. - Chronic oral corticosteroids are not indicated in this situation, and may require a therapeutic use exemption by the sports authority overseeing athletic competitions. - Immunotherapy is useless with asthma

A 42-year-old female is found to have a thyroid nodule during her annual physical examination. Her TSH level is normal. Ultrasonography of her thyroid gland shows a solitary nodule measuring 1.2 cm. Which one of the following would be most appropriate at this point? (check one) A. A radionuclide thyroid scan B. A fine-needle aspiration biopsy of the nodule C. Partial thyroidectomy D. Total thyroidectomy E. Reassurance

B. A fine-needle aspiration biopsy of the nodule 1. have their TSH measured. 2. Patients with a suppressed TSH should be evaluated with a radionuclide thyroid scan' - "hot" (show increased isotope uptake) are almost never malignant and fine-needle aspiration biopsy is not needed. - all other nodules, fine-needle aspiration biopsy to determine whether the lesion is malignant (SOR B).

A 39-year-old female presents with lower abdominal/pelvic pain. On examination, with the patient in a supine position, you palpate the tender area of her lower abdomen. When you have her raise both legs off the table while you palpate the abdomen, her pain intensifies. Which one of the following is the most likely diagnosis? (check one) A. Appendicitis B. A hematoma within the abdominal wall musculature C. Diverticulitis D. Pelvic inflammatory disease E. An ovarian cyst

B. A hematoma within the abd wall muscles - reduction of the pain caused by abdominal palpation when the abdominal muscles are tightened is known as Carnett's sign - If the cause of the pain is visceral, the taut abdominal muscles may protect the locus of pain. - In contrast, intensification of pain with this maneuver points to a source of pain within the abdominal wall itself.

A 16-year-old female presents for follow-up after a tibial stress fracture. The fracture was diagnosed 5 weeks ago by characteristic physical examination findings and radiographs showing a transverse fracture of the tibial diaphysis. She was placed on non-weight-bearing status for 2 weeks and after that was advised to limit activities that caused discomfort. In addition, she was placed on appropriate calcium and vitamin D supplementation based on results of her laboratory workup. The patient is a basketball player and would like to begin practicing with the team in 1 week. She says she is now able to walk without discomfort but has not tried running or jumping. Which one of the following is necessary for this patient to be able to return to basketball participation next week? (check one) A. Consultation with a sports medicine physician B. A normal physical examination of the affected area C. Normal radiographs of the tibia D. A normal hydroxyvitamin D level

B. A normal physical examination of the affected area Stress fractures are common in teenage athletes. Because this patient has a normal physical examination and can walk without pain, she can return to basketball as long as her symptoms do not return. Most stress fractures heal in 6-10 weeks with conservative management such as non-weight bearing and activity limitation. Athletes can return to play once they are pain free and have a normal physical examination, even if the time since diagnosis is less than 6 weeks. However, they should refrain from all high-impact activities such as running and jumping until they can walk without pain.

A 45-year-old male with diabetes mellitus returns to your office for follow-up. He is on metformin (Glucophage), 1000 mg/day, as well as atorvastatin (Lipitor), 40 mg daily for hyperlipidemia. There is no diagnosis of hypertension, and his blood pressure at today's visit is 120/70 mm Hg. Laboratory results include a hemoglobin A1c of 6.4% and an LDL-cholesterol level of 105 mg/dL. His urine albumin/creatinine ratio is in the microalbuminuric range for the first time. Which one of the following would be most appropriate at this point? (check one) A. Renal ultrasonography B. A repeat urine albumin/creatinine ratio C. 24-hour urine for microalbumin D. Increasing the atorvastatin dosage E. Stopping metformin

B. A repeat urine albumin/creatinine ratio This normotensive diabetic patient, appropriately screened for microalbuminuria, should have this finding confirmed on at least one of two additional spot tests, since temporary factors other than nephropathy can also result in microalbuminuria. - Not U/S; only after CKD is confirmed - Not 24-hour urine: the urine microalbumin/creatinine ratio correlates well with a 24-hour urine for albumin. - Metformin is not contraindicated in the presence of microalbuminuria alone without a decline in the glomerular filtration rate. - Not increase atorvostatin dosage: already on high-intensity statin therapy; treatment targeting LDL no longer favored

Which one of the following injection sites for insulin administration is best for preventing hypoglycemia in a 14-year-old male with diabetes mellitus who wishes to participate in track and field running events? (check one) A. Arm B. Abdomen C. Hip D. Calf E. Thigh

B. Abdomen The use of a nonexercised injection site for insulin administration, such as the abdomen, may reduce the risk of exercise-induced hypoglycemia.

A 75-year-old female is admitted to the hospital with a change in mental status. The initial workup includes a chemistry profile that reveals a plasma potassium level of 6.4 mEq/L (N 3.7-5.2). Which one of the following should be given now to rapidly lower the plasma potassium level? (check one) A. Corticosteroids B. Albuterol C. Furosemide (Lasix) D. 0.45% saline

B. Albuterol - Severe hyperkalemia (>7.0 mEq/L) requires aggressive treatment. - Calcium chloride or gluconate has no effect on the plasma potassium level, but it should be given first, as it rapidly stabilizes the membranes of cardiac myocytes, reducing the risk of cardiac dysrhythmias. - Therapies that translocate potassium from the serum to the intracellular space should be instituted next, as they can quickly (albeit temporarily) lower the plasma concentration of potassium. - These interventions include sodium bicarbonate, glucose with 1. insulin 2. albuterol. Total body potassium can be lowered with sodium polystyrene sulfonate, but this takes longer to affect the plasma potassium level than translocation methods. In the most severe cases, acute hemodialysis can be instituted.

A 3-year-old male is brought to the urgent-care clinic on a Monday morning by his mother with a 1-day history of complaining of ear pain. The child's mother says she has not noticed any fever during this time. He is up to date on all immunizations and has no previous history of ear infections or history of recent illness. The history is negative for medication allergies. On examination the child has a temperature of 38.2°C (100.8°F) and seems to be uncomfortable. When you examine his ears you note moderate bulging of the tympanic membrane in both ears. All other findings are normal. According to the guidelines published by the American Academy of Pediatrics, which one of the following would be the most appropriate initial management? (check one) A. Amoxicillin, 40-50 mg/kg, for 10 days B. Amoxicillin, 80-90 mg/kg, for 10 days C. Amoxicillin/clavulanate (Augmentin), 90 mg/kg/day of amoxicillin and 6.4 mg/kg/day of clavulanate, divided into two doses, for 7 days D. Cefdinir, 14 mg/kg/day for 10 days E. Ciprofloxacin (Cipro), 10-20 mg/kg for 7 days

B. Amoxicillin, 80-90 mg/kg, for 10 days - acute otitis media The usual treatment for AOM is amoxicillin, but an antibiotic with additional B-lactamase coverage, such as amoxicillin/clavulanate, should be given if the child has received amoxicillin within the past 30 days, has concurrent purulent conjunctivitis, or has a history of AOM unresponsive to amoxicillin (SOR C). Penicillin-allergic patients should be treated with an alternative antibiotic such as cefdinir, cefuroxime, cefpodoxime, or ceftriaxone.

When considering a diagnosis of pancreatitis, amylase levels (check one) A. can help determine the severity of the disease B. are less likely to be elevated in alcoholics C. are more sensitive and specific than serum lipase levels D. are less likely to be affected by nonpancreatic conditions such as renal insufficiency

B. Amylase and lipase levels are used to help make the diagnosis of acute pancreatitis. - amylase is less sensitive & specific

A 64-year-old white male presents to the emergency department with a 48-hour history of left lower quadrant pain. After a thorough history and a physical examination you conclude that the patient has diverticulitis. The patient is allergic to metronidazole (Flagyl). You recommend a clear-liquid diet, a follow-up visit with his primary care physician in 48 hours, and treatment with: (check one) A. amoxicillin B. amoxicillin/clavulanate (Augmentin) C. ciprofloxacin (Cipro) D. doxycycline E. azithromycin (Zithromax)

B. An accepted regimen for outpatient treatment of diverticulitis is amoxicillin/clavulanate, 875 mg every 12 hours. - The other regimens are not optimal treatments because they do not include anaerobic coverage.

Which one of the following accurately describes the classic rash of erythema migrans? (check one) A. Scattered individual purple macules on the ankles and wrists B. An annular rash with a bright red outer border and partial central clearing C. A dry, scaling, dark red rash in the groin, with an active border and central clearing D. A diffuse eruption with clear vesicles surrounded by reddish macules E. A migratory pruritic, erythematous, papular eruption

B. An annular rash with a bright red outer border and partial central clearing is - Lyme disease Scattered individual purple macules on the ankles and wrists: Rocky Mountain Spotted Fever A dry, scaling, dark red rash in the groin, with an active border and central clearing: tinea cruris A diffuse eruption with clear vesicles surrounded by reddish macules - chicken pox A migratory pruritic, erythematous, papular eruption: urticaria

A 60-year-old female has a strong family history of breast cancer and is considering tamoxifen (Soltamox) to reduce her risk. Which one of the following is an effect associated with this treatment that should be included in the shared decision-making discussion with the patient? (check one) A. An increased risk of bone fractures B. An increased risk of endometrial cancer C. A reduction in leg cramps D. A decreased risk of thromboembolic events E. A reduction in vasomotor symptoms

B. An increased risk of endometrial cancer . Tamoxifen has been shown to be more beneficial than raloxifene; also reduce fracture Potential harms - increase of 4-7 events of VTE per 1000 women over 5 years. - increases the incidence of endometrial cancer, leg cramps, bladder control issues, vasomotor symptoms, and vaginal dryness, itching, and discharge.

You evaluate an 18-month-old male with fecal impaction and determine that disimpaction is indicated. Which one of the following would be most appropriate initially? (check one) A. An oral stimulant such as sennosides (Senokot) B. An oral osmotic agent such as polyethylene glycol 3350 (MiraLax) C. An enema using saline, mineral oil, or phosphate soda D. A bisacodyl (Dulcolax) rectal suppository E. Manual disimpaction

B. An oral osmotic agent such as polyethylene glycol 3350 (MiraLax) Oral osmotics such as polyethylene glycol-based solutions are recommended as an appropriate initial approach to constipation in children Rectal therapies are similar in terms of effectiveness but are more invasive and less commonly used as first-line treatment (SOR A). Not Oral stimulants and bisacodyl rectal suppositories: not for children under 2 years of age. Not Enemas: second-line therapy only, does not improve outcome Not manual disimpaction: not 1st line

A 20-year-old nonsmoker presents to your office with a sudden onset of chest pain. You order a chest radiograph, which shows a small (<15%) pneumothorax. He is in no respiratory distress and vital signs are normal. Pulse oximetry shows a saturation of 98% on room air. which one of the following would be most appropriate initially? (check one) A. CT of the affected lung B. Analgesics and a follow-up visit in 48 hours C. Chest tube insertion D. Hospital admission and a repeat chest film in 24 hours

B. Analgesics and a follow-up in 48 hours - patient without apparent lung disease who develops a spontaneous "small" pneumothorax (<15% of lung volume) can be managed as an outpatient with analgesics and follow-up within 72 hours. - CT of the lung is needed in complicated cases, including patients with known lung disease or recurrent pneumothoraces. - A chest tube is required only when the pneumothorax involves >15% of lung volume.

Which one of the following is most appropriate for patients with asplenia? (check one) A. Lifelong daily antibiotic prophylaxis B. Antibiotics for any episode of fever C. An additional dose of Hib vaccine D. Avoiding live attenuated influenza vaccine E. Withholding pneumococcal vaccine

B. Antibiotics for any episode of fever - PCN PO bid esp. child under 5 & 1-2 yrs in older adults - Lifelong daily antibiotics may be considered following post-splenectomy sepsis. - risk for Haemophilus influenzae type b infection is not increased in asplenic patients - Live attenuated influenza vaccine may be used in asplenic patients, unless they have sickle cell disease. - Do not avoid pneumococcal polysaccharide vaccine, but give PCV13 & 23 8 wks apart

A long-term care resident is admitted to the hospital. The patient has a living will which specifies that "treatment be withheld or withdrawn and that I be permitted to die naturally with only the administration of medication or the performance of any medical treatment deemed necessary to alleviate pain." The patient has appointed his wife as his health care surrogate. He has mild Alzheimer's disease and scored 26 out of 30 on a Mini-Mental State Examination performed within the last month. He is alert and pleasant and responds appropriately to questions but cannot remember the current date. His wife is with him. Which one of the following would be most appropriate with regard to decision making and ordering related to the patient's code status? (check one) A. Determine the patient's competence B. Assess the patient's decision-making capacity C. Confirm the code status with the patient's wife D. Write a Do Not Resuscitate (DNR) order E. Order comfort measures only

B. Assess the patient's decision-making capacity - patient with dementia can still have decision-making capacity. With Mini-Mental State Examination scores <20 (maximum score = 30) there is an increased likelihood of incapacity, but this varies from case to case and is situation dependent. - Competence is a legal term and is a judicial decision made by a court. Any licensed physician can make a determination of capacity, and a psychiatrist is not required.

A 68-year-old male with end-stage lung cancer is being treated for pain secondary to multiple visceral and skeletal metastases. He has been on oral ibuprofen and parenteral morphine. However, over the past few weeks he reports progressive worsening of his pain. In order to achieve better pain control his morphine dosage has been continuously titrated up. In spite of this increase he continues to report severe pain that is now diffuse and occurs even when his caregivers touch him. Which one of the following would be most appropriate at this time? (check one) A. Increase the morphine dosage until continuous sedation is obtained B. Attempt a reduction in the morphine dosage C. Add an anxiolytic to help relieve anxiety D. Advise the family that nothing more can be done for his pain

B. Attempt a reduction in the morphine dosage Opioid-induced hyperalgesia is characterized by a paradoxical increase in sensitivity to pain despite an increase in the opioid dosage. It is seen in patients who are receiving high doses of parenteral opioids such as morphine. Patients report the development of diffuse pain away from the site of the original pain. Allodynia, a perception of pain in the absence of a painful stimulus, is also typical in opioid-induced hyperalgesia.

An 82-year-old female is hospitalized for pneumonia and sepsis. She has advance directives in place. Should it become necessary, the patient's decision-making capacity is determined by: (check one) A. The spouse or next of kin B. The attending physician C. A consulting psychiatrist D. A judge, at the request of hospital social services or the

B. Attending physician - responsible for determining capacity and incapacity for decision making - The extent, cause, and probable duration of any incapacity should be documented in the clinical record.

A 22-year-old female with a 2-week history of paroxysmal cough is found to have pertussis confirmed by a polymerase chain reaction test and a nasal swab culture. Which one of the following is the antibiotic of choice for this patient? (check one) A. Amoxicillin B. Azithromycin (Zithromax) C. Ciprofloxacin (Cipro) D. Clindamycin (Cleocin) E. Doxycycline

B. Azithromycin (Zithromax) - treatment and prophylaxis of pertussis; - 6 wks from onset for peds under 1 y/o, 3 wks for all others - Trimethoprim/sulfamethoxazole is an alternative in cases of allergy or intolerance to macrolides.

Overweight and obesity in children should be determined by which one of the following? (check one) A. Body weight B. BMI percentile for age and gender C. Individual BMI D. Abdominal girth E. Percentage of body fat

B. BMI percentile for age and gender In children, overweight and obesity is determined by the BMI percentile for age and gender. In adults, BMI, body fat percentage, and abdominal girth are used to determine a patient's classification (SOR B).

An anxious 62-year-old white male comes to the emergency department complaining of extreme shortness of breath and a cough producing blood-tinged sputum. The patient denies chest pain and fever. On examination he is afebrile and has expiratory wheezes and a few rales throughout the chest. The heart is normal except for a rapid rate and an S3 gallop. A chest radiograph reveals a right pleural effusion with enlargement of the cardiac silhouette and redistribution of blood flow to the upper lobes. Which one of the following tests would be best for confirming the diagnosis? (check one) A. Troponin I B. BNP C. D-dimer D. CT angiography of the chest E. Arterial blood gases

B. BNP - heart failure with a bronchospastic component. - S3 gallop occurs with a dilated left ventricle and a right-sided pleural effusion, which are common in heart failure - A BNP level is useful in differentiating cardiac and pulmonary diseases Not troponin - for cardiac ischemia Not CT angiogram - for pulmonary embolism Not D-dimer - DVT

Which one of the following community health programs best fits the definition of secondary prevention? (check one) A. A smoking education program at a local middle school B. A blood pressure screening at a local church C. A condom distribution program D. Screening diabetic patients for microalbuminuria .

B. Blood pressure screening Primary prevention - targets individuals who may be at risk to develop a medical condition and intervenes to prevent the onset of that condition (e.g., childhood vaccination programs, water fluoridation, antismoking programs, and education about safe sex). Secondary prevention - targets individuals who have developed an asymptomatic disease and institutes treatment to prevent complications (e.g., routine Papanicolaou smears, and screening for hypertension, diabetes, or hyperlipidemia). Tertiary prevention - targets individuals with a known disease, with the goal of limiting or preventing future complications (e.g., screening diabetics for microalbuminuria, rigorous treatment of diabetes mellitus, and post-myocardial infarction prophylaxis with β-blockers and aspirin)

Question 7 of 10 Which one of the following is true concerning Paget's disease of bone? (check one) A. It is a precursor of multiple myeloma B. Both bone formation and bone resorption are increased C. The treatment of choice for symptomatic disease is a calcium channel blocker D. Pagetic bone pain is difficult to relieve and resistant to medical treatment E. Extracellular calcium homeostasis is typically abnormal

B. Both bone formation and bone resorption area focal disorder of skeletal metabolism in which all elements of skeletal remodeling (resorption, formation, and mineralization) are increased. increased - Treatment: one of the newer bisphosphonate - despite the massive bone turnover, extracellular calcium homeostasis is almost invariably normal.

A 31-year-old African-American female presents with the chief complaint of bilateral galactorrhea of 3 months' duration. She also has not menstruated for 1 year despite changing birth control pills several times. A review of systems is otherwise noncontributory. Except for a milky discharge with stimulation of the breasts, her examination is within normal limits. Serum prolactin on two occasions is >200 μg/L (N 0-20). Which one of the following would be most appropriate at this point? (check one) A. Stop her oral contraceptive and repeat the serum prolactin level in 1 month B. Order a brain MRI with enhancement and emphasis on the pituitary fossa C. Order bilateral mammography D. Start the patient on risperidone (Risperdal)

B. Brain MRI for pituitary fossa - Galactorrhea - prolactin level >200 μg/L usually indicates a prolactinoma, and requires MRI of the pituitary with gadolinium enhancement. - Many drugs can cause galactorrhea, including oral contraceptives and risperidone, but they would not elevate serum prolactin to this level. - Treatment *Dopamine agonists such as bromocriptine or cabergoline*

A 60-year-old male complains of multiple episodes of lightheadedness over the past 3 months, saying he felt as if he might "pass out" while sitting at his desk. His past medical history and a physical examination are unremarkable. An EKG shows right bundle branch block and left anterior hemiblock. Which one of the following would be the most appropriate next step? (check one) A. Echocardiography B. Cardiac event monitoring C. Hospital admission for pacemaker insertion D. Immediate initiation of aspirin and metoprolol (Lopressor)

B. Cardiac event monitoring - *trifascicular block* - right bundle branch block w. left anterior hemiblock - at risk for arrhythmias (fast & slow) - with syncope, heart monitoring study would be most appropriate Not echo: not next best step Not hospitalization: not indicated

A 9-month-old white male is brought to your office for a well-child visit. You note that the child's weight gain has been flat over the last several months. He has fallen from the 75th percentile to the 15th for weight, and his percentile for length is beginning to decline as well. The mother states that the child began having diarrhea as soon as she began giving him various grain cereals and baby foods 5 months ago. The remainder of a review of systems and a social and family history is unremarkable. Physical examination reveals an undernourished infant with mild abdominal distention. A check of the infant's hemoglobin shows a microcytic anemia with a low serum ferritin level. Which one of the following is the most likely diagnosis? (check one) A. Thalassemia minor B. Celiac sprue C. Cystic fibrosis D. Congenital megacolon (Hirschsprung's disease) E. Inborn error of metabolism

B. Celiac Sprue - acquired malabsorption that resolves when the patient is exposed to a gluten-free diet. - Gluten is a substance found in wheat, rye, and barley, but not in corn or rice products - Children with this sensitivity will develop inflammation and destruction of the microvilli in the small intestine as a result of an immune response to gluten. - Patients with celiac sprue often present as this child has, between 4 and 24 months of age with impaired growth, diarrhea, and abdominal distention - An iron deficiency anemia can occur with impairment of iron absorption from the small intestine. -Serologic tests, and ultimately a biopsy of the small intestine, can confirm the diagnosis.

A 4-week-old white male is brought to your office with a 2-week history of increasing dyspnea, cough, and poor feeding. The child appears nontoxic and is afebrile. On examination you note conjunctivitis, and a chest examination reveals tachypnea and crackles. A chest film shows hyperinflation and diffuse interstitial infiltrates and a WBC count reveals eosinophilia. What is the most likely etiologic agent? (check one) A. Staphylococcus species B. Chlamydia trachomatis C. Respiratory syncytial virus D. Parainfluenza virus

B. Chlamydial pneumonia - usu in infants 3 - 16 wks; pt sick for weeks - infant nontoxic & afebrile, tachypneic w/ prominent cough - physical examination: diffuse crackles with few wheezes, and conjunctivitis (50% of cases). - CXR: hyperinflation and diffuse interstitial or patchy infiltrates. Not Staphylococcal pneumonia - fever, initial expiratory wheeze like bronchiolitis - PE: abdominal distress, tachypnea, dyspnea, and localized or diffuse bronchopneumonia or lobar disease - CBC: leukocytosis. Not RSV - rhinorrhea and pharyngitis, followed in 1-3 days by a cough and wheezing. Auscultation of the lungs will reveal diffuse rhonchi, fine crackles, and wheezes, but the chest film is often normal. - If the illness progresses, coughing and wheezing increase, air hunger and intercostal retractions develop, and evidence of hyperexpansion of the chest is seen. - WBC count will be normal or elevated, and the differential may be normal or shifted either to the right or left. - Chlamydial infections can be differentiated from respiratory syncytial virus infections by a history of conjunctivitis, the subacute onset and absence of fever, and the mild wheezing. There may also be eosinophilia. Not Parainfluenza virus infection - typical cold symptoms. - Eight percent of infections affect the upper respiratory tract. - In children hospitalized for severe respiratory illness, parainfluenza viruses account for about 50% of the cases of laryngotracheitis and about 15% each of the cases of bronchitis, bronchiolitis, and pneumonia.

Which one of the following conditions can affect hemoglobin A1c levels? (check one) A. Heart failure B. Chronic hemolytic anemia C. COPD D. Hypothyroidism

B. Chronic hemolytic anemia; the hemoglobin A1c (HbA1c) blood test provides information regarding average glucose levels over the past 3 months. Any condition that shortens erythrocyte survival or decreases mean erythrocyte age, such as recent acute blood loss or hemolytic anemia, will falsely lower HbA1c levels. Hemoglobin variants and iron deficiency, kidney failure, and liver disease can also affect HbA1c results. Heart failure, COPD, and hypothyroidism do not influence HbA1c values.

A 30-year-old female presents with dysuria and flank pain. She reports a fever of 102°F yesterday morning. She has not taken any antipyretics since that time, and today her temperature is 36.7°C (98.1°F). She has a pulse rate of 93 beats/min, a respiratory rate of 16/min, and a blood pressure of 116/58 mm Hg. The remainder of her physical examination is unremarkable, except for marked costovertebral angle tenderness. A CBC reveals a WBC count of 14,590/mm3 (N 4300-10,800) with 85% neutrophils, 12% lymphocytes, and 3% basophils, but is otherwise normal. A urine $-hCG is negative. A urine dipstick is positive for leukocyte esterase, and urine microscopic analysis is notable for <1 RBC and >50 WBCs/hpf. Urine culture results are pending. You confirm she has no medication allergies. Which one of the following oral antibiotics would be most appropriate for empiric therapy? (check one) A. Amoxicillin B. Ciprofloxacin (Cipro) C. Erythromycin D. Metronidazole (Flagyl) E. Nitrofurantoin (Furadantin)

B. Ciprofloxacin (Cipro) Acute pyelonephritis (renal pelvis and kidney) - most often seen in young women - most commonly caused by Escherichia coli. - Outpatient oral fluoroquinolone e.g. ciprofloxacin is usually the first-line therapy - If the community fluoroquinolone resistance rate exceeds 10%, a one-time dose of a parenteral antimicrobial such as ceftriaxone or a consolidated dose of an aminoglycoside should be given, followed by an oral fluoroquinolone regimen (SOR B). - Alternative oral agents include trimethoprim/sulfamethoxazole (bactrim) and beta-lactam (not 1st line due to high levels of resistance); wait for culture Not Amoxicillin and nitrofurantoin - for uncomplicated cystitis Not Erythromycin and metronidazole - not appropriate

A 30-year-old female with a history of prolonged QT syndrome presents with severe acute bacterial sinusitis. Which one of the following antibiotics should be avoided? (check one) A. Amoxicillin B. Clarithromycin (Biaxin) C. Amoxicillin/clavulanate (Augmentin) D. Moxifloxacin (Avelox) E. Cefuroxime (Ceftin)

B. Clarithromycin (Biaxin) - Clarithromycin interferes with the delayed rectifier potassium current, which results in the accumulation of potassium ions in cardiac myocytes and thereby delays cardiac repolarization. Clarithromycin is - metabolized by the cytochrome P450 3A enzyme. When using clarithromycin it is important to avoid any other medications that may inhibit this enzyme, leading to higher clarithromycin levels. The other antibiotics listed do not have this effect.

A 40-year-old white male presents with a 5-year history of periodic episodes of severe right-sided headaches. During the most recent episode the headaches occurred most days during January and February and lasted about 1 hour. The most likely diagnosis is which one of the following? (check one) A. Migraine headache B. Cluster headache C. Temporal arteritis D. Trigeminal neuralgia

B. Cluster headache - predominantly a male disorder - onset is 27-30 years - Attacks often occur in cycles and are unilateral Migraine headaches - are more common in women, start at an earlier age (second or third decade), and last longer (4-24 hours). Temporal arteritis - above age 50 Trigeminal neuralgia - usually in paroxysms lasting 20-30 seconds.

A 91-year-old white male presents with a 6-month history of a painless ulcer on the dorsum of the proximal interphalangeal joint of the second toe. Examination reveals a hallux valgus and a rigid hammer toe of the second digit. His foot has mild to moderate atrophic skin changes, and the dorsal and posterior tibial pulses are absent. Appropriate treatment includes which one of the following? (check one) A. Surgical correction of the hammer toe B. Custom-made shoes to protect the hammer toe C. Bunionectomy D. A metatarsal pad

B. Custom-made shoes to protect the hammer toe - Surgical correction of a hammer toe and bunionectomy could be disastrous in an elderly patient with a small ulcer and peripheral vascular disease. - The best approach with this patient is to prescribe custom-made shoes and a protective shield with a central aperture of foam rubber placed over the hammer toe. Metatarsal pads are not useful in the treatment of hallux valgus and a rigid hammer toe.

Which one of the following conditions is the leading cause of death for patients with rheumatoid arthritis? A. Most infections in the United States occur between August and December B. Coronary artery disease C. Thromboembolic disease D. Lymphoma E. Lung cancer

B. Coronary artery disease RA patients have accelerated atherosclerosis related to a chronic inflammatory state. It is thus particularly important to address modifiable risk factors for coronary disease in these patients, including tobacco use, hypertension, and dyslipidemia.

A 43-year-old female complains of easy bruising. She is otherwise asymptomatic. A CBC reveals a platelet count of 23,000/mm3 (N 150,000-450,000). A peripheral smear reveals giant platelets. A workup is negative for autoimmune causes, including Graves disease, HIV, Epstein-Barr virus, cytomegalovirus, varicella zoster, hepatitis C, and Helicobacter pylori. She is on no prescription or over-the-counter medications and denies alcohol or drug use. Which one of the following would be the most appropriate initial management? (check one) A. Platelet transfusion B. Corticosteroids C. Thrombopoietin-receptor agonists D. A bone marrow biopsy E. Splenectomy

B. Corticosteroids - Immune (idiopathic) thrombocytopenic purpura - acquired immune-mediated disorder with isolated thrombocytopenia not found to have another cause - Treat severe thrombocytopenia (platelet count <50,000/mm3) unless there is evidence of acute bleeding. - Corticosteroids first-line therapy or Intravenous immunoglobulin and rituximab - thrombopoietin-receptor agonists 2nd line or splenectomy. - bone marrow biopsy, to rule out myelodysplastic syndrome and lymphoproliferative disorders is indicated in patients over the age of 60 - no platelet transfusion

A 14-year-old male with Tanner stage 1 pubic hair has prepubertal-size testes. His height is at the 3rd percentile. The physical examination is otherwise unremarkable. Which one of the following additional findings would be most consistent with constitutional delay of growth and puberty? (check one) A. Impairment of the sense of smell B. Delayed bone age C. Elevated LH and FSH D. Elevated thyrotropin E. Elevated prolactin

B. Delayed bone age - Constitutional delay of growth and puberty (CDGP) - tends to be inherited - Bone age is delayed, but growth potential is often normal Not LH and FSH elevation - hypergonadotropic hypogonadism, not CDGP Not Thyrotropin elevation - hypothyroidism, which can cause a secondary delay in growth and puberty but not CDGP Not Anosmia (impaired smell) - Kallmann syndrome, not CDGP -Puberty is also delayed in Kallmann i.e. hypogonadotropic hypogonadism Not Prolactin - pituitary tumors and by dopamine-blocking agents (SOR C).

A 72-year-old white female who is otherwise healthy complains of occasional incontinence. She reports that this occurs mainly at night when she awakens with an intense desire to void, and by the time she is able to get to the bathroom she has "wet herself." The most likely diagnosis is: (check one) A. Sphincter incompetence B. Detrusor instability C. Detrusor hypotonia D. Uninhibited neurogenic bladder

B. Detrusor instability - uninhibited contractions of the detrusor muscle. - causes an intense urge to void, which overcomes the patient's voluntary attempt to hold the sphincter closed - hence, the common term urge incontinence. -Other common causes of incontinence: - weak sphincter (sphincter incompetence), which leads to leakage associated with ordinary activities such as coughing or lifting (stress incontinence) - overflow of urine from an abnormally distended, hypotonic, poorly contractile bladder (detrusor hypotonia). - more common in males with longstanding obstruction due to prostatic hypertrophy. Spinal cord damage (rare) - reflex incontinence - patient unable to sense the need to void.

A 72-year-old white female is admitted to the hospital with her first episode of acute heart failure. She has a history of hypertension treated with a thiazide diuretic. An echocardiogram reveals no evidence of valvular disease and no segmental wall motion abnormalities. Left ventricular hypertrophy is described, and the ejection fraction is 55%. Her pulse rate is 72beats/min. The most likely cause of her heart failure is: (check one) A. systolic dysfunction B. diastolic dysfunction C. hypertrophic cardiomyopathy D. high-output failure

B. Diastolic dysfunction - left ventricular hypertrophy as a response to chronic systolic hypertension - ventricle becomes stiff and unable to relax or fill adequately, thus limiting its forward output - The typical patient is an elderly person who has systolic hypertension, left ventricular hypertrophy, and a normal ejection fraction (50%-55%).

A 34-year-old white female who works as an engineer for a major corporation complains of fatigue, low energy, and a depressed mood. She states that she has felt this way for most of her life. She feels depressed most of the time but denies any recent stresses or significant losses in her life. She reports that she is doing well at work and that she recently received a promotion. She has no interests other than her job and states that she has no happy thoughts and that her self-esteem is very low. She denies suicidal thoughts but states that she does not care if she dies. She has had no sleep disturbance, change in appetite, or difficulty concentrating. She is taking no medications and denies substance abuse. Results of a recent medical evaluation required by her employer were all normal, including a physical examination, EKG, multiple chemical profile, CBC, urinalysis, and TSH level. Which one of the following is the most likely diagnosis? (check one) A. Major depression B. Dysthymic disorder C. Bipolar disorder D. Cyclothymia E. Adjustment disorder with depressed mood

B. Dysthymic disorder - depressed mood for at least 2 years in addition to at least two of the following: change in appetite, alteration in sleep, low energy, low self-esteem, poor concentration, or feelings of hopelessness. - There must be no history of a manic or hypomanic episode, substance abuse, a chronic psychotic disorder, or an organic cause. - Not MDD: no triggers and no objective signs

A 62-year-old female with numbness in the lower extremities and macrocytosis has a normal serum folate level and a serum B12 level of 200 pg/mL (N 150-800). Which one of the following laboratory findings would confirm the diagnosis of B12 deficiency? (check one) A. Elevated angiotensin converting enzyme B. Elevated methylmalonic acid C. Elevated free erythrocyte protoporphyrin D. Low haptoglobin E. Low homocysteine

B. Elevated methylmalonic acid - Vitamin B12 is a cofactor in the synthesis of both methionine and succinyl coenzyme A, and vitamin B12 deficiency leads to the accumulation of *methylmalonic acid and homocysteine*, which are the precursors of these compounds. - Homocysteine is also elevated in folic acid deficiency, check methylmalonic acid if serum vitamin B12 levels are 150-400 pg/mL. Not reduced haptoglobin level - hemolytic anemia. Not elevated free erythrocyte protoporphyrin level - lead poisoning or iron deficiency. Not elevated angiotensin converting enzyme - sarcoidosis.

For which one of the following respiratory infections should antibiotic therapy be initiated immediately upon diagnosis? A. Bronchitis B. Epiglottitis C. Laryngitis D. Rhinosinusitis E. Tracheitis

B. Epiglottitis Many infections of the respiratory tract are viral Epiglottitis is one exception - Haemophilus influenzae type b - rapidly worsening, potentially fatal airway - hoarseness, dysphagia, stridor, drooling, fever, chills, and respiratory distress - IV antibiotic: beta-lactam against methicillin-resistant Staphylococcus aureus.

An otherwise healthy 37-year-old male presents to your office with a 2-week history of redness and slight irritation in his groin. On examination a tender erythematous plaque with mild scaling is seen in his right crural fold. The area fluoresces coral-red under a Wood's light. Which one of the following would be the most appropriate treatment at this time? (check one) A. Amoxicillin B. Erythromycin C. Ketoconazole D. Nystatin (Mycostatin) E. Triamcinolone (Kenalog)

B. Erythromycin - Corynebacterium infection causing erythrasma - Erythromycin, either systemic or topical, is the treatment of choice. - coral-red fluorescence under a Wood's lights Not Tinea cruris (Microsporum) - fluoresces green Not intertrigo (Epidermophyton) or tinea cruris (Trichophyton) infections - do not fluoresce

An incidental 2-cm adrenal nodule is discovered on renal CT performed to evaluate hematuria in a 57-year-old female with flank pain. She has no past medical history of palpitations, headache, hirsutism, sweating, osteoporosis, diabetes mellitus, or hypertension. A physical examination is normal, with the exception of a blood pressure of 144/86 mm Hg. Laboratory evaluation reveals a serum sodium level of 140 mmol/L (N 135-145) and a serum potassium level of 3.8 mmol/L (N 3.5-5.0). What is the most appropriate next step in the evaluation of this patient? (check one) A. Repeat CT in 12 months B. Evaluation for adrenal hormonal secretion C. Fine-needle aspiration of the nodule D. MRI of the abdomen E. Referral to a general surgeon for exploratory laparotomy

B. Evaluation for adrenal hormonal secretion - adrenal masses are found on abdominal CT in 4% of cases, and the incidence of adrenal masses increases to 7% in adults over 70 years of age. - as many as 11% are hypersecreting tumors and 7% are malignant - adrenal function must be assessed with an *overnight dexamethasone* suppression test. - A morning cortisol level >5 μg/dL after a 1-mg dose indicates adrenal hyperfunction. - Additional testing should include *24-hour fractionated metanephrines* and catecholamines to rule out pheochromocytoma. - If the patient has hypertension, *morning plasma aldosterone activity and plasma renin activity* should be assessed to rule out a primary aldosterone-secreting adenoma.

Which one of the following is more typical of a keloid rather than a hypertrophic scar? (check one) A. Location on an extensor surface B. Expansion beyond the margins of the inciting injury C. Development soon after the inciting trauma D. Regression over time

B. Expansion beyond the margins of the inciting injury

An 80-year-old male presents with the chief complaint of a "bone spur," describing mid-heel pain that worsens as the day progresses. The pain is not relieved with ibuprofen. Examination reveals tenderness in the central aspect of the heel and a radiograph of the foot is unremarkable. The most likely diagnosis is: (check one) A. Multiple myeloma B. Fat-pad atrophy C. Tarsal tunnel syndrome D. S1 radiculopathy E. Plantar fasciitis

B. Fat pad atrophy - a common cause of heel pain in the geriatric patient - and in contrast to plantar fasciitis, causes pain as the day progresses. Not Plantar fasciitis - morning pain Not Tarsal tunnel syndrome - in the distribution of the posterior tibial nerve, radiating into the plantar aspect of the foot toward the toes Not Lumbar radiculopathy - pain radiating down the leg into the heel - associated with weakness of dorsiflexion of the big toe and a decreased ankle reflex Not Multiple myeloma - no heel pain; if yes, nocturnally.

You are evaluating a 68-year-old male with obstructive urinary symptoms. Which one of the following medications may lead to falsely depressed levels of prostate-specific antigen (PSA)? (check one) A. Terazosin (Hytrin) B. Finasteride (Proscar) C. Tamsulosin (Flomax) D. Doxazosin (Cardura) E. Lycopene

B. Finasteride (Proscar)

A 24-year-old female with a 2-year history of dyspnea on exertion has been diagnosed with exercise-induced asthma by another physician. Which one of the following findings on pulmonary function testing would raise concerns that she actually has vocal cord dysfunction? (check one) A. A good response to an inhaled β-agonist B. Flattening of the inspiratory portion of the flow-volume loop, but a normal expiratory phase C. Flattening of the expiratory portion of the flow-volume loop, but a normal inspiratory phase D. Flattening of both the inspiratory and expiratory portion of the flow-volume loop E. A decreased FEV1 and a normal FVC

B. Flattening of the inspiratory portion of the flow-volume loop, but a normal expiratory phase Pulmonary function testing with a flow-volume loop typically shows a normal expiratory portion but a flattened inspiratory phase (SOR C).

A 5-month-old infant has had several episodes of wheezing, not clearly related to colds. The pregnancy and delivery were normal; the infant received phototherapy for 1 day for hyperbilirubinemia. He had an episode of otitis media 1 month ago. There is no chronic runny nose or strong family history of asthma. He spits up small amounts of formula several times a day, but otherwise appears well. His growth curve is normal. An examination is unremarkable except for mild wheezing. Which one of the following is the most likely diagnosis? (check one) A. Benign reactive airway disease of infancy B. Gastroesophageal reflux C. Unresolved respiratory syncytial virus infection D. Early asthma E. Cystic fibrosis

B. Gastroesophageal reflux - common cause of wheezing in infants - At 5 months of age, most infants no longer spit up several times a day, and this is a major clue that this child's wheezing may be from the reflux - In addition, there is no family history of asthma and the wheezing is not related to infections Not cystic fibrosis - will have recurrent infections and failure to thrive

Which one of the following dietary supplements has the best evidence of efficacy in the treatment of osteoarthritis of the knee? (check one) A. Methylsulfonylmethane (MSM) B. Glucosamine sulfate C. Harpagophytum procumbens (devil's claw) D. Curcuma longa (turmeric) E. Zingiber officinale (ginger)

B. Glucosamine sulfate

A 6-year-old male is brought to your office for a well child check. His vital signs are normal; he is 117 cm (46 in) tall and weighs 19 kg (42 lb). The patient has grown out of his car seat, and his mother recently was told by a friend that he can now sit in the front seat of a car. She asks you if this is true. According to recommendations from the American Academy of Pediatrics, which one of the following is true for this patient? (check one) A. He should be using a rear-facing child safety seat in the back seat of the car B. He should be using a belt-positioning booster seat in the back seat of the car C. He should be using a belt-positioning booster seat and can ride in the front or back seat of the car D. He no longer needs a safety seat but should always ride in the back seat of the car E. He no longer needs a safety seat and can ride in the front or back seat of the car

B. He should be using a belt-positioning booster seat in the back seat of the car - after he reaches weight limit of a safety seat

A 45-year-old male with diabetes mellitus sees you for the first time. If the patient has not previously received it, which one of the following vaccines is recommended for him by the Advisory Committee on Immunization Practices? (check one) A. Hepatitis A B. Hepatitis B C. Meningococcal D. Varicella zoster

B. Hepatitis B Late in 2012, the Advisory Committee on Immunization Practices of the Centers for Disease Control and Prevention recommended hepatitis B vaccine for all previously unvaccinated adults between the ages of 19 and 59 with diabetes mellitus, as soon as possible after the diagnosis of diabetes is made

An otherwise healthy 10-year-old female presents with a papulovesicular eruption on one leg.It extends from the lateral buttock, down the posterolateral thigh, to the lateral calf. It is mildly painful. The patient's immunizations are up to date, including varicella and MMR. Her family has a pet cat at home, and another child at her school was sent home with a rash earlier in the week Which one of the following is the most likely diagnosis? (check one) A. Contact dermatitis B. Herpes zoster dermatitis C. Tinea corporis D. Scabies

B. Herpes Zoster dermatitis - Herpes zoster can occur from either a wild strain or a vaccine strain of varicella-zoster virus in vaccinated children, but the incidence is low. All cases are mild and uncomplicated.

A 75-year-old female presents with a 2-month history of bilateral headache, diffuse myalgias, and diplopia. On examination she has substantially diminished vision in her left eye, but no other neurologic findings. A moderately tender, cordlike structure is palpable just anterior to her ear and extending up to her lateral scalp. Blood tests show a markedly elevated erythrocyte sedimentation rate. Which one of the following would be most appropriate at this point? (check one) A. Clopidogrel (Plavix) B. High-dose corticosteroids C. NSAIDs E. Dipyridamole/aspirin (Aggrenox)

B. High dose corticosteroids: temporal arteritis - age over 50, new-onset headache, abnormalities of the temporal artery, and an elevated erythrocyte sedimentation rate - A temporal artery biopsy is needed to confirm the diagnosis, but when the findings are this compelling, corticosteroids should be started even before a biopsy, to prevent further vision loss. - Temporal arteritis is the most common clinical pattern of giant cell arteritis, which can also involve other branches of the carotid artery.

The parents of a 40-day-old infant bring her to your clinic because she has had a persistent fever for the past 2 days with rectal temperatures between 38.1°C (100.5°F) and 38.9°C (102.0°F). She has been fussy and wants to be held, but has been nursing well. She is crying when you enter the room, and on examination she has good skin turgor and capillary refill. The examination does not reveal any obvious source of infection. By the time you complete the examination the infant is resting quietly in her father's arms. You obtain a CBC and urinalysis. The WBC count is 12,500/mm3 (N 5000-19,500) with an absolute neutrophil count of 9500/mm3 (N 1000-9000). The urinalysis is within normal limits. Which one of the following would be most appropriate at this time? (check one) A. Home care and parental observation only, as long as the temperature remains under 39.0°C (102.2°F) B. Home care and reevaluation in 24 hours C. Oral antibiotics and reevaluation in 24 hours D. A complete sepsis workup, including blood cultures, stool studies, a chest radiograph, and cerebrospinal fluid studies

B. Home care and reevaluation in 24 hours - Most children will be evaluated for a febrile illness before 36 months of age, with the majority having a self-limited viral illness. - Nontoxic-appearing febrile infants 29-90 days of age who have a negative screening laboratory workup, including a CBC with differential and a normal urinalysis, can be sent home and followed up in 24 hours (SOR B). - Observation with no follow-up is an appropriate strategy in nontoxic children, but *only if the child is 3-36 months* of age and the *temperature is under 39°C* (SOR B). - Nontoxic children 3-36 months of age should be reevaluated in 24-48 hours if the temperature is over 39°C. - Any infant *younger than 29 days*, and any infant or child with a *toxic appearance* regardless of age, should undergo a complete sepsis workup and be admitted for observation until culture results are obtained or the source of the fever is found and treated (SOR A)

A 59-year-old male reports nausea, vomiting, and progressive fatigue for the past few months. At his last visit, 6 months ago, his blood pressure was poorly controlled and hydrochlorothiazide was added to his β-blocker therapy. At this visit he appears moderately dehydrated on examination. Laboratory testing reveals a serum calcium level of 12.5 mg/dL (N 8.0-10.0), a BUN level of 36 mg/dL (N 6-20), and a creatinine level of 2.2 mg/dL (N 0.6-1.1). A CBC, albumin level, and electrolyte levels are normal. His intact parathyroid hormone level is reported a few days later, and is 60 pg/mL (N 10-65). What is the most likely cause of his hypercalcemia? (check one) A. Renal failure B. Hyperparathyroidism C. Milk alkali syndrome D. Sarcoidosis

B. Hyperparathyroidism - Many patients have mild hyperparathyroidism that becomes evident only with an added calcium load. - Thiazide diuretics reduce calcium excretion and can cause overt symptoms in a patient whose hyperparathyroidism would otherwise have remained asymptomatic. - The finding of a normal parathyroid hormone (PTH) level in a patient with hypercalcemia is diagnostic for hyperparathyroidism, since PTH should be suppressed in the presence of elevated calcium. - Symptomatic hypercalcemia causes dehydration because of both intestinal symptoms and diuresis. Reversible renal insufficiency can result, and can become permanent if it is long-standing and severe. Not renal failure - usually causes hypocalcemia, but can cause hypercalcemia resulting from tertiary hyperparathyroidism from sever high Phosphate & low vitamin D - This patient's renal insufficiency is not severe enough to cause tertiary hyperparathyroidism. Not Milk alkali syndrome - hypercalcemia resulting from a chronic overdose of calcium carbonate - becoming more common as more patients take calcium and vitamin D supplements Not milk alkali syndrome, and other causes of hypercalcemia such as sarcoidosis - the PTH level is suppressed.

Three members of the same family present with a high fever and cough that began abruptly yesterday. All three report having fevers over 40° C (104° F), painful coughs, moderate sore throats, and prostration. They have loss of appetite, but no vomiting or diarrhea. Two other family members have similar symptoms. On examination the patients appear ill and flushed. There is no cervical adenopathy, no visible pharyngeal inflammation, and no significant findings on examination of the chest. Which one of the following is the most likely diagnosis? (check one) A. Mycoplasma pneumonia B. Influenza-like illness C. Bacterial bronchitis D. Upper respiratory infection E. Legionnaires disease

B. Influenza-like illness - has a very abrupt onset, and a fever with a nonproductive cough is almost always present. Unconfirmed cases are referred to as influenza-like illness (ILI) or suspected influenza. Patients with confirmed cases tend to say they have never been so ill.

An 82-year-old male nursing-home resident is sent to the emergency department with lower abdominal pain and bloody diarrhea. He has a history of multi-infarct dementia, hypertension, and hyperlipidemia. On examination he is afebrile, and a nasogastric aspirate is negative for evidence of bleeding. Which one of the following is the most likely cause of this patient's bleeding? (check one) A. Peptic ulcer disease B. Ischemic colitis C. Diverticular bleeding D. Angiodysplasia E. Infectious colitis

B. Ischemic colitis Diverticular bleeding and angiodysplasia are painless Infectious colitis is associated with fever.

A 45-year-old male presents with a 3-month history of hoarseness. He denies any other complaints and has not been ill recently. He is not on any medication, has no history of chronic medical problems, and does not smoke cigarettes or drink alcohol. Which one of the following would be the most appropriate management of this patient? (check one) A. Voice rest for 1 month B. Laryngoscopy C. A trial of a proton pump inhibitor D. A trial of inhaled corticosteroids E. Oral corticosteroids

B. Laryngoscopy should be performed to visualize the larynx and evaluate for vocal cord pathology in a patient whose hoarseness does not resolve within 3 months (SOR C).

A 45-year-old male is hospitalized for the management of alcohol withdrawal syndrome. His symptoms include tachycardia, diaphoresis, tremors, and visual hallucinations. His CIWA-Ar (Clinical Institute Withdrawal Assessment for Alcohol, Revised) score is 18, indicating moderate alcohol withdrawal. Which one of the following medications has been shown to reduce the risk of developing seizures in this situation? (check one) A. Carbamazepine (Tegretol) B. Lorazepam (Ativan) C. Gabapentin (Neurontin) D. Phenytoin (Dilantin) E. Valproic acid (Depakene)

B. Lorazepam (Ativan)

Which one of the following comorbid conditions increases the risk that latent tuberculosis infection will progress to active disease? (check one) A. Hypertension B. Lung cancer C. Obesity D. Coronary artery disease E. Hyperlipidemia

B. Lung cancer Risk factors for progression from latent to active tuberculosis include: 1. diabetes mellitus, alcoholism, recent contact with a person who has an active tuberculosis infection 2. immunosuppressive therapy 3. lung parenchymal diseases such as COPD, silicosis, or lung cancer 4. under 5 y/o 5. weighing less than 90% of ideal body weight

A 12-month-old male is brought to your office for a routine well child visit. His father has epilepsy and takes seizure medication. Which one of the following vaccines will slightly increase the child's risk of a febrile seizure for up to 2 weeks after administration? (check one) A. Hepatitis B B. MMR C. HiB D. Pneumococcal E. Polio

B. MMR Fever and febrile seizures may occur after administration of several vaccines. Postimmunization seizures, especially febrile seizures, occur at a higher rate in children who have a past history of seizures or a first-degree relative with a history of seizures. Of the vaccines listed, the only one likely to put the child at risk for a seizure up to 2 weeks after administration is the MMR vaccine. Specifically, it is the measles component of the vaccine that is the potential culprit. A temperature of 39.4°C (103°F) or higher develops in approximately 5%-15% of susceptible vaccine recipients, usually 6-12 days after receipt of MMR vaccine. The fever generally lasts 1-2 days but may last up to 5 days.

A 25-year-old female is in active labor at term and is dilated to 7 cm. An electronic fetal monitoring tracing is shown in Figure 5. Which one of the following is a possible etiology for this fetal heart rate pattern? (check one) A. Normal progress of labor B. Maternal fever C. Effects of epidural anesthesia D. Post-dates pregnancy E. Umbilical cord prolapse

B. Maternal fever - fetal tachycardia, - baseline fetal heart rate >160 beats/min for at least 15 minutes. - a nonreassuring pattern. - Causes of fetal tachycardia 1. maternal fever 2. fetal hypoxia 3. hyperthyroidism 4. maternal or fetal anemia 5. medication effects 6. parasympatholytic or sympathomimetic drugs 7. chorioamnionitis 8. fetal tachyarrhythmia 9. prematurity. Fetal bradycardia: Epidural anesthesia, post-dates pregnancy, and umbilical cord prolapse would all be causes of fetal bradycardia.

A 67-year-old female has started receiving home hospice care. Her attending physician can bill through which one of the following? (check one) A. Medicare Part A B. Medicare Part B C. Medicare Part C D. Medicare Part D E. The attending physician cannot bill Medicare

B. Medicare Part B

A 78-year-old female has chronic symptomatic orthostatic hypotension, likely related to diabetic autonomic dysfunction, which has failed to respond to nonpharmacologic treatment. Her current medications include metformin (Glucophage), 1000 mg twice daily; atorvastatin (Lipitor), 40 mg daily; aspirin, 81 mg daily; and insulin glargine (Lantus), 24 units at bedtime. Which one of the following would be the most effective therapy for her orthostatic hypotension? (check one) A. Clonidine (Catapres) B. Midodrine C. Pseudoephedrine D. Terbutaline E. Theophylline

B. Midodrine - Effective treatments for chronic orthostatic hypotension include fludrocortisone, midodrine, and physostigmine (SOR B)

An elderly couple is having trouble paying for the considerable number of medications they require. They ask you about the safety of obtaining drugs from Canada. Which one of the following is true concerning Canadian drugs? (check one) A. Few of the drugs available from Canada have been approved by the Food and Drug Administration (FDA) B. Most of the drugs available from Canada come from the same manufacturers as in the U.S. C. The approval process for a drug by Health Canada is shorter than the FDA's process D. Many drugs discontinued for safety reasons by the FDA are still available in Canada E. Drugs obtained through websites advertising Canadian drugs are well regulated

B. Most of the drugs available from Canada come from the same manufacturers as in the U.S. - The FDA has approved more than 90% of the drugs available from Canada. - Most of these drugs come from the same manufacturers as drugs in the U.S. - Health Canada takes longer, on average, to approve a drug for release than does the FDA - most drugs discontinued for safety reasons by the FDA between 1992 and 2001 had not been approved for use in Canada. - Websites advertising Canadian drugs may be selling counterfeit drugs from unregulated sources..

A 49-year-old white female comes to your office complaining of painful, cold finger tips which turn white when she is hanging out her laundry. While there is no approved treatment for this condition at this time, which one of the following drugs has been shown to be useful? (check one) A. Propranolol (Inderal) B. Nifedipine (Procardia) C. Ergotamine/caffeine (Cafergot) D. Methysergide (Sansert)

B. Nifedipine (Procardia) - no approved treatment for Raynaud's disease. - However, patients with this disorder reportedly experience subjective symptomatic improvement with calcium channel blockers esp. nifedipine - Beta Blockers can produce arterial insufficiency of the Raynaud type, so propranolol and atenolol are contraindicated. - Ergotamine preparations and methysergide can produce cold sensitivity, and should therefore be avoided in patients with Raynaud's disease

A 3-year-old male was treated for acute otitis media last month. His mother brings him in for follow-up because she believes his hearing has not been normal since then. He attends day care and has had several upper respiratory infections. On examination the tympanic membranes are not inflamed, but the membrane is retracted on the right side. An office tympanogram shows a normal peak (type A) on the left side, but a flat tracing (type B) on the right side. Which one of the following would be the most appropriate recommendation? (check one) A. Audiometry B. Observation with follow-up C. An antihistamine/decongestant combination D. Intranasal corticosteroids E. Systemic corticosteroids

B. Observe - unilateral serous otitis, so unlikely to have delayed language from decreased hearing on one side. - Hearing loss of longer than 3 months may indicate a need for tympanostomy tubes Not Surgical treatmen - only for patients with chronic effusion. Not Audiometry - not needed to make a decision about surgery at this point. Not hearing test - would not add any useful information. Not meds - ineffective, including antihistamine and decongestant therapy, and corticosteroids by any route.

A 3-year-old male is brought to the emergency department by his parents, who report seeing him swallow a handful of adult ibuprofen tablets 20 minutes ago. Which one of the following would be the most appropriate initial management of this patient? (check one) A. Oral ipecac B. Oral activated charcoal C. Gastric lavage D. Whole-bowel irrigation E. Close observation

B. Oral activated charcoal - A single dose of activated charcoal is the decontamination treatment of choice for most medication ingestions. - should be used within 1 hour of ingestion of a potentially toxic amount of medication (SOR C). - Gastric lavage, cathartics, or whole bowel irrigation is best for ingestion of medications that are poorly absorbed by activated charcoal (iron, lithium) or medications in sustained-release or enteric-coated formulations

Which one of the following is recommended in all patients with croup, including those with mild disease? (check one) A. Humidification therapy B. Oral dexamethasone as a single dose C. Oral diphenhydramine (Benadryl) every 6 hours until improvement D. Subcutaneous epinephrine as a single dose E. Intramuscular ceftriaxone (Rocephin) as a single dose

B. Oral dexamethasone as a single dose - 0.15-0.60 mg/kg Nebulized epinephrine is an accepted treatment in patients with moderate to severe croup.

A 58-year-old male with COPD presents with a 5-day history of increased dyspnea and purulent sputum production. He is afebrile. His respiratory rate is 24/min, heart rate 90 beats/min, blood pressure 140/80 mm Hg, and oxygen saturation 90% on room air. Breath sounds are equal, and diffuse bilateral rhonchi are noted. He is currently using albuterol/ipratropium by metered-dose inhaler three times daily. In addition to antibiotics, which one of the following would be most appropriate for treating this exacerbation? (check one) A. A single dose of intramuscular dexamethasone B. Oral prednisone for 5 days C. Daily inhaled fluticasone (Flovent) D. Hospital admission for intravenous methylprednisolone sodium succinate (Solu-Medrol) E. No corticosteroids at this time

B. Oral prednisone for 5 days

A 69-year-old female sees you for an annual examination. She asks you to look at her toes, and you note a fungal infection in five toenails. She says the condition is painful and limits her ability to complete her morning walks. She asks for treatment that will allow her to resume her daily walks as soon as possible. Her only other medical problem is allergic rhinitis which is well controlled. (check one) A. Oral griseofulvin ultramicrosize (Gris-PEG) daily for 12 weeks B. Oral terbinafine (Lamisil) daily for 12 weeks C. Topical terbinafine (Lamisil AT) daily for 12 weeks D. Topical ciclopirox (Penlac Nail Lacquer) daily for 12 weeks E. Toenail removal

B. Oral terbinafine (Lamisil) daily for 12 weeks - has the highest cure rate and best long-term resolution rate of the therapies listed. - Topical creams are not appropriate for onychomycosis because the infection resides in the cell of the toenail. - Antifungal nail lacquers have a lower cure rate than systemic therapy and should be used only when oral agents would not be safe. - Toenail removal is reserved for patients with an isolated infected nail or in cases involving a dermatophytoma.

A 25-year-old white female presents to your office with the following complaints: sudden onset of intense apprehension, fear, terror associated with impending doom, dyspnea, palpitations, and a feeling of loss of control. Which one of the following is the most likely diagnosis? (check one) A. Pheochromocytoma B. Panic attack C. Hypochondriasis D. Hypoglycemia E. Hyperthyroidism

B. Panic attacks - ages 17 and 30, and 80% are women. - sudden onset of intense apprehension, fear, or terror, often associated with thoughts of impending doom - with at least four of the following somatic symptoms: dyspnea (patients often hyperventilate); palpitations, chest pain, or discomfort; choking or smothering sensations; dizziness; a feeling of unreality; paresthesias; diaphoresis; faintness; trembling or shaking; hot and cold flashes; - fears of dying, going crazy, or losing control during an attack.

A 45-year-old Hispanic male with schizophrenia presents with an exacerbation of his COPD. He currently takes only ziprasidone (Geodon). He asks for a prescription for clarithromycin (Biaxin) because it has worked well for previous exacerbations. Which one of the following effects of this drug combination should you be alert for? (check one) A. Stevens-Johnson syndrome B. Prolonged QT interval C. Seizures D. Diarrhea E. Hypoglycemia

B. Prolonged QT interval - Second-generation antipsychotic used in the treatment of schizophrenia - This risk is further increased when these drugs are combined with certain antibiotics (e.g., clarithromycin), antiarrhythmics (class I and III), and tricyclic antidepressants.

Which one of the following is considered first-line therapy for migraine prophylaxis in adults? (check one) A. Gabapentin (Neurontin) B. Propranolol (Inderal) C. Fluoxetine (Prozac) D. Vitamin B2 (riboflavin) F. Naproxen (Naprosyn)

B. Propranolol - Other first-line agents include timolol, amitriptyline, divalproex sodium, sodium valproate, and topiramate. - Gabapentin, fluoxetine, vitamin B , and naproxen are considered second-line therapies for migraine prophylaxis in adults (SOR B), and should be used when no first-line agent or combination is effective or tolerable

A 45-year-old white male develops disabling tremulousness, loss of voice, and a marked sense of forceful and rapid heartbeat whenever he must speak to a large group. Which one of the following drugs is likely to be of most value in enabling him to give presentations at sales and stockholders' meetings? (check one) A. Desipramine (Norpramin) B. Propranolol (Inderal) C. Alprazolam (Xanax) D. Amantadine (Symmetrel) E. Buspirone (BuSpar)

B. Propranolol (Inderal)

An 84-year-old male is walking across the street and has to hurry to avoid oncoming traffic. He suddenly develops extreme pain in his knee and falls to the street, and has to be carried to the sidewalk. The following day he comes to the emergency department. He is comfortable when placed in a knee immobilizer, but is very tender just above the patella. He can bend his knee, but when he tries to straighten his leg it is so weak that he cannot move it at all. Radiographs of the knee are shown in Figure 4. Figure 4A Figure 4B What is the most likely diagnosis? (check one) A. Patellar tendon rupture B. Quadriceps tendon rupture C. Tibial plateau fracture D. Patellar subluxation E. Lumbar radiculopathy

B. Quadriceps tendon rupture - can be partial or complete complete (this case ) - cannot actively strengthen the leg - sulcus sign: painful indentation above patella; may be filled w/ blood and not palpable Not patellar tendon rupture - Xray: small shard of the patella has been pulled off and has migrated superiorly with the quadriceps; especially in lateral view - the hematoma filling the gap in the quadriceps is the same density as the muscle, but wrinkling of the fascia over the distal quadriceps provides a clue that it is no longer attached to the superior margin of the patella. - patella is retracted superiorly by the quadricepts Not Tibial plateau fractures - intra-articular, so they produce a large hemarthrosis; seen on Xray - will have pain but not weakness Not Patellar subluxation - obvious acutely; the patella is displaced laterally. - if patella is replaced, findings then include tenderness along the medial retinaculum, sometimes a joint effusion, and a positive apprehension sign when the patella is pushed gently laterally

A 5-year-old male fell while playing and complained that his wrist hurt. The next day he is brought to your office because he refuses to use his arm. Which one of the following best describes the condition seen in the radiographs shown in Figure 9? Figure 9A Figure 9B (check one) A. A normal appearance B. A radial fracture C. An ulnar fracture D. A radioulnar fracture E. Indeterminate result

B. Radial fracture - most common fracture i - easily missed - The bend in the cortex of the distal radius indicates the fracture. - Sometimes referred to as a buckle or torus fracture, it will heal with almost any choice of treatment - Most clinicians opt for casting x 1 week to reduce the chance of reinjury - the parents' preferences in this regard are important.

A 67-year-old white female has a DXA scan with a resulting T-score of -2.7. She has a strong family history of breast cancer. Which one of the following would be the most appropriate treatment for this patient? (check one) A. A bisphosphonate B. Raloxifene (Evista) C. Calcitonin nasal spray (Miacalcin) D. Teriparatide (Forteo) E. Conjugated estrogens (Premarin)

B. Raloxifene - selective estrogen receptor modulator. - increases the risk of venous thromboembolism - decrease the risk of invasive breast cancer (SOR A). Bisphosphonates - inhibit osteoclastic activity - Zoledronic acid, alendronate, and risedronate - decrease both hip and vertebral fractures - Ibandronate decreases fracture risk at the spine Calcitonin nasal spray - antiresorptive spray - decreases the incidence of vertebral compression fractures Teriparatide - recombinant human parathyroid hormone with potent bone anabolic activity - protect from vertebral and nonvertebral fractures. Hormone replacement therapy - only in women with moderate or severe vasomotor symptom - The lowest possible dose should be used for the shortest amount of time possible (SOR C).

A 35-year-old white male who has had diabetes mellitus for 20 years begins having episodes of hypoglycemia. He was previously stable and well controlled and has not recently changed his diet or insulin regimen. Which one of the following is the most likely cause of the hypoglycemia? (check one) A. Spontaneous improvement of β-cell function B. Renal disease C. Reduced physical activity D. Insulin antibodies

B. Renal disease - The most common cause of hypoglycemia in a previously stable, well-controlled diabetic patient who has not changed his or her diet or insulin dosage is diabetic renal disease Not reduction in physical activity or the appearance of insulin antibodies (unlikely after 20 years of therapy) - would increase insulin requirements and produce hyperglycemia Not Spontaneous improvement β -cell functio - after 20 years would be very rare.

As part of routine care for a 31-year-old female you obtain a Papanicolaou (Pap) test for cervical cancer screening. The cytology results are normal, and the sample is positive for the presence of HPV but negative for serotypes 16 and 18. Which one of the following is the most appropriate management for this patient? (check one) A. Immediate colposcopy B. Repeat Pap and HPV testing in 3 months C. Repeat Pap and HPV testing in 6 months D. Repeat Pap and HPV testing in 1 year E. Repeat Pap and HPV testing in 3 years

B. Repeat Pap & Cotest in 1 year - 30-64 - HPV 16 are at particular risk for CIN 3+. - HPV 18 is association with cervical adenocarcinomas, which are less efficiently detected by cytology than squamous cell cancers.

A 37-year-old graphic designer presents to your office with a history of several months of radial wrist pain. She does not recall any specific trauma but notes that it hurts to hold a coffee cup. Finkelstein's test is positive and a grind test is negative, and there is tenderness to palpation over the radial tubercle. Which one of the following would be most appropriate at this point? (check one) A. Plain radiography focusing on the scaphoid B. Rest and a thumb spica wrist splint C. MRI of the wrist D. A short arm cast

B. Rest and a thumb spica wrist splint de Quervain's tenosynovitis. Finkelstein's test has good sensitivity and specificity (SOR C) in patients with a negative grind test.

One day after a nurse performs CPR on an emergency-department patient, she learns that the patient had meningococcal meningitis. Which one of the following is the most appropriate chemoprophylaxis for this condition? (check one) A. Penicillin G benzathine (Bicillin LA), 1.2 million units intramuscularly B. Rifampin, 600 mg every 12 hours for 2 days C. Oral prednisone, 40 mg daily for 5 days D. Quadrivalent meningococcal vaccine E. No prophylaxis

B. Rifampin, 600 mg q12 x 12 days - Rifampin has been shown to be 90% effective in eliminating meningococcus from the nasopharynx. - also minocycline and ciprofloxacin - even high doses of penicillin may not eradicate nasopharyngeal meningococci. - Meningococcal vaccine appears to have clinical efficacy, but it usually takes more than 5 days to become effective.

A 47-year-old male is preparing for a 3-day trip to central Mexico to present the keynote address for an international law symposium. He asks you for an antibiotic to be taken prophylactically to prevent bacterial diarrhea. Which one of the following would you recommend? (check one) A. Trimethoprim/sulfamethoxazole (Bactrim, Septra) B. Rifaximin (Xifaxan) C. Doxycycline D. Nitrofurantoin (Macrobid)

B. Rifaximin - prophylaxis for those on critical short trips for which even a short period of diarrhea might cause undue hardship. - Rifaximin, a nonabsorbable antibiotic, has been shown to reduce the risk for traveler's diarrhea by 77%. - Trimethoprim/sulfamethoxazole and doxycycline are no longer considered effective antimicrobial agents against enteric bacterial pathogens; Increasing resistance to the fluoroquinolones, especially among Campylobacter species, is limiting their use as prophylactic agents.

A 25-year-old female is concerned about recurrent psychological and physical symptoms that occur during the luteal phase of her menstrual cycle and resolve by the end of menstruation. She wants help managing these symptoms, but does not want to take additional estrogen or progesterone. Which one of the following management strategies is supported by the best clinical evidence? (check one) A. Cognitive-behavioral therapy B. Spironolactone during the luteal phase C. Bright light therapy during the luteal phase D. Evening primrose oil started 2-4 days prior to the luteal phase E. Black cohosh

B. Spironolactone during the luteal phase - improves psychological and physical symptoms of premenstrual syndrome over 2-6 months compared with placebo.

Which one of the following is appropriate at the routine postpartum visit? (check one) A. A CBC B. Screening for depression C. Thyroid function tests D. Glucose tolerance testing E. A urine dipstick

B. Screen for depression - screening tool: Edinburgh Postnatal Depression Scale - other tests should be symptomatic, not screening

A 64-year-old white male appears to be depressed 2 weeks after hospital discharge for a myocardial infarction. He experienced short runs of ventricular tachycardia during his hospitalization, and echocardiography revealed an ejection fraction of 40% at the time of discharge, with no symptoms of heart failure. He has a history of depression in the past. His current symptoms include depressed mood, sleep disturbance, feelings of hopelessness, and anhedonia. He denies suicidal ideation.Which one of the following would be most appropriate at this point? (check one) A. Low-dose amitriptyline at bedtime B. Sertraline (Zoloft) C. Referral for electroconvulsive therapy D. Referral for intense interpersonal psychotherapy

B. Sertraline (Zoloft) - SSRIs are safe and effective in treating depression in patients with coronary disease, particularly those with a history of previous episodes of depression - while it may be effective for sleep disturbance, amitriptyline has potential cardiac side effects and is unlikely to be effective for the treatment of depression in low doses.

A 30-year-old otherwise healthy female has concerns about her menses and fertility. Her last menstrual period was 8 months ago when she stopped taking oral contraceptive pills (OCPs). In her teens and early twenties she had irregular, sporadic periods. Four years ago she developed menometrorrhagia and resultant iron deficiency anemia; this was corrected with the use of OCPs. She is now interested in becoming pregnant. Her physical examination, including a gynecologic examination, is normal. A urine pregnancy test is negative and her TSH level is in the normal range. Which one of the following is the most appropriate next step? (check one) A. A CBC and metabolic panel B. Serum LH and FSH levels C. Karyotype analysis D. Hydrochlorothiazide/triamterene (Dyazide) E. Pelvic ultrasonography

B. Serum LH and FSH levels - secondary amenorrhea (defined as the cessation of regular menses for 3 months or irregular menses for 6 months). - The most common cause: polycystic ovary syndrome, primary ovarian failure, hypothalamic amenorrhea, and hyperprolactinemia. - normal physical examination, negative pregnancy test, and no history of chronic disease, a hormonal workup is indicated, including TSH, LH, and FSH levels (SOR C). - A hormonal challenge with medroxyprogesterone to provoke withdrawal bleeding is used to assess functional anatomy and estrogen levels (SOR C). However, it has poor specificity and sensitivity for ovarian function and a poor correlation with estrogen levels.

A 70-year-old white female presents with a pruritic rash on her sacrum that has occurred intermittently over the last 6 years. She reports that the area is always very tender just before the blister-like lesions erupt. She is otherwise in good health, and takes no medications. Her past medical history is unremarkable. You provide appropriate treatment for the condition. You should advise the patient to avoid which one of the following during future outbreaks? (check one) A. Excessive intake of green, leafy vegetables B. Sexual contact C. Perfumed soaps or body lotions D. Sun exposure E. Prolonged sitting

B. Sexual contact - genital hearpes - most common sexually transmitted genital ulcer disease in the U.S. - may be the most common sexually transmitted disease in women over the age of 50 years. - Extragenital sites are involved in one-fourth of infected women, and the sacrum and buttocks are frequent locations.

When treating acute adult asthma in the emergency department, using a metered-dose inhaler (MDI) with a spacer has been shown to result in which one of the following, compared to use of a nebulizer? (check one) A. Higher hospitalization rates B. Shorter stays in the emergency department C. Higher relapse rates D. Less improvement in peak-flow rates E. Increases in the total dose of albuterol

B. Shorter stays in the emergency department - lower pulse rates - better peak-flow rates - greater improvement in arterial blood gases - decrease required albuterol doses - lower costs

A 44-year-old female presents with a complaint of increasingly dry eyes over the past 3-4 months, and says she can no longer wear contacts due to the discomfort and itching. She also apologizes for chewing gum during the visit, explaining that it helps keep her mouth moist. On examination you note decreased tear production, decreased saliva production, and new dental caries. She stopped taking a daily over-the-counter allergy medication about 1 month ago. Which one of the following is the most likely diagnosis? (check one) A. Sarcoidosis B. Sjögren's syndrome C. Ocular rosacea D. Allergic conjunctivitis E. Medication side effect

B. Sjögren's syndrome - one of the three most common systemic autoimmune diseases. - lymphocytic infiltration of exocrine glands and leads to acinar gland degeneration, necrosis, atrophy, and decreased function. - A positive anti-SS-A or anti-SS-B antigen test or a positive salivary gland biopsy is a criterion for classification of this diagnosis. - oral complaints, also include arthralgias, thyroiditis, pulmonary disease, and GERD. Sarcoidosis - shortness of breath or skin manifestations Lupus - generally have fatigue and joint pain Ocular rosacea - eye symptoms very similar to those of Sjögren's syndrome, but no oral findings Drugs such as anticholinergics can cause a dry mouth - unlikely a month after the medication was discontinued (SOR B).

A 59-year-old male with known cirrhosis is beginning to show some lower abdominal distention. Ultrasonography confirms your suspicion that he has developed moderate ascites for the first time. Which one of the following is recommended as the initial treatment of choice for this condition? (check one) A. Chlorthalidone B. Spironolactone (Aldactone) C. Furosemide (Lasix) D. Ramipril (Altace) E. Large-volume paracentesis

B. Spironolactone (Aldactone) - grade 2 ascites (visible clinically by abdominal distention) - the initial treatment: diuretics along with salt restriction - Aldosterone antagonists such as spironolactone are more effective than loop diuretics (SOR A) - Chlorthalidone, a thiazide diuretic, is not recommended - Large-volume paracentesis is the recommended treatment of grade 3 ascites (gross ascites with marked abdominal distention), and is followed by salt restriction and diuretics.

Which one of the following situations is most likely to result in immunity from court-awarded damages for personal injuries occurring as a result of reasonable and ordinary emergency care? (check one) A. Evaluating a football injury as a volunteer team physician at a local high-school game B. Stabilizing an injured victim at the scene of an automobile accident until EMS arrives C. Providing emergency care to your office nurse after he collapses while on the job D. Responding to the collapse of one of your patients in the hospital parking lot E. Treating an asthma attack while staffing the first-aid shelter at an outdoor rock concert

B. Stabilizing an injured victim at the scene of an automobile accident until EMS arrives

A patient with chronic atrial fibrillation treated with dabigatran (Pradaxa) sees you for follow-up. She says she can no longer afford the dabigatran and would like to switch to warfarin (Coumadin). She has normal renal function. Which one of the following would be the most appropriate approach? (check one) A. Start warfarin and stop dabigatran when her INR is 2.0-3.0 B. Start warfarin now and stop dabigatran in 3 days C. Stop dabigatran, start warfarin, and start low molecular weight heparin and enoxaparin (Lovenox) every 12 hr until her INR is 2.0-3.0 D. Stop dabigatran for 24 hr and then start warfarin E. Hospitalize the patient, stop dabigatran, start warfarin, and treat with heparin until her INR is 2.0-3.0

B. Start warfarin now and stop dabigatran in 3 days - Bridging with a parenteral agent is not necessary. - Dabigatran is known to increase the INR, so the INR will not reflect warfarin's effect until dabigatran has been withheld for at least 2 days.

A 63-year-old male with type 2 diabetes mellitus is seen in the emergency department for an acute, superficial, previously untreated infected great toe. Along with Staphylococcus aureus, which one of the following is the most common pathogen in this situation? (check one) A. Pseudomonas B. Streptococcus C. Clostridium D. Escherichia coli E. Adenovirus

B. Streptococcus The most common pathogens in previously untreated acute superficial foot infections in diabetic patients are aerobic gram-positive Staphylococcus aureus and β-hemolytic streptococci (groups A, B, and others). Previously treated and deep infections are often polymicrobial.

A 40-year-old runner complains of gradually worsening pain on the lateral aspect of his foot. He runs on asphalt, and has increased his mileage from 2 miles/day to 5 miles/day over the last 2 weeks. Palpation causes pain over the lateral 5th metatarsal. The pain is also reproduced when he jumps on the affected leg. When you ask about his shoes he tells you he bought them several years ago. Which one of the following is the most likely diagnosis? (check one) A. Ligamentous sprain of the arch B. Stress fracture C. Plantar fasciitis D. Osteoarthritis of the metatarsal joint

B. Stress fracture - Running injuries are primarily caused by overuse due to training errors. - Runners should be instructed to increase their mileage gradually. - A stress fracture causes localized tenderness and swelling in superficial bones, and the pain can be reproduced by having the patient jump on the affected leg Not Plantar fasciitis - burning pain in the heel - tenderness of the plantar fascia where it inserts onto the medial tubercle of the calcaneus.

A 19-year-old female runner has a 1-week history of constant groin pain. There is limited hip motion on flexion and internal rotation of the right hip. Radiographs of the hip and pelvis are normal. Which one of the following is the most likely diagnosis? (check one) A. Iliotibial band syndrome B. Stress fracture of the right femoral neck C. Osteitis pubis D. Pelvic inflammatory disease

B. Stress fractures of the femoral neck - most commonly seen in military recruits and runners. - persistent groin pain, and limited hip flexion and internal rotation. - Radiographs may be normal early. Not Iliotibial band syndrome - in runners and presents with stinging pain over the lateral femoral epicondyle Not Osteitis pubis - distance runners and presents with pain in the anterior pelvic area and tenderness over the symphysis pubis. Not Pelvic inflammatory disease - associated with abdominal pain and fever

Of the following, which is the most frequent cause of seizures in the elderly? (check one) A. Alcohol withdrawal B. Stroke C. Head trauma D. Hypoglycemia E. Dementia

B. Stroke in the geriatric population, cerebrovascular disease is the most common cause of seizures, with about 10% of stroke victims developing epileptic seizures. Seizures are more common following hemorrhagic strokes compared to nonhemorrhagic strokes. Ref: Hazzard WR, Blass JP, Halter JB, et al (eds): Principles of Geriatric Medicine and Gerontology,

A 60-year-old female with moderate COPD presents with ongoing dyspnea in spite of treatment with both an inhaled long-acting β-agonist and a long-acting anticholinergic agent. Your evaluation reveals an oxygen saturation of 88% and a PaO2 of 55%. Echocardiography reveals a normal ejection fraction but moderate pulmonary hypertension. Which one of the following would be most appropriate at this time? (check one) A. No changes in the current medical regimen B. Supplemental oxygen C. Low-dose sildenafil (Revatio) D. Nifedipine (Procardia) E. Low-dose prednisone

B. Supplemental oxygen - moderate COPD and moderate nonpulmonary arterial hypertension pulmonary hypertension is hypoxic - meets the criteria for use of supplemental oxygen (SOR A). - Low-dose prednisone may be a future option.

A factory worker sustains a forced flexion injury of the distal interphalangeal (DIP) joint, resulting in a small bone fragment at the dorsal surface of the proximal distal phalanx (mallet fracture). Which one of the following is the most appropriate management strategy? A. Buddy taping and early range of motion B. Splinting the DIP joint in extension C. Splinting the DIP joint in flexion D. Referral for surgical repair

B. The recommended treatment for a mallet fracture is splinting the distal interphalangeal (DIP) joint in extension (SOR B). The usual duration of splinting is 8 weeks. - If the finger fracture involves >30% of the intra-articular surface, referral to a hand or orthopedic surgeon can be considered. - However, conservative therapy appears to have outcomes similar to those of surgical treatment and therefore is generally preferred.

A 42-year-old female presents with a 2-month history of right-sided shoulder pain. A history reveals that her job requires repetitive motion, including abduction of the shoulder. Ibuprofen has not been helpful and the pain interferes with her sleep. The physical examination suggests rotator cuff tendinitis. A radiograph of the shoulder is normal. You discuss treatment options and the patient decides to proceed with a corticosteroid injection. Which one of the following is the appropriate anatomic location for the injection? (check one) A. The acromioclavicular joint B. The subacromial space C. The intra-articular shoulder joint under fluoroscopy D. The area of insertion of the deltoid muscle E. The area of insertion of the long head of the biceps

B. The subacromial space

Which one of the following is true regarding NSAIDs? (check one) A. They are cardioprotective B. They should be avoided in persons with cirrhotic liver disease C. They are not safe in pregnancy D. They are not safe in lactating women

B. They should be avoided in persons with cirrhotic liver disease - can increase the risk of bleeding in cirrhotic patients, as they further impair platelet function - decrease blood flow to the kidneys and can increase the risk of renal failure in patients with cirrhosis. - increase cardiovascular morbidity, worsen heart failure, increase blood pressure, and increase events such as ischemia and MI - no known teratogenic effects of NSAIDs

A 77-year-old male presents with significant postherpetic neuralgia in a chest wall distribution. Which one of the following is most likely to be effective in diminishing his discomfort? (check one) A. Oral valacyclovir (Valtrex) B. Topical lidocaine (Xylocaine) patches C. Thoracic epidural corticosteroid injections D. Herpes zoster vaccine E. Acupuncture

B. Topical lidocaine (Xylocaine) patches

A 14-year-old male has open and closed comedones without evidence of surrounding inflammation on his face and upper back. Which one of the following is the most appropriate initial treatment? (check one) A. Topical antibiotics B. Topical retinoids C. Oral antibiotics D. Oral isotretinoin

B. Topical retinoids - Comedones - noninflammatory acne lesions include papules, pustules, and nodules. - Grading acne based on the type of lesion and severity helps guide therapy. - Topical retinoids prevent the formation of comedones and reduce their number Not Topical antibiotics - for mild to moderate inflammatory or mixed acne. Not Oral antibiotics - for moderate to severe acne Not Oral isotretinoin - severe, recalcitrant acne.

You see a 4-year-old male in your office for evaluation of persisting fever, rash, and red eyes. In a discussion with his father you learn that the child has had temperatures in the 99°F-102°F range for 6 days, along with what the father describes as "pink eye." Today the child broke out in a rash on his chest and back and also has cracked red lips. On examination you confirm that he has bilateral nonpurulent conjunctival injection and a generalized maculopapular rash, as well as erythema of his hands and feet. Which one of the following is recommended at this time to evaluate for cardiac complications? (check one) A. An EKG B. Transthoracic echocardiography C. Cardiac CT D. Magnetic resonance (MR) coronary angiography E. A radionuclide myocardial perfusion scan

B. Transthoracic echocardiography - Kawasaki disease Diagnostic criteria include fever for at least 5 days and at least 4 of the 5 principal clinical features: • changes of the oral cavity and lips • polymorphous rash • bilateral nonpurulent conjunctivitis • changes in the extremities (erythema followed by desquamation) • cervical lymphadenopathy - Coronary abnormalities, including coronary aneurysms, are the most concerning sequelae of Kawasaki disease and may occur in the first week. - MR coronary angiography can be used to assess response to treatment over time. - Intravenous immunoglobulin and corticosteroids reduce the risk of coronary abnormalities and should be administered as soon as the disease is suspected.

A 58-year-old postmenopausal female presents with a recent onset of painless vaginal bleeding. Her last menses occurred 8 years ago and she has had no bleeding until now. She reports that her Papanicolaou smears have always been normal, with the last one obtained a year ago. A pelvic examination today is normal. Which one of the following management options is the preferred next diagnostic step? (check one) A. Colposcopy with endocervical curettage B. Transvaginal ultrasonography C. Meperidol (Demerol) for pain control D. Saline infusion sonohysterography E. Hysteroscopy

B. Transvaginal ultrasonography - painless postmenopausal bleeding - an endometrial thickness <3-4 mm would essentially rule out endometrial carcinoma (SOR C) Not endometrial biopsy - only if transvaginal ultrasonography is not available. - invasive and has low sensitivity for focal lesions. Not Saline infusion hysterography -if the endometrial thickness is greater than the threshold, or if an adequate measurement cannot be obtained by ultrasonography. - If hysterography shows a global process, then a histologic diagnosis can usually be obtained with an endometrial biopsy - if a focal lesion is present hysteroscopy should be considered as the next diagnostic step. No Colposcopy - not indicated given the patient's normal Papanicolaou smear.

A hospitalized patient is being treated with vancomycin for an infection due to methicillin-resistant Staphylococcus aureus (MRSA). Which one of the following is most important to monitor? (check one) A. Hepatic function B. Trough serum levels C. Peak serum levels D. Audiograms

B. Trough serum levels - best predictor of vancomycin efficacy - should be over 10 mg/L to prevent development of bacterial resistance

A 62-year-old male is admitted to the hospital with acute renal failure. A renal biopsy confirms the diagnosis of acute interstitial nephritis (AIN). Infection and immune-associated causes are ruled out, and you consider medications as a potential cause. Which one of the following would be most likely to cause AIN? (check one) A. Chronic daily use of metoprolol (Lopressor) B. Twice-daily use of ibuprofen for 2 weeks C. Initiation of lisinopril (Prinivil, Zestril) therapy 1 week ago D. A 5-day course of azithromycin (Zithromax) 6 months ago E. Intermittent use of acetaminophen, up to 4 g/day

B. Twice daily use of ibuprofen x 2 wks - all NSAIDs are known to be associated with AIN - usually approximately 2 weeks after starting a medication and is not dose-related. - Other medications strongly associated with AIN include various antibiotics (particularly cephalosporins, penicillins, sulfonamides, aminoglycosides, and rifampin), diuretics, and miscellaneous medications such as allopurinol.

A 44-year-old obese female complains of intermittent right upper quadrant pain that is worse after fatty meals. Which one of the following is the preferred initial imaging modality for evaluating her complaint and confirming the diagnosis? (check one) A. A plain radiograph B. Ultrasonography C. Cholescintigraphy D. Contrast-enhanced CT E. Contrast-enhanced MRI

B. Ultrasonography - cholecystitis or cholelithiasis

A 26-year-old gravida 2 para 1 presents at 30 weeks gestation with a complaint of severe itching. She has excoriations from scratching in various areas. She says that she had the same problem during her last pregnancy, and her medical records reveal a diagnosis of intrahepatic cholestasis of pregnancy. Elevation of which one of the following is most characteristic of this disorder? (check one) A. γ-Glutamyltransferase (GGT) B. Bile acids C. Direct bilirubin D. Indirect bilirubin E. Prothrombin time

B. bile acids - Intrahepatic cholestasis of pregnancy - severe pruritus in the third trimester. - no primary skin lesions - elevation of serum levels of total bile acids. - no jaundice; usu normal bilirubin, GGT so not a liver disease

A 42-year-old previously healthy white female presents to your office with her third episode of abdominal pain. This episode began 2 hours ago, and the pain is improving. She describes colicky right upper quadrant and epigastric pain. On examination you note mild right upper quadrant tenderness, with otherwise unremarkable findings. Renal function tests are normal. Which one of the following would be most appropriate at this point? (check one) A. KUB films B. Ultrasonography of the right upper quadrant C. Abdominal CT with intravenous contrast D. Abdominal CT with intravenous and oral contrast E. MRI of the abdomen

B. Ultrasound of RUQ RLQ - Abdominal CT with intravenous contrast is the preferred test for right lower quadrant pain LLQ - abdominal CT with intravenous tumor - MRI

A 35-year-old white male presents with dyspepsia. He has had no symptoms that suggest gastroesophageal reflux or bleeding, but a test for Helicobacter pylori is positive. After 2 weeks of treatment with omeprazole (Prilosec), amoxicillin, and clarithromycin (Biaxin), he is asymptomatic. Which one of the following is recommended to test for the eradication of H. pylori in this patient? (check one) A. Immunoglobulin G serology B. A urea breath test C. Upper endoscopy with a biopsy D. An upper gastrointestinal series

B. Urea breath test - A test-and-treat approach is recommended for most patients with undifferentiated dyspepsia. - This strategy reduces the need for antisecretory medications, as well as the number of endoscopies - The currently recommended test for eradication of H. pylori in this clinical setting is either the urea breath test or H. pylori stool antigen.

Which one of the following is true concerning Norwalk virus? (check one) A. Outbreaks occur mostly in settings with large numbers of children, such as schools and day-care centers B. Viral shedding continues long after the acute illness C. The virus does not survive long on most environmental surfaces D. An episode of Norwalk gastroenteritis leads to long-lasting immunity E. It is a less common cause of diarrhea in adults than Shigella

B. Viral shedding continues long after the acute illness - Outbreaks occurs at all settings and all ages esp. elderly & immunocompromised - Viral shedding b/f and after illness - Virus persists on environmental surfaces and can tolerate a broad range of temperatures - Multiple strains of the virus, so a single infection does not confer immunity - the most common cause of diarrhea in adults.

Which one of the following is associated with an increased risk of delirium in hospitalized patients? (check one) A. Not having family members present at the time of admission B. Vision or hearing impairment C. Repeated reorientation for patients with cognitive impairment D. Early mobilization

B. Vision or hearing impairment; also - dehydration - immobility - cognitive impairment - sleep deprivation

A 70-year-old white male presents with fatigue, weakness, and foot paresthesias. His hemoglobin level is 10.5 g/dL (N 12.6-17.4). His peripheral smear is shown in Figure 2. Figure 2 Which one of the following is the most likely diagnosis? (check one) A. Iron deficiency anemia B. Vitamin B12 deficiency anemia C. Hemolytic anemia D. Acute myelogenous leukemia E. Chronic myelogenous leukemia

B. Vitamin B12 deficiency anemia - hypersegmented polymorphonuclear (PMN) white blood cell - typical of vitamin B12 deficiency with pernicious anemia. - no evidence of hemolysis or leukemia. - It is important to note that elderly patients with vitamin B12 deficiency may have neurologic signs and symptoms before developing hematologic abnormalities.

A mother brings in her 2-month-old infant for a routine checkup. The baby is exclusively breastfed, and the mother has no concerns or questions. In addition to continued breastfeeding, which one of the following would you recommend continuing or adding at this time? (check one) A. Iron supplementation B. Vitamin D supplementation C. A multivitamin D. 8 oz of water daily E. 4 oz of cereal daily

B. Vitamin D supplementation Although breast milk is the ideal source of nutrition for healthy term infants, supplementation with 400 IU/day of vitamin D is recommended beginning in the first few days of life and continuing until the child is consuming at least 500 mL/day of formula or milk containing vitamin D (SOR B). The purpose of supplementation is to prevent rickets. For exclusively breastfed infants, iron supplementation should begin at 4 months of age. The introduction of cereal is recommended at 6 months of age.

A 65-year-old white female presents with weight loss and fatigue. On examination, she has lymphadenopathy, hepatomegaly, and mild splenomegaly. Her hemoglobin level is 9.0 g/dL (N 12.0-16.0), and a chemistry panel reveals a serum protein level of 9.0 g/dL (N 6.0-8.0). You order a chest radiograph, which shows clear lung fields and no evidence of lytic lesions in the thoracic spine. Serum protein electrophoresis reveals a monoclonal gamma-globulin spike, which on immunoelectrophoresis is found to be due to IgM kappa-protein. Urine for Bence-Jones protein is positive. A bone marrow biopsy from the iliac crest demonstrates hypercellularity, with a large number of lymphocytes, but normal-appearing plasma cells. Which one of the following is the most likely diagnosis? (check one) A. Multiple myeloma B. Waldenström's macroglobulinemia C. Sarcoidosis D. Monoclonal gammopathy of undetermined significance E. Non-Hodgkin's lymphoma

B. Waldenström's macroglobulinemia. - an uncontrolled proliferation of lymphocytes and plasma cells, which produce IgM proteins with kappa light chains. -ave age: 65 years - Weakness, fatigue, weight loss, bleeding, and recurrent infections are common presenting symptoms. - pallor, hepatosplenomegaly, and lymphadenopathy. - moderate anemia and monoclonal IgM peaks on serum electrophoresis. - Bence-Jones protein is seen in 80% of cases, but is typically absent in monoclonal gammopathy of undetermined significance. Not multiple myeloma - will have lytic bone lesions are not seen Not Sarcoidosis - hilar lymphadenopathy and a polyclonal gammopathy. Not Non-Hodgkin's lymphoma - with similar symptoms, lymphadenopathy, and hepatosplenomegaly - but generally lacks a monoclonal gammopathy and Bence-Jones proteinemia, and has distinctive malignant lymphocytes on bone marrow biopsy

According to both the Centers for Disease Control and the American College of Sports Medicine, in order to burn fat stores obese patients should exercise: (check one) A. a minimum of 30 minutes 3 days/week B. a minimum of 30 minutes at least 5 days/week D. a minimum of 20 minutes every day E. a minimum of 10 minutes at least 3 times daily

B. a minimum of 30 mins at least 5 times/week - Because glycogen is the primary energy source for muscles during the first 20 minutes of exercise, at least 30 minutes of exercise is necessary to begin burning fat stores. - The CDC and the American College of Sports Medicine recommend a minimum of 30 minutes of exercise 5 days per week. - Twenty minutes of exercise daily or three 10-minute sessions daily does improve cardiovascular fitness, but does not cause significant weight loss. Walking, on land or in water, and stationary biking are equivalent in benefit.

The physician counseling a 4-year-old child about the death of a loved one should keep in mind that children in this age group: (check one) A. often feel no sense of loss B. often believe they are somehow responsible for the death C. should not attend a funeral D. should usually be told the loved one is having a long sleep E. usually accept the finality of death with little question

B. believe they are responsible for the death - Children from the ages of 2 to 6 often believe they are somehow responsible for the death of a loved one. - The emotional pain may be so intense that the child may react by denying the death - If children wish to attend a funeral, or if their parents want them to, they should be accompanied by an adult who can provide comfort and support. - Telling a child the loved one is asleep or that he or she "went away" usually creates false hopes for return, or it may foster a sleep phobia.

Terminally ill cancer patients who receive palliative chemotherapy (check one) A. survive longer B. are less likely to die at home C. are less likely to undergo CPR D. are less likely to undergo mechanical ventilation E. are referred to hospice earlier in their disease

B. are less likely to die at home Patients who receive palliative chemotherapy for end-stage cancers are less likely to die at home, more likely to undergo CPR, and more likely to undergo mechanical ventilation. In addition, these patients are referred to hospice later and there is no survival benefit.

A 55-year-old male is brought to the emergency department with a complaint of pain in the right eye and reduced vision of about 10 minutes' duration. His eye was injured while he was hitting a metal stake with a sledge hammer. He was not wearing safety goggles. On examination you note a subconjunctival hemorrhage completely surrounding the cornea. The iris is irregular. Which one of the following is contraindicated prior to emergency transfer to an ophthalmologist? (check one) A. Administering an analgesic B. Attempting tonometry C. A visual acuity test D. Use of an eye shield E. Administering an antiemetic

B. attempting tonometry - Because of a risk of extruding intraocular fluid, tonometry should not be attempted if globe rupture is suspected. - raise the possibility of globe rupture due to a fragment of steel penetrating through the cornea and pupil and into the globe. - Relief of pain with an analgesic is appropriate before transfer. - A rapid assessment of gross visual acuity (e.g., counting fingers, seeing light versus dark) may be performed. - An eye shield should be placed over the affected eye to avoid putting pressure on the eye during transport to the ophthalmologist. - Because the Valsalva effect from vomiting may lead to extrusion of intraocular contents, an antiemetic would be appropriate before transfer as well.

A 63-year-old male presents for a preoperative evaluation prior to total knee arthroplasty. He underwent coronary artery angioplasty and stent placement 3 years ago following an episode of angina and an abnormal exercise stress test. His current medications include aspirin, atorvastatin (Lipitor), and lisinopril (Prinivil, Zestril). He runs 2 miles three times/week without difficulty. He has no history of heart failure, diabetes mellitus, kidney disease, or cerebrovascular disease. An EKG and preoperative blood tests, including kidney function tests, are normal. Which one of the following is indicated prior to surgery to decrease this patient's perioperative risk? (check one) A. Performing noninvasive cardiac stress testing B. Continuing his statin therapy C. Discontinuing aspirin D. Starting a β-blocker E. Starting clopidogrel (Plavix)

B. continue statin - For patients undergoing vascular therapy, statins have been associated with an improvement in postoperative cardiac outcomes. - This patient has a Revised Cardiac Risk Index (Goldman Index) score of 1, Score 1 - low-risk group for perioperative cardiac complications - are able to walk for 2 blocks or climb a flight of stairs without stopping to rest (4 METS) - do not need noninvasive cardiac testing. - should continue β-blocker; but adding one may increase risk Do not stop ASA - places patient at risk for perioperative cardiac events Do not start plavix, but don't stop it either - stopping clopidogrel markedly increases risk in patients who has undergone coronary stent placement (6 weeks for bare-metal stents, 1 year for drug-eluting stents) - no need to start clopidogrel in other patients.

The preferred site for an emergency airway is: (check one) A. the thyrohyoid membrane B. the cricothyroid membrane C. immediately below the cricoid cartilage D. through the first and second tracheal rings E. at the level of the thyroid isthmus

B. cricothyroid membrane - directly above the cricoid cartilage, through the cricothyroid membrane. - Strictly speaking, this is not a tracheotomy, because it is actually above the trachea - However, it is below the vocal cords and bypasses any laryngeal obstruction Not thyrohyoid membrane - above the vocal cords, impractical site Not area directly below the cricoid cartilage - i.e. second, third, and fourth tracheal rings, as well as the thyroid isthmus - preferred tracheotomy site under controlled circumstances - excessive bleeding and difficulty finding the trachea may significantly impede the procedure in an emergency.

An anxious and agitated 18-year-old white male presents to your office with a 2-hour history of severe muscle spasms in the neck and back. He was seen 2 days ago in a local emergency department with symptoms of gastroenteritis, treated with intravenous fluids, and sent home with a prescription for prochlorperazine (Compazine) suppositories. The best therapy for this problem is intravenous administration of: (check one) A. atropine B. diphenhydramine (Benadryl) C. haloperidol D. succinylcholine (Anectine) E. carbamazepine (Tegretol)

B. diphenhydramine (Benadryl) - acute dystonic reaction - can be frightening and painful to the patient and confusing to the treating physician who may be unaware of what medications the patient is taking. - Dystonia can be caused by any agent that blocks dopamine, including prochlorperazine, metoclopramide, and typical neuroleptic agents such as haloperidol. The acute treatment of choice is diphenhydramine or benztropine.

A 40-year-old white female lawyer sees you for the first time. When providing a history, she describes several problems, including anxiety, sleep disorders, fatigue, persistent depressed mood, and decreased libido. These symptoms have been present for several years and are worse prior to menses, although they also occur to some degree during menses and throughout the month. Her menstrual periods are regular for the most part. The most likely diagnosis at this time is: (check one) A. premenstrual syndrome B. dysthymia C. dementia D. menopause E. anorexia nervosa

B. dysthymia - Psychological disorders, including anxiety, depression, and dysthymia, are frequently confused with premenstrual syndrome (PMS), and must be ruled out before initiating therapy. - PMS is trully cyclic

A 54-year-old female presents with a 2-month history of intense vulvar itching that has not improved with topical antifungal treatment. On examination you note areas of white, thickened, excoriated skin. Concerned about malignancy you perform punch biopsies, which reveal lichen sclerosus. The treatment of choice for this condition is topical application of: (check one) A. conjugated estrogens B. fluorinated corticosteroids C. petrolatum D. 2% testosterone E. fluorouracil (Efudex)

B. fluorinated corticosteroids - Lichen sclerosus is a chronic, progressive, inflammatory skin condition found in the anogenital region. It is characterized by intense vulvar itching. The treatment of choice is high-potency topical corticosteroids.

A 40-year-old male respiratory therapist presents for a health examination prior to hospital employment. His history indicates that as a child he lived on a farm in Iowa. His examination is unremarkable, but a chest radiograph shows that both lung fields have BB-sized calcifications in a miliary pattern. No other findings are noted. A PPD skin test is negative. The findings in this patient are most likely a result of (check one) A. HIV infection B. histoplasmosis C. coccidioidomycosis D. tuberculosis E. cryptococcosis

B. histoplasmosis Asymptomatic patients in excellent health often present with this characteristic chest radiograph pattern, which is usually due to histoplasmosis infection, especially if the patient has been in the midwestern United States.

Which one of the following is more likely to occur with glipizide (Glucotrol) than with metformin (Glucophage)? (check one) A. Lactic acidosis B. Hypoglycemia C. Weight loss D. Gastrointestinal distress

B. hypoglycemia Metformin - biguanide used as an oral antidiabetic agent - One of its main advantages over some other oral agents is that it does not cause hypoglycemia - Lactic acidosis, while rare, can occur in patients with renal impairment. - metformin reduces insulin levels and more frequently has a weight-maintaining or even a weight loss effect - Gastrointestinal distress is a common side effect of metformin, particularly early in therapy.

You see a 9-year-old female for evaluation of her asthma. She and her mother report that she has shortness of breath and wheezing 3-4 times per week, which improves with use of her albuterol inhaler. She does not awaken at night due to symptoms, and as long as she has her albuterol inhaler with her she does not feel her activities are limited by her symptoms. About once per year she requires prednisone for an exacerbation, often triggered by a viral infection. Based on this information you classify her asthma severity as: (check one) A. intermittent B. mild persistent C. moderate persistent D. severe persistent

B. mild persistent her symptoms occurring more than 2 days per week, but not daily, and use of her albuterol inhaler more than 2 days per week, but not daily.

A 35-year-old nulligravida sees you for preconception counseling. She has hypothyroidism treated with levothyroxine (Synthroid), and her most recent TSH level was in the therapeutic range. She has no symptoms of hypothyroidism. Which one of the following is the patient most likely to require if she becomes pregnant? (check one) A. A decreased dosage of levothyroxine B. An increased dosage of levothyroxine C. The addition of liothyronine (Cytomel) D. Substitution of desiccated thyroid hormone preparation

B. increase dosage of levothyroxine Thyroid hormone requirements increase during pregnancy. Most women with hypothyroidism who become pregnant require an increased levothyroxine dosage (SOR A). A common recommendation is to have women on fixed daily doses of levothyroxine begin taking nine doses weekly (one extra dose on 2 days of the week) as soon as the pregnancy is confirmed (SOR B). Thyroid function tests should be repeated regularly throughout the pregnancy to guide additional dosage adjustments.

Which one of the following causes rhinitis medicamentosa with prolonged use in the treatment of rhinitis? (check one) A. Intranasal antihistamines B. Intranasal decongestants C. Intranasal anticholinergics D. Intranasal mast cell stabilizers E. Leukotriene antagonists

B. intranasal decongestants - Intranasal decongestants such as phenylephrine should not be used for more than 3 days, as they cause rebound congestion on drug withdrawal. - When used for several months or more, these agents can cause a form of rhinitis, rhinitis medicamentosa, that can be extremely difficult to treat. (rhinitis medicamentosa = rebound nasal dripping)

When a screening test identifies a cancer earlier, thereby increasing the time between diagnosis and death without prolonging life, this is called: (check one) A. length-time bias B. lead-time bias C. a false-positive screening test D. increasing the positive predictive value of the screening test E. attributable risk

B. lead-time bias when a screening test identifies a cancer earlier, thereby increasing the time between diagnosis and death without actually prolonging life.

A 31-year-old gravida 1 para 0 presents for a routine visit at 32 weeks gestation. She has gestational diabetes mellitus (GDM) and has been following the dietary guidelines from her dietitian. However, her blood glucose is still elevated. She is adamant about not starting insulin but is willing to consider taking metformin (Glucophage). Before making a decision she would like to know the specific benefits to her and her baby. You would tell her that one benefit of treatment of GDM is a decreased risk for (check one) A. maternal type 2 diabetes mellitus after delivery B. maternal preeclampsia C. perinatal death D. a small-for-gestational-age infant

B. maternal pre-eclampsia Glucose control for GDM decrease risk for 1. operative delivery, 2. large-for-gestational-age infants 3. shoulder dystocia 4. maternal preeclampsia.

Which one of the following is the recommended duration of thromboprophylaxis following total hip arthroplasty, starting from the day of surgery and including outpatient prophylaxis? (check one) A. 7 days B. 14 days C. 35 days D. 60 days E. 90 days

B. outpatient thromboprophylaxis for a duration of up to 35 days. - Older recommendations for 10-14 days of prophylaxis were based on studies performed when this was the usual hospital stay. - This is still recommended as the minimum length for prophylaxis, but a longer period of time is preferred.

A 40-year-old white female presents with pain on inspiration and dyspnea since this morning. She has no chronic medical problems, takes no medications, has not traveled, and has no history of trauma. On examination the patient is afebrile, has a heart rate of 90 beats/min and a respiratory rate of 20/min, and her lungs are clear to auscultation. The pain is worse in the supine position. Which one of the following would you do initially? (check one) A. Order a CBC with differential B. Order a chest film and EKG C. Prescribe ibuprofen D. Prescribe omeprazole (Prilosec) E. Prescribe a bronchodilator

B. order a CXR and CKG - pleuritic chest pain, and - worse when supine and is accompanied by dyspnea creates additional concern. - Supine pain could be due to pericarditis (EKG) - Dyspnea increases suspicion for pneumonia, pulmonary embolism, pneumothorax, and myocardial infarction (EKG/CXR) - Once these problems have been ruled out, a diagnosis of pleurisy would be reasonable and can be treated with an NSAID. Not CBC - for infection or anemia is the cause of the problem Not Omeprazole or a bronchodilator - inappropriate treatment, as asthma and reflux are not likely in this patient.

A 45-year-old male was admitted to the hospital for nausea resulting from chemotherapy for colon cancer. He has no other chronic diseases and takes no routine medications. He was mildly dehydrated on admission and has been receiving intravenous fluids (D5 ½-normal saline with potassium chloride) at slightly higher than maintenance rates through an indwelling port for the last 24 hours. The nausea is being controlled by antiemetics, and his condition is improving. Results of routine blood work at the time of admission and from the following morning are shown below. Admission Following Morning Glucose 109 mg/dL (N 65-110) 371 mg/dL BUN 13 mg/dL (N 7-21) 9 mg/dL Creatinine 0.9 mg/dL (N 0.6-1.6) 0.9 mg/dL Sodium 143 mmol/L (N 136-144) 129 mmol/L Potassium 3.7 mmol/L (N 3.6-5.1) 6.6 mmol/L Chloride 110 mmol/L (N 101-111) 108 mmol/L Total CO2 20 mmol/L (N 22-32) 22 mmol/L Which one of the following would be the most appropriate next step? (check one) A. Start an intravenous insulin drip B. Order blood work taken from a peripheral vein C. Restrict the patient's free water intake D. Switch from normal saline to hypertonic saline E. Treat with diuretics

B. order blood work from peripheral vein - Patient's condition is improving but his lab is out of range - his sugar is 371 but he has no diabetes, his potassium is so high he should be symptomatic - likely the blood is from indwelling catheter w/out discarding the first few c.c.

A 30-year-old African-American female is being evaluated because of absent menses for the last 6 months. Menarche was at age 12. Her menstrual periods have frequently been irregular, and are accompanied only occasionally by dysmenorrhea. She had her first child 4 years ago, but has not been able to become pregnant since. A physical examination and pelvic examination are unremarkable. A serum pregnancy test is negative, prolactin levels are normal, and LH and FSH levels are both three times normal on two occasions. These findings are consistent with: (check one) A. hypothalamic amenorrhea B. ovarian failure C. pituitary microadenoma D. polycystic ovary syndrome

B. ovarian failure the elevated FSH and LH indicate an ovarian problem

A 4-year-old male has a BMI of 17.5 kg/m2, which places him between the 90th and 95th percentiles for BMI. According to the CDC, he should be classified as being (check one) A. at a healthy weight B. overweight C. obese D. morbidly obese

B. overweight The recommended terminology for weight classification are BMI (for children ages 2-18 years) or weight-for-length ratio (for children ages 0-2 years). Children under the age of 2 years are identified as being overweight when their weight-for-length ratio exceeds the 95th percentile for their sex. Obese is not used for children under the age of 2 years. Children age 2-18 years Underweight: below 5th percentile healthy weight: 5th and 85th percentile Overweight: 85th and 94th percentile Obese: 95th or greater percentile. No current standard for childhood morbid obesity, but obesity is sometimes classified as severe or extreme when a child's BMI is at the 99th percentile or greater.

Which one of the following is associated with testosterone supplementation in men with hypogonadism? (check one) A. Muscle wasting B. Polycythemia C. Osteoporosis D. An increased risk of benign prostatic hypertrophy

B. polycythemia Testosterone increases hematocrit and can cause polycythemia. In patients receiving testosterone supplementation, hematocrit should be monitored every 6 months for the first 18 months, then annually. Testosterone should be discontinued if there is more than a 50% rise in hematocrit

A patient presenting with severe carbon monoxide poisoning should be treated with: (check one) A. inhaled helium B. supplemental oxygen C. intravenous calcium gluconate D. intravenous iron E. intravenous magnesium

B. supplemental oxygen

An elevation of serum alkaline phosphatase combined with an elevation of 5'-nucleotidase is most suggestive of conditions affecting (check one) A. bone B. the liver C. the placenta D. the small intestine

B. the liver Measuring 5'-nucleotidase to determine whether the alkaline phosphatase elevation is due to a hepatic problem is well substantiated, practical, and cost effective

American Urological Association guidelines define asymptomatic microscopic hematuria as which one of the following in the absence of an obvious benign cause? (check one) A. ≥ 1 RBCs/hpf B. ≥ 3 RBCs/hpf C. ≥ 10 RBCs/hpf D. A positive dipstick reading for blood

B. ≥ 3 RBCs/hpf The American Urological Association guidelines define asymptomatic microscopic hematuria (AMH) as ≥3 RBCs/hpf on a properly collected urine specimen in the absence of an obvious benign cause (SOR C). A positive dipstick does not define AMH, and evaluation should be based solely on findings from microscopic examination of urinary sediment and not on a dipstick reading. A positive dipstick reading merits microscopic examination to confirm or refute the diagnosis of AMH.

A 27-year-old Korean female consults you regarding several painful ulcers she has developed in the vaginal area. Your examination reveals multiple 0.5-cm to 1.5-cm oval ulcers with sharply defined borders and a yellowish-white membrane. She denies recent sexual activity. Except for recurring aphthous ulcers of her mouth, her past history is unremarkable. You obtain blood for a CBC and serology. A Tzanck smear and culture of her ulcer is negative for herpes simplex virus. Two days later she returns to discuss her laboratory findings. She draws your attention to a pustule with an erythematous margin at the site where the venipuncture was done. At this time the most likely diagnosis is: (check one) A. Reiter's syndrome B. Behçet's syndrome C. syphilis D. mucocutaneous lymph node syndrome (Kawasaki disease) E. AIDS

B.Behçet's syndrome - recurring genital and oral ulcerations and relapsing uveitis (2/3 will go blind) - more common in Japan, Korea, and the Eastern Mediterranean area, and affects primarily young adults - cause is unknown - Patients may develop arthritis, vasculitis, intestinal manifestations, or neurologic manifestations - also associated with cutaneous hypersensitivity - 60%-70% of patients will develop a sterile pustule with an erythematous margin within 48 hours of an aseptic needle prick. Not Reiter's syndrome - nogenital ulcers Not syphilis - painless ulcers Mucocutaneous lymph node syndrome (Kawasaki disease) - primarily affects children under 6 years of age. AIDS - distinctive skin lesions - genital ulcers are not a common manifestation of this disease.

You test a patient's muscle strength and find that his maximum performance consists of the ability to move with gravity neutralized. This qualifies as which grade of muscle strength, on a scale of 0 to 5? (check one) A. 0 B. 1 C. 2 D. 3 E. 4

C. 2 Muscle strength is scored on a scale of 0 to 5. 0 - inability to contract a muscle is scored as 0. 1 - Contraction without movement constitutes 2- Movement with the effect of gravity neutralized 3- while movement against gravity only 4 - Movement against gravity plus some additional resistance indicates grade 4 5 - normal movement against substantial resistance.

How many arteries and veins are normally found in the umbilical cord on a newborn examination? (check one) A. 1 artery, 1 vein B. 1 artery, 2 veins C. 2 arteries, 1 vein D. 2 arteries, 2 vein E. 2 arteries, 3 veins

C. 2 arteries, 1 vein

The recommended time to screen for gestational diabetes in asymptomatic women with no risk factors for this condition is: (check one) A. in the first trimester B. at 16-20 weeks gestation C. at 24-28 weeks gestation D. at 35-37 weeks gestation

C. 24-28 weeks gestation. The patient may be given a 50-g oral glucose load followed by a glucose determination 1 hour later.

A study finds that the positive predictive value of a new test for breast cancer is 75%, which means that (check one) A. among patients with known breast cancer who had the test, 75% had a positive test B. among patients with no breast cancer who had the test, 75% had a negative test C. 75% of patients who tested positive actually had breast cancer D. 75% of patients who tested negative did not have breast cancer

C. 75% of patients who tested positive actually had breast cancer

Slipped capital femoral epiphysis is most likely in which one of the following patients with no history of trauma? (check one) A. A 3-day-old male with a subluxable hip B. A 7-year-old male with groin pain and a limp C. A 13-year-old male with knee pain D. A 16-year-old female with lateral thigh numbness

C. A 13-year-old male with knee pain Slipped capital femoral epiphysis (SCFE) occurs most commonly during the adolescent growth spurt (11-13 for girls, 13-15 for boys). While the cause is unknown, associated factors include anatomic variables e.g. femoral retroversion or steeper inclination of the proximal femoral physis, in addition to being overweight. African-Americans are affected more commonly as well. The patient may present with pain in the groin or anterior thigh, but also may present with pain referred to the knee. That is also the case for Legg-Calvé-Perthes disease, also known as avascular or aseptic necrosis of the femoral head. This condition most commonly occurs in boys 4-8 years of age. In addition to hip (or knee) pain, limping is a prominent feature.

In which one of the following patients should a creatine kinase level be obtained to detect Duchenne muscular dystrophy? (check one) A. A 2-month-old male who is unable to roll over from prone to supine B. A 7-month-old male who is unable to get into a sitting position unassisted C. A 15-month-old male who is walking but is unable to stand up from a supine position without support D. A 16-month-old male who is not walking unassisted E. A 6-month-old with high neuromuscular tone on physical examination

C. A 15-month-old male who is walking but is unable to stand up from a supine position without support Infants should roll from prone to supine by 4 months of age and supine to prone by 6 months of age. They should be able to get themselves into a sitting position by 9 months of age. While low muscular tone in an infant suggests muscular dystrophy, high muscle tone is concerning for an upper motor neuron condition and should be evaluated with MRI.

Based on U.S. Preventive Services Task Force guidelines, screening for lung cancer with low-dose CT of the chest is indicated for which one of the following patients with a 30-pack-year smoking history? (check one) A. 50-year-old current smoker B. n 85-year-old current smoker C. A 60-year-old who quit smoking 10 years ago D. A 75-year-old who quit smoking 20 years ago

C. A 60-year-old who quit smoking 10 years ago Low-dose computed tomography (LDCT) in adults 55-80 years of age who have a 30-pack-year smoking history and currently smoke or have quit within the past 15 years.

A 26-year-old pet groomer sustained a dog bite to her left hand 2 hours ago. On examination a 4-cm × 2.5-cm laceration is noted on the thenar eminence of her palm. Although the wound shows some gaping there is minimal active bleeding. No neurovascular injury is noted. Which one of the following is an indication for antibiotics in this patient? (check one) A. A wound size >2 cm B. The presence of wound gaping C. A bite involving the hand D. The patient's occupation

C. A bite involving the hand high risk include - bite on an extremity with underlying venous and/or lymphatic compromise - bite involving the hand - bite near or in a prosthetic joint, - cat bites, crush injuries, delayed presentation, puncture wounds, - underlying diabetes mellitus and immunosuppression.

A 25-year-old white male who has a poorly controlled major seizure disorder and a 6-week history of recurrent fever, anorexia, and persistent, productive coughing visits your office. On physical examination he is noted to have a temperature of 38.3°C (101.0°F), a respiratory rate of 16/min, gingival hyperplasia, and a fetid odor to his breath. Auscultation of the lungs reveals rales in the mid-portion of the right lung posteriorly. Which one of the following is most likely to be found on a chest radiograph? (check one) A. Sarcoidosis B. Miliary calcifications C. A lung abscess D. A right hilar mass E. A right pleural effusion

C. A lung abscess - anaerobic lung abscess Physical examination usually reveals - poor dental hygiene - a fetid odor to the breath and sputum - rales, and pulmonary findings consistent with consolidation Sarcoidosis - no productive cough and have bilateral physical findings Disseminated TB (miliary calcification) - A persistent productive cough is not a striking finding Right hilum mass or Right pleural effusion - The clinical presentation and physical findings are not consistent

A 25-year-old female kindergarten teacher comes to your office for evaluation of a cough she has had for 2 weeks. The preceding week she had symptoms of rhinorrhea, mild malaise, low-grade fever, and lacrimation. She reports that episodes of coughing are so severe that vomiting is induced. She was evaluated at a walk-in clinic 1 week ago and was diagnosed with bronchitis. Treatment with hydrocodone cough syrup and amoxicillin has not helped. On examination she has mild rhinorrhea and injected conjunctivae, but her lungs are clear. A chest radiograph is normal and her laboratory results reveal a mild lymphocytosis. Which one of the following is the most appropriate next step in the management of this patient? (check one) A. Corticosteroid therapy B. A sputum culture C. A nasopharyngeal culture and polymerase chain reaction testing D. Direct fluorescent antibody testing E. Serologic testing

C. A nasopharyngeal culture and polymerase chain reaction testing - whooping cough - reemerged over the past few years. - Stages 1. catarrhal stage - nonspecific symptoms similar to those of a viral upper respiratory illness; 1-2 weeks; highly contagious. 2. paroxysmal stage - severe coughing spells followed by the inspiratory whoop (much more likely in,children); Post-tussive emesis is another classic sign. - There are no characteristic findings on examination other than signs induced by extreme coughing. - Serologic testing is useful only in research settings, and direct fluorescent antibody testing is not recommended. - Azithromycin should be used as initial therapy, but this is to decrease transmission of the illness and does not improve symptoms.

A 67-year-old male presents with a 10-day history of bilateral shoulder pain and stiffness accompanied by upper arm tenderness. On examination there is soreness about both shoulders and the patient has great difficulty raising his arms above his shoulders. There is no visual disturbance, and no tenderness over the temporal arteries. C-reactive protein is elevated and the erythrocyte sedimentation rate is 65 mm/hr (N 0-17). Which one of the following would help to confirm the most likely diagnosis? (check one) A. The use of published validated diagnostic criteria B. Synovitis of the glenohumeral joint on ultrasonography C. A response to treatment with prednisone D. A response to NSAIDs E. A lack of systemic symptoms

C. A response to treatment with prednisone polymyalgia rheumatica: mostly in older adults, almost never b/f 50 symptoms: fatigue, weight loss, low-grade fever, a decline in appetite, and depression. No validated diagnostic criteria available Treatment: 15 mg of prednisone daily is dramatic, often within 24-48 hours, and if this response is not seen, alternative diagnoses must be considered. :( NSAIDs are not useful in the management and associated with high drug morbidity. Ultrasonography may be useful in making the diagnosis, with typical findings of subdeltoid bursitis and tendon synovitis of the shoulders, but synovitis of the glenohumeral joint is less common.

A 70-year-old Asian male presents with hematochezia. He has stable vital signs. Lower endoscopy is performed, but is unsuccessful due to active bleeding. Which one of the following would be most appropriate at this point? (check one) A. Abdominal CT B. A barium enema C. A technetium-99m blood pool scan D. Exploratory laparotomy E. A small-bowel radiograph

C. A technetium-99m blood pool scan - localizing the bleeding site, rather than diagnosing the cause of the bleeding, is the most important task Lower GI series - usually nondiagnostic during heavy, active bleeding Blood pool scan - allows repeated scanning over a prolonged period of time - allow enough accumulation of the isotope to direct the arteriographer to the most likely source of the bleeding - If the scan is negative, arteriography would be unlikely to reveal the active source of bleeding, and is also a more invasive procedure Exploratory laparotomy - may be indicated if a blood pool scan or an arteriogram is nondiagnostic and the patient continues to bleed heavily.

While playing basketball, a 29-year-old male falls on his outstretched hand with his wrist fully extended. He sees you the following day because of diffuse wrist pain and decreased range of motion. The point of maximal tenderness is on the dorsal aspect of the wrist between the extensor pollicis brevis and extensor pollicis longus tendons. There is no visible deformity. Radiographs show no fracture. Which one of the following is the most appropriate initial treatment of this patient? (check one) A. A wrist extension splint B. An ulnar gutter splint C. A thumb spica splint D. A short arm cast E. Physical therapy

C. A thumb spica splint - falling on an outstretched hand with the wrist extended; suspect an occult fracture of scaphoid - tenderness in the anatomic snuff box (between the extensor pollicis longus and extensor pollicis brevis tendons) - even if radiographs are negative - To prevent vascular necrosis and non-union; both wrist and thumb must be immobilized - thumb spica splint is the best option initially - should be worn continuously until a follow-up evaluation, including radiographs, in 1-2 weeks.

A 26-year-old male presents with hand pain. He tells you he was out drinking with friends last night and does not remember sustaining any injuries. On examination, there is diffuse swelling and tenderness across the dorsal and medial aspects of the hand. Radiographs are shown in Figures 8 and 9. Which one of the following would be the most appropriate treatment? (check one) A. A wrist extension splint B. A molded finger splint C. A ulnar gutter splint D. A short arm cast E. Surgical pin fixation

C. A ulnar gutter splint x 3 - 4 wks - fracture of the fifth metacarpal head, commonly known as a boxer's fracture - only slight volar angulation and no displacement; so ulnar gutter splint, which immobilizes the wrist, hand, and fourth and fifth digits in the neutral position - Surgical pinning is indicated in cases of significant angulation (35°-40° or more of volar angulation) or in fractures with significant rotational deformity or displacement. - other options listed are not appropriate treatments for this injury. This injury most commonly results from "man-versus-wall" pugilistics, but other mechanisms of injury are possible.

Which one of the following is found most consistently in patients diagnosed with irritable bowel syndrome? (check one) A. Passage of blood per rectum B. Passage of mucus per rectum C. Abdominal pain D. Constipation E. Diarrhea

C. Abdominal pain abdominal pain as the most consistent feature found in irritable bowel syndrome (IBS), and its absence makes the diagnosis less likely.

A 62-year-old female with type 2 diabetes mellitus routinely has fasting blood glucose levels in the 80-100 mg/dL range and her hemoglobin A1c level is 7.8%. She has been diligently monitoring her blood glucose levels and all are acceptable with the exception of elevated bedtime readings. She currently is on insulin glargine (Lantus), 18 U at night. Which one of the following changes would be most appropriate for this patient? (check one) A. Adding rapid-acting insulin at breakfast B. Adding rapid-acting insulin at lunch C. Adding rapid-acting insulin at dinner D. Increasing the nightly insulin glargine dose E. Increasing the insulin glargine dosage and giving two-thirds in the morning and one-third at night

C. Adding rapid-acting insulin at dinner - addition of a rapid-acting insulin at dinner would be the next step in management. - For patients exhibiting blood glucose elevations before dinner, the addition of rapid-acting insulin at lunch is preferred. - For patients with elevations before lunch, rapid-acting insulin with breakfast would most likely improve glucose control. - Increasing or splitting the insulin glargine would be unlikely to improve management.

A 20-year-old patient comes to the emergency department complaining of shortness of breath. On examination his heart rate is 180 beats/min, and his blood pressure is 122/68 mm Hg. An EKG reveals a narrow complex tachycardia with a regular rhythm. Which one of the following would be the most appropriate initial treatment? (check one) A. Amiodarone (Cordarone) B. Diltiazem (Cardizem) C. Adenosine (Adenocard) D. Magnesium E. Synchronized cardioversion

C. Adenosine (Adenocard)

A 79-year-old female had a total knee replacement yesterday. She has mild dementia as a result of a stroke 10 years ago, but her dementia has been stable since then. Last night she became confused and agitated, striking out at nurses, and could not be consoled. Which one of the following would be most appropriate at this time? (check one) A. Soft restraints B. CT of the head C. Adequate pain control D. A sedating SSRI such as paroxetine (Paxil) E. Lorazepam (Ativan) intravenously as needed

C. Adequate pain control - postoperative delirium - associated with an increased mortality rate. - Reorientation and pain management are important management - Benzodiazepines, antipsychotics, antidepressants, and restraints are not helpful and may make the situation worse.

A 21-year-old primigravida at 10 weeks gestation has a negative titer for rubella. The best procedure to follow is to? (check one) A. Institute a (-globulin regimen and maintain it throughout her pregnancy B. Administer rubella vaccine after 12 weeks gestation C. Administer rubella vaccine immediately post partum D. Administer rubella vaccine 12 weeks post partum

C. Administer rubella immediately postpartum Rubella has been directly responsible for inestimable pregnancy wastage, as well as for severe congenital malformations. Identification and vaccination of unimmunized women immediately after childbirth or abortion is recommended. The use of beta-globulin to prevent viremia in nonimmune subjects exposed to rubella is not recommended. The vaccine should be avoided shortly before or during pregnancy since it is an attenuated live virus. Because of herd immunity there is a very low likelihood that this patient will be exposed to rubella.

A 26-year-old gravida 2 para 1 at 10 weeks' gestation presents to the emergency department with abdominal pain and vaginal spotting. Ultrasonography reveals an ectopic pregnancy. Her blood type is A-negative, antibody-negative. Appropriate management with regard to her Rh status includes: (check one) A. Administration of 50 µg of RHO immune globulin (RhoGAM) B. Administration of 300 µg of RhoGAM C. Administration of 50 µg of RhoGAM only if she requires laparoscopic intervention D. Administration of 300 µg of RhoGAM only if she requires laparoscopic intervention E. No RhoGAM, as it is not indicated in an Rh-negative woman with an ectopic pregnancy

C. Administration of 50 µg of RHO immune globulin (RhoGAM) - Both ectopic pregnancy and spontaneous or therapeutic abortion pose a significant risk for fetomaternal hemorrhage - Rh-negative patient needs Rhogam who is unsensitized (D antibody screen-negative prior to administration of RhoGAM) - 12 weeks or less, 50 mcg of RhoGAM is recommended; If the estimated gestational age is greater than 12 weeks, 300 µg of RhoGAM is recommended.

A 2-year-old Hispanic male with a 3-day history of nasal congestion presents with a barking cough and hoarseness. He is afebrile. The examination reveals tachypnea, inspiratory and expiratory stridor, noticeable intercostal retractions, and good color. Which one of the following is indicated? (check one) A. Albuterol syrup and the use of a humidifier B. Inhaled albuterol (Proventil, Ventolin) C. Aerosolized epinephrine and intramuscular dexamethasone D. Visualization of the epiglottis, and ceftriaxone (Rocephin)

C. Aerosolized epinephrine and intramuscular dexamethasone - laryngotracheobronchitis, or croup - In rare instances, this illness can be complicated by critical upper airway obstruction. - Humidification of inspired air is sometimes beneficial, but the child should not be sent home until improvement is demonstrated - Because this child has stridor and intercostal retractions, aerosolized epinephrine is indicated, along with intramuscular dexamethasone, and hospitalization may be required for observation and continued treatment - Antibiotics do not have a role in the treatment of viral croup - Attempted visualization of the epiglottis is not indicated since it will increase the child's anxiety and worsen the symptoms.

A 20-month-old male presents with a history of a fever up to 38.5°C (101.3°F), pulling at both ears, drainage from his right ear, and a poor appetite following several days of nasal congestion. This is his first episode of acute illness, and he has no history of drug allergies. The fever is confirmed on examination and the child is found to be fussy but can be distracted. He is eating adequately and shows no signs of dehydration. Positive findings include mild nasal congestion, a purulent discharge from the right auditory canal, and a red, bulging, immobile tympanic membrane in the left auditory canal. Which one of the following would be first-line treatment for this patient? (check one) A. Ceftriaxone (Rocephin) B. Amoxicillin/clavulanate (Augmentin) C. Amoxicillin D. Azithromycin (Zithromax) E. Penicillin VK

C. Amoxicillin This patient has acute bilateral otitis media, with presumed tympanic membrane perforation, and qualifies by any criterion for treatment with antibiotics. Amoxcillin, 80-90 mg/kg/day, should be the first-line antibiotic for most children with acute otitis media (SOR B).

A 50-year-old white female comes to you because she has found a breast mass. Your examination reveals a firm, fixed, nontender, 2-cm mass. No axillary nodes are palpable, nor is there any nipple discharge. You send her for a mammogram, and fine-needle aspiration is performed to obtain cells for cytologic examination. The mammogram is read as "suspicious" and the fine-needle cytology report reads, "a few benign ductal epithelioid cells and adipose tissue." Which one of the following would be the most appropriate next step? (check one) A. A repeat mammogram in 3 months B. Repeat fine-needle aspiration in 3 months C. An excisional biopsy of the mass D. Referral for breast irradiation E. Referral to a surgeon for simple mastectomy

C. An excisional biopsy of the mass - In the ideal setting, the accuracy of fine-needle aspiration may be over 90%. - Although the report from the mammogram and the biopsy are not ominous in this patient, they do not explain the clinical findings - Immediate repeat fine-needle aspiration or, preferably, a tissue biopsy is indicated

The preferred antibiotic treatment for community-acquired pneumonia in a young adult in the ambulatory setting is: (check one) A. trimethoprim/sulfamethoxazole (Bactrim, Septra) B. cephalexin (Keflex) C. azithromycin (Zithromax) D. penicillin V E. ciprofloxacin (Cipro)

C. Azithromycin (Zithromax) - covers the *atypical organism Mycoplasma pneumoniae*, most common causes of community-acquired pneumonia. - Certain fluoroquinolones such as levofloxacin also cover atypical causes, but ciprofloxacin does not - The other antibiotics listed are also ineffective against Mycoplasma.

A 2-year-old male is brought to your office for a well child examination. Developmental screening reveals that he has about a 10-word vocabulary. His mother attributes this to their bilingual home but admits she is concerned about autism. Which one of the following behaviors would provide additional evidence that the child may have autism? (check one) A. Use of gestures rather than words to communicate ideas B. Frequently being engrossed in pretend play with dolls C. Becoming upset by normal noises D. Seemingly excessive attempts to attract attention with his behavior E. Repeated copying of parental facial expressions

C. Becoming upset by normal noises Delayed social development is typically the first sign of autism. Language delay can be another finding, but it is less specific. - Of the behaviors listed, only abnormal sensitivity to sound is consistent with autism. Gesturing, pretend play, mimicking, and attempting to attract caregiver attention all suggest other diagnoses.

A 35-year-old female asks you about options for weight loss. She weighs 104 kg (229 lb) and has a BMI of 34 kg/m2. Her health problems include hypertension and depression. According to the U.S. Preventive Services Task Force, which one of the following is the most appropriate initial recommendation for weight-loss management in this patient? (check one) A. A high-protein diet B. A low-carbohydrate diet C. Behavioral counseling D. Bariatric surgery E. Polypeptides

C. Behavioral counseling the most effective behavioral counseling interventions for obesity management were comprehensive and of high intensity (12-26 sessions in a year) and involved multiple behavioral management activities, such as group and individual sessions, setting weight-loss goals, addressing barriers to change, and active use of self-monitoring.

A 23-year-old female sees you for the first time for a routine health maintenance evaluation. She tells you that her father just had a "heart valve replacement" at age 47. On examination you note a harsh 3/6 systolic murmur at the right upper sternal border. She feels well and her exercise tolerance is normal. Her history indicates that she has been well throughout her life and received appropriate childhood vaccinations and care for routine illnesses. She denies tobacco, alcohol, and drug use now and in the past. Her blood pressure today is 132/84 mm Hg. You are concerned about aortic valve disease and order an echocardiogram for further evaluation. Which one of the following is the most likely cause of aortic valve disease in this patient? (check one) A. Hypertension B. Endocarditis C. Bicuspid aortic valve D. Rheumatic heart disease E. Coronary atherosclerosis

C. Bicuspid aortic valve - most common congenital heart defect in the United States 1%-2%. - autosomal dominant pattern of inheritance and incomplete penetrance - children of a patient with a bicuspid aortic valve have about a 10% chance of having it, get an echo - will eventually have significant aortic valve dysfunction (stenosis or insufficiency) and/or aortopathy such as aortic root dilation.

A 56-year-old male with diabetes mellitus, hypertension, and chronic renal insufficiency presents for follow-up of his chronic medical conditions. Results of his most recent metabolic panel included an estimated glomerular filtration rate of 30 mL/min/1.73 m2 (N >60) and a calcium level of 10.4 mg/dL (N 8.5-10.2). Medication reconciliation reveals he is not taking the sevelamer (Renagel, Renvela) prescribed by the consulting nephrologist. You explain to the patient that he should be taking sevelamer to lower his serum calcium. The drug accomplishes this by? (check one) A. Blocking the effect of parathyroid hormone B. Blocking excessive vitamin D levels, thus decreasing intestinal calcium absorption and increasing renal calcium excretion C. Blocking intestinal absorption of phosphate, which lowers parathyroid hormone secretion D. Directly blocking excessive calcium absorption in the intestines E. Directly increasing the renal excretion of both calcium and phosphate

C. Blocking intestinal absorption of phosphate - Sevelamer is a newer synthetic agent in the therapeutic class of phosphate binders, which includes calcium acetate - Decreasing serum phosphate lowers the feedback stimulation of parathyroid hormone secretion by the parathyroid gland, which is often excessive in chronic renal insufficiency. Normalizing parathyroid levels improves serum calcium levels.

According to the Diagnostic and Statistical Manual of Mental Disorders, 5th edition (DSM-5), the severity of anorexia nervosa is based on which one of the following? (check one) A. Refusal to eat B. The frequency of episodes of binge eating or purging behavior C. Body mass index (BMI) D. The presence or absence of amenorrhea E. Orthostatic changes in pulse or blood pressure

C. Body mass index (BMI)

You make a diagnosis of depression in a 26-year-old female. Her BMI is 32 kg/m² and she has been trying to lose weight. Which one of the following antidepressants would be LEAST likely to cause her to gain weight? (check one) A. Mirtazapine (Remeron) B. Amitriptyline C. Bupropion (Wellbutrin) D. Paroxetine (Paxil) E. Citalopram (Celexa)

C. Bupropion (Wellbutrin) - can help weight loss - Among SSRIs, paroxetine is associated with the most weight gain and fluoxetine with the least. - Mirtazapine has been associated with more weight gain than the SSRIs.

An appendectomy and cholecystectomy were performed in the past, and the patient has since been free of gastrointestinal disease. On physical examination his blood pressure is 100/80 mm Hg, pulse 100 beats/min and regular, and temperature 37.0°C (98.6°F). He has mild jugular venous distention and crackles at both lung bases. Examination of his heart is unremarkable, although there is 1+ dependent edema. His abdomen is diffusely tender without masses or organomegaly. Findings on a rectal examination are normal. The results of routine laboratory tests, including a CBC, chemistry profile, EKG, and urinalysis, are all normal. The stool examination shows numerous white blood cells. Of the following, the most likely diagnosis is: (check one) A. viral gastroenteritis B. Clostridium difficile colitis C. ulcerative colitis D. gluten-sensitive enteropathy (celiac sprue) E. digoxin toxicity

C. C. diff colitis - *semiformed stool* rather than watery, fecal leukocytes (not seen in viral gastroenteritis or sprue), and a hospital stay greater than 2 weeks. - used to be associated with antibiotic use; now no abx is needed - primary sources for infection in such cases have been toilets, bedpans, floors, and the hands of hospital personnel - Prompt recognition and treatment is essential to prevent patient relapse and to minimize intramural epidemics Not ulcerative colitis - contains blood and occurs intermittently over a protracted course Not Digoxin toxicity - abnormal EKG & lab - esp. potassium high or low

Which one of the following is the best radiographic test for confirming the diagnosis of renal colic? (check one) A. A KUB radiograph B. Ultrasonography C. CT D. Intravenous pyelography E. MRI

C. CT - renal colic. Noncalcium stones may be missed by plain radiography but visualized by CT. MRI is a poor tool for visualizing stones.

In patients with type 2 diabetes mellitus, intensive glycemic control has not been shown to be beneficial for which one of the following diabetic complications? (check one) A. Peripheral neuropathy B. Foot infections C. Cardiovascular disease D. Proliferative retinopathy E. Nephropathy

C. CVD - only in Type 1, not Type 2 diabetes - if HbA1c <6.5, harmful to certain population e.g. elderly & heart pt - Intensive management helps reduce microvascular complications such as retinopathy, nephropathy, and neuropathy. - Foot infections are less common in patients without neuropathy and in patients with good glycemic control

A 54-year-old male presents to the emergency department with an acute onset of chest pain. His cardiac risk factors include hypertension, hyperlipidemia, and a positive family history. His temperature is 37.0°C (98.6°F), pulse rate 80 beats/min, blood pressure 155/86 mm Hg, and respiratory rate 22/min. His oxygen saturation is 95% on room air. An EKG shows rare unifocal PVCs and nonspecific ST-T-wave changes. Initial cardiac markers are negative. Which one of the following would be most appropriate at this point? (check one) A. Helical (spiral) CT of the chest B. Echocardiography C. PA and lateral chest films D. A ventilation-perfusion scan E. Magnetic resonance angiography

C. CXR - still valuable in the early evaluation of patients with chest pain - While they do not confirm or rule out the presence of myocardial ischemia, other causes of chest pain may be evident, such as pneumothorax, pneumonia, or heart failure. - The chest film may also provide clues about other possible diagnoses, such as pulmonary embolism, aortic disease, or neoplasia. - other tests listed often have a role in the evaluation of chest pain, but none has supplanted the plain chest film as the best initial imaging study.

An 11-year-old male is brought to your office for evaluation of bilateral posterior heel pain that has occurred for the past few months. He plays basketball and soccer several times a week and the pain begins several minutes into each of these activities. There is no pain at rest or with walking. The patient has not noticed any numbness, tingling, or weakness. On examination you find no swelling or tenderness of the heel or Achilles tendon. Reflexes, strength, and range of motion at the ankle are intact, but he does have bilateral posterior heel pain when you passively dorsiflex the ankles. Which one of the following is the most likely diagnosis? (check one) A. Achilles tendinopathy B. Calcaneal apophysitis C. Plantar fasciitis D. Heel pad syndrome E. Tarsal tunnel syndrome

C. Calcaneal apophysitis - Correct aka Sever disease - most common etiology of heel pain in children between 5 and 11 years of age - in these children the bones grow faster than the muscles and tendons - A tight Achilles tendon then pulls on its insertion site at the posterior calcaneus with repetitive running or jumping activities, causing microtrauma to the area - may have swelling and tenderness in this area - passive dorsiflexion may increase the pain - Radiography is usually normal a - Treatment involves decreasing pain-inducing activities, anti-inflammatory or analgesic medication if needed, ice, stretching and strengthening of the gastrocnemius-soleus complex, and the use of orthotic devices. Not Plantar fasciitis and heel pad syndrome - plantar surface of the heel rather than posteriorly. Not Achilles tendinopathy - tenderness to palpation of the Achilles tendon. Not Tarsal tunnel syndrome - compression of the posterior tibial nerve causes neuropathic pain and numbness in the posteromedial ankle and heel.

A 74-year-old male presents with a 4-day history of diarrhea that he had initially thought was "a 24-hour virus." He states that the onset of his illness included nausea, one episode of vomiting, and profuse diarrhea. He has felt feverish and has been having abdominal cramps. He does not recall eating anything unusual and has not traveled recently. On examination he appears uncomfortable, but in no real distress. His oral temperature is 37.1°C (98.8°F), blood pressure 134/82 mm Hg, and pulse rate 100 beats/min. He has lost 4 kg (9 lb) since his last visit 2 months earlier. His abdomen is soft, with hyperactive bowel sounds and mild diffuse tenderness on palpation. A CBC and basic metabolic profile are normal. Which one of the following is the most likely cause of this patient's illness? (check one) A. Norwalk-like virus (Norovirus) B. Shigella C. Campylobacter D. Escherichia coli O157:H7 E. Staphyloccocus aureus

C. Campylobacter jejuni - most common causes of bacterial foodborne illnesses,affect 1 million Americans annually - Undercooked or improperly handled chicken - infection generally isolated and sporadic, occurs more frequently at the extremes of age, affects males disproportionately - Symptoms typically begin 2-5 days following exposure. - Diarrhea is the predominant symptom, with a lesser degree of nausea and vomiting. Up to 10 days is required for full recovery.

A 20-year-old male presents with a complaint of pain in his right testis. The onset of pain has been gradual and has been associated with dysuria and urinary frequency. The patient has no medical problems and is sexually active. On examination he has some swelling and mild tenderness of the testis. The area posterior to the testis is swollen and very tender. He has a normal cremasteric reflex, and the pain improves with elevation of the testicle. Which one of the following would be the most appropriate management of this patient? (check one) A. Surgical evaluation B. Doppler ultrasonography C. Ceftriaxone (Rocephin) and doxycycline D. Levofloxacin (Levaquin) E. Ciprofloxacin (Cipro)

C. Ceftriaxone (Rocephin) and doxycycline - epididymitis. - In males 14-35 years of age - the most common causes are Neisseria gonorrhoeae and Chlamydia trachomatis. - treatment is ceftriaxone, 250 mg intramuscularly, and doxycycline, 100 mg twice daily for 10 days (SOR C). - A single 1-g dose of azithromycin may be substituted for doxycycline. - treatment under 14 or over age 35 - the infection is usually caused common urinary tract pathogens, and levofloxacin, 500 mg once daily for 10 days, would be the appropriate treatment (SOR C).

A 14-year-old male presents to your office with a high fever that began suddenly. He has a diffuse petechial rash and some nuchal rigidity on examination. A lumbar puncture is performed, and gram-negative diplococci are found. You admit him to the hospital for treatment. Which one of the following would be most appropriate for prevention of secondary disease at this time? (check one) A. Immediate chemoprophylaxis for his entire school B. Immediate vaccination of all contacts C. Chemoprophylaxis for family members and very close contacts only D. Isolation of all family members for 1 week E. No preventive measures until culture results are available

C. Chemoprophylaxis for family members and very close contacts only The risk for secondary disease among close contacts is highest during the first few days after the onset of illness in the index patient, mandating immediate chemoprophylaxis of those exposed.

A 54-year-old male presents to your office with a 10-day history of increasing cough. A physical examination reveals coarse crackles in the left lower lobe. You make a diagnosis of pneumonia. The patient's only current medication is simvastatin (Zocor). Which one of the following is CONTRAINDICATED in this patient? (check one) A. Amoxicillin/clavulanate (Augmentin) B. Azithromycin (Zithromax) C. Clarithromycin (Biaxin) D. Doxycycline E. Levofloxacin (Levaquin)

C. Clarithromycin - In older adults, coprescription of clarithromycin or erythromycin with a statin that is metabolized by CYP 3A4 (atorvastatin, simvastatin, lovastatin) increases the risk of statin toxicity. The other antibiotics listed do not interact with statins

A 35-year-old male with a toothache presents to a local clinic for uninsured patients. On examination you find a decayed left lower molar that is tender when tapped lightly, and surrounding gingival inflammation and tenderness. There is no obvious mandibular swelling, but he does have a tender submandibular lymph node. The earliest available dental appointment is in 1 week. He is allergic to penicillin. Which one of the following would be the best antibiotic treatment for this patient? (check one) A. Doxycycline B. Trimethoprim/sulfamethoxazole (Bactrim, Septra) C. Clindamycin (Cleocin) D. Ciprofloxacin (Cipro) E. Cephalexin (Keflex)

C. Clindamycin - periodontitis of the tooth's roots with cellulitis, complicated by an apical abscess. - *anaerobic oral bacteria*. - Penicillin VK, amoxicillin or amoxicillin/clavulanate is preferred for antibiotic treatment - Pt is allergic to penicillin - Clindamycin is a good choice to cover the likely pathogens.

A 27-year-old male with a diagnosis of depression prefers to avoid pharmacologic treatment. You agree to engage in a trial of therapy in your office. During the treatment process, you help the patient realize that some of his perceptions and interpretations of reality may be false and lead to negative thoughts. Next, you help him discover alternative thoughts that reflect reality more closely, and to learn to discard his previous distorted thinking. By learning to substitute healthy thoughts for negative thoughts, he finds his mood, behavior, and physical reaction to different situations are improved. Which one of the following best categorizes this type of therapy? (check one) A. Psychoanalysis B. Biofeedback C. Cognitive therapy D. Group psychotherapy E. Hypnosis therapy

C. Cognitive therapy - helps patients correct false self-beliefs that can lead to negative moods and behaviors - shown to effectively treat patients with unipolar major depression, and is particularly useful in patients who do not respond to medication or who prefer nonpharmacologic therapy. Not Psychoanalysis - process of free association where repressed memories are recovered. Not Biofeedback - involves instrumentation that gives feedback about a patient's physiologic response to various situations in order to bring the autonomic nervous system under voluntary control. Not Group psychotherapy - people who are emotionally ill meet in a group guided by a trained therapist and help one another effect personality change. Not Hypnosis - patient enter a state of heightened focal concentration and receptivity

A 14-year-old female bumped heads with another player in a soccer game. She was knocked down, appeared briefly dazed, and now has a headache and mild dizziness while seated on the sidelines. Which one of the following would be most appropriate at this point? (check one) A. Return to play after symptoms have resolved for at least 30 minutes B. Immediate neuroimaging to rule out intracranial injury C. Complete cognitive and physical rest for 24 hours before returning to normal activities D. Initial complete cognitive and physical rest followed by an individualized graded return to activity E. No sports participation until symptoms have been absent for 1 week 21

C. Complete cognitive and physical rest for 24 hours before returning to normal activities mild concussion without loss of consciousness: standard neuroimaging can be expected to be normal. The evaluation should include a standard concussion assessment tool, and if concussion is suspected the athlete should be removed from play. Complete physical and cognitive rest are required for the first 1-2 days, but return to normal activity must be individualized depending on the course of symptoms and response to gradually increasing activity.

During a routine prenatal visit, a patient at 28 weeks gestation describes a worsening pain in her lower back and pelvic area. She is averse to analgesics but is eager to try exercise to relieve the pain. Additional patient history and an examination confirm that the pain is not due to underlying medical problems. Which one of the following would be the most appropriate exercise prescription for this patient? (check one) A. Isometric exercise B. Concentric exercise C. Core stability exercise D. Closed kinetic chain exercise E. Isotonic exercise

C. Core exercise - The most appropriate exercises for pregnancy related pelvic pain and low back pain target the low back, trunk, and abdominal muscles to increase core stability - e.g. Pilates, back extension exercises, and abdominal crunches. Not Isometric and isotonic exercises - work muscle groups against either an external force or opposing muscle groups, and are best suited for the development of muscle tone, strength, and conditioning in the extremities Not concentric and closed kinetic chain exercises - work muscles against resistance - for rehabilitating and strengthening the extremities.

Which one of the following is a risk factor for intermittent claudication? (check one) A. Hyperthyroidism B. Hypercalcemia C. Diabetes mellitus D. Hypogonadism E. Elevated angiotensin-converting enzyme

C. Diabetes mellitus; also - cigarette smoking - hypertension - dyslipidemia Not elevation of angiotensin-converting enzyme - sarcoidosis.

The mother of a 16-year-old male calls to report that her son has a severe sore throat and has been running a fever of 102°F. Which one of the following additional findings would be most specific for peritonsillar abscess? (check one) A. A 1-day duration of illness B. Ear pain C. Difficulty opening his mouth D. Hoarseness E. Pain with swallowing

C. Difficulty opening his mouth - Trismus (universally present with peritonsillar abscess) - Peritonisillar abscess: usu. has 3 days hx of sore throat - voice changes, otalgia, and odynophagia may or may not be present; also they can be from other disease e.g. Pharyngotonsillitis and peritonsillar cellulitis - Otalgia: common with peritonsillar abscess, otitis media, temporomandibular joint disorders, and a variety of other conditions

Of the following, which one is the most common adverse event to complicate the hospital course of patients age 65 and over? (check one) A. Falls B. Wound infections C. Drug-related events D. Procedure-related events E. Anesthesia-related events

C. Drug-related problems are the most common type of adverse event - 11.46 for drug-related events - 6.15 for wound infection, - 3.85 for procedure-related events - 3.19 for falls - 0.09 for anesthesia-related events.

A 52-year-old male presents with a small nodule in his palm just proximal to the fourth metacarpophalangeal joint. It has grown larger since it first appeared, and he now has mild flexion of the finger, which he is unable to straighten. He reports that his father had similar problems with his fingers. On examination you note pitting of the skin over the nodule. The most likely diagnosis is: (check one) A. degenerative joint disease B. trigger finger C. Dupuytren's contracture D. a ganglion E. flexor tenosynovitis

C. Dupuytren's contracture - characterized by changes in the palmar fascia, with progressive thickening and nodule formation - can progress to a contracture of the associated finger - The fourth finger is most commonly affected - Pitting or dimpling can occur over the nodule because of the connection with the skin. Degenerative joint disease - no palmar nodule Trigger finger & Ganglions - affect tendon, not the palmar fascia, and causes the finger to lock and release Flexor tenosynovitis - an inflammation, is associated with pain, which is not usually seen with Dupuytren's contracture.

A 42-year-old female presents to the emergency department with pleuritic chest pain. Her probability of pulmonary embolism is determined to be low. Which one of the following should be ordered to further evaluate this patient? (check one) A. Brain natriuretic peptide (BNP) B. CT pulmonary angiography C. ELISA-based D-dimer D. A cardiac troponin level E. A ventilation-perfusion lung scan

C. ELISA-based D-dimer Patients who have a low or moderate pretest probability of pulmonary embolism should have d-dimer testing as the next step in establishing a diagnosis.

A severely depressed 77-year-old male is hospitalized after an intentional drug overdose. He was found by chance when his housekeeper returned to retrieve something she had left behind. The patient has been severely depressed since he suffered a myocardial infarction 1 year ago, and the recent death of his wife has increased his despondency. He had left a note apologizing to his family and his physician, who has treated him with multiple medications for depression over the past year. He has been treated with SSRIs, SNRIs, and atypical antipsychotics in high doses and in various combinations without significant improvement. Which one of the following would be most likely to improve this patient's depression at this point? (check one) A. Cognitive-behavioral therapy B. Psychoanalysis C. Electroconvulsive therapy D. Goal-directed psychotherapy E. Limbic stimulation

C. Electroconvulsive therapy

A 53-year-old female is concerned about a skin lesion that has recently been changing in size and shape. On examination she is found to have a 7-mm, asymmetric, darkly pigmented lesion with some color variegation and irregular borders. Which one of the following skin biopsy techniques is most appropriate for confirming the diagnosis? (check one) A. A shave biopsy B. Electrodesiccation and curettage C. Elliiptical excision D. Mohs surgery .

C. Elliiptical excision - based on the asymmetry, irregular border, color variegation, and size larger than 6 mm. - preferred method of biopsy for any lesion suspicious for melanoma is complete elliptical excision with a small margin of normal-appearing skin

A 45-year-old male is seen for a well-demarcated, nonpruritic rash in the right axilla. It is fine-scaled with a cigarette-paper appearance. The rash has a coral-red fluorescence under a Wood's light. Which one of the following is the most likely diagnosis? (check one) A. Candidiasis B. Tinea cruris C. Erythrasma D. Inverse psoriasis

C. Erythrasma: fluoresces with Wood's light. - superficial gram-positive bacterial infection caused by Corynebacterium minutissimum. - fluorescence is caused by porphyrins. - often between the toe web spaces, followed by the groin and axillae. - treat w/ oral erythromycins and clindamycins

An 80-year-old female is admitted to your service at a skilled nursing facility 5 days after repair of a hip fracture. When you review her records you note that she has not received any previous treatment for osteoporosis. You are considering ordering zoledronic acid (Reclast) to reduce her risk of another fracture. Which one of the following should be evaluated before administering zoledronic acid to this patient? (check one) A. Vitamin D levels B. Liver enzyme levels C. Estimated glomerular filtration rate D. A CBC

C. Estimated glomerular filtration rate Contraindications to zoledronic acid include hypocalcemia and a creatinine clearance <35 mL/min or other evidence of acute renal impairment.

A 4-year-old female has had three urinary tract infections in the past 6 months. She complains of difficulty with urination and on examination is noted to have labial adhesions that have resulted in near closing of the introitus. Which one of the following is the most appropriate management? (check one) A. No treatment at this time B. Reporting your suspicion of child abuse to the appropriate authorities C. Application of estrogen cream to the site D. Gentle insertion of progressively larger dilators over a period of several days E. Referral to a gynecologist for surgical correction

C. Estrogen cream - prepubertal labial adhesions - idiopathic - may be partial or complete

A 78-year-old male presents for a routine follow-up visit for hypertension. He is a smoker, but has no known coronary artery disease and is otherwise healthy. On examination you note an irregular pulse. An EKG reveals multiple premature ventricular contractions (PVCs), but no other abnormalities. Current guidelines recommend which one of the following? (check one) A. Amiodarone (Cordarone) for suppression of PVCs B. Flecainide (Tambocor) for suppression of PVCs C. Evaluation for underlying coronary artery disease D. No further evaluation or treatment

C. Evaluation for underlying coronary artery disease

A 16-year-old male is brought to your office by his mother for "stomachaches." On the review of systems he also complains of headaches, occasional bedwetting, and trouble sleeping. His examination is within normal limits. His mother says that he is often in the nurse's office at school, and doesn't seem to have any friends. When you discuss these problems with him, he admits to being teased and called names at school. Which one of the following would be most appropriate? (check one) A. Explain that he must try to conform to be more popular B. Explain that these symptoms are a stress reaction and will lessen with time C. Explore whether his school counselor has a process to address this problem D. Order a TSH level

C. Explore whether his school counselor has a process to address this problem

In a patient with microcytic anemia, which one of the following patterns of laboratory abnormalities would be most consistent with iron deficiency as the underlying cause? (check one) A. Ferritin low, total iron binding capacity (TIBC) low, serum iron low B. Ferritin low, TIBC low, serum iron high C. Ferritin low, TIBC high, serum iron low D. Ferritin high, TIBC low, serum iron low

C. Ferritin low, TIBC high, serum iron low - Ferritin and serum iron levels fall with iron deficiency. - Total iron binding capacity rises, indicating a greater capacity for iron to bind to transferrin (the plasma protein that binds to iron for transport throughout the body) when iron levels are low.

A 53-year-old male presents to your office with a several-day history of hiccups. They are not severe, but have been interrupting his sleep, and he is becoming exasperated. What should be the primary focus of treatment in this individual? (check one) A. Drug treatment to prevent recurrent episodes B. Decreasing the intensity of the muscle contractions in the diaphragm C. Finding the underlying pathology causing the hiccups D. Improving the patient's quality of sleep E. Suppressing the current hiccup symptoms

C. Finding the pathology of hiccup - respiratory reflex that originates from the phrenic and vagus nerves, as well as the thoracic sympathetic chain. - Hiccups lasting hours: self-limiting - treatment: interrupting the reflex loop of the hiccup e.g. mechanical means (e.g., stimulating the pharynx with a tongue depressor) or medical treatment w/ chlorpromazine i - Hiccups lasting over 2 days: may be an underlying pathology causing the hiccups - must treat pathology

Which one of the following is the preferred treatment for patients with obsessive-compulsive disorder? (check one) A. Lithium carbonate B. Alprazolam (Xanax) C. Fluoxetine (Prozac) D. Amitriptyline (Elavil) E. Valproic acid (Depakene)

C. Fluoxetine (Prozac) - SSRIs such as fluoxetine and fluvoxamine are FDA-approved and considered first-line agents in the treatment of this condition.

A 49-year-old male brings you a copy of his laboratory results obtained during an insurance examination. The patient says he feels fine, but his bilirubin level was 2.5 mg/dL (N <1.0). He says he averages 5 alcoholic beverages per week and takes no medications other than occasional ibuprofen. On examination he is not jaundiced and has no scleral icterus, and the remainder of the examination is within normal limits, including palpation of the liver and spleen. Laboratory testing reveals a normal CBC, normal liver enzyme levels, and normal serum haptoglobin. Bilirubin fractionation reveals an indirect level of 2.0 mg/dL and a direct level of 0.5 mg/dL (N <0.4). The most likely diagnosis is (check one) A. asymptomatic cholecystitis B. alcoholic liver disease C. Gilbert's syndrome D. hemolytic anemia

C. Gilbert's syndrome Gilbert's syndrome is a hereditary condition associated with unconjugated hyperbilirubinemia (usually with a bilirubin level <5.0 mg/dL). The bilirubin level increases with infection, exertion, and fasting. Patients are asymptomatic and have otherwise normal liver function studies. Not hemolytic anemias, which cause a decrease in serum haptoglobin, an increase in lactate dehydrogenase and/or CBC abnormalities, particularly on the peripheral smear.

A 75-year-old African-American female is diagnosed with macular degeneration. She is being treated for type 2 diabetes mellitus, hypothyroidism, hypertension, hypercholesterolemia, and gout. Which one of her conditions is associated with macular degeneration? (check one) A. Type 2 diabetes mellitus B. Hypothyroidism C. Hypertension D. Gout

C. HTN- Age-related macular degeneration - most common cause of blindness in the older population - more frequently in light-skinned individuals than in dark-skinned individuals - Risk factors include smoking and hypertension.

Routine vaccination against which one of the following organisms has significantly reduced the risk of bacterial meningitis among young children? (check one) A. Borrelia burgdorferi B. Escherichia coli C. Haemophilus influenzae D. Listeria monocytogenes

C. Haemophilus influenzae

A 6-year-old white male visits your office with chief complaints of a recent onset of fever, bilateral knee and ankle pain, colicky abdominal pain, and rash. On examination, his temperature is 38.3 degrees C (101.0 degrees F), and there is a prominent palpable reddish-brown rash on the buttocks and thighs. There is pain on motion of his knees and ankles, and mild diffuse abdominal tenderness. The stool is positive for occult blood. Laboratory Findings Hemoglobin 11.0 g/dL (N 11.5-13.5) Hematocrit 33% (N 34-40) WBCs 14,500/mm3 (N 5500-15,000); 85% segs, 15% lymphs Platelets 345,000/mm3 (N 150,000-400,000) Prothrombin time 12 sec (N 11-15) Which one of the following is the most likely diagnosis? (check one) A. Systemic onset juvenile rheumatoid arthritis B. Rocky Mountain spotted fever C. Henoch-Schonlein purpura D. Disseminated anthrax E. Acute iron ingestion

C. Henoch-Schonlein purpura - typically follows an upper respiratory tract infection, and presents with low-grade fever, fatigue, arthralgia, and colicky abdominal pain. -*pink maculopapules* rash progresses to *petechiae or purpura* - rash are palpable - Arthritis, usually involving the knees and ankles - gastrointestinal tract involvement results in heme-positive stools in 50% of cases. - Laboratory findings are not specific or diagnostic, and include indications of mild to moderate thrombocytosis, leukocytosis, and anemia, and an elevated erythrocyte sedimentation rate. - Treatment: corticosteroids are indicated in the rare patient with life-threatening gastrointestinal or central nervous system manifestations Not Systemic juvenile-onset rheumatoid arthritis - evanescent salmon-pink rash. Not Rocky Mountain spotted fever - no arthritis and the rash begins distally on the legs. Not Iron ingestion - no rash, no fever, no arthritis Not Disseminated anthrax - no rash and joint symptoms

A 55-year-old female has severe symptoms of gastroesophageal reflux disease. Upper endoscopy with a biopsy shows severe esophagitis and Barrett's esophagus. Which one of the following is true regarding this patient? (check one) A. The severity of her symptoms is due to the presence of Barrett's esophagus B. Follow-up screening endoscopy will reduce her risk of death from esophageal cancer C. Her risk of developing esophageal adenocarcinoma is >90% D. Her risk of developing esophageal adenocarcinoma is <1%

C. Her risk of developing esophageal adenocarcinoma <1% The actual risk of adenocarcinoma from Barrett's esophagus is less than 1%. Endoscopy does nothing to reduce the risk of death. Patients with Barrett's esophagus can have minimal symptoms.

A mother meets you in the emergency department with her 3-week-old infant. The infant was delivered at term, with an uneventful prenatal and postnatal course to this point. The mother reports that the infant stopped breathing for 20-25 seconds, and that his lips and tongue appeared bluish. There was no coughing, choking, or congestion, but the child seemed "limp." The episode ended when the mother vigorously stimulated her child and he started crying. On examination, the child appears normal. Which one of the following would be most appropriate at this point? A. Reassurance and no further evaluation B. Discharge with a home apnea monitor C. Hospital admission for observation D. Mandatory referral to child protective services E. Direct laryngoscopy to rule out a foreign body

C. Hospital admission for observation Hospital admission is not always necessary for a short, self-correcting episode associated with feeding. Given the history of not breathing for 20-25 seconds, having a blue tongue and lips, and being limp, admitting the child for observation is appropriate. - referral to child protective services is not mandatory. - Laryngoscopy would not be routine - Many groups recommend home apnea monitoring after discharge for patients with more severe or undiagnosed cases.

You are asked to medically manage a 66-year-old patient who is scheduled for an elective cholecystectomy. He is also being treated for panhypopituitarism secondary to a pituitary macroadenoma resection many years ago. His medications include levothyroxine (Synthroid), 125 :g/day; prednisone, 10 mg in the morning and 5 mg in the evening; and fludrocortisone, 10 mg/day. Preoperative orders for this patient should include which one of the following? (check one) A. Normal saline intravenously as a bolus B. ACTH daily while on intravenous fluids C. Hydrocortisone, 25 mg intravenously every 8 hr D. Levothyroxine, 250 :g intravenously daily

C. Hydrocortisone, 25 mg intravenously every 8 hr A patient with known adrenal insufficiency secondary to hypopituitarism who is undergoing a period of stress such as illness or surgery should be given intravenous corticosteroids. For moderate-risk procedures such as vascular or orthopedic operations, 50 mg of hydrocortisone is recommended x 8 hrs For major surgery, such as open heart surgery or an esophagectomy, 100 mg of hydrocortisone would be needed x 8 hrs

A 35-year-old white male presents to the emergency department with chest pain of 30 minutes duration. He describes the pain as feeling like pressure on his chest, and says it radiates into his left arm. It is accompanied by dyspnea, diaphoresis, anxiety, and palpitations. His past medical history is unremarkable and he has no family history of premature heart disease. He smokes 2 packs of cigarettes per day and *admits to intranasal cocaine* use 2 hours ago. Vital signs include a blood pressure of 180/110 mm Hg, a pulse rate of 110 beats/min, a respiratory rate of 24/min, and a temperature of 37.2°C (99.0°F). Other than the anxiety and diaphoresis, the general examination is unremarkable. An EKG shows sinus tachycardia with an early repolarization pattern. Aspirin and nitroglycerin have been administered, as well as oxygen via nasal cannula. Which one of the following would be most appropriate at this point? (check one) A. Nifedipine (Procardia) B. Enalapril intravenously C. Lorazepam (Ativan) intravenously D. Metoprolol (Toprol) intravenously E. Thrombolytic therapy

C. IV lorazepam (Ativan) - cocaine induced CP - hypertension, tachycardia, and chest pain will often respond to IV benzodiazepines Not beta-blocker - can exacerbate coronary artery spasm in cocaine associated chest pain Not Fibrinolytic therapy - only w/ ST elevation Not Calcium channel blocker - has not been studied - but may be considered if there is no response to benzodiazepines and nitroglycerin\ Not ACE inhibitors - does not address the tachycardia.

A 4-year-old male has a fever of 1 week's duration. It has been at or slightly above 38° C (101° F) and has responded poorly to antipyretics. The patient complains of photophobia, burning in his eyes, and a sore throat. His mother also notes that his eyes look red, his lips are red and cracked, and he has a "strawberry tongue." The child's palms and soles are erythematous and the periungual regions show desquamation of the skin. He has minimally painful nodes located in the anterior cervical region, about 2×2 cm in size. A Streptococcus screen is negative. The most appropriate management at this time would be: (check one) A. Intramuscular benzathine penicillin G (Bicillin L-A), 600,000 U B. Intravenous nafcillin (Nallpen) C. Intravenous immune globulin and aspirin D. Prednisone, 2-3 mg/kg daily E. A fine-needle biopsy of the lymph nodes

C. IVIG and ASA - Kawasaki Disease - most common under 5 y/o - aka mucocutaneous lymph node syndrome - common form of vasculitis in childhood - self-limited, with fever and acute inflammation lasting 12 days on average without therapy. - if untreated, this illness can result in heart failure, coronary artery aneurysm, myocardial infarction, arrhythmias, or occlusion of peripheral arteries. - Diagnose: fever + 1. Conjunctivitis (bilateral, non purulent) - exudative 2. Rash ( truncal, non-specific) - erythematous polymorphous rash 3. Adenopathy (at least 1 cervical node > 1 cm) 4. Strawberry tongue (or any change in oropharyngeal mucosa) 5. Hand, feet swelling/desquamation. Treatment: hospitalize the patient Not prednisone - unsafe in Kawasaki disease, as a previous study showed an extraordinarily high rate of coronary artery aneurysm with its use

A 41-year-old male trips on a curb while running, sustaining an inversion ankle injury. According to the Ottawa ankle rules, which one of the following would be an indication for radiographic evaluation? (check one) A. Tenderness at the anterior talofibular ligament B. Point tenderness over the cuboid C. Inability to take four steps either immediately after the injury or while in your office D. Bony tenderness at the anterior aspect of the distal tibia E. Point tenderness over the base of the fourth metatarsal

C. Inability to take 4 steps either immediately after or while in your office - Ottawa ankle rule Radiographs should be obtained for all patients with an acute ankle injury who meet any of the following criteria: 1. inability to take four steps, either immediately after the injury or when being evaluated 2. localized tenderness of the navicular bone or the base of the fifth metatarsal 3. localized tenderness at the posterior edge or tip of either malleolus.

Which one of the following is the major mechanism of action of metformin (Glucophage)? (check one) A. Stimulation of pancreatic insulin release B. Inhibition of glucose production by the liver C. Inhibition of carbohydrate absorption in the small intestine D. Improved insulin sensitivity of skeletal muscle

C. Inhibition of carbohydrate absorption in the small intestine Metformin - multiple mechanisms of action, but its main effect on serum glucose results from inhibition of gluconeogenesis in the liver. Sulfonylureas and meglitinides - stimulate insulin release from the pancreas thiazolidinediones - peripheral tissues to insulin "-glucosidase inhibitors. Carbohydrate absorption in the small intestine is inhibited by the

A 47-year-old female presents to your office with a complaint of hair loss. On examination she has a localized 2-cm round area of complete hair loss on the top of her scalp. Further studies do not reveal an underlying metabolic or infectious disorder. Which one of the following is the most appropriate initial treatment? (check one) A. Topical minoxidil (Rogaine) B. Topical immunotherapy C. Intralesional triamcinolone (Kenalog) D. Oral finasteride (Proscar) E. Oral spironolactone (Aldactone)

C. Intralesional triamcinolone (Kenalog) - alopecia areata - localized autoimmune reaction to hair follicles - occasionally spreads to involve the entire scalp (alopecia totalis) or the entire body (alopecia universalis) - Spontaneous recovery usually occurs within 6-12 months, although areas of regrowth may be pigmented differently - Recovery is less likely if the condition persists for longer than a year, worsens, or begins before puberty Intralesional corticosteroid injection - older than 10 years of age - affects less than 50% of the scalp Not Minoxidil - alternative for children younger than 10 years of age - alopecia areata affects more than 50% of the scalp. Not topical immunotherapy - most effective treatment for chronic severe alopecia areata but severe side effects Not Finasteride - for adreogenic alopecia (male-pattern baldness) - inhibits 5 ß-reductase type 2 - resulting in a decrease in dihydrotestosterone levels, and is used in the treatment of androgenic alopecia Not spironolactone - for androgenic alopecia because it is an aldosterone antagonist with antiandrogenic effects.

A 34-year-old white mechanic felt a slight impact on his left eye while hammering on an axle 2 days ago. He has experienced some discomfort since that time, and complains of blurring of vision. Physical examination discloses no local erythema or other evidence of injury to the eye. Fluorescein staining is negative. His visual acuity is 20/40 in the affected eye. The most likely diagnosis is: (check one) A. Traumatic iritis B. Corneal abrasion C. Intraocular foreign body D. Bacterial corneal ulcer E. Retinal detachment

C. Intraocular foreign body

Over the past year, a 32-year-old white female has experienced increasing hair growth on her chin and chest, acne, and irregular menstrual periods. She takes no medications. Which one of the following would be the most appropriate course of action at this point? (check one) A. Empiric treatment with metformin (Glucophage) B. CT of the adrenal glands C. Laboratory testing D. Brain MRI E. Pelvic ultrasonography

C. Laboratory testing

Which one of the following medications should be started at a low dosage and titrated slowly to minimize the risk of Stevens-Johnson syndrome? (check one) A. Carbamazepine (Tegretol) B. Divalproex (Depakote) C. Lamotrigine (Lamictal) D. Lithium E. Ziprasidone (Geodon)

C. Lamotrigine (Lamictal) Lamotrigine is an anti-epileptic medication that is often used in bipolar disorder. It can cause Stevens-Johnson syndrome, which is a severe disorder of the skin and mucous membranes. This most commonly occurs in children or when the drug is initiated at a high dosage, and is also more likely to occur in patients taking divalproex. To decrease the risk of Stevens-Johnson syndrome, it is recommended that lamotrigine therapy be started at a dosage of 25 mg daily and titrated every 2 weeks until the goal dosage is reached.

A 13-year-old male presents with a 3-week history of left lower thigh and knee pain. There is no history of a specific injury, and his past medical history is negative. He has had no fevers, night sweats, or weight loss, and the pain does not awaken him at night. He tried out for the basketball team but had to quit because of the pain, which was worse when he tried to run. Which one of the following physical examination findings would be pathognomonic for slipped capital femoral epiphysis? (check one) A. Excessive forward passive motion of the tibia with the knee flexed B. Lateral displacement of the patella with active knee flexion C. Limited internal rotation of the flexed hip D. Reduced hip abduction with the hip flexed E. Inability to extend the hip past the neutral position

C. Limited internal rotation of flexed hip - Slipped capital femoral epiphysis (SCFE) - typically occurs in young adolescents during the growth spurt - Pain with physical activity, most commonly in the upper thigh anteriorly, but one-third of patients present with referred lower thigh or knee pain *Hallmark* of *SCFE* 1. limited internal rotation of the hip 2. even greater limitation of internal rotation when the hip is flexed to 90°

Which one of the following should be given intravenously in the initial treatment of status epilepticus? (check one) A. Propofol (Diprivan) B. Phenobarbital C. Lorazepam (Ativan) D. Midazolam (Versed)

C. Lorazepam (Ativan)

A 35-year-old male presents with acute low back pain after he spent a weekend building a storage shed in his backyard. He has no neurologic symptoms, and the pain does not radiate into either leg. Which one of the following has been shown to be a useful treatment in this situation? A) Bed rest B) Acupuncture C) Lumbar traction D) Cyclobenzaprine (Flexeril) E) Methylprednisolone (Medrol) (check one) A. Bed rest B. Acupuncture C. Lumbar traction D. Cyclobenzaprine (Flexeril) E. Methylprednisolone (Medrol)

C. Lumbar traction - Nonbenzodiazepine muscle relaxants such as cyclobenzaprine are beneficial for the relief of acute low back pain for the first 7-14 days after the onset of symptoms. - Unsupported treatment options for acute low back pain include oral corticosteroids, acupuncture, lumbar support, massage, chiropractic spinal manipulation, and traction. - bed rest is terrible

A 67-year-old male with moderate macrocytosis complains of paresthesias of his feet. If the patient has a borderline low vitamin B12 level, elevated levels of which one of the following would suggest vitamin B12 deficiency? (check one) A. Serum gastrin B. Reticulocytes C. Methylmalonic acid D. Serum ferritin E. Serum folate

C. Methylmalonic acid - also homocysteine levels Not Gastrin: high in pernicious anemia, but are not diagnostic alone. Not High ferritin levels: increased iron stores in the liver Not ferritin levels: hemochromatosis.

36-year-old obese female presents to your office with a chief complaint of amenorrhea. On examination you note hirsutism and body acne. She is on no medications and a pregnancy test is negative. Serum testosterone is at the upper limits of normal and TSH is within normal limits. In addition to weight loss and exercise, which one of the following would be the most appropriate initial management? (check one) A. High-dose combined oral contraceptives B. Progestin-only contraceptives C. Metformin (Glucophage) D. Levothyroxine (Synthroid)

C. Metformin (Glucophage) PCOS characterized by hyperandrogenism, polycystic ovaries on pelvic ultrasonography, and ovulatory dysfunction. Hyperandrogenism and either polycystic ovaries or ovulatory dysfunction are necessary to make the diagnosis. The first-line recommendation in obese patients is lifestyle modification, but metformin may improve abnormal menstruation (SOR A). Low-dose combined oral contraceptives are more frequently used to reduce the risk of endometrial cancer in patients with chronic anovulation and the resulting unopposed estrogen secretion.

A 57-year-old previously healthy menopausal female presents to your office with a 1-year history of palpitations and an unintentional 10-lb weight loss. A review of systems is negative for tremors or visual changes. Vital signs include a blood pressure of 129/85 mm Hg and a heart rate of 110 beats/min. A physical examination is otherwise unremarkable except for a nontender, diffusely enlarged thyroid with no distinct nodules, and mild proptosis. Laboratory studies are significant for a TSH level<0.01 :U/mL (N 0.60-3.30), a free T3 level of 14.51 pg/mL (N 2.0-3.5), and a free T4 level of 4.52 ng/dL (N 0.71-1.40). A thyroid-stimulating immunoglobulin test is positive. In addition to a β-blocker, which one of the following is the most appropriate initial management? (check one) A. Radioactive iodine ablation B. Thyroidectomy C. Methimazole (Tapazole) D. Propylthiouracil (PTU)

C. Methimazole (Tapazole) Methimazole and propylthiouracil (PTU) are the two oral antithyroid medications available. However, because of reports of severe hepatocellular damage, methimazole should be used instead of PTU unless it is contraindicated. Radioactive iodine treatment (131I) - if patients who do not achieve remission with antithyroid medications - :( .worsening of preexisting orbitopathy; - - :( transient increase in thyroid hormone & thyroid storm. - patients should be treated with methimazole prior to 131I administration. Thyroidectomy - pt suspected of having cancer - do not tolerate or refuse alternative forms of therapy - antithyroid medication should be given to achieve a euthyroid state prior to surgery in most patients.

A 68-year-old female presents with a 2-month history of painful, swollen wrists and knees. The pain is always present and is accompanied by stiffness in these joints for 2-3 hours every morning. Her past medical history, family history, and social history are unremarkable. She takes a daily multivitamin. A complete physical examination is notable only for symmetric, moderately swollen, slightly erythematous, and very tender wrists and knees. Range of motion is intact but increases her pain. Plain radiographs of these joints show erosions at the ulnar styloids. Lyme disease serologies are negative. Anti-cyclic citrullinated peptide (CCP) antibody testing is positive. Which one of the following would be appropriate for this patient as a sole therapy for her joint condition? (check one) A. Aspirin therapy B. Doxycycline C. Methotrexate D. Naproxen E. Prednisone

C. Methotrexate - Rheumatoid Arthritis

A 55-year-old nonsmoking African-American female with diabetes mellitus sees you for a routine visit. She has no other cardiac risk factors. Her blood pressure is 120/74 mm Hg and she has a fasting total cholesterol level of 180 mg/dL, an HDL-cholesterol level of 52 mg/dL, and an LDL-cholesterol level of 100 mg/dL. Her calculated 10-year risk of atherosclerotic cardiovascular disease is 5.8%. According to the 2013 American College of Cardiology/American Heart Association cholesterol guidelines, which one of the following is recommended for this patient? (check one) A. No statin therapy B. Low-intensity statin therapy C. Moderate-intensity statin therapy D. High-intesity statin thera

C. Moderate intensity statin 4 major groups needing *moderate intensity statin* (1) individuals with clinical atherosclerotic cardiovascular disease (ASCVD) (2) those with primary elevations of LDL >190 mg/dL (3) patients age 40-75 with diabetes mellitus, an LDL-C level of 70-189 mg/dL, and no clinical ASCVD (4) patients age 40-75 without clinical ASCVD or diabetes, an LDL-C level of 70-189 mg/dL, and an estimated 10-year ASCVD risk >7.5%. For patients age 40-75 with diabetes, an LDL-C level of 70-189 mg/dL, and no clinical ASCVD, a moderate-intensity statin is recommended.

A 45-year-old female presents to your office because she has had a lump on her neck for the past 2 weeks. She has no recent or current respiratory symptoms, fever, weight loss, or other constitutional symptoms. She has a history of well-controlled hypertension, but is otherwise healthy. On examination you note a nontender, 2-cm, soft node in the anterior cervical chain. The remainder of the examination is unremarkable. Which one of the following would be most appropriate at this point? (check one) A. Immediate biopsy B. Treatment with antibiotics, then a biopsy if the problem does not resolve C. Monitoring clinically for 4-6 weeks, then a biopsy if the node persists or enlarges D. Serial ultrasonography to monitor for changes in the node

C. Monitoring clinically for 4-6 weeks, then a biopsy if the node persists or enlarges Immediate biopsy - if the patient does not have inflammatory symptoms and the lymph node is >3 cm - if the node is in the supraclavicular area - coexistent constitutional symptoms such as night sweats or weight loss. Immediate evaluation - if the patient has risk factors for malignancy. Treatment with antibiotics - inflammatory symptoms such as pain, erythema, fever, or a recent infection. If node continues to enlarge - biopsy or imaging with CT or ultrasonography (not serial)

A 70-year-old white male with hypertension has several abnormal liver function tests on routine testing. He says he does not drink alcohol, and the prescription medications he is taking are unlikely to cause hepatotoxicity. However, during more extensive history taking, he tells you that he does use some over-the-counter medications. Which one of these is most likely responsible for the abnormal laboratory findings? (check one) A. Aspirin, used occasionally for headache B. A fiber supplement taken to promote regular bowel habits C. One long-acting niacin tablet per day D. One 250-mg vitamin C tablet daily E. Chewable simethicone after meals, almost daily

C. Niacin Hepatotoxicity resulting from timed-release formulations of niacin has been reported in elderly individuals Patients may be taking this supplement without their physician's knowledge, feeling it is safe because it is a vitamin Not Aspirin and vitamin C - gastrointestinal iron loss and anemia

A 39-year-old male with a history of alcoholism presents to your office with complaints of abdominal pain, vomiting, and nausea following a recent binge. He has eaten little since the onset of his symptoms 3 days ago. Laboratory findings suggest alcoholic ketoacidosis. His serum bicarbonate level is 16.3 mEq/L (N 22.0-26.0). In addition to thiamine, what other treatment should be provided for this patient? (check one) A. Bicarbonate and insulin B. Glucagon (GlucaGen) and hydrocortisone C. Normal saline and glucose D. N-acetylcysteine and pyridoxine (vitamin B6)

C. Normal saline and glucose - Alcoholic ketoacidosis - Blood glucose levels are usually low or normal, and volume depletion associated with nausea, vomiting, and abdominal pain - high osmolal and anion gaps - insulin usu low, but pt not usu hypoglycemic - Treatment: vigorous volume repletion, thiamine and glucose Not Bicarb: Only in marked acidemia (pH <7.10) is the administration of bicarbonate thought to be necessary. Not N-acetylcysteine and pyridoxine: Not the treatment of alcoholic ketoacidosis Not glucagon and hydrocortisone: levels are typically high

Which one of the following is true regarding electronic cigarettes? (check one) A. They release lower concentrations of particulate matter than tobacco cigarettes B. They are a nicotine-free alternative to tobacco C. They are not regulated by the FDA D. They have been proven to be effective for smoking cessation E. They have been shown to be safe in pregnancy

C. Not regulated by the FDA

You receive a telephone call from the mother of a 5-year-old female. The child has had diarrhea and a decreased appetite for the past 2 days. She is still playing some. The mother reports no vomiting, but says her daughter has complained of a dry mouth and does not have tears when she cries. You suspect that the child may be mildly dehydrated. Which one of the following would you advise? (check one) A. Increased water intake B. Clear liquids with sodium, such as chicken broth C. An over-the-counter oral rehydration solution D. Intravenous fluids in the emergency department E. Loperamide (Imodium)

C. OTC oral rehydration solution - Water and other clear liquids, even those with sodium, such as chicken broth, should not replace an oral rehydration solution because they are hyperosmolar. - These fluids do not adequately replace potassium, bicarbonate, or sodium, and can sometimes cause hyponatremia. four factors predicted dehydration: 1. a capillary refill time >2 seconds 2. the absence of tears 3. dry mucous membranes 4. an ill general appearance This child has two of the signs, but does not require intravenous fluids at this point.

A 45-year-old male is seen in the emergency department with a 2-hour history of substernal chest pain. An EKG shows an ST-segment elevation of 0.3 mV in leads V4-V6. In addition to evaluation for reperfusion therapy, which one of the following would be appropriate? (check one) A. Enteric aspirin, 81 mg B. Intravenous metoprolol (Lopressor) C. Oral clopidogrel (Plavix) D. Warfarin (Coumadin), after blood is drawn to establish his baseline INR

C. Oral clopidogrel (Plavix) - STEMI - most important goal is to begin fibrinolysis less than 30 minutes after the first contact with the health system. - The patient should be given oral clopidogrel, and should also chew 162-325 mg of aspirin (not 81mg) β-blockers - such as metoprolol shou"trifascicular block"ld not be routinely given, Warfarin - not indicated

A 35-year-old male consults you about a vague chest pain he developed while sitting at his desk earlier in the day. The pain is right-sided and was sharp for a brief time when it began, but it rapidly subsided. There was no hemoptysis and the pain does not seem pleuritic. His physical examination, EKG, and oxygen saturation are unremarkable. A chest film shows a 10% right pneumothorax. Which one of the following is true in this situation? (check one) A. He is likely to be an overweight smoker with a chronic cough B. Rupture of subpleural bullae would be an unlikely cause of his problem C. Outpatient observation with a repeat chest radiograph in 24 hours is indicated D. A chest tube should be placed expeditiously E. After treatment his probability of recurrence is less than 15%

C. Outpatient observation with a repeat chest radiograph in 24 hours is indicated - spontaneous pneumothorax - tall, thin individuals under 40 years of age - no clinically apparent lung disease, and the chest pain is sometimes minimal at onset and may resolve within 24 hours even if untreated - may have a normal physical examination, although tachycardia is occasionally noted. - Dx: chest radiographs - 30% of patients will have a recurrence within 6 months to 2 years. - The treatment of an initial pneumothorax of less than 20% may be monitored; f/u 24-48 hours. - Indications for treatment include progression, delayed expansion, or the development of symptoms. T - perhaps almost all of them, will have subcutaneous bullae on a CT scan.

A 47-year-old female presents with progressive difficulty hearing. She is employed as an office worker, has no significant past medical history, and takes no medications. Physical examination shows no gross abnormalities of her outer ears. The external ear canals are free of cerumen, and the tympanic membranes move well to insufflation. Weber's test and the Rinne test have results that are compatible with a conductive hearing loss. Which one of the following is the most likely cause of this patient's hearing loss? (check one) A. Noise-induced hearing loss B. Meniere's disease C. Otosclerosis D. Acoustic neuroma E. Perilymphatic fistula

C. Otosclerosis - between the third and fifth decades - more common in women. - a progressive conductive hearing loss. O All of the other choices are exclusively sensorineural in character. Meniere's disease - causes fluctuating hearing loss Noise-induced hearing loss - frequently and characteristically is accompanied by tinnitus. Perilymphatic fistula - sudden unilateral hearing loss with tinnitus and vertigo. Acoustic neuroma - associated with tinnitus and gradual hearing impairment.

Which one of the following treatmens is associated with a herald patch? (check one) A. Pityriasis alba B. Pityriasis lichenoides C. Pityriasis rosea D. Pityriasis rubra pilaris E. Pityriasis (tinea) versicolor

C. Pityriasis rosea

A patient is sent to you by his employer after falling down some steps and twisting his ankle and foot. Which one of the following would be the most appropriate reason to obtain foot or ankle radiographs? (check one) A. Notable swelling and discoloration over the anterior talofibular ligament B. A complaint of marked pain with weight bearing as he walks into the examining room C. Pain in the maleolar zone and bone tenderness of the posterior medial malleolus D. The absence of passive plantar foot flexion when the calf is squeezed (Thompson test)

C. Pain in maleolar zone and bone tenderness of the posterior medial malleolus - Ottawa ankle & foot rule An ankle radiograph series is required only if - pain in the malleolar zone and bone tenderness of either the distal 6 cm of the posterior edge or the tip of either the lateral malleolus or the medial malleolus; Inability to bear weight for four steps, both immediately after the injury and in the emergency department, is also an indication for ankle radiographs. -Pain in the midfoot zone and bone tenderness at the base of the 5th metatarsal or the navicular; if the patient is unable to bear weight both immediately after the injury and in the emergency department. A positive Thompson sign - Achilles tendon rupture, is the absence of passive plantar foot flexion when the calf is squeezed.

A previously healthy 29-year-old pediatric nurse has a 3-day history of malaise, arthralgias, and a nonpruritic rash. The rash is a faint, maculopapular, irregular, reticulate exanthem that covers her thighs and the inner aspects of her upper arms. Symmetric synovitis is present in several distal and proximal interphalangeal joints and in her metacarpophalangeal joints. Small effusions, warmth, and tenderness are noted in her left wrist and right elbow. No other joints are affected. The most likely cause of this problem is: (check one) A. Varicella-zoster virus B. Measles (rubeola) virus C. Parvovirus B19 D. Human immunodeficiency virus (HIV)

C. Parvovirus B19 - erythema infectiosum or fifth disease, - common cause of an exanthematous rash and arthritis in younger women. - esp. in health-care workers who have frequent contact with children.

Which one of the following is the most common cause of recurrent and persistent acute otitis media in children? (check one) A. Haemophilus influenzae B. Moraxella catarrhalis C. Penicillin-resistant Streptococcus pneumoniae D. Pseudomonas aeruginosa E. Staphylococcus aureus

C. Penicillin-resistant Streptococcus pneumoniae Streptococcus pneumoniae, Haemophilus influenzae, and Moraxella catarrhalis are the most common bacteria

The induction or inhibition of the cytochrome P450 (CYP) enzyme is responsible for many adverse drug reactions. Which one of the following is an inducer of the cytochrome P450 enzyme? (check one) A. Ciprofloxacin (Cipro) B. Fluconazole (Diflucan) C. Phenytoin (Dilantin) D. Clarithromycin (Biaxin) E. Grapefruit juice

C. Phenytoin (Dilantin) - everything else are inhibitor

A 42-year-old male presents with anterior neck pain. His thyroid gland is markedly tender on examination, but there is no overlying erythema. He also has a bilateral hand tremor. His erythrocyte sedimentation rate is 82 mm/hr (N 1-13) and his WBC count is 11,500/mm3 (N 4300-10,800). His free T4 is elevated, TSH is suppressed, and radioactive iodine uptake is abnormally low. Which one of the following treatment options would be most helpful at this time? (check one) A. Levothyroxine (Synthroid) and NSAIDs B. Propylthiouracil C. Prednisone D. Nafcillin E. Thyroidectomy

C. Prednisone - painful subacute thyroiditis - hyperthyroidism, elevated erythrocyte sedimentation rate - It is unclear whether there is a viral etiology to this self-limited disorder. - Thyroid function returns to normal in most patients after several weeks, and may be followed by a temporary hypothyroid state. - Treatment is symptomatic. Although NSAIDs can be helpful for mild pain, high-dose glucocorticoids provide quicker relief for the more severe symptoms.

A 23-year-old female presents with recurrent unprovoked epistaxis. The patient's mother is known to have hereditary hemorrhagic telangiectasia. Contrast echocardiography is recommended to screen for which one of the following frequently associated conditions? (check one) A. Atrioseptal defect B. Ventricular septal defect C. Pulmonary arteriovenous malformations E. Myocardial perfusion defects

C. Pulmonary arteriovenous malformations

You are writing a prescription for amoxicillin for a 6-year-old female with acute otitis media. Her mother has had an anaphylactic reaction to penicillin in the past and is concerned that she may have passed this trait down to her daughter. You reassure her that this is not usually the case but warn her about potential signs of an allergic reaction. Which one of the following is the most concerning early symptom of a dangerous drug reaction? (check one) A. Tachycardia and elevated blood pressure B. Small, bright, erythematous macules diffusely over the trunk C. Pruritus around the mouth and on the palms of the hands and soles of the feet D. Eczematous patches in the antecubital and popliteal fossae E. Diarrhea with blood on the tissue paper

C. Pruritus around the mouth and on the palms of the hands and soles of the feet

During a preparticipation examination of a 5-year-old male for summer soccer camp, his mother states that he frequently awakens during the night with complaints of cramping pain in both legs, and that he seems to experience this after a day of heavy physical activity. She says that he appears to drag his legs at times, but she has never noticed a definite limp. A physical examination of the hips, knees, ankles, and leg musculature is entirely normal. Which one of the following would be the most appropriate next step in the evaluation and management of this patient? (check one) A. Plain films of both hips and knees B. Serum electrolyte levels C. Recommending that he not participate in running sports D. Reassurance, with no activity restrictions or treatment E. Referral to a pediatric orthopedist

C. Reassurance - growing pain - Benign nocturnal limb pains of childhood (growing pains) occur in as many as one-third of children, most often between 4 and 6 years of age. -The pain often awakens the child within hours of falling asleep following an active day. - The pain is generally localized around the knees, most often in the shins and calves, but also may affect the thighs and the upper extremities - A characteristic history coupled with a normal physical examination will confirm the diagnosis.

A 70-year-old female presents to your office as a new patient. She is healthy and has no complaints. She walks for exercise 30-45 minutes daily and takes no prescription medications. Her blood pressure is 125/75 mm Hg, heart rate 72 beats/min, and respiratory rate 14/min. On examination she has a systolic crescendo-decrescendo murmur heard loudest at the right upper sternal border. An EKG in the office is within normal limits. Echocardiography shows mild aortic stenosis based on peak aortic jet velocity, aortic valve area, and mean pressure gradient. Her ejection fraction is 55%. At a follow-up visit she states that she continues to be symptom free. Which one of the following should be the next step in the evaluation and management of her aortic stenosis? (check one) A. Exercise treadmill testing B. Right heart catheterization C. Repeat echocardiography in 3 years D. Cardiothoracic surgery consultation E. Initiation of statin therapy

C. Repeat echocardiography in 3 years - Although aortic stenosis can result in adverse cardiac events, most of these events occur in patients who are symptomatic. Thus, the American Heart Association and the American College of Cardiology recommend that asymptomatic patients with mild aortic stenosis undergo repeat echocardiography every 3-5 years.

A 70-year-old female presents to your office as a new patient. She is healthy and has no complaints. She walks for exercise 30-45 minutes daily and takes no prescription medications. Her blood pressure is 125/75 mm Hg, heart rate 72 beats/min, and respiratory rate 14/min. On examination she has a systolic crescendo-decrescendo murmur heard loudest at the right upper sternal border. An EKG in the office is within normal limits. Echocardiography shows mild aortic stenosis based on peak aortic jet velocity, aortic valve area, and mean pressure gradient. Her ejection fraction is 55%. At a follow-up visit she states that she continues to be symptom free. Which one of the following should be the next step in the evaluation and management of her aortic stenosis? (check one) A. Exercise treadmill testing B. Right heart catheterization C. Repeat echocardiography in 3 years D. Cardiothoracic surgery consultation E. Initiation of statin therapy

C. Repeat echocardiography in 3 years Thus, the American Heart Association and the American College of Cardiology recommend that asymptomatic patients with mild aortic stenosis undergo repeat echocardiography every 3-5 years. - Further workup or treatment is not indicated for patients who have mild AS and are asymptomatic. - - Exercise treadmill testing may be indicated in patients with severe AS based on echo even if they are asymptomatic.

A 24-year-old African-American male presents with a history of several weeks of dyspnea, cough productive of bloody streaks, and malaise. His examination is normal except for bilateral facial nerve palsy. A CBC and urinalysis are normal. A chest radiograph reveals bilateral lymph node enlargement.This presentation is most consistent with: (check one) A. polyarteritis nodosa B. Goodpasture's syndrome C. sarcoidosis D. pulmonary embolus .

C. Sarcoidosis - young to middle-age adults (predominantly 20-29 years old) esp. African-Americans. - asymptomatic in 30%-50% of patients - diagnosed on a routine chest film. - 1/3 will present with fever, malaise, weight loss, cough, and dyspnea - pulmonary system is the main organ system affected: bilateral hilar lymphadenopathy and discrete, noncaseating epithelial granulomas. *Facial nerve palsy* - <5% of patients, and usually late in the process. - Before Lyme disease was recognized, bilateral facial nerve palsy was almost always due to sarcoidosis *Hemoptysis* - late in the course of sarcoidosis - usually related to Aspergillus infection or cavitation. *Renal* - rarely results in significant proteinuria or hematuria Not Polyarteritis nodosa - may involve the lungs - chest radiograph is more likely to reveal granulomatous lesions rather than patchy infiltrates Not Goodpasture's syndrome - pulmonary hemorrhage, glomerulonephritis, and antiglomerular basement membrane antibodies - Hemoptysis, pulmonary alveolar infiltrates, dyspnea, and iron-deficiency anemia are frequent presenting features - Within days or weeks, the pulmonary findings are generally followed by hematuria, proteinuria, and the rapid loss of renal function Not Pulmonary embolus - acute event - dyspnea and possibly hemoptysis, but not hilar lymphadenopathy

58-year-old male complains of leg claudication. Subsequent tests reveal that he has significant bilateral peripheral arterial disease. His current medications include atenolol (Tenormin), 50 mg/day, and aspirin, 325 mg/day. His blood pressure is 128/68 mm Hg, and his pulse rate is 64 beats/min. His LDL-cholesterol level is 123 mg/dL. The addition of which one of the following could reduce this patient's symptoms? (check one) A. Epoetin alfa (Epogen) B. Nifedipine (Procardia) C. Simvastatin (Zocor) D. Testosterone supplementation E. Warfarin (Coumadin) titrated to an INR of 2.0-3.0

C. Simvastatin (Zocor) - Peripheral arterial disease (PAD) - common malady that has several proven treatments. - goal: reducing PAD symptoms & preventing death due to systemic cardiovascular events (CVEs), especially - Statin drugs (specifically simvastatin and atorvastatin) have been shown to be beneficial for treatment of PAD symptoms and prevention of CVEs through the reduction of cholesterol, but they also appear to have other properties that help reduce leg pain in patients with PAD. Not lowering blood - recommended but only only ACE inhibitors have been shown to reduce symptoms of PAD directly. - combination of atenolol and nifedipine has actually been shown to worsen symptoms of PAD. Not adding warfarin to aspirin - no additional benefit in either reduction of PAD symptoms or prevention of CVEs - may have a role in preventing clots in patients who have undergone revascularization.

Which one of the following is a proven strategy to reduce anterior cruciate ligament tears in high-school athletes? (check one) A. The use of neoprene knee sleeves by all athletes competing in high-risk sports B. Consistent inclusion of long-distance running in practice sessions C. Structured exercises stressing balance, muscle strength, and proprioception E. Increased enforcement of penalties involving dangerous plays

C. Structured exercises stressing balance, muscle strength, and proprioception

Which one of the following has the best evidence of effectiveness for preventing fractures in postmenopausal women with osteoporosis? (check one) A. Home-hazard assessment B. Daily supplementation with vitamin D C. Treatment with calcitonin D. Treatment with alendronate (Fosamax)

D. Treatment of alendronate (Fosamax) - only supported evidence

A 65-year-old female is admitted to the hospital for a carotid endarterectomy and you are asked to make preoperative recommendations in advance of her surgery scheduled for tomorrow. She takes only low-dose aspirin. The physical examination is normal, including her blood pressure, as is an EKG. She has good exercise capacity and denies any symptoms of angina. You judge her to be stable for surgery. Which one of the following should you recommend that the patient start today? (check one) A. An ACE inhibitor B. A β-blocker C. A statin D. A diuretic

C. Statin If recommended prior to surgery, β-blockers should be started several weeks beforehand and carefully titrated. They may be harmful if initiated in the immediate perioperative period. Statins are recommended in the perioperative period for vascular surgery regardless of other cardiac risk factors

A 67-year-old male is admitted to your inpatient service with a week-long acute exacerbation of COPD. He also has hypertension and type 2 diabetes mellitus. After 24 hours of intravenous fluids and intravenous methylprednisolone, he is now tolerating oral intake. Which one of the following corticosteroid regimens is best for this patient at this time? (check one) A. Continue intravenous methylprednisolone until his COPD is back to baseline, then switch to oral methylprednisolone for a 14-day total course of treatment B. Switch to oral prednisone for a 14-day total course of treatment, including the initial 24-hour intravenous treatment C. Switch to oral prednisone for 4 more days of treatment D. Use only inhaled corticosteroids by nebulizer E. Discontinue corticosteroid treatment altogether after 24 hours

C. Switch to oral prednisone for 4 more days of treatment - randomized, controlled trial has demonstrated that 5-day courses of systemic corticosteroid therapy are at least as effective as 14-day courses (SOR A)

A 30-year-old female is referred to you by a local optometrist after she was treated several times for anterior uveitis. You are concerned about an associated systemic disease. She feels well otherwise, and denies back or joint pain, rash, cough, or fever. A chest radiograph reveals enlarged mediastinal lymph nodes. Which one of the following is most likely to be associated with her recurrent uveitis? (check one) A. Cat-scratch disease B. Lyme disease C. Syphilis D. Tuberculosis

C. Syphilis Many patients with uveitis have an associated systemic disease. Some medications may cause secondary uveitis, and conditions such as ocular lymphoma and bloodborne infection may masquerade as primary uveitis. In North America, the most common conditions associated with uveitis are the seronegative spondyloarthropathies, sarcoidosis, syphilis, rheumatoid arthritis, and reactive arthritis. All of the conditions listed may be associated with uveitis, but given the chest radiograph findings and clinical scenario in this case, sarcoidosis is most likely.

Which one of the following is the preferred treatment for scabies? (check one) A. Topical benzoyl peroxide, 10% B. Topical crotamiton (Eurax), 10% C. Topical permethrin (Elimite), 5% D. Topical lindane, 1% E. Oral ivermectin (Stromectol), 200 mg

C. Topical permethrin (Elimite) Permethrin and lindane - two most studied topical treatments for scabies. - a single overnight application of permethrin is more effective than lindane (odds ratio for clinical failure, 0.66; 95% confidence interval, 0.46-0.95). - The potential neurotoxicity of lindane, especially with repeated applications, has limited its use.

A 52-year-old female presents with vulvovaginal dryness and pain with intercourse. She has not menstruated for 6 months and denies hot flashes, insomnia, or other vasomotor symptoms. She has no past history of cancer or surgery. Her examination is consistent with vaginal atrophy. Which one of the following is the recommended first-line treatment for this patient? (check one) A. Oral estrogen B. Oral estrogen and progestogen C. Vaginal estrogen D. Vaginal estrogen and oral progestogen

C. Vaginal estrogen

An elevation of serum methylmalonic acid is both sensitive and specific for a cellular deficiency of which vitamin? (check one) A. Vitamin A B. Vitamin B 6 C. Vitamin B 12 D. Vitamin D E. Folate

C. Vitamin B 12

A 24-year-old gravida 2 para 1 at 9 weeks gestation sees you for a routine prenatal check. She complains of significant nausea, and recommended dietary modifications have not helped. She drives a school bus so she would like to avoid sedating medications. She appears well-hydrated and her examination is otherwise normal. Which one of the following would be best for relieving this patient's nausea? (check one) A. Auricular acupressure B. A scopolamine patch (Transderm Scop) C. Vitamin B6 (pyridoxine) D. Methylprednisolone (Medrol)

C. Vitamin B6 (pyridoxine)

An 18-year-old female basketball player comes to your office the day after sustaining an inversion injury to her ankle. She says she treated the injury overnight with rest, ice, compression, and elevation. You examine her and diagnose a moderate to severe lateral ankle sprain. In addition to rehabilitative exercises, you advise (check one) A. a short-term cast B. a posterior splint that allows no flexion or extension C. a semi-rigid stirrup brace (Air-Stirrup, "Aircast") D. an elastic bandage E. no external brace or support

C. a semi-rigid stirrup brace - semi-rigid brace that allows flexion and extension, or a soft lace-up brace is recommended over immobilization. - External ankle support has been shown to improve proprioception

A 42-year-old white male develops respiratory distress 12 hours after he sustained a closed head injury and a femur fracture. A physical examination reveals a respiratory rate of 40/min. He has a pO2 of 45 mm Hg (N 75-100), a pCO2 of 25 mm Hg (N 35-45), and a blood pH of 7.46 (N 2 2 7.35-7.45). His hematocrit is 30.0% (N 37.0-49.0). Of the following, the most likely diagnosis is: (check one) A. respiratory depression due to central nervous system damage B. heart failure C. adult respiratory distress syndrome (ARDS) D. hypovolemic shock E. tension pneumothorax

C. adult respiratory distress syndrome (ARDS) - acute resp failure s/p trauma - injury at the alveolar-capillary interface, with resulting leakage of proteinaceous fluid from the intravascular space into the interstitium and subsequently into alveolar spaces - It has become acceptable to describe this entire spectrum of acute diffuse injury as adult respiratory distress syndrome (ARDS). - The syndrome of ARDS can occur under a variety of circumstances and produces a spectrum of clinical severity from mild dysfunction to progressive, eventually fatal, pulmonary failure.

A 29-year-old white female is hospitalized following a right middle cerebral artery stroke confirmed by MRI. Her past medical history is remarkable only for a history of an uncomplicated tonsillectomy during childhood and a second-trimester miscarriage 3 years ago. The only remarkable finding on physical examination is left hemiplegia.The initial laboratory workup reveals normal hematocrit and hemoglobin levels, a normal prothrombin time, and a platelet count of 200,000/mm3 (N 140,000-440,000). The activated partial thromboplastin time is 95 sec (N 23.6-34.6), and it does not normalize when the patient's serum is mixed with normal plasma. A serum VDRL is positive, and a serum FTA-ABS is nonreactive. Which one of the following is the most likely diagnosis? (check one) A. Hemophilia B. Neurosyphilis C. Antiphospholipid syndrome D. Thrombotic thrombocytopenic purpura E. Protein C deficiency

C. antiphospholipid syndrome - to the appearance of a heterogeneous group of circulating antibodies to negatively charged phospholipids - most commonly a lupus anticoagulant and anticardiolipin antibodies - The antibodies are usually detected by a false-positive serologic test for syphilis - Clinical features include 1. venous and arterial thrombosis 2. fetal wastage 3. thrombocytopenia 4. activated partial thromboplastin time (aPTT) inhibitor. important differential for all patients with unexplained thrombosis or cerebral infarction, particularly in young patients. Not hemophilia: - PTT would normalize when pt's plasma mix with normal plasma Not neurosyphillis - negative serum FTA-ABS result Not Thrombotic thrombocytopenic purpura - associated with prolongation of the aPTT - a hemolytic anemia Not protein C deficiency - is a hypercoagulable state that can lead to stroke - none of the laboratory abnormalities suggests this diagnosis.

The pruritic lesions on the arm shown in Figure 6 are typical of: (check one) A. poison ivy dermatitis B. brown recluse spider bites C. bedbug bites D. Hymenoptera stings E. molluscum contagiosum

C. bedbug bites

Which one of the following causes of anemia is associated with a normal red cell distribution width? (check one) A. Vitamin B12 deficiency B. Iron deficiency C. β-Thalassemia trait D. Sideroblastic anemia E. Myelofibrosis

C. beta Thalassemia - Red cell distribution width (RDW) is a measure of the variability of size of the red cells. - particularly useful in distinguishing anemic disorders - especially iron deficiency anemia (high RDW, normal to low mean corpuscular volume) - uncomplicated heterozygous thalassemia (normal RDW, low mean corpuscular volume).

A mother brings in her 10-year-old son because of a swollen area in his neck that she first noticed yesterday. He has also had symptoms of an upper respiratory infection. On examination the child has a runny nose but otherwise appears well. Palpation reveals a soft, 1.5-cm, slightly tender mass, inferior to the angle of the mandible and anterior to the sternocleidomastoid muscle. The most likely diagnosis is (check one) A. thyroglossal duct cyst B. dermoid cyst C. branchial cleft cyst D. thyroid tumor

C. branchial cleft cyst found when a previously unrecognized cyst becomes infected; anterior to the sternocleidomastoid muscle or preauricular. Not Thyroglossal duct cysts - in the midline over the hyoid bone. Frequently, they elevate when the patient swallows. Not Dermoid cysts - mobile, moving with the overlying skin in the submental or midline region. Not Thyroid tumors - midline. Malignant masses are usually hard, irregular, nontender, and fixed.

The test of choice for immediate evaluation of an acutely swollen scrotum is: (check one) A. a pelvic radiograph B. radionuclide imaging C. color Doppler ultrasonography D. CT E. MRI

C. color doppler ultrasound - avoid damage such as testicular torsion, or of life-threatening diseases such as testicular carcinoma. - Color Doppler ultrasonography is the test of choice for immediate evaluation of scrotal masses (SOR B) because it can be done quickly and has a high sensitivity (86%-88%) and specificity (90%-100%) for detecting testicular torsion, which is a surgical emergency Not Radionuclide imaging - involves too much of a time delay to be useful. Not CT and MRI - only if ultrasonography is inconclusive or carcinoma is suspected - useful for staging testicular tumors Not Pelvic radiographs - not for scrotal masses.

A mother brings her 2-month-old infant to the emergency department because of profuse vomiting and severe diarrhea. The infant is dehydrated, has a cardiac arrhythmia, appears to have ambiguous genitalia, and is in distress. This presentation suggests a diagnosis of: (check one) A. acute gastroenteritis B. hypertrophic pyloric stenosis C. congenital adrenal hyperplasia D. congenital intestinal malrotation E. Turner's syndrome

C. congenital adrenal hyperplasia - inherited deficiency of any of the enzymes necessary for the biosynthesis of cortisol. In patients with the salt-losing variant - Vomiting is prominent, and anorexia is also present. - Disturbances in cardiac rate and rhythm may occur, along with cyanosis and dyspnea. - In the male, various degrees of hypospadias may be seen, with or without a bifid scrotum or cryptorchidism.

An 80-year-old male presents with a 10-day history of intermittent colicky abdominal pain. The pain is low and central and seems to be worse after eating. He has no associated fever or vomiting but does feel nauseated when the pain is present. He says that prior to this episode he had hard stools once or twice a week that were difficult to pass. For the past several days he has had only watery stools, several times a day. On examination there is fullness in his left lower quadrant with nonspecific tenderness diffusely and no guarding or rebound. A urine dipstick is normal. Which one of the following is the most likely diagnosis? (check one) A. Viral gastroenteritis B. Acute colitis C. Constipation D. Urinary tract infection E. Nephrolithiasis

C. constipation - Rome criteria define constipation as the presence of two or more of the following: 1. straining on defecation 2. hard stools 3. incomplete evacuation 4. less than three bowel movements per week. This patient has multiple symptoms on this list. The presence of watery bowel movements does not rule out the diagnosis of constipation, as it is common for liquid stool to pass an obstructive source.

A 30-year-old female who had a deep venous thrombosis in her left leg during pregnancy has an uneventful delivery. During the pregnancy she was treated with low molecular weight heparin. Just after delivery her left leg is pain free and is not swollen. She plans to resume normal activities soon. Which one of the following would be most appropriate with regard to anticoagulation? (check one) A. Discontinuing treatment, with no further evaluation B. Discontinuing treatment if venous Doppler ultrasonography is negative for thrombus C. Continuing low molecular weight heparin for 6 more weeks D. Switching to low-dose unfractionated heparin for 6 weeks E. Switching to aspirin for 6 weeks

C. continue Low Molecular Weight Heparin x 6 wks - risk of pulmonary embolism continues in the postpartum period, and may actually increase during that time. - For patients who have had a deep-vein thrombosis during pregnancy, treatment should be continued for 6 weeks after delivery, with either warfarin or low molecular weight heparin.

A 70-year-old white female comes to your office for an initial visit. She has taken levothyroxine (Synthroid), 0.3 mg/day, for the last 20 years. Although a recent screening TSH was fully suppressed at <0.1 μU/mL, she claims that she has felt "awful" when previous physicians have attempted to lower her levothyroxine dosage. You explain that a serious potential complication of her current thyroid medication is: (check one) A. adrenal insufficiency B. carcinoma of the ovary C. carcinoma of the thyroid D. hip fracture E. renal failure

C. hip fracture - Women older than 65 years of age who have low serum TSH levels, indicating physiologic hyperthyroidism, are at increased risk for new hip and vertebral fractures. - Use of thyroid hormone itself does not increase the risk of fracture if TSH levels are normal.

One week after returning from a Caribbean vacation, a 43-year-old female presents to a walk-in clinic with a complaint of redness and itching on the sole of her foot She recalls experiencing a stinging sensation in the same area while she was wading in the surf on the day before she was to return home, but was unable to see any sign of injury immediately following the incident. Since her return the itching has intensified and the red area has enlarged. The most likely cause of this condition is a: (check one) A. filarial nematode B. jellyfish C. hookworm D. roundworm E. tapeworm

C. hookworm - characteristic serpiginous, red larva tracts - When third-stage hookworm larvae, most commonly of the species infecting dogs and cats, penetrate the skin and migrate through the dermis, they create the tracks . - most often in tropical and semitropical regions such as the Caribbean, Africa, Asia, and South America. - Travelers to beaches with pet feces are most at risk because of the direct contact of bare skin with the sand - a stinging or itching sensation may be noted upon penetration; followed by the the creeping eruption 1-5 days later (may delay up to 1 mo) a - The larvae will not develop in the human host, so the infection is self-limited - Treatment: antihelminthic drugs can greatly reduce the clinical course.

While performing a routine physical examination on a 42-year-old female you discover an apparent nodule in the left lobe of the thyroid measuring approximately 1 cm in diameter, which is confirmed on ultrasonography. The most appropriate next step in the evaluation of this finding is a A. serum calcitonin level B. serum free T3 level C. serum TSH level D. serum thyroglobulin leve E. radionuclide thyroid scan

C. serum TSH level - Nodules <1 cm should also be fully evaluated when found in patients with a family history of thyroid cancer, a personal history of head and neck irradiation, or a finding of cervical node enlargement. - 1st steps: measurement of TSH or ultrasound - The American Thyroid Association's guidelines recommend that TSH be the initial evaluation (SOR A) and that this be followed by a radionuclide thyroid scan if results are abnormal. - Diagnostic ultrasonography is recommended for all patients with a suspected thyroid nodule, a nodular goiter, or a nodule found incidentally on another imaging study (SOR A).

The 1990 Patient Self-Determination Act requires that: (check one) A. the process for advance directives be standardized for all 50 states B. a living will be implemented for patients upon admission to the hospital C. hospitals ask patients about advance directives D. verbally expressed wishes be honored for individuals who do not have a written advance directive

C. hospitals ask patients about advance directives - then incorporate the information into medical records - The living will, a written advance directive, allows a competent person to indicate his or her health care preferences while cognitively and physically healthy. - A living will may list medical interventions the patient wishes to have withheld or withdrawn when he or she becomes unable to communicate. - Another type of advance directive, the durable power of attorney for health care, allows persons to designate a proxy (or surrogate) to make decisions for them if they become incapacitated.

You are examining a patient with a chronically painful shoulder. You forward flex the arm to 90° with the elbow bent to 90°. You then internally rotate the arm, which causes pain in the shoulder. This finding suggests: (check one) A. glenohumeral instability B. anterior shoulder dislocation C. impingement/rotator cuff disorder D. acromioclavicular joint osteoarthritis E. acromioclavicular joint separation

C. impingement/rotator cuff issue - maneuver described is Hawkins' impingement test - Pain with this maneuver may signify subacromial impingement, including a rotator cuff tendinopathy or tear.

A 19-year-old college freshman consults you at the request of her cross-country coach because she has not had a period in 2 of the last 3 months. She notes that her current training regimen is much more intense than in high school last year. She has an appropriate body image and denies caloric restriction. A pregnancy test at the student health center was negative. On examination she is lean and highly trained. Her examination is otherwise normal. Which one of the following would be the most appropriate recommendation for this patient? (check one) A. Estrogen supplementation B. Cyclic oral contraceptive pills C. Increased caloric intake D. Bisphosphonate therapy

C. increase calorie intake - exercise-related oligomenorrhea - Menstrual problems in athletes do correlate with bone density loss and impaired recovery from exercise - menstrual irregularity of varying severity is extremely common in female distance runners - The main issue in well-nourished female athletes seems to be that energy intake is not increased to match energy expenditures at high levels of training Not Hormonal manipulation - does not affect bone density, though it may produce withdrawal bleeding Not Bisphosphonate therapy - not recommended in women of child-bearing age.

A 29-year-old gravida 2 para 1 presents for pregnancy confirmation. Her last menstrual period began 6 weeks ago. Her medical history is significant for hypothyroidism, which has been well-controlled on levothyroxine (Synthroid), 150 μg daily, for the past 2 years. Which one of the following would be the most appropriate next step in the treatment of this patient's hypothyroidism during her pregnancy? (check one) A. Add liothyronine (Cytomel) to her current regimen B. Decrease the levothyroxine dosage C. Increase the levothyroxine dosage D. Continue her current regimen

C. increase the levothyroxine dosage - Maternal hypothyroidism can have serious effects on the fetus - Because of hormonal and metabolic changes in early pregnancy, the levothyroxine dosage often needs to be increased at 4-6 weeks gestation - the patient eventually may require a 30%-50% increase in dosage in order to maintain her euthyroid status.

A 55-year-old male presents to your office for evaluation of increasing dyspnea with exertion over the past 2 weeks. He has smoked 2 packs of cigarettes per day since the age of 20. He has had a chronic cough for years, along with daily sputum production. He was given an albuterol inhaler for wheezing in the past, which he uses intermittently. On examination he has a severe decrease in breath sounds, no evidence of jugular venous distention, no cardiac murmur, and no peripheral edema. A chest film shows hyperinflation, but no infiltrates or pleural effusion. Office spirometry shows that his FEV1 is only 55% of the predicted value. You consider using inhaled corticosteroids as part of the treatment regimen for this patient. This has been shown to: (check one) A. increase cataract formation B. increase the incidence of fracture C. increase the risk of pneumonia D. slow the progression of the disease E. improve overall mortality from the disease

C. increase the risk of pneumonia Inhaled corticosteroids - will not reduce mortality or affect long-term progression of COPD. - reduce the number of exacerbations and the rate of decline in the quality of life - no increase in cataract formation or rate of fracture - side effects: candidal infection of the oropharynx, hoarseness, and an increased risk of developing pneumonia.

Which one of the following immunizations is indicated for all pregnant women at any stage of pregnancy? (check one) A. MMR B. Varicella C. Influenza D. HPV

C. influenza

For 2 weeks, a 62-year-old male with biopsy-documented cirrhosis and ascites has had diffuse abdominal discomfort, fever, and night sweats. His current medications are furosemide (Lasix) and spironolactone (Aldactone). On examination his temperature is 38.0°C (100.4°F), his blood pressure is 100/60 mm Hg, and his heart rate is 92 beats/min and regular. Examination of the heart and lungs is normal. The abdomen is soft with vague tenderness in all quadrants. There is no rebound or guarding. The presence of ascites is easily verified. Bowel sounds are quiet. The rectal examination is normal, and the stool is negative for occult blood. You perform diagnostic paracentesis and send a sample of the fluid for analysis. Which one of the following findings would best support the suspected diagnosis of spontaneous bacterial peritonitis? (check one) A. pH <7.2 B. Bloody appearance C. Neutrophil count >250/mL D. Positive cytology E. Total protein >1 g/dL

C. neutrophil count >250/mL - Diagnostic paracentesis; for pt w/ newly onset ascites - for chronic ascites w/ new findings e.g. fever & abd pain Spontaneous Bacterial Peritonitis - neutrophil count >250/mL is diagnostic for peritonitis Hepatoma - bloody ascites w/ abnormal cytology Not protein > 1g/dL - evidence against SBP Not pH - useless

A 56-year-old female presents for a health maintenance examination. She has a history of a total hysterectomy for benign disease 4 years ago. You are able to document that the hysterectomy pathology was benign and that she has had normal Papanicolaou (Pap) tests for 10 years. The patient asks about regular Pap smears. Which one of the following would be the most appropriate recommendation? (check one) A. Routine Pap smears should be continued until age 70 B. A Pap smear should be done every 3 years C. A Pap smear is not indicated D. A Pap smear should be done yearly for 3 years and only if indicated thereafter

C. not indicated After a hysterectomy for documented benign disease, cytologic screening may be discontinued. Papanicolaou (Pap) smears in this population are low yield and may cause unnecessary testing due to false-positives. Pap smears may be continued if the reason for the hysterectomy is uncertain. If there is a history of invasive cervical cancer or DES exposure, screening should be continued, although there is not a great deal of data to support this practice

A 45-year-old white male undergoes a health screening at his church. He has a carotid Doppler study, abdominal ultrasonography, heel densitometry, and a multiphasic blood panel. He receives a report indicating that all of the studies are normal, but a 0.7-cm thyroid nodule was noted. The TSH level is normal. He schedules a visit with you and brings you the report. A neck examination and ENT examination are normal, and you do not detect a nodule. You recommend (check one) A. a radionuclide thyroid scan B. T3, T4, and calcitonin levels C. repeat ultrasonography in 6-12 months D. a fine-needle biopsy E. hemithyroidectomy

C. repeat ultrasound in 6-12 mos - classic incidentaloma. - Nodules are detected in up to 50% of thyroid sonograms and carry a low risk of malignancy (<5%). - If the TSH level is normal, nuclear scanning and further thyroid studies are not necessary. - it is reasonable to follow small nodules with clinical examinations and periodic sonograms

A 73-year-old male sees you for evaluation of a tremor. Based on the history and examination, you suspect Parkinson's disease. Which one of the following would be most helpful for confirming the diagnosis? (check one) A. CT of the brain B. MRI of the brain C. A positive response to levodopa D. Confirming that the tremor occurs with movement E. Confirming that the tremor had a symmetric onset

C. response to Levodopa Patients with Parkinson's disease should respond to an adequate therapeutic challenge of levodopa or a dopamine agonist. The diagnosis of idiopathic Parkinson's disease is clinical, not radiographic. Cardinal signs of Parkinson's disease include an asymmetric tremor onset and a distal resting tremor of 3-6 Hz.

A 15-year-old white female who has had regular periods since age 12 comes to your office because of secondary amenorrhea and a milky discharge from her breasts. A pregnancy test is negative. The best test for initial evaluation of the pituitary in this patient is: (check one) A. plasma antidiuretic hormone B. plasma ACTH C. serum prolactin D. serum FSH and LH E. fasting growth hormone

C. serum prolactin - Anterior pituitary hormone overproduction - most to least common 1. Prolactinomas. 2. Growth hormone-secreting tumors, - cause acromegaly or gigantism. 3. corticotropic (ACTH-secreting) adenomas - cause cortisol excess (Cushing's disease) 4. Glycoprotein hormone-secreting pituitary adenomas (secreting TSH, LH, or FSH) - TSH-secreting adenomas are a rare cause of hyperthyroidism. Paradoxically, most patients with gonadotropin-secreting adenomas have hypogonadism.

A 2-year-old child stumbles, but his mother keeps him from falling by pulling up on his right hand. An hour later the child refuses to use his right arm and cries when his mother tries to move it. The most likely diagnosis is (check one) A. dislocation of the ulna B. dislocation of the olecranon epiphysis C. subluxation of the head of the radius D. subluxation of the head of the ulna E. anterior dislocation of the humeral head

C. subluxation of the head of the radius - nurse maid elbow

A 45-year-old female presents with a complaint of pain and swelling in her right index finger of 2 days' duration. She reports that 5 days ago she had artificial nails applied, which she removed yesterday due to the pain. She used hydrogen peroxide on the finger, but it did not help. She denies any systemic symptoms or fever. On examination there is erythema and swelling in the lateral nail fold of the right index finger, with purulent material noted. Which one of the following would be the most appropriate treatment for this patient? (check one) A. Removal of the proximal nail fold B. Topical corticosteroids C. Topical antibiotics D. Topical antifungals

C. topical abx - acute paronychia - which typically is caused by local trauma to the nail fold or cuticle, with resulting inoculation and infection. - Topical antibiotics, with or without topical corticosteroids, is one treatment option. - Other options include warm compresses, oral antibiotics Not incision and drainage - not always necessary Not Removal of the proximal nail fold - for chronic paronychia that is not responsive to other treatments Not topical corticosteroids - should be combined with abx Not topical antifungals - chronic paronychia, which can be associated with a fungal infection, but not for acute paronychia.

A 61-year-old female presents to your office with a sudden painless loss of vision in her right eye. Her past medical history includes both hypertension and type 2 diabetes mellitus. Which one of the following would make you suspect retinal vein occlusion as the cause of her sudden visual loss? (check one) A. An afferent pupillary defect in the contralateral eye B. Right eye redness C. Tortuous retinal veins on funduscopic examination D. Macular drusen on funduscopic examination

C. tortuous retinal veins on funduscopic exam - retinal vein occlusion: sudden painless loss of vision or distortion of vision. - Redness is not typical - Tortuous and dilated retinal veins are the most common finding on funduscopic examination. - Patients also often have multiple cotton-wool spots, although these are not specific to retinal vein occlusion. - An afferent pupillary defect often occurs on the affected side. - Diabetes mellitus and hypertension are both risk factors for retinal vein occlusion

A 70-year-old white female complains of two episodes of urinary incontinence. On both occasions she was unable to reach a bathroom in time to prevent loss of urine. The first episode occurred when she was in her car and the second while she was in a shopping mall. She is reluctant to go out because of this problem. The most likely cause of her problem is: (check one) A. overflow incontinence B. stress incontinence C. urge incontinence D. functional incontinence

C. urge incontinence - when patients sense the urge to void but are unable to inhibit leakage long enough to reach the toilet - treatment: 1st behavioral, then anticholinergics Overflow incontinence - when the bladder cannot empty normally and becomes overdistended Functional incontinence - lower urinary tract function is intact but other factors such as immobility and severe cognitive impairment lead to incontinence.

Sympathomimetic decongestants such as pseudoephedrine and phenylephrine can be problematic in elderly patients because they can (check one) A. decrease blood pressure B. cause bradycardia C. worsen existing urinary obstruction D. enhance the anticholinergic effects of other medications E. enhance the sedative effects of other medications

C. worsen existing urinary obstruction Sympathomimetic agents can elevate blood pressure and intraocular pressure, may worsen existing urinary obstruction, and adversely interact with β-blockers, methyldopa, tricyclic antidepressants, oral hypoglycemic agents, and MAOIs. Elevate heart rate as well.

A 55-year-old male has New York Heart Association Class II heart failure. He becomes dyspneic with significant exertion. His only medication is an ACE inhibitor. Which one of the following additional medications has been shown to improve longevity in this situation? (check one) A. Digitalis B. Warfarin (Coumadin) C. β-Blockers D. Amiodarone (Cordarone) E. Non-dihydropyridine calcium channel blockers

C. β-Blockers - reduce mortality in symptomatic patients with heart failure (SOR A). Not digoxin - trend toward increased mortality among women with heart failure who were taking digoxin - but digoxin levels were higher among women than men. Not warfarin - no evidence Not amiodarone - no evidence in pt without A-fib Not Calcium channel blockers - cause peripheral vasodilation, decreased heart rate, decreased cardiac contractility, and decreased cardiac conduction.

A 26-year-old female has a 4-month history of continuous right-sided headache. The headache is associated with tearing and nasal congestion only on the right, and has not responded to over-the-counter analgesics. The patient went to the emergency department a few nights ago because of the pain, and CT of the head at that visit was normal. On examination you note conjunctival injection on the right. Findings are otherwise normal. Which one of the following would be the most appropriate treatment at this time? (check one) A. Sumatriptan (Imitrex) B. Amitriptyline C. Indomethacin D. Topiramate (Topamax)

Cluster headache: - Males - Occurs at night - Unilateral HA, orbital pain, blocked nasal passages, rhinorrhea, conjunctival injection, miosis. -Treated with O2, verapamil, ppx with ergotamine or sumatriptan. Chronic paroxysmal hemicrania - Women - Unilateral HAand w/ conjunctival hyperemia and rhinorrhea - Treat with indomethacin

A critically ill adult male is admitted to the intensive-care unit because of sepsis. He has no history of diabetes mellitus, but his glucose level on admission is 215 mg/dL and insulin therapy is ordered. Which one of the following is the most appropriate target glucose range for this patient? (check one) A. 80-120 mg/dL B. 100-140 mg/dL C. 120-160 mg/dL D. 140-180 mg/dL E. 160-200 mg/dL

D. 140-180 mg/dL American Diabetes Association recommend insulin infusion with a target glucose level of 140-180 - no reduction in mortality from intensive treatment targeting near-euglycemic glucose levels compared to conventional management with a target glucose level of <180 mg/dL. - There also were reports of harm resulting from intensive glycemic control, including higher rates of severe hypoglycemia and even increased mortality.

A new drug treatment is shown to reduce the incidence of a complication of a disease by 50%. If the usual incidence of this complication were 1% per year, how many patients with this disease would have to be treated with this medication for 1 year to prevent one occurrence of this complication? (check one) A. 20 B. 50 C. 100 D. 200 E. 500

D. 200 - relative risk reduction without also considering the absolute rate can distort the importance of a therapy. - determine the number needed to treat to benefit one patient. - NNT = the percentage of absolute risk reduction of a particular therapy is divided into 100. - In the case in question, the absolute risk reduction is 0.5% (0.5×.01). Thus, the number-needed-to-treat for the example cited is 200

A 26-year-old gravida 3 para 2 was diagnosed with gestational diabetes mellitus at 24 weeks gestation. She was prescribed appropriate nutritional therapy and an exercise program. After 4 weeks, her fasting plasma glucose levels remain in the range of 105-110 mg/dL. Which one of the following would be the most appropriate treatment for this patient at this time? (check one) A. Continuation of the current regimen B. Long-acting insulin glargine (Lantus) once daily C. Pioglitazone (Actos) once daily D. A combination of intermediate-acting insulin (e.g., NPH) and a short-acting insulin (e.g., lispro) twice daily E. Sliding-scale insulin 4 times daily using ultra-short-acting insulin

D. A combination of intermediate-acting insulin (e.g., NPH) and a short-acting insulin (e.g., lispro) twice daily - pharmacologic therapy with fasting plasma glucose levels remain above 100 mg/dL despite diet and exercise. - fasting plasma glucose levels below 95 mg/dL and 1-hour postprandial levels below 140 mg/dL results in improved fetal well-being and neonatal outcomes. - metformin or glyburide is considered safe but insulin therapy is the best option - Thiazolidinediones such as pioglitazone have not been shown to be effective or safe in pregnancy.

A 2-year-old white male is seen for a well child visit. His mother is concerned because he is not yet able to walk. The routine physical examination, including an orthopedic evaluation, is unremarkable. Speech and other developmental landmarks seem normal for his age. Which one of the following tests would be most appropriate? (check one) A. A TSH level B. Random urine for aminoaciduria C. Phenylketonuria screening D. A serum creatine kinase level E. Chromosome analysis

D. A serum creatine kinase level - Duchenne muscular dystrophy - most common neuromuscular disorder of childhood (age 4-5) years - X-linked inheritance may have had additional children who would also be at risk. - can be diagnosed earlier by testing for elevated creatine kinase in boys who are slow to walk - mean age for walking in affected boys is 17.2 months, whereas over 75% of developmentally normal children in the United States walk by 13.5 months. - Massive elevation of creatine kinase (CK) from 20 to 100 times normal occurs in every young infant with the disease

A 72-year-old previously healthy female comes in for evaluation of recent headaches. She describes the pain as generalized all over her head and persisting over the past several months. She reports feeling more achy and fatigued in the past several weeks, with a decreased appetite and unintentional weight loss of 4 lb in the past 2 months. She denies any other symptoms including sinus congestion, nausea, vomiting, numbness, tingling, weakness, or vision changes. Acetaminophen has been minimally helpful for the pain. On examination you note a temperature of 37.9°C (100.2°F), normal cranial nerves, a normal eye examination, and no tenderness to palpation of the head. She is mildly tender to palpation of the shoulders and upper arms. Laboratory testing reveals an erythrocyte sedimentation rate of 88 mm/hr (N 1-25). Which one of the following is necessary to confirm the most likely diagnosis? (check one) B. CT of the head C. MRI of the head D. A temporal artery biopsy E. A lumbar puncture

D. A temporal artery biopsy - temporal arteritis - most common in the elderly and is twice as common in women as in men. - Because of its inflammation, patients commonly have systemic symptoms, including fever. - temporal artery may be thickened, tender, or lacking pulsation, although a normal artery does not rule out the diagnosis. - Jaw claudication is a fairly specific but nonsensitive finding. - an elevated erythrocyte sedimentation rate (ESR) is sensitive; but nonspecific and may be caused by other conditions - Not MRI and CT: for cerebral hemorrhage or mass - Not lumbar puncture: for benign intracranial hypertension or meningitis

A 25-year-old gravida 1 para 0 sees you for a routine prenatal visit. This is a planned pregnancy and you calculate her to be at approximately 14 weeks gestation based on the dates of her last menstrual period. She is healthy without any medical problems, takes no medication, and does not use tobacco products. She is adopted and does not know her family history. She feels well today and has no specific concerns. Her vital signs are stable, her weight is normal, and fetal heart tones are auscultated with a Doppler stethoscope at approximately 140 beats/min. Which one of the following should be completed today? (check one) A. A 1-hour glucose tolerance test B. A group B Streptococcus screen C. A TSH level D. A urinalysis and urine culture E. Evaluation for bacterial vaginosis

D. A urinalysis and urine culture: during 11 and 16 weeks gestation for asymptomatic bacteruria Not bacterial vaginosis: treating decreases the risk of low birth weight and premature rupture of membranes; but universal screening is not recommended. Not group B Streptococcus (GBS): w 35-37 wks Not diabetes mellitus: 24 and 28 weeks gestation with 50-g glucose load.

You are evaluating a 25-year-old gravida 5 para 1 at 6 weeks estimated gestation. She has a history of three consecutive spontaneous miscarriages. Her workup has been negative except for a positive lupus anticoagulant on two occasions, separated by 6 weeks. You make the diagnosis of antiphospholipid antibody syndrome. She has no previous history of venous or arterial thrombosis. The best medical management at this time is: (check one) A. Acetaminophen B. Warfarin (Coumadin) C. Prednisone D. Aspirin and heparin combined E. Progesterone

D. ASA & heparin - an increased risk of thromboembolism, fetal loss, thrombocytopenia, and poor pregnancy outcome. Not prednisone - increased risk of premature rupture of membranes, preterm delivery, fetal growth restriction, infection, preeclampsia, diabetes, osteopenia, and avascular necrosis. Not Progesterone - may be useful for recurrent spontaneous abortion related to a luteal phase defect, but not antiphospholipid antibody syndrome.

A 72-year-old white male presents with a complaint of headache, blurred vision, and severe right eye pain. His symptoms began acutely about 1 hour ago. Examination of the eye reveals a mid-dilated, sluggish pupil; a hazy cornea; and a red conjunctiva. Which one of the following is the most likely diagnosis? (check one) A. Retinal detachment B. Central retinal artery occlusion C. Mechanical injury to the globe D. Acute angle-closure glaucoma

D. Acute angle-closure glaucoma - an acute onset of severe pain, blurred vision, halos around lights, increased intraocular pressure, red conjunctiva, a mid-dilated and sluggish pupil, and a normal or hazy cornea. Retinal detachment - normal vision or peripheral or central vision loss; no pain; increasing floaters; and a normal conjunctiva, cornea, and pupil. Central retinal artery occlusion - amaurosis fugax, a red conjunctiva, a pale fundus, a cherry-red spot at the fovea, and "boxcarring" of the retinal vessels. Mechanical injury to the globe - moderate to severe pain, normal or decreased vision, subconjunctival hemorrhage completely surrounding the cornea, and a pupil that is irregular or deviated toward the injury (SOR B). amaurosis fugas - temporary vision loss to both eyes

A 30-year-old female presents to your office for an initial visit. She reports a long history of asthma that currently awakens her three times per month, necessitating the use of an albuterol inhaler (Proventil, Ventolin). According to current guidelines, which one of the following would be optimal treatment? (check one) A. Continued use of a short-acting β-agonist only as needed B. Adding a long-acting β-agonist C. Adding a leukotriene receptor antagonist D. Adding a low-dose inhaled corticosteroid E. Adding theophylline

D. Adding a low-dose inhaled corticosteroid - mild persistent asthma - Inhaled corticosteroids improve asthma control more effectively in children and adults than any other single long-term controller medication (SOR A).

A hospitalized 55-year-old male has developed the tachyarrhythmia shown in Figure 6. He is alert and denies chest pain, although he complains of palpitations and is mildly dyspneic since the onset of this sustained dysrhythmia. His blood pressure is 116/76 mm Hg and pulse oximetry shows 93% saturation on 2L of oxygen. Figure 6 Which one of the following would be the best treatment for this patient? (check one) A. Synchronized DC cardioversion B. Metoprolol (Lopressor) C. Amiodarone (Cordarone) D. Adenosine (Adenocard) E. Atropine

D. Adenosine - paroxysmal supraventricular tachycardia (PSVT) - a heart rate of approximately 170 Amiodarone - hemodynamically stable ventricular tachycardia

Which one of the following nutritional interventions should be recommended to accelerate pressure ulcer healing in the elderly? (check one) A. Supplemental arginine B. Oral vitamin C and zinc C. High-dose multivitamins D. Adequate protein intake

D. Adequate protein intake - Very few nutritional interventions have been shown to accelerate pressure ulcer healing in the elderly. p Maintaining a protein intake of at least 1.2-1.5 g/kg/day is recommended, and some authorities recommend 2 g/kg/day with stage III or IV ulcers. - Increased caloric intake is also necessary to promote healing

A patient in the first trimester of pregnancy has just learned that her husband has acute hepatitis B. She feels well, and her screening test for hepatitis B surface antigen (HBsAg) was negative last month. She has not been immunized against hepatitis B. Which one of the following would be the most appropriate management of this patient? (check one) A. No further workup or immunization at this time, a repeat HBsAg test near term, and treatment of the newborn if the test is positive B. Use of condoms for the remainder of the pregnancy, and administration of immunization after delivery C. Testing for hepatitis B immunity (anti-HBs), and immunization if needed D. Administration of hepatitis B immune globulin (HBIG) now and hepatitis B vaccine after the first trimester E. Administration of both HBIG and hepatitis B vaccine now

D. Administer Hep B Immunoglobulin & vaccine now - Hepatitis B vaccine is a killed-virus vaccine and can be used safely in pregnancy, with no need to wait until after organogenesis. - The patient is unlikely to be previously immune to hepatitis B, given that she has no history of hepatitis B infection, immunization, or carriage. - Because the patient's HBsAg is negative, she is not the source of her husband's infection. - Full treatment for this patient has an efficacy of only 75% so follow-up testing is still needed. Not condom - too late; she has been exposed for at least 6 weeks

A previously healthy 59-year-old male is brought to the emergency department by his wife, who describes symptoms of confusion and ataxia. She also says that he has had a fever and cough for the past 2 weeks. On examination he has a temperature of 39.0°C (102.2°F), a heart rate of 125 beats/min, a respiratory rate of 25/min, a blood pressure of 85/46 mm Hg, and an O2 saturation of 88%. Laboratory findings include a WBC count of 15,500/mm3 (N 4300-10,800), a glomerular filtration rate of 45%, and a hemoglobin level of 9.1 g/dL (N 13.0-18.0). A chest radiograph reveals a large left lower lobe infiltrate. You start the patient on an appropriate antibiotic regimen. Which one of the following is the most appropriate initial treatment of this patient's hypotension? (check one) A. Dobutamine B. Dopamine C. Norepinephrine D. Aggressive fluid resuscitation E. Packed red blood cell transfusion

D. Aggressive fluid resuscitation SIRS - fever >38.5°C - heart rate >90 beats/min - respiratory rate >20/min - WBC count >12,000/mm3) also meets severe sepsis criteria - positive chest radiograph - evidence of organ hypoperfusion (mental status changes) - septic shock (mean arterial pressure <60 mm Hg). - initial treatment for patients with hypotension in septic shock is fluid resuscitation (SOR A) - vasopressor: only after fluid resuscitation fails to restore mean arterial pressure (>65 mm Hg) or when there is evidence of continued organ hypoperfusion.

Which one of the following serum proteins is typically DECREASED in a hospitalized patient with sepsis? (check one) A. Complement C3 B. Ferritin C. C-reactive protein (CRP) D. Albumin E. Fibrinogen

D. Albumin - acute phase response - liver makes less albumin & transferrin - serum level goes up for other proteins e.g. ceruloplasmin, complement, haptoglobin, fibrinogen, and C-reactive protein - Serum levels of ferritin may be extremely high in certain conditions, but are also influenced by total-body iron stores.

A 30-year-old female reports that she and her husband have not been able to conceive after trying for 15 months. She takes no medications, has regular menses, and has no history of headaches, pelvic infections, or heat/cold intolerance. Her physical examination is unremarkable. Her husband recently had a normal semen analysis. Which one of the following would be the most appropriate next step? (check one) A. Observation for 1 year B. TSH, free T4, and prolactin levels C. Hysterosalpingography D. An estradiol level E. A luteal-phase progesterone level

D. An estradiol level For women who are having regular menstrual cycles, ovulation is very likely. Ovulation can be confirmed by a progesterone level >5 ng/mL on day 21 of the cycle. If this is the case, tubal patency should be confirmed with hysterosalpingography or laparoscopy. Should the progesterone level be <5 ng/mL, anovulation should be investigated with TSH, estradiol, FSH, and prolactin levels. If they are unremarkable it is reasonable to try clomiphene to induce ovulation.

A 7-year-old male is brought to your office with a 10-day history of cough and fever. A chest radiograph shows no acute air-space process but four posterior healing rib fractures. The child's past medical history is unremarkable. Which one of the following would be most appropriate at this point? (check one) A. A skeletal survey B. Studies to evaluate for osteogenesis imperfecta C. Studies to evaluate for rickets D. An immediate referral to initiate a child abuse investigation

D. An immediate referral to initiate a child abuse investigation A skeletal survey is appropriate in a child 2 years of age or younger suspected of being physically abused. It is not thought to be necessary in children 4 years of age or older, especially in a case where suspicious fractures have already been discovered. Posterior or posteromedial rib fractures are secondary to child abuse. Osteogenesis imperfecta can cause continuous beading of the ribs and crumpled long bones such as accordina femora, and is often associated with blue sclerae, skin fragility, or brittle teeth. Rickets usually is associated with long bone bowing deformities. In the chest it can cause prominence of the costochondral junctions (rachitic rosary) and indentation of the lower ribs where the diaphragm attaches (Harrison's grooves).

A 27-year-old white male construction worker suffers from severe plaque-type psoriasis that has required systemic therapy. Which one of the following is associated with this condition? (check one) A. A reduced overall risk of cardiovascular mortality B. A decreased risk of skin cancer with successful treatment C. A low likelihood of recurrence with successful treatment D. An increased risk for the condition in the children of affected individuals E. Low body mass index and difficulty maintaining weight

D. An increased risk for the condition in the children of affected individuals

In a patient presenting with unstable angina, which one of the following findings would denote the highest risk for death or myocardial infarction? (check one) A. New-onset angina beginning 2 weeks to 2 months before presentation B. Angina with hypotension C. Angina provoked at a lower threshold than in the past D. Increased anginal frequency

D. Angina w/ low BP Unstable angina patients at high risk include those with at least one of the following: • Angina at rest with dynamic ST-segment changes over 1 mm • Angina with hypotension • Angina with a new or worsening mitral regurgitation murmur • Angina with an S3 or new or worsening crackles • Prolonged (>20 min) anginal pain at rest • Pulmonary edema most likely related to ischemia

Which one of the following is most characteristic of hoarding disorder? (check one) A. Collecting eccentric or bizarre items B. Collecting only seemingly worthless items C. Deriving pleasure from collected items D. Anxiety and emotional distress if collected items are disposed of

D. Anxiety and emotional distress if collected items are disposed of

You have just received test results confirming that a 78-year-old patient has metastatic lung cancer. She informs you she does not want to know the results of the tests and is "leaving it in God's hands." You know that additional issues need to be explored, such as her desire for chemotherapy and hospice care. Which one of the following is the most appropriate strategy for determining her wishes? (check one) A. Acknowledge her concerns, but proceed with a discussion of her diagnosis and prognosis B. Ask family members to gently break the news to your patient and tell them you will return later to discuss the details and answer questions C. Have a hospice representative visit and discuss the diagnosis and options for care D. Ask the patient to designate someone with whom you can discuss the results and prognosis

D. Ask the patient to designate someone with whom you can discuss the results and prognosis

Screening for osteoporosis should be done in which one of the following groups? (check one) A. Postmenopausal women B. Women over age 50 with a BMI ≥30 kg/m2 C. Men over age 50 with type 2 diabetes mellitus D. Men over age 70

D. Men over age 70 - All women ≥65 (SOR A) - all men ≥70 (SOR C) - men and women age 50-69: factors associated w/ low bone density e.g. low body weight, previous fracture, a family history, a history of falls, physical inactivity, low vitamin D or calcium intake, and the use of certain meds or the presence of certain medical conditions

A mother brings her 2-year-old daughter to your office because the child is not using her left arm. Earlier in the day the mother left the toddler under the supervision of her 12-year-old sister while she went to the store. When she returned the toddler was playing with toys using only her right arm, and was holding the left arm slightly pronated, flexed, and close to her body. The older daughter was unaware of any injury to the girl's arm, and the child does not seem distressed or traumatized. Physical examination of the child's clavicle, shoulder, wrist, and hand do not elicit any signs of pain or change in function. She does seem to have some tenderness near the lateral elbow and resists your attempts to examine that area. There is no ecchymosis, swelling, or deformity of the elbow. Which one of the following would be most appropriate at this point? A. Plain radiographs of the affected elbow B. Ultrasonography of the affected elbow C. Evaluation by an orthopedic surgeon within 24 hours D. Attempted reduction of the subluxed radial head E. Placement in a splint and follow-up in the office if there is no improvement in the next 1-2 weeks

D. Attempted reduction of the subluxed radial head

A 26-month-old child presents with a 2-day history of 6-8 loose stools per day and a low-grade fever. When evaluating the child to determine whether he is dehydrated, which one of the following would NOT be useful? (check one) A. Skin turgor B. Capillary refill time C. Respiratory rate and pattern D. The BUN/creatinine ratio E. The serum bicarbonate level

D. BUN/Cr ratio most useful findings - prolonged capillary refill time, abnormal skin turgor, and abnormal respiratory pattern (SOR C) - Capillary refill time is not affected by fever and should be less than 2 seconds. BUN/creatinine ratio - not useful in children. - normal serum creatinine level changes with age in children Serum bicarbonate level (<17 mmol/L) - helpful in identifying children who are dehydrate - level <13 mmol/L is associated with an increased risk of failure of outpatient rehydration efforts.

The swelling appeared after a sudden painful "pop" as he was lifting a heavy box. A physical examination reveals a soft to firm, nontender mass in the anterior aspect of the arm, and weakness of forearm supination. Shoulder radiographs are normal. Which one of the following is the most likely diagnosis? (check one) A. Acute anterior shoulder dislocation B. Lateral epicondylitis C. Biceps tendinitis D. Biceps tendon rupture

D. Biceps tendon rupture - from eccentric load to a flexed elbow. - Risk factors: age >40, deconditioning, contralateral biceps tendon rupture, a history of rotator cuff tear, rheumatoid arthritis, and cigarette smoking - Weakness in forearm supination and elbow flexion - The biceps squeeze test and the hook test are both sensitive and specific for diagnosing the condition.

In a 15-year-old female with no known chronic medical conditions, which one of the following is essential before initiating oral combined hormonal contraception? (check one) A. Bimanual pelvic examination B. Clinical breast examination C. Cervical cytology and HPV screening D. Blood pressure measurement E. Weight measurement

D. Blood pressure measurement the only medical evaluation identified as essential before prescription of combined hormonal contraception is measurement of blood pressure, as severe hypertension is a contraindication to oral contraception.

A 25-year-old female comes to your office requesting a referral to an otolaryngologist for surgery on her nose. She states that her nose is too large and that "something must be done." She has already seen multiple family physicians, as well as several otolaryngologists. She is 168 cm (66 in) tall and weighs 64 kg (141 lb). A physical examination is normal, and even though she initially resists a nasal examination, it also is normal. The size of her nose is normal. Which one of the following is the most likely cause of this patient's concern about her nose? (check one) A. Obsessive-compulsive disorder B. Anorexia nervosa C. Depression D. Body dysmorphic disorder

D. Body dysmorphic disorder

A 44-year-old male sees you for evaluation of an episode of pink-tinged urine last week. He denies any flank or abdominal pain, as well as frequency, urgency, and dysuria. He has no prior history of renal or other urologic disease, and no other significant medical problems. He has a 24-pack-year smoking history. A urinalysis today reveals 8-10 RBCs/hpf. You refer him to a urologist for cystoscopy. Which one of the following would be the most appropriate additional evaluation? (check one) A. KUB radiography B. Transabdominal ultrasonography C. Voiding cystourethrography D. CT urography E. Magnetic resonance urography

D. CT urography - or intravenous pyelography is recommended by the American College of Radiology as the most appropriate imaging procedure for hematuria in all patients - with the exception of those with generalized renal parenchymal disease, young women with hemorrhagic cystitis, children, and pregnant females.

For which type of renal calculus is acidification of the urine indicated? (check one) A. Cystine B. Uric acid C C. Calcium oxalate D. Calcium phosphate

D. Calcium phosphate - Urine should be acidified for prevention of *calcium phosphate and struvite stones*. Cranberry juice or betaine can lower urine pH - Uric acid, cystine, and calcium oxalate stones tend to form in acidic urine - struvite (magnesium ammonium phosphate) and calcium phosphate stones form in alkaline urine.

A 30-year-old male presents to your office because he thinks he may be suffering from alcohol withdrawal. He was dependent on alcohol for at least 10 years and has completed treatment programs twice. He had been abstinent for over a year until he began drinking heavily after his wife filed for divorce 2 weeks ago. A friend found him in a bar last night and has kept him from consuming alcohol for the past 12 hours. The patient is now nauseated, miserable, restless, shaky, and sweating, and says he can feel his heart pounding. He has not had any seizures or episodes of delirium tremens. His temperature is 37.5°C (99.6°F), pulse rate 100 beats/min, and blood pressure 150/92 mm Hg. His palms are moist and he has a mild tremor on arm extension. He is oriented but cannot perform serial additions. A CBC, basic metabolic panel, and urine drug screen are normal. You decide that outpatient treatment would be appropriate. Which one of the following alcohol withdrawal management options is supported by the best evidence? (check one) A. Thiamine and magnesium B. Carbamazepine (Tegretol) C. Phenytoin (Dilantin) D. Chlordiazepoxide E. Clonidine (Catapres)

D. Chlordiazepoxide Benzodiazepines are the preferred medication for treating alcohol withdrawal (SOR A) and preventing alcohol withdrawal seizures; long acting preferred Not anticonvulsants - less addictive but do not prevent seizures or delirium tremens. Not Clonidine and beta-blockers: do not prevent alcohol withdrawal seizures. Not Thiamine or magnesium - address nutritional deficiencies resulting from alcoholism but would not reduce withdrawal symptoms.

A 16-year-old afebrile, otherwise healthy female presents with a 4-day history of right ear pain. She says she has spent a fair amount of time swimming recently. Traction on the pinna causes pain. The erythema and inflammation is limited to the ear canal but there is too much edema to easily visualize the tympanic membrane. Which one of the following would be the most appropriate treatment? (check one) A. Amoxicillin B. Amoxicillin/clavulanate (Augmentin) C. Amoxicillin/clavulanate plus acetic acid 2% topically D. Ciprofloxacin 0.3%/dexamethasone 0.1% (Ciprodex) topically

D. Cipro 0.3%/dexamethasone 0.1% (Ciprodex) topical Acute otitis externa is treated with topical antibiotics. Although no preparation has been shown to be most effective, a fluoroquinolone does not create a risk of ototoxicity if the tympanic membrane is not intact. Topical corticosteroids may hasten symptom reduction. Oral antibiotics are not appropriate unless the infection has spread beyond the ear canal, or if the patient has diabetes mellitus or is immunocompromised.

Which one of the following is a criterion for gastric bypass surgery, according to recommendations of the National Institutes of Health? (check one) A. A Framingham risk score >25% B. Severe insulin resistance C. Failed pharmacotherapy D. Clearance by a mental health professional E. A BMI >30 kg/m2 with comorbidities

D. Clearance by a mental health professional - Indications for laparoscopic bariatric surgery for morbid obesity include a BMI >40 kg/m2 or a BMI of 35-40 kg/m2 with significant obesity-related comorbidities.

A 45-year-old male presents with a complaint of recent headaches. He has had four headaches this week, and his description indicates that they are moderate to severe, bilateral, frontal, and nonthrobbing. There is no associated aura. He has had similar episodes of recurring headachesin the past. Based on this limited history, which one of the following headache types can be eliminated from the differential diagnosis? (check one) A. Tension-type headache B. Sinus headache C. Migraine headache D. Cluster headache E. Headache of intracranial neoplasm

D. Cluster headache - can be removed from the differential - always unilateral, although the affected side can vary

A 35-year-old male with a 4-month history of pain in the medial aspect of his right knee sees you for follow-up. He has been doing physical therapy for the past month with minimal benefit. A plain radiograph is negative and MRI shows a tear in the medial meniscus. Which one of the following is most likely to yield the best long-term result? (check one) A. Referral for meniscectomy B. Corticosteroid injection C. Hylan GF 20 (Synvisc) injection D. Continued physical therapy E. A knee brace

D. Continue PT Arthroscopic partial meniscectomy is the most common orthopedic procedure performed in the United States, but not beneficial

While performing a digital rectal examination of the prostate on a 67-year-old patient with diabetes mellitus, you note the findings shown below. The patient confirms that the area has been itchy for some time but he has been reluctant to seek care. He has tried a variety of over-the-counter moisturizing lotions with limited success. Of the following topical treatments, which one is most likely to provide significant improvement? (check one) A. Antibacterial ointment B. Antifungal cream C. Antiviral ointment D. Corticosteroid cream E. Rubbing alcohol

D. Corticosteroid cream

Over the last 6 months a developmentally normal 12-year-old white female has experienced intermittent abdominal pain, which has made her quite irritable. She also complains of joint pain and general malaise. She has lost 5 kg (11 lb) and has developed an anal fissure. Which one of the following is the most likely cause of these symptoms? (check one) A. Celiac disease (gluten enteropathy) B. Irritable bowel syndrome C. Hepatitis A D. Crohn's disease E. Giardiasis

D. Crohn's disease - The most common age of onset: adolescence then 50-80 years of age - systemic signs and symptoms are more common than with ulcerative colitis - Perianal disease is also common IBS & viral gastroenteritis - no weight loss and anal lesions are extremely uncommon - historical and epidemiologic findings in this case are not consistent with either of these infections Celiac disease and giardiasis - can have diarrhea and weight loss - no anal fissures.

A 62-year-old female undergoes elective surgery and is discharged on postoperative day 3. A week later she is hospitalized again with pneumonia. A CBC shows that her platelet count has dropped to 150,000/mm3 (N 150,000-300,000) from 350,000 /mm3 a week ago. She received prophylactic heparin postoperatively during her first hospitalization. The patient is started on intravenous antibiotics for the pneumonia and subcutaneous heparin for deep-vein thrombosis prophylaxis. On hospital day 2, she has an acute onset of severe dyspnea and hypoxia; CT of the chest reveals bilateral pulmonary emboli. Her platelet count is now 80,000/mm3 . Which one of the following would be most appropriate at this point? (check one) A. Continue subcutaneous heparin B. Discontinue subcutaneous heparin and start a continuous intravenous heparin drip C. Discontinue heparin and give a platelet transfusion D. Discontinue heparin and start a non-heparin anticoagulant such as argatroban or desirudin (Iprivask) E. Discontinue unfractionated heparin and start a low molecular weight heparin such as enoxaparin (Lovenox)

D. Discontinue heparin and start a non-heparin anticoagulant such as argatroban or desirudin (Iprivask) - Heparin-induced thrombocytopenia HIT - within 1-2 weeks of heparin administration - will have HIT antibodies in the serum - unexplained 30%-50% decrease in the platelet count - may have arterial or venous thrombosis, anaphylactoid reactions immediately following heparin administration, or skin lesions at the site of heparin injections - Postop patients receiving subcutaneous unfractionated heparin prophylaxis are at highest risk for HIT

A 69-year-old male presents with a 2-week history of fever, fatigue, weight loss, and mild diarrhea. He is found to have a mildly tender mass in the left lower quadrant of the abdomen. The most likely diagnosis is: (check one) A. Crohn's disease B. ulcerative colitis C. celiac disease D. diverticulitis E. lymphoma

D. Diverticulitis - left lower quadrant in the elderly and may present as an abscess Crohn's disease - distal small intestine (regional enteritis) - most typically in a young person - usually in the second or third decade of life. Ulcerative colitis - usually presents with a longer history - does not typically present with a mass Lymphoma 2-week history of a palpable mass is not a typical presentation Celiac disease - does not cause a palpable left lower quadrant mass.

Which one of the following has been shown to be effective for Lyme disease prophylaxis after removal of an engorged deer tick? (check one) A. Amoxicillin B. Ceftriaxone (Rocephin) C. Cefuroxime axetil (Ceftin) D. Doxycycline E. Clarithromycin (Biaxin)

D. Doxycycline a single 200-mg dose of doxycycline was 87% effective for preventing Lyme disease if given within 72 hours after removal of a deer tick.

A 65-year-old female presents to the emergency department as directed by her primary care physician because of "high potassium" that was found today during routine laboratory monitoring. The patient has a past medical history significant for diet-controlled diabetes mellitus, hypertension, and asthma. She feels well and specifically denies palpitations, fatigue, changes in urine output, and muscle cramps. You do not have access to the patient's outpatient medical records and order a chemistry panel in the emergency department with the following results: Sodium. . 143 mEq/L (N 135-145) Potassium. . 6.3 mEq/L (N 3.5-5.0) CO2. . ..27 mEq/L (N 22-30) Creatinine.. .1.6 mg/dL (N 0.6-1.0) BUN. . 30 mg/dL (N 7-21) Which one of the following is the first additional test that should be obtained in the diagnostic evaluation of this patient? (check one) A. A urinalysis B. A CBC C. Arterial blood gases D. An EKG E. Renal ultrasonography

D. EKG takes precedent

In a patient with chronic hepatitis B, which one of the following findings suggests that the infection is in the active phase? (check one) A. A normal liver biopsy B. Detectable levels of HBeAb C. Detectable levels of HBsAb D. Elevated levels of ALT E. Undetectable levels of HBV DNA

D. Elevated levels of ALT - also HBeAg, indicating high levels of HBV DNA anti-HBeAg, Carrier Stat - Anti-HBe - normalization of liver enzymes - greatly reduced levels of hepatitis B virus in the bloodstream.

A 32-year-old female comes to your office because she has had increasing difficulty performing her daily tasks for the past 6 months. She says she worries excessively about routine events every day and constantly feels tense, restless, jittery, "on edge," irritable, and unable to relax. She also reports that sometimes her heart pounds and races, her hands feel sweaty and clammy, and her mouth feels like it is "full of cotton." She reports difficulty concentrating and falling asleep at night. A recent TSH level was normal. During your examination you note frequent sighing, a fine tremor in her hands, bitten nails, and clammy hands. Based on her symptoms and examination, which one of the following pharmacologic agents for managing her condition is supported by the best available evidence? A. Alprazolam (Xanax) B. Buspirone C. Hydroxyzine D. Escitalopram (Lexapro) E. Quetiapine (Seroquel)

D. Escitalopram (Lexapro) - Generalized Anxiety Disorder

A 12-year-old male who lives on a farm presents with lesions on his toes (shown in Figure 7). Which one of the following items from the patient's history is relevant to the diagnosis? Figure 7 (check one) A. Recent tooth extraction and gingival surgery B. A family history of systemic lupus erythematosus C. Recurrent fevers for the past 2 weeks D. Exposure to cold temperatures E. Vaccination of the sheep he is raising for a 4-H project

D. Expose to cold temperature - pernio, or chilblains - localized inflammatory lesion of the skin - in the extremities following exposure to nonfreezing cold temperatures. - benign condition, - red-purple plaques with deep swelling, and are accompanied by itching or burning. - not associated with infections or connective tissue disease.

A mother brings her 12-month-old son to your clinic, concerned that he is repeatedly banging his head against the floor, wall, or crib. She reports that this behavior began about 2 months ago. It now occurs several times per week, and at times is incited when the child is frustrated with a toy or when he does not get what he wants from his parents. The mother notes that she is sometimes awakened at night by the sound of her son rhythmically banging his head against the rail of his crib. Physical examination reveals a normal child with some soft-tissue swelling of the forehead, but no broken skin, ecchymosis, or signs of bony damage. Developmental milestones and growth have been normal, and the child is not on any medications. Children with this presentation are most likely to have which one of the following? (check one) A. A history of child abuse B. A skin laceration or skull fracture C. An eventual diagnosis of Lesch-Nyhan syndrome D. Extinction of this habit by age 3 E. Future cognitive delay when compared with children without this habit

D. Extinction by age 3 - 3%-15% of normal children and usually begins between the ages of 5 and 11 months. - The vast majority of these children will engage in this activity for only a few months, and most will stop by age 3.

A 61-year-old female tells you that her brother was recently diagnosed with hereditary hemochromatosis and his physician suggested that she get tested. She feels well and has no significant health problems. Which one of the following would be most appropriate for initial screening? (check one) A. Serum transaminases B. A CBC and a serum iron level C. Testing for the HFE gene D. Ferritin and transferrin saturation E. Total iron binding capacity

D. Ferritin and transferrin saturation The transferrin saturation is calculated by dividing the serum iron level by the total iron binding capacity. If the serum ferritin level is elevated (>200 ng/mL in women) or the transferrin saturation is ³45% the HFE gene should be checked.

A 70-year-old male sees you for a routine annual evaluation. He complains of fatigue but has no other symptoms. He has a history of hypertension but has not fully adhered to his drug regimen, which includes hydrochlorothiazide, amlodipine (Norvasc), and lisinopril (Prinivil, Zestril). Laboratory Findings Hemoglobin 9.0 g/dL (N 13.5-17.2) Serum creatinine 2.2 mg/dL (N 0.6-1.2) Glomerular filtration rate 26 mL/min/1.73 m2 Serum iron 30 g/dL (N 60-170) Total iron binding capacity 300 :g/dL (N 240-450) Ferritin 55 ng/mL (N 46-100) Mean corpuscular volume 77 :m3 (N 80-100) One year ago the patient had a serum creatinine level of 2.0 mg/dL. A colonoscopy 6 months ago was unremarkable and a stool test for occult blood is negative. Which one of the following would be most appropriate at this point? (check one) A. An erythropoietin level B. Transfusion of packed RBCs C. Epoetin alfa (Procrit) D. Ferrous sulfate orally E. Intravenous iron therapy

D. Ferrous sulfate orally CKD 5 Stage: Stage 1 is defined as a GFR >90 mL/min/1.73 m2, Stage 5 (kidney failure) GFR <15 mL/min/1.73 m2. Anemia is associated with all stages: Hgb: 13.5 g/dL in men or 12.0 g/dL in women. CKD anemia from decreased erythropoietin, but testing not needed, just CBC, reticulocyte, ferritin, B12, folate, transferrin serum ferritin level <25 ng/mL is indicative of low iron stores Treat: iron Patients with depleted iron stores will benefit from replenishment, which serves to correct an isolated iron orally with ferrous sulfate, 325 mg 3 times a day; repeat CBC, transferrin, ferritin in 1-3 mos For patients who do not respond to iron replacement, erythropoiesis-stimulating agents such as epoetin alfaor darbepoetin alfa should be used. The goal should be to relieve symptoms such as fatigue and to achievea hemoglobin level of 11-12 g/dL. Levels >13 g/dL increase the mortality rate, particularly from cardiovascular disease.

A 63-year-old female with community-acquired pneumonia is being treated with appropriate antibiotics. The only abnormality on a basic metabolic panel is a serum sodium level of 121 mEq/L (N 135-145). Her shortness of breath and cough are improving. She has no other complaints on a review of systems. On examination the patient is noted to have normal vital signs and mucous membranes are moist. She has crackles in her right lower lobe. Skin turgor is normal. The remainder of the physical examination is normal. Further testing reveals the following: Urine sodium. . . . . . . . . . . . . . . . . . . . . . . . . . 50 mEq/L Serum osmolality. . . . . . . . . . . . . . . . . . . . . . . 276 mOsm/kg (N 280-285) Urine osmolality. . . . . . . . . . . . . . . . . . . . . . . ..300 mOsm/kg Which one of the following would be most appropriate at this point? (check one) A. Intravenous diuretics B. Intravenous hypertonic saline C. Intravenous isotonic saline D. Fluid restriction E. No further interventions

D. Fluid restriction Hypotonic hyponatremia, manifested by low serum osmolality, no signs of hypovolemia. High U(Na) + low U(Osmolality) = SIADH Fluid restriction = mild euvolemic hyponatremia IV isotonic saline = mild hypovolemic hyponatremia IV hypertonic saline = severe hyponatremia IV diuretics = hypervolemic hyponatremia

which one of the following effects of antioxidant supplementation has been demonstrated in randomized clinical trials? (check one) A. Decreased mortality with vitamin A supplementation B. Decreased mortality with β-carotene supplementation C. Decreased mortality with vitamin E supplementation D. Increased mortality with some antioxidant supplements

D. Increased mortality with some antioxidant supplements

A 58-year-old male presents with recent behavior and personality changes, and you suspect dementia. Which one of the following is most likely to present in this manner? (check one) A. Alzheimer's disease B. Vascular dementia C. Mixed Alzheimer's disease and vascular dementia D. Frontotemporal dementia E. Progressive supranuclear palsy

D. Frontotemporal dementia - personality changes

A 60-year-old white female with type 1 diabetes mellitus presents with early satiety, nausea, bloating, and postprandial fullness. Laboratory tests are normal, as are upper endoscopy and biliary ultrasonography. Which one of the following would help confirm the most likely diagnosis? (check one) A. Pelvic ultrasonography B. An exercise stress test C. Psychiatric consultation D. Gastric emptying scintigraphy E. Colonoscopy

D. Gastric emptying scintigraphy - gastroparesis - autonomic neuropathy more commonly seen in type 1 diabetics and in women - The initial evaluation should include a patient history and examination, a CBC to rule out infection, a metabolic panel, endoscopy, and a biliary tract evaluation - diagnosis is best confirmed by scintigraphy. Not Pelvic ultrasonography and colonoscopy - not indicated because the patient's symptoms are upper intestinal Not Cardiac evaluation and psychiatric consultation - are not warranted with these symptoms.

Which one of the following is the most likely cause of hearing loss in newborns? (check one) A. Intraventricular hemorrhage B. Anomalies of the external ear canal C. Congenital cholesteatoma D. Genetic disorders E. Infectious diseases

D. Genetic disorders - (e.g., Waardenburg syndrome, Usher's syndrome, Alport syndrome, and Turner's syndrome) are responsible for more than 50% of hearing impairments in children. - Intraventricular hemorrhage is a central cause of hearing loss, and is rare. - Conductive abnormalities such as external canal anomalies and congenital cholesteatoma, and sensorineural causes other than genetic disorders (e.g., infectious diseases) are important but less frequent.

A 71-year-old female with end-stage lung cancer was recently extubated and is awaiting transfer to hospice. She is awake and confused and has significant respiratory secretions. Which one of the following medications used for reducing respiratory secretions is LEAST likely to cause central nervous system effects such as sedation? (check one) A. Atropine B. Transdermal scopolamine (Transderm Scop) C. Hyoscyamine (Levsin) D. Glycopyrrolate (Robinul)

D. Glycopyrrolate (Robinul) Glycopyrrolate does not cross the blood-brain barrier, and is therefore least likely to cause central nervous system effects such as sedation. The other medications listed do cross the blood-brain barrier.

A 25-year-old female who is 3 months post partum presents with multiple complaints, including increasing weakness and fatigue, intolerance to warm environments, a weight loss of 30 lb despite an increased appetite, difficulty sleeping, awareness that her heart is beating faster and "pounding" in her chest, increasing restlessness and difficulty concentrating, increased tremulousness, and a significant swelling in her neck. She takes no medication, has experienced no recent trauma, and has not ingested large amounts of iodine. When you examine her you find no exophthalmos or lid lag and no pretibial edema, but her skin is warm, smooth, and moist. You also find a smooth, non-nodular, nontender, enlarged thyroid gland, clear lungs, a resting tremor, and hyperactive reflexes. Laboratory testing reveals a low TSH level, elevated free T3 and free T4, and high uptake on a radioactive iodine uptake scan. Which one of the following is the most likely diagnosis? (check one) A. Postpartum thyroiditis B. Silent thyroiditis C. Subacute thyroiditis D. Graves disease E. Exogenous thyroid ingestion

D. Graves disease This patient has symptoms consistent with hyperthyroidism, which could be caused by any of the options listed. But only Graves disease will cause high radioactive iodine uptake on a thyroid scan

Which one of the following is recommended to reduce the risk of sudden infant death syndrome (SIDS)? (check one) A. The use of home cardiorespiratory monitors B. The use of soft bedding materials C. Having the infant sleep in a prone position D. Having the infant sleep in a separate bed E. Maintaining a room temperature of 78°F-80°F when the infant is sleeping

D. Having the infant sleep in a separate bed

A white male adolescent is concerned because he is the shortest boy in his class. His age is 14.3 years and his parents are of normal height. He has a negative past medical history and no symptoms. On physical examination you note that he is 151 cm (59 in) tall. The average height for his age is 165 cm. His weight is 43 kg (95 lb). His sexual maturity rating is 3 for genitalia and 2 for pubic hair. A wrist radiograph shows a bone age of 12.2 years (the average height is 152 cm for this bone age). On the basis of this evaluation you can tell the patient and his parents that (check one) A. he should have a growth hormone stimulation test B. his adult height will be below average C. his sexual development is about average for his age D. he will begin to grow taller within a year or so E. an underlying nutritional deficiency may be the cause of his short stature

D. He will begin to grow taller in a year or so - Constitutional growth delay - delayed but eventually normal growth in an adolescent, is usually genetic. - If no chronic disease, if his sexual maturity rating is 2 or 3, and if his height is appropriate for skeletal age, he can be told without endocrinologic testing that he will begin to grow taller within a year or so. - Adult height may be below average, but cannot be predicted reliably. - Average sexual maturity ratings for a male of 14.3 years are 4 for genitalia and 3 to 4 for pubic hair. - +The history and physical examination would have given clues to any illnesses or nutritional problems.

Brain natriuretic peptide (BNP) is a marker for which one of the following? (check one) A. Renal failure B. Acute adrenal insufficiency C. Cerebrovascular accident D. Heart failure E. Ureteral obstruction

D. Heart Failure

An 86-year-old female presents to your office with a complaint of increasing cough, especially at night, over the past 2-3 weeks. On examination you hear crackles at the bases of both lungs. The chest radiograph shown in Figure 1 is consistent with which one of the following causes of this patient's cough? Figure 1 (check one) A. Bilateral pneumonia B. Asbestosis C. Tuberculosis D. Heart failure E. Emphysema

D. Heart failure - cardiomegaly, with a cardiothoracic ratio >6.50, as well as some enlargement of pulmonary veins due to pulmonary venous hypertension - no infiltrate, so Not community-acquired pneumonia. - no pleural plaques so Not asbestosis - no upper-lobe so Not tuberculosis. - Not emphysema, there is typically a small vertical heart and evidence of hyperexpansion.

Which one of the following should be monitored during testosterone replacement therapy? (check one) A. Patient Health Questionnaire 9 (PHQ-9) scores B. Fasting glucose levels C. Fasting lipid profiles D. Hematocrit E. Overnight polysomnography

D. Hematocrit - testosterone may cause erythrocytosis

A 68-year-old female with diabetes mellitus, coronary artery disease, fibromyalgia, and dyspepsia presents for follow-up. She has been taking omeprazole (Prilosec) for 10 years. It was started during a hospitalization, and her symptoms have returned with previous trials of discontinuation. Which one of the following adverse events is this patient at risk for as a result of her omeprazole use? (check one) A. Hypermagnesemia B. Urinary tract infections C. Nephrolithiasis D. Hip fractures

D. Hip fractures

A 56-year-old female comes in for evaluation of gradually worsening right hip pain. She describes her pain as located in the groin and dull in nature, and with activity often notes a clicking sensation associated with sharp pain. On examination her hip range of motion is intact but pain is elicited with extremes of internal and external rotation and her groin pain is exacerbated with the FABER test (knee flexion, abduction and external rotation of the leg until the ankle rests proximal to the contralateral knee) and FADIR test (knee flexion, adduction, and internal rotation of the leg). Which one of the following is the most likely diagnosis? (check one) A. Femoral neck fracture B. Femoral hernia C. Trochanteric bursitis D. Hip labral tear

D. Hip labral tear - Dull or sharp groin pain radiates to the lateral hip, anterior thigh, or buttock. - insidious onset - may have mechanical symptoms e.g. catching or painful clicking with activity Not fracture - pt has no history Not femoral hernia - pain is supposed to be worse with straining of lifting; with bulge in upper thigh Not Trochanteric bursitis - lateral hip pain with point tenderness over the greater trochanter of the femur.

A 55-year-old male is brought to the emergency department because of confusion and seizures. He has a history of hypertension and obstructive sleep apnea due to obesity. He is not conscious and no other history is available. An examination shows no focal neurologic findings, but a general examination is limited because of his size. Breath sounds are diminished, and heart sounds are difficult to hear. He has venous insufficiency changes on his lower extremities, with brawny-type edema. Laboratory testing reveals a sodium level of 116 mmol/L (N 135-145), but normal renal and liver functions. A chest radiograph shows mild cardiomegaly. A BNP level is pending, but immediate treatment is felt to be indicated. Which one of the following is the treatment of choice for this patient? (check one) A. Valsartan (Diovan) B. Furosemide C. Vasopressin (Pitressin) D. Hypertonic saline E. Conivaptan (Vaprisol)

D. Hypertonic saline - severe hyponatremia manifested by confusion and seizures, a life-threatening situation warranting urgent treatment with hypertonic (3%) saline. - The serum sodium level should be raised by only 1-2 mmol/L per hour, to prevent serious neurologic complications - Saline should be used only until the seizures stop - Some authorities recommend concomitant use of furosemide, especially in patients who are likely to be volume overloaded, as this patient is, but it should not be used alone. - The arginine vasopressin antagonist conivaptan is approved for the treatment of euvolemic or hypervolemic hyponatremia, but not in patients who are obtunded or in a coma, or who are having seizures.

Occlusion of the circumflex artery is most likely to cause EKG changes in: (check one) A. V1 and V2 B. V3 and V4 C. II, III, and AVF D. I and AVL

D. I an AVL Circumflex occlusion causes changes in I, AVL, and possibly V5 and V6 as well. Left anterior descending coronary artery occlusion causes changes in V1 to V6. Right coronary occlusion causes changes in II, III, and AVF.

Which one of the following is the best INITIAL management for hypercalcemic crisis? (check one) A. Intravenous furosemide B. Intravenous pamidronate (Aredia) C. Intravenous plicamycin (Mithramycin) D. Intravenous saline

D. IV saline The initial management of hypercalcemic crisis involves volume repletion and hydration. The combination of inadequate fluid intake and the inability of hypercalcemic patients to conserve free water can lead to calcium levels over 14-15 mg/dL

A 30-year-old ill-appearing male presents with right hand and arm pain and a rapidly expanding area of redness. On examination he temp is 38.9°C (102.0°F), a pulse rate of 120beats/min, and a blood pressure of 116/74 mm Hg. He also has erythema from the dorsal hand to the elbow, violaceous bullae on the dorsal hand and wrist, and severe pain with dorsiflexion of the wrist or fingers. Which one of the following is the most appropriate initial step in the management of this patient? A. Oral dicloxacillin and outpatient follow-up within the next 24 hours B. Intravenous metronidazole C. Consultation with an infectious disease specialist D. Immediate surgical consultation for operative debridement E. Incision and drainage with wound cultures in the emergency department

D. Immediate surgical consultation for operative debridement - Severe pain and skin changes outside the realm of cellulitis, including bullae and deeper discoloration, are strong indications of necrotizing fasciitis. - Antimicrobial therapy is essential but is not sufficient by itself; aggressive surgical debridement within 12 hours reduces the risk of amputation and death.

A 35-year-old male amateur rugby player seeks your advice because right hip pain of several months' duration has progressed to the point of interfering with his athletic performance. The pain is accentuated when he transitions from a seated to a standing position, and especially when he pivots on the hip while running, but he cannot recall any significant trauma to the area and finds no relief with over-the-counter analgesics. On examination his gait is stable. The affected hip appears normal and is neither tender to palpation nor excessively warm to touch. Although he has a full range of passive motion, obvious discomfort is evident with internal rotation of the flexed and adducted right hip. Which one of the following is most strongly suggested by this clinical picture? (check one) A. Osteoarthritis B. Avascular necrosis C. Bursitis D. Impingement E. Pathologic fracture

D. Impingement - femoroacetabular impingement. - Gradually worsening anterolateral hip joint pain that is sharply accentuated when pivoting laterally on the affected hip or moving from a seated to a standing position - is consistent with Reproduction of the pain on range-of-motion examination by manipulating the hip into a position of flexion, adduction, and internal rotation (FADIR test) is the most sensitive physical finding. - Special radiographic imaging of the flexed and adducted hip can emphasize the anatomic abnormalities associated with impingement that may go unnoticed on standard radiographic series views. Avascular necrosis - pain is similarly insidious and heightened when bearing weight, tenderness is usually evident with hip motion in any direction. Osteoarthritis of the hip - generally occurs in individuals of more advanced age - the pain produced is typically localized to the groin area and can be elicited by flexion, abduction, and external rotation (FABER test) of the affected hip Bursitis - soreness after exercise and tenderness over the affected bursa.

A 78-year-old white male is scheduled to undergo CT with contrast. His current diagnoses include type 2 diabetes mellitus, heart failure, anemia of chronic disease, and renal insufficiency. Evidence supports the use of which one of the following to reduce the risk of contrast-induced nephropathy in this patient? (check one) A. Intravenous furosemide B. Ascorbic acid C. Calcium antagonists D. Isotonic bicarbonate infusion E. High osmolar contrast media

D. Isotonic bicarb infusion - Low-osmolar or iso-osmolar contrast media should be used to prevent contrast-induced nephropathy in at-risk patients. - hydration before the procedure, preferably with isotonic saline or isotonic sodium bicarbonate solution is best

A 20-year-old college wrestler is seen for an examination prior to the wrestling season. He tells you that some friends have told him he should start taking dehydroepiandrosterone (DHEA), and he asks for your advice. Which one of the following is true about the effects of this drug? (check one) A. It enhances performance but not muscle strength B. It enhances muscle strength but not performance C. It enhances both performance and muscle strength D. It does not enhance either performance or muscle strength

D. It does not enhance either performance or muscle strength - Like androstenedione, DHEA is a precursor to testosterone, but neither of these substances has been shown to enhance either performance or strength. In fact, they increase serum estrogen and luteinizing hormone levels - Dehydroepiandrosterone (DHEA) is illegal under the Anabolic Steroid Control Act of 2004, and is prohibited by the NCAA and the International Olympic Committee.

Which one of the following is true concerning breast cancer screening? (check one) A. It is useful for detecting premalignant conditions B. It can predict which of the discovered cancers are indolent, with a low potential for harm C. The decrease in mortality from breast cancer can be attributed almost entirely to early detection D. It has resulted in an increase in the diagnosis of localized disease E. It has resulted in a significant decrease in the incidence of regional and metastatic disease

D. It has resulted in an increased in the dx of localized disease - it cannot predict which of the discovered cancers are more aggressive, and cannot accurately detect premalignant lesions. - The decrease in the mortality rate of breast cancer is due both to earlier detection and better follow-up medical care.

Which one of the following is true regarding respiratory syncytial virus (RSV) infection? (check one) A. Most infections in the United States occur between August and December B. Corticosteroids should be a routine part of treatment C. The diagnosis is usually based on positive serology D. It is rarely associated with bacterial co-infection

D. It is rarely associated with bacterial co-infection Respiratory syncytial virus (RSV) is a common cause of respiratory tract infections in children. The infections are usually self-limited and are rarely associated with bacterial co-infection, but in very young infants, prematurely born infants, or those with pre-existing heart/lung conditions, the infection can be severe. In North America, RSV season is November to April. Treatment is primarily supportive, including a trial of bronchodilators, with continued use only if there is an immediate response. Corticosteroids and antibiotics are not routinely indicated (SOR B). Routine laboratory and radiologic studies should not be used in making the diagnosis, as it is based on the history and physical examination (SOR C).

You see a 27-year-old male with autosomal dominant polycystic kidney disease. He has no other medical problems and his renal function has always been normal on annual testing. Today the patient reports his blood pressure at home has been 142-150/84-90 mm Hg. His blood pressure at this visit is 145/88 mm Hg. Which one of the following medications is preferred for the initial management of hypertension in this patient? (check one) A. Amlodipine (Norvasc) B. Chlorthalidone C. Furosemide (Lasix) D. Lisinopril (Prinivil, Zestril)

D. Lisinopril (Prinivil, Zestril)

Which one of the following is an appropriate rationale for antibiotic treatment of Bordetella pertussis infections? (check one) A. It delays progression from the catarrhal stage to the paroxysmal stage B. It reduces the severity of symptoms C. It reduces the duration of illness D. It reduces the risk of transmission to others E. It reduces the need for hospitalization

D. It reduces the risk of transmission to others - effective for eradicating bacterial infection but not for reducing the duration or severity of the disease. - The eradication of infection is important for disease control because it reduces infectivity - most effective if started early in the course of the illness i.e. catarrhal phase. - Paroxysmal stage follows the catarrhal phase. - Tx: macrolides - azithromycin for 3-5 days or clarithromycin for 7 days - Children under 1 month of age should be treated with azithromycin b/c erythromycin causes hypertrophic pyloric stenosis in young infants - Trimethoprim/sulfamethoxazole can be used in patients with macrolide allergy or resistance (not for peds under 2 mo)

A generally healthy 35-year-old female has mild generalized anxiety, but is not depressed. She does not want to take a prescription medication, and asks if an herbal or dietary supplement might be helpful. Which one of the following botanical medications has the best clinical evidence of potential benefit for anxiety disorders when used for a short time (up to 24 weeks)? (check one) A. St. Johns wort B. Valerian C. Passionflower D. Kava E. Chamomile Correct.

D. Kava (no explanation)

An asymptomatic 60-year-old male sees you for a health maintenance visit. His past medical history is significant for hypertension and hyperlipidemia. His medications include chlorthalidone, 25 mg daily, and atorvastatin (Lipitor), 20 mg daily. He smoked 2 packs of cigarettes a day for 20 years but quit 5 years ago. The physical examination is normal. Laboratory findings include a normal basic metabolic panel, a cholesterol level of 210 mg/dL, an HDL-cholesterol level of 34 mg/dL, an LDL-cholesterol level of 150 mg/dL, and a triglyceride level of 200 mg/dL. Which one of the following screening tests is recommended by the U.S. Preventive Services Task Force for this patient? (check one) A. Prostate-specific antigen (PSA) B. A bone density test C. Abdominal ultrasonography D. Low-dose chest CT E. Carotid ultrasonography

D. Low-dose chest CT - 55-80 and healthy - current smokers or have quit within the past 15 years, and have a 30-pack-year history of smoking

A 36-year-old female has been seen multiple times in the past several months for various pain-related complaints. On each occasion, no physical or laboratory findings were found to explain the symptoms. The patient is involved in a workers compensation case and could make a significant amount of money if it is demonstrated that her physical complaints are related to work conditions. Which one of the following diagnoses characterizes her unexplained physical symptoms? (check one) A. Somatization disorder B. Conversion disorder C. Hypochondriasis D. Malingering

D. Malingering - purposefully feign physical symptoms for external gain. Factitious disorder - adopting physical symptoms for unconscious internal gain, such as deriving comfort from taking on the role of being sick Somatization disorder - related to numerous unexplained physical symptoms that last for several years and typically begin before 30 years of age. Conversion disorder - a single voluntary motor or sensory dysfunction suggestive of a neurologic condition, but not conforming to any known anatomic pathways or physiologic mechanisms.

Electrosurgical destruction is contraindicated for which one of the following skin lesions? (check one) A. Cherry angiomata B. Pyogenic granuloma C. Basal cell carcinoma D. Melanoma E. Actinic keratosis

D. Melanoma Contraindications to treatment with electrosurgery include the use of a pacemaker and the treatment of melanoma.

A 57-year-old female with a past medical history significant for well-controlled type 2 diabetes mellitus, hypertension, and hyperthyroidism presents to your office with a chief complaint of a sore throat and a fever to 101.5°F at home. She has had chills and night sweats but has not had a cough, chest pain, or abdominal pain. Physical Findings General.. . . . ill appearing HEENT. . . . . . diffuse tender anterior cervical adenopathy; thyroid nontender; oropharynx erythematous with some pus on her tonsils Cardiovascular.. .tachycardia without murmur Skin. . . . mild jaundice Laboratory Findings Rapid strep test. . . . . . negative Total WBC count. . . . . 3000/mm3 (N 4500-11,000) and absolute neutrophil count 0 Total bilirubin. . . . . . . . 5 mg/dL (N 0-1.0) Alkaline phosphatase. . 151 U/L (N 38-126) Which one of the following medications is most likely to cause these laboratory abnormalities? (check one) A. Amlodipine (Norvasc) B. Aspirin C. Metformin (Glucophage) D. Methimazole (Tapazole)

D. Methimazole (Tapazole): agranulocytosis + cholestatic jaundice 0.3% methimazole pt develop agranulocytosis, usually within the first 60 days of starting therapy - Other rare complications: serum sickness, cholestatic jaundice, alopecia, nephrotic syndrome, hypoglycemia, and loss of taste. - also increased risk of fetal anomalies, so propylthiouracil (PTU) is preferred in pregnancy.

A 42-year-old male with a 4-year history of multiple sclerosis (MS) presents with an acute attack manifested by ataxia, incoordination, and dysarthria. Which one of the following is indicated for managing this flare-up of his MS? (check one) A. Fingolimod (Gilenya) B. Glatiramer (Copaxone) C. Interferon-β (Avonex, Betaseron) D. Methylprednisolone (Medrol) E. Pramipexole (Mirapex)

D. Methylprednisolone (Medrol) Corticosteroids, either orally or parenterally, are the first-line treatment for acute exacerbations of multiple sclerosis (MS) (SOR A). Disease-modifying agents such as interferon beta, glatiramer, and immunosuppressants such as fingolimod may decrease the frequency of exacerbations and slow the progression of MS but are not the agents of first choice for treatment of acute flareups. Pramipexole does not have a primary role in the treatment of MS, although it might be used to treat certain specific symptoms as an adjunct therapy.

A 45-year-old female with no significant past medical history presents to your office with 2 weeks of worsening pain in her right arm. For the past 2 months she has worked on a plastics manufacturing assembly line. A physical examination reveals no swelling and a normal range of motion. She has normal strength in the upper extremity but she experiences increased pain with extension of her right wrist against resistance. Palpation reveals marked tenderness over the lateral epicondyle of the right arm. Which one of the following is most likely to improve the patient's long-term outcome? (check one) A. Physical therapy B. Regular physical activity using her hands and arms C. Use of an inelastic, nonarticular proximal forearm strap D. Modifying her work routines

D. Modifying her work routines lateral epicondylitis, aka tennis elbow - overuse tendinopathy of the common extensor tendon origin of the lateral elbow - Conservative care: offloading the involved tendons improving outcomes at 1 year - Physical therapy can improve pain and function in the short term only (SOR A). - The evidence is weaker for bracing, with some studies showing improved pain and function at 3-6 weeks (SOR B). - corticosteroid injections reduce acute pain for up to 6 weeks, their use increases rates of poor long-term outcomes (SOR A).

A 35-year-old male presents with a 4-month history of pain in the lower lumbar region without radiation. He works in retail sales, and the pain and stiffness prevent him from working. He estimates the pain to be 7 on a 10-point scale. He has been under the care of a chiropractor and has experienced some relief with spinal manipulation. His history is negative for red flags indicating a serious cause for his pain. The only positive findings on a physical examination are diffuse mild tenderness over the lumbar region and mild limitation of lumbar mobility on forward and lateral flexion/extension maneuvers. Appropriate laboratory tests and imaging studies are all within normal limits. In addition to appropriate analgesics, which one of the following modalities has the best evidence of long-term benefit in this situation? (check one) A. Transdermal electric nerve stimulation (TENS) B. Epidural corticosteroid injections C. SSRIs D. Multidisciplinary rehabilitation

D. Multidisciplinary rehabilitation - lumbar strain or sprain. In addition to analgesics (e.g., acetaminophen or NSAIDs) (SOR A) and spinal manipulation (SOR B), a multidisciplinary rehabilitation program is the best choice for management (SOR A)

A 45-year-old male presents to the emergency department with a complaint of acute, sharp chest pain relieved only by leaning forward. On examination you hear a pericardial friction rub. An EKG shows diffuse ST elevations. Echocardiography reveals a small pericardial effusion. Which one of the following is the most appropriate initial treatment? (check one) A. β-Blockers B. Nitrates C. Glucocorticoids D. NSAIDs

D. NSAIDs Acute pericarditis: colchicine and/or NSAIDs for the first episode of mild to moderate pericarditis. Not beta-Blockers would only be appropriate if the cause of the patient's chest pain were an infarction or ischemia. Not Nitrates: do not relieve the pain of pericarditis. Not: Glucocorticoids - for use in patients with severe or refractory cases or in cases where the likely cause of the pericarditis is connective tissue disease, autoreactivity, or uremia (SOR C).

A 56-year-old female has a 35-pack-year smoking history. She is concerned that she may have COPD, although she has no history of chronic cough, chest pain, or other pulmonary symptoms. Her family history is remarkable for a mother with COPD who was a smoker, but there is no family history of α1-antitrypsin disease. Which one of the following would you recommend with regard to screening spirometry? (check one) A. Screening, based on her age B. Screening, based on her family history C. Screening, based on her smoking history D. No screening, based on lack of benefit

D. No screening, based on lack of benefit USPSTF: "moderate certainty" that screening asymptomatic patients for COPD using spirometry has little or no benefit and is not recommended. This recommendation applies to otherwise healthy individuals without a family history of α1-antitrypsin disease.

A 65-year-old female develops gram-negative septicemia from a urinary tract infection. Despite the use of fluid resuscitation she remains hypotensive, with a mean arterial pressure of 50 mm Hg. Which one of the following would be the most appropriate treatment for this patient? (check one) A. Vasopressin (Pitressin) B. Phenylephrine (Neo-Synephrine) C. Epinephrine D. Norepinephrine (Levophed) E. Low-dose dopamine

D. Norepi (Levophed) - sepsis - vasopressors are indicated when fluid resuscitation does not restore organ perfusion and blood pressure. - Norepinephrine and dopamine currently are the preferred pressor agents - Norepinephrine appears to be more effective and has a lower mortality rate - Vasopressin is employed after high-dose conventional vasopressors have failed.

Which one of the following is a significant risk factor for esophageal adenocarcinoma? (check one) A. Aspirin therapy B. Ibuprofen therapy C. Helicobacter pylori infection D. Obesity E. Crohn's disease

D. Obesity Esophageal adenocarcinoma has become the predominant type of esophageal cancer in North America and Europe, and gastroesophageal reflux and obesity are the main risk factors.

A 75-year-old white female presents with back pain of several months' duration, which is worsened by movement. A physical examination is unremarkable except for mild pallor. She takes furosemide (Lasix) for hypertension. Laboratory Findings Hemoglobin.. . . . . . . . . 10.0 g/dL (N 12.0-16.0) Serum creatinine.. . . . . .2.0 mg/dL (N 0.6-1.5) BUN. . . . . . . . . . . . . . . . . . . 40 mg/dL (N 8-25) Serum uric acid. . . . . . . . . 8.0 mg/dL (N 3.0-7.0) Serum calcium. . . . . . . . . .12.0 mg/dL (N 8.5-10.5) Total serum protein. . . . 9.8 g/dL (N 6.0-8.4) Globulin. . . . . . . 6.1 g/dL (N 2.3-3.5) Albumin. . . . . . . .3.7 g/dL (N 3.5-5.0) Serum IgG.. . . . .3700 mg/dL (N 639-1349) Urine protein . . positive for Bence-Jones protein Which one of the following would be most appropriate at this point? (check one) A. Repeat the physical examination and laboratory evaluation every 6 months B. Discontinue the diuretic and repeat the laboratory evaluation in 1 month C. Obtain a bone scan D. Obtain a bone marrow examination E. Begin therapy with tamoxifen (Soltamox), 20 mg daily

D. Obtain a bone marrow examination - multiple myeloma - 1% of all malignant diseases - mean age at diagnosis of 61 years - dx by bone marrow examination showing >10% plasma cells in the marrow - The serum level of monoclonal immunoglobulin is typically >3 g/dL Not Tamoxifen: for breast cancer

A 76-year-old female presents with a history of bilateral shoulder pain for the past month. She reports stiffness in the morning for about 1 hour and also reports difficulty getting up when seated in a chair. Acetaminophen is ineffective for her pain. Her erythrocyte sedimentation rate is 65 mm/hr (N 1-25). Which one of the following is the best initial treatment for this condition? (check one) A. Oral dexamethasone, 8 mg twice a day B. Oral methylprednisolone (Medrol Dosepak), taper 24 mg to 0 mg over 7 days C. Oral prednisolone (Orapred), 60 mg daily D. Oral prednisone, 15 mg/day E. Intravenous methylprednisolone, 120 mg every 6 hours

D. Oral prednisone, 15 mg/day Alternative treatment includes intramuscular methylprednisolone, 120 mg every 3 weeks.

Which one of the following is a contraindication to immunization with MMR? (check one) A. The patient had a tuberculosis skin test (PPD) within the previous 2 days B. A household member of the patient has an immunodeficiency C. The mother of the patient is pregnant D. The patient is pregnant E. The patient is breastfeeding her newborn infant

D. Patient is pregnant - A PPD may be falsely negative if administered 2-30 days after MMR administration, not the reverse. - If the patient is immunodeficient or pregnant, rather than a household contact, then MMR is contraindicated. - Breastfeeding is not a contraindication.

In the United States, the number of deaths has increased in recent years for which one of the following vaccine-preventable illnesses? (check one) A. Tetanus B. Hepatitis C C. Rubella D. Pertussis E. West Nile virus

D. Pertussis - 4 deaths in 1996 to - 17 deaths in 2001 - 56 deaths from 2001 to 2003 - Immunity has decreased in previously vaccinated adolescents and adults, and now they are a reservoir for infection - Tdap vaccine is recommended as a single booster for patients age 19-65, and those between the ages of 11 and 18 years should receive Tdap rather than a Td booster - Tdap vaccine protects against pertussis, in addition to tetanus and diphtheria - There are no vaccines for hepatitis C or West Nile virus.

A 30-year-old white gravida 2 para 1 who has had no prenatal care presents for urgent care at 33 weeks gestation. Her symptoms include vaginal bleeding, uterine tenderness, uterine pain between contractions, and fetal distress. Her first pregnancy was uncomplicated, with a vaginal delivery at term. Which one of the following is the most likely diagnosis? (check one) A. Uterine rupture B. Vasa previa C. Placenta previa D. Placental abruption E. Cervical cancer

D. Placental abruption

A 16-year-old male accompanied by his mother presents to your outpatient clinic with concerns about his short stature and "boyish" looks. He is a sophomore in high school but is frequently mistaken for someone much younger. Radiographs reveal a bone age of 14.7 years. Which one of the following would suggest the need for further evaluation? (check one) A. A family history of delayed growth B. Height below the fifth percentile for age C. Weight below the fifth percentile for age D. Prepubescent testicular size

D. Prepubescent testicular size - Most cases of short stature are due to constitutional growth delay - i.e. child is normal but delayed in his development - hallmark: below the fifth percentile for height for most of childhood - Usually these children are thin and have a family history of delayed development. - Bone age would be expected to be at least 2.5 standard deviations below the mean for age matched peers - if no signs of puberty are seen by 14 years of age (no breast development in girls, no testicular enlargement in boys), then further workup for a more serious condition should be sought. - Other indications for evaluation would be no menarche in a girl by 16 years of age and underdeveloped genitalia in a boy 5 years after his first pubertal changes.

Which one of the following decreases the absorption of orally administered calcium supplements? (check one) A. Taking calcium carbonate with meals B. Taking calcium citrate with meals C. Vitamin D supplementation D. Proton pump inhibitors

D. Proton Pump Inhibitor - Long-term histamine H2 -blocker or proton pump inhibitor use is associated with decreased absorption of calcium carbonate. - Patients taking these medications who require calcium supplementation should use calcium citrate to improve absorption. - Calcium carbonate preparations should be given with a meal to improve absorption. - Vitamin D is important in calcium absorption

4-week-old full-term male is brought to your office by his parents. They report that their child started vomiting just after his 1-week visit. The parents are concerned because they think the vomiting is worsening, occurring after every feeding, and "shooting across the room." You note that the baby is afebrile, but has not gained any weight since birth. Based on this information, the most likely diagnosis is: (check one) A. Formula intolerance B. Meningitis C. Viral gastroenteritis D. Pyloric stenosis E. Inappropriate feeding

D. Pyloric stenosis fits the described scenario; it is characterized by the early onset of worsening projectile vomiting and poor weight gain, and occurs most often in full-term male infants who are otherwise healthy. Formula intolerance causes regurgitation, as would inappropriate feeding. Meningitis, whether viral or bacterial, would be associated with fever. Viral gastroenteritis is a common cause of vomiting in older children, and is usually associated with fever and diarrhea.

Staff members of an assisted-living facility ask for your advice regarding aerobic exercise programs for their older residents. The evidence is greatest for which one of the following benefits of physical activity in the elderly? (check one) A. Maintaining weight after weight loss B. Improving quality of sleep C. Increasing bone density D. Reducing the risk of falls

D. Reducing the risk of falls

Which one of the following is most consistent with a diagnosis of iron deficiency anemia? (check one) A. Low iron-binding capacity B. An elevated methylmalonic acid level C. Increased serum ferritin D. Reticulocytosis about 1 week after administration of iron

D. Reticulocytosis 1 week after iron administration iron deficiency anemia - serum iron is low but iron-binding capacity is high - Serum ferritin is one-tenth of normal - Bone marrow iron stores are depleted - Oral replacement, which is safer than parenteral administration should raise the hemoglobin level by 0.2 g/dL/day - A reticulocyte response should be seen in a week to 10 days unless factors such as a concomitant folic acid deficiency prevent a full response to therapy.inhibiting the Na+/K+-ATPase, cardiac glycosides

A 7-year-old Hispanic female has a 3-day history of a fever of 40.0°C (104.0°F), muscle aches, vomiting, anorexia, and headache. Over the past 12 hours she has developed a painless maculopapular rash that includes her palms and soles but spares her face, lips, and mouth. She has recently returned from a week at summer camp in Texas. Her pulse rate is 140 beats/min, and her blood pressure is 80/50 mm Hg in the right arm while lying down. Which one of the following is the most likely diagnosis? (check one) A. Mucocutaneous lymph node syndrome B. Leptospirosis C. Rocky Mountain spotted fever D. Scarlet fever E. Toxic shock syndrome

D. Rocky Mountain spotted fever (RMSF) (SOR C) - tick-borne. - mostly found in the South Atlantic and south central states. - the summer and with exposure to tall vegetation (e.g., while camping, hiking, or gardening) - clinical dx: fever, hypotension, rash, myalgia, vomiting, and headache (sometimes severe) - rash associated with RMSF usually appears 2-4 days after the onset of fever and begins as small, pink, blanching macules on the ankles, wrists, or forearms that evolve into maculopapules - occur anywhere on the body, including the palms and soles, but the face is usually spared Not Mucocutaneous lymph node syndrome - similar condition in children (usually <2 year) - changes in the lips and oral cavity, such as strawberry tongue, redness and cracking of the lips, and erythema of the oropharyngeal mucosa. Not Leptospirosis - severe cutaneous hyperesthesia Not scarlet fever - prominent pharyngitis - fine, papular, erythematous rash. Toxic shock syndrome - may have similar symptoms, but usually in postmenarchal females.

Medicare pays for which one of the following? (check one) A. Routine dental care B. Custodial nursing-home care C. Hearing aids D. Screening mammography

D. Screening mammography

An 82-year-old male resident of a nursing home has developed symptoms of depression including withdrawal and sadness. The staff also reports that he doesn't want to leave his room, and often expresses a desire to stay in bed all day. After performing an appropriate evaluation and recommending nonpharmacologic interventions, you also decide that pharmacologic treatment is indicated. Which one of the following would be the most appropriate antidepressant for this patient? (check one) A. Amitriptyline (Elavil) B. Doxepin (Sinequan) C. Trazodone (Desyrel) D. Sertraline (Zoloft) E. Olanzapine (Zyprexa)

D. Sertraline (Zoloft) - Amitriptyline, doxepin, MAO inhibitors, and clomipramine should be avoided in nursing-home patients. - SSRIs are the most appropriate first-line pharmacologic treatment for depression in nursing-home residents. - Other classes of non-tricyclic antidepressants may be effective and appropriate, but the evidence for this is not as good as the evidence for SSRIs.

A copper T 380A intrauterine device (ParaGard) would be preferred over a levonorgestrel-releasing intrauterine device (Mirena) in a patient with a history of which one of the following? (check one) A. Nulliparity B. Current smoking C. Acute deep vein thrombosis D. Severe cirrhosis E. Heart failure

D. Severe cirrhosis Levonorgestrel-releasing IUD (Mirena) contraindicated - severe liver disease > hepatocellular adenoma - Breast cancer DVT, smoking, severe HTN is ok - IUD does not have estrogen

Which one of the following best describes the condition seen in the radiograph in Figure 6? A. Osgood-Schlatter disease B. Legg-Calvé-Perthes disease C. Blount's disease D. Slipped capital femoral epiphysis E. A normal hip

D. Slipped capital femoral epiphysis - epiphysis displaced - usually occurs in late childhood or adolescence. Osgood-Schlatter disease - anterior tibial tubercle Legg-Calvé-Perthes disease - avascular necrosis of the femoral head. Blount's disease - medial portion of the proximal tibia. All of these conditions cause leg pain in children.

Which one of the following is true concerning anterior cruciate ligament (ACL) tears? (check one) A. The incidence of ACL tears is higher in males than in females B. ACL tears are not associated with early-onset osteoarthritis C. The majority of ACL tears are caused by physical contact D. Strength training can prevent ACL tears

D. Strength training can prevent ACL tears

A 78-year-old white female presents with a 3-day history of lower thoracic back pain. She denies any antecedent fall or trauma, and first noted the pain upon arising. Her description of the pain indicates that it is severe, bilateral, and without radiation to the arms or legs. Her past medical history is positive for hypertension and controlled diabetes mellitus. Her medications include hydrochlorothiazide, enalapril (Vasotec), metformin (Glucophage), and a general multivitamin. She is a previous smoker but does not drink alcohol. She underwent menopause at age 50 and took estrogen for "a few months" for hot flashes. Physical examination reveals her to be in moderate pain with a somewhat stooped posture and mild tenderness over T12-L1. She has negative straight-leg raising and normal lower extremity sensation, strength, and reflexes. Which one of the following is true regarding this patient's likely condition? (check one) A. An MRI or nuclear medicine bone scan should be performed B. Prolonged (approximately 2 weeks) bed rest will increase the chance of complete recovery C. Investigation for an underlying malignancy is indicated D. Subcutaneous or intranasal calcitonin (Calcimar, Miacalcin) may be very helpful for pain relief

D. Subcutaneous or intranasal calcitonin (Calcimar, Miacalcin) may be very helpful for pain relief - osteoporotic vertebral compression fracture. - should be confirmed with a plain radiograph.

Which one of the following classes of diabetes medications acts primarily by stimulating pancreatic insulin secretion? (check one) A. Biguanides, such as metformin (Glucophage) B. Thiazolidinediones, such as pioglitazone (Actos) C. DPP-4 inhibitors, such as sitagliptin (Januvia) D. Sulfonylureas, such as glipizide (Glucotrol) E. Amylin analogs, such as pramlintide (Symlin)

D. Sulfonylureas, such as glipizide (Glucotrol) Biguanides and thiazolidinediones - insulin sensitizers that decrease hepatic glucose production and increase insulin sensitivity Sulfonylureas and meglitinides - stimulate pancreatic insulin secretion DPP-4 inhibitors - prevent GLP-1 breakdown and slow the breakdown of some sugars GLP-1 mimetics - stimulate insulin secretion, suppress glucagon secretion, and promote β-cell production Amylin analogs -acts with insulin to delay gastric emptying and they also inhibit glucagon release.

A 32-year-old male smoker presents with a 4-day history of progressive hoarseness. He is almost unable to speak, and associated symptoms include a cough slightly productive of yellow sputum, as well as tenderness over the ethmoid sinuses. He is afebrile and has normal ear and lung examinations. His oropharynx is slightly red with no exudate, and examination of his nasal passages reveals mucosal congestion. Which one of the following would be the most appropriate treatment? (check one) A. Amoxicillin for 10 days B. Omeprazole (Prilosec), 40 mg daily C. Azithromycin (Zithromax) for 5 days D. Symptomatic treatment only

D. Symptomatic treatment only Acute laryngitis most often has a viral etiology and symptomatic treatment is therefore most appropriate.

Which one of the following is true regarding treatment of pressure ulcers? (check one) A. Multiple controlled trials have shown that nutritional supplements hasten ulcer healing B. Keeping the head of the bed elevated to 45° during the day promotes healing by minimizing shearing forces C. Systemic antibiotics are most helpful when used intermittently to reduce bacterial counts D. Topical antibiotics should not be used for more than 2 weeks at a time

D. Topical antibiotics should not be used for more than 2 weeks at a time Nutritional supplements - does not speed ulcer healing Head of the should be kept less than 30° - reduce shearing forces Systemic antibiotics - should only be used for cellulitis, osteomyelitis, and bacteremia - Topical antibiotics may be used for periods of up to 2 weeks (SOR C).

A 4-year-old male presents with a 3-day history of sores on his right leg. The sores began as small red papules but have progressed in size and now are crusting and weeping. Otherwise he is in good health and is up to date with immunizations. On examination he has three lesions on the right anterior lower leg that are 0.5-1.5 cm in diameter, with red bases and honey-colored crusts. There is no regional lymphangitis or lymphadenitis. Which one of the following is the preferred first-line therapy? (check one) A. Oral erythromycin (Erythrocin) B. Oral penicillin V C. Topical hexachlorophene (pHisoHex) D. Topical mupirocin (Bactroban)

D. Topical mupirocin - nonbullous impetigo, - *Staphylococcus aureus or Streptococcus pyogenes* - Topical antibiotics, such as mupirocin, but not compounds containing neomycin, are the preferred first-line therapy for impetigo involving a limited area. Not Oral antibiotics - for patients with more extensive impetigo or with systemic symptoms or signs. Not Penicillin V and hexachlorophene - ineffective Not erythromycin - same effect as topical - common adverse effect of nausea.

Which one of the pharmacologic effects of transdermal medications changes the LEAST with aging? (check one) A. Liver metabolism of the drugs B. Renal excretion of the drugs C. Distribution within the body D. Transdermal absorption of the drugs

D. Transdermal absorption of the drugs - Due to an increase in the ratio of fat to lean body weight, the volume of distribution changes with aging, especially for fat-soluble drugs. - Both liver metabolism and renal excretion of drugs decrease with aging, increasing serum concentrations.

A 48-year-old male sees you for a routine health maintenance examination. His blood pressure is 142/90 mm Hg and you recommend that he return for a repeat blood pressure measurement. Eight weeks later his blood pressure is 138/88 mm Hg. He denies any symptoms on a review of systems. He tells you that on his 40th birthday he abruptly stopped smoking after smoking a pack of cigarettes a day since his early twenties. He is adopted and cannot provide a family history. According to U.S. Preventive Services Task Force guidelines, which one of the following conditions should this patient be screened for now? (check one) A. Abdominal aortic aneurysm B. Peripheral arterial disease C. Colon cancer D. Type 2 diabetes mellitus E. Hemochromatosis

D. Type 2 diabetes mellitus Asymptomatic adults with sustained blood pressure >135/80 mm Hg be screened for type 2 diabetes mellitus using fasting plasma glucose, a 2-hour glucose tolerance test, or hemoglobin A1c measurements (USPSTF B recommendation). colon cancer - 50-75 AAA - 65-75 none for PAD

A right-hand-dominant 38-year-old male comes to your office because of right elbow pain. He recently began participating in a highly competitive adult volleyball league, and 2 weeks after he first began playing he developed mild pain in the medial elbow of his right arm. While completing an overhead serve last night he felt an acute worsening of the elbow pain. After the match he noted bruising over his medial elbow. When you examine him you find bruising and pain to palpation around the medial elbow. With his shoulder in 90° of abduction and external rotation you rapidly flex and extend the elbow while maintaining valgus torque on the elbow (the moving valgus stress test). The patient reports pain between 70° and 120° of flexion. This clinical presentation is most consistent with which one of the following causes of elbow pain? (check one) A. Medial epicondylitis B. Biceps tendinopathy C. Cubital tunnel syndrome D. Ulnar collateral ligament injury E. Triceps tendinopathy

D. Ulnar collateral ligament injury This patient has injured his ulnar collateral ligament (UCL). The UCL is the primary restraint to valgus stress on the elbow during overhead throwing. These injuries often occur in athletes participating in sports that require overhead throwing, such as baseball, javelin, and volleyball. Patients often report a pop followed by immediate pain and bruising around the medial elbow. The moving valgus stress test has 100% sensitivity and 75% specificity for diagnosing UCL injuries. Not Medial epicondylitis: insidious onset of pain related to a recent increase in occupational or recreational activities. Patients also often report weakened grip strength. The point of maximal tenderness is 5-10 mm distal to and anterior to the medial epicondyle. It is most often a tendinopathy of the flexor carpi radialis and the pronator teres. Not Biceps tendinopathy - vague anterior elbow pain and a history of repeated elbow flexion with forearm supination and pronation, such as dumbbell curls. Resisted supination produces pain deep in the antecubital fossa. Not Cubital tunnel syndrome - neuropathy of the ulnar nerve caused by compression or traction as it passes through the cubital tunnel of the medial elbow. The onset of pain is more insidious than UCL injury, occurring with repetitive activity, and is usually accompanied by numbness and tingling in the ulnar border of the forearm and hand. If it has existed for some time, the intrinsic hand muscle may become weak. Not triceps tendinopathy - more common in weight lifters or athletes who repetitively extend their elbows against resistance. Pain occurs at the posterior elbow with resisted extension, and tenderness is located over the triceps insertion.

An 8-year-old female is brought to your office because she has begun to limp. She has had a fever of 38.8°C (101.8°F) and says that it hurts to bear weight on her right leg. She has no history of trauma. On examination, she walks with an antalgic gait and hesitates to bear weight on the leg. Range of motion of the right hip is limited in all directions and is painful. Her sacroiliac joint is not tender, and the psoas sign is negative. Laboratory testing reveals an erythrocyte sedimentation rate of 55 mm/hr (N 0-10), a WBC count of 15,500/mm 3 (N 4500-13,500), and a C-reactiveprotein level of 2.5 mg/dL (N 0.5-1.0). Which one of the following will provide the most useful diagnostic information to further evaluate this patient's problem? (check one) A. MRI B. CT C. A bone scan D. Ultrasonography E. Plain-film radiography

D. Ultrasonography - septic arthritis - detecting effusion of the hip joint. - If an effusion is present, urgent ultrasound-guided aspiration should be performed. Bone scintigraphy - a limping child when the history, physical examination, and radiographic and sonographic findings fail to localize the pathology. CT - for cortical bone MRI - joints, soft tissues, cartilage, and medullary bone. - osteomyelitis, extent of malignancies, identifying stress fractures, and diagnosing early Legg-Calvé-Perthes disease. Plain film radiography - an initial imaging modality in any child with a limp. - However, films may be normal in patients with septic arthritis, providing a false-negative result.

Which one of the following should be used first for ventricular fibrillation when an initial defibrillation attempt fails? (check one) A. Amiodarone (Cordarone) B. Lidocaine (Xylocaine) C. Adenosine (Adenocard) D. Vasopressin (Pitressin) E. Magnesium

D. Vasopressin

Activated protein C resistance (factor V Leiden) is most commonly found in patients with? (check one) A. Hemolytic anemia B. Carcinoma of the lung C. Familial hypercholesterolemia D. Venous thrombotic disease E. Cystic fibrosis

D. Venous thrombotic disease Venous thrombosis, both acute and recurrent, is associated with several hematologic abnormalities, in addition to the well-known factors of trauma, surgery, malignancy, sepsis, and oral contraceptive use. Notably, activated protein C resistance (factor V Leiden) has been found to be one of the most common hereditary causes of thrombophilia.

A 24-year-old female presents to your clinic with a 5-day history of fever to 103°F. She has no localizing symptoms or overt physical findings. Initial testing shows an elevated WBC count with a disproportionate number of reactive lymphocytes. Which one of the following conditions is the most likely cause of these findings? (check one) A. Bacterial infection B. Connective tissue disease C. Lymphoma D. Viral infection

D. Viral infection - β-Blockers do diminish the effectiveness of inhaled β-agonists, but this effect is not severe enough to contraindicate using these drugs together.

A 67-year-old female has a bone density study that indicates a T score of -3.5. You prescribe alendronate (Fosamax) but at her next visit she says she cannot tolerate the side effects and asks about other therapies. Which one of the following has the best evidence for prevention of both vertebral fractures and hip fractures? (check one) A. Calcitonin-salmon (Miacalcin) B. Raloxifene (Evista) C. Teriparatide (Forteo) D. Zoledronic acid (Reclast)

D. Zoleronic acid (Reclast) - Intravenous zoledronic acid has been shown to reduce both hip fracture risk and vertebral fracture risk. Not Teriparatide, raloxifene & calcitonin salmon - only for vertebral fracture, not hip - reduces vertebral fracture risk but not hip fracture risk.

You are caring for a 60-year-old female with Crohn's disease that is well controlled by infliximab (Remicade). As your staff updates her immunization status, which one of the following should be kept in mind? (check one) A. Hepatitis A vaccine is contraindicated B. Pneumococcal vaccine is contraindicated C. Tetanus toxoid is contraindicated D. Zoster vaccine is contraindicated

D. Zoster vaccine is contraindicated- live attenuated virus vaccine Immunosuppression is defined as: • Treatment with glucocorticoids (treatment with the equivalent of 20 mg/day of prednisone for 2 weeks or more, and discontinuation within the previous 3 months) • Ongoing treatment with effective doses of 6-MP/azathioprine or discontinuation within the previous 3 months • Treatment with methotrexate or discontinuation within the previous 3 months • Treatment with infliximab or discontinuation within the previous 3 months • Significant protein-calorie malnutrition

A 42-year-old African-American male recently traveled to the Caribbean for a scuba diving trip. Since his return he has noted brief intermittent episodes of vertigo not associated with nausea or vomiting. He is concerned, however, because these episodes occurred after sneezing or coughing and then a couple of times after straining while lifting something. He has had no hearing loss, and no vertigo with positional changes such as bending over or turning over in bed. The most likely cause of this patients vertigo is (check one) A. vestibular neuronitis B. Menieres disease C. benign paroxysmal positional vertigo D. a perilymphatic fistula E. multiple sclerosis triggered by a rapid change in climate

D. a perilymphatic fistula - fistula between the middle and inner ear - may be caused by barotrauma from scuba diving, direct blows, heavy weight bearing, and excessive straining (e.g., with sneezing or bowel movements.) Vestibular neuronitis - more sudden, unremitting syndrome. ' Menieres disease - episodes of vertigo, associated with hearing loss and often with nausea and vomiting. Benign paroxysmal positional vertigo - more likely in older individuals, associated with postural change. Multiple sclerosis - requires symptoms in multiple areas and is not thought to be provoked by climate change.

While playing tennis, a 55-year-old male tripped and fell, landing on his outstretched hand with his elbow in slight flexion at impact. Pronation and supination of the forearm are painful on examination, as are attempts to flex the elbow. There is tenderness of the radial head without significant swelling. A radiograph of the elbow shows no fracture, but a positive fat pad sign is noted. Appropriate management would include: (check one) A. a long arm cast for 2 weeks, followed by use of a brace B. mobilization of the elbow beginning 3 weeks after the injury C. a posterior splint for 6 weeks D. a posterior splint and a repeat radiograph in 1-2 weeks

D. a posterior splint and a repeat radiograph in 1-2 weeks - Nondisplaced radial head fractures - Conservative therapy: placing the elbow in a posterior splint for 5-7 days, followed by early mobilization and a sling for comfort. - joint effusion may be aspirated for pain relief and to increase mobility. - Radiographs should be repeated in 1-2 weeks to make sure that alignment is appropriate.

A 12-year-old female is brought to your office with an 8-day history of sore throat and fever, along with migratory aching joint pain. She is otherwise healthy and has no history of travel, tick exposure, or prior systemic illness. A physical examination is notable for exudative pharyngitis; a blanching, sharply demarcated macular rash over her trunk; and a III/VI systolic ejection murmur. Joint and neurologic examinations are normal. A rapid strep test is positive and her C-reactive protein level is elevated. Of the following, the most likely diagnosis is: (check one) A. juvenile rheumatoid arthritis B. infective endocarditis C. Kawasaki syndrome D. acute rheumatic fever E. Lyme disease

D. acute rheumatic fever - JONES criteria 2 major criteria or 1 major criterion and 2 minor criteria, plus evidence of a preceding streptococcal infection, indicate a high probability of the disease. Major criteria include - carditis, migratory polyarthritis, erythema marginatum, chorea, and subcutaneous nodules. Minor criteria include - fever, arthralgia, an elevated erythrocyte sedimentation rate or C-reactive protein (CRP) level, and a prolonged pulse rate interval on EKG. The differential diagnosis is extensive and there is no single laboratory test to confirm the diagnosis.

You are a member of a committee at your local hospital that has been asked to develop measures to reduce the incidence of postoperative methicillin-resistant Staphylococcus aureus (MRSA) infections. Which one of the following would be most effective for preventing these infections? (check one) A. Give preoperative antibiotics to all surgical patients to eradicate bacteria B. Screen all admitted patients for MRSA and use antibiotics pre- and postoperatively in positive cases C. Culture the nares of all hospital employees upon hiring and on a routine basis thereafter D. Institute an intensive program of good hand washing for all employees

D. always hand-washing

A 24-year-old male sustains a boxer's fracture of the fifth metacarpal. A radiograph shows no rotational deformity and 25° of volar angulation. After an attempt at closed reduction the angulation remains unchanged. Which one of the following would be most appropriate at this time? (check one) A. Open reduction B. Placement of a pin to prevent further displacement C. A short arm-thumb spica cast D. An ulnar gutter splint

D. an ulnar gutter splint Up to 40° of volar angulation is acceptable for fifth metacarpal fractures. For second and third metacarpal fractures, less angulation is acceptable. Appropriate treatment is a gutter splint.

A 16-year-old white male is seen for a preparticipation sports examination. His height is 183 cm (72 in), his weight is 64 kg (141 lb), and he appears to have long arms. A physical examination reveals a high arched palate, kyphosis, myopia, and pectus excavatum. Which one of the following valvular abnormalities is most likely in this patient? (check one) A. Mitral stenosis B. Pulmonic stenosis C. Aortic stenosis D. Aortic insufficiency E. Bicuspid aortic valve

D. aortic insufficiency - Marfan syndrome - arachnodactly, arm span greater than height, a high arched palate, kyphosis, lenticular dislocation, mitral valve prolapse, myopia, and pectus excavatum. - aortic insufficiency murmur or a murmur associated with mitral valve prolapse - Cardiovascular defects are progressive, and aortic root dilation occurs in 80%-100% of affected individuals. - Aortic regurgitation becomes more common with increasing age.

A 67-year-old female presents with the inability to smell. She is in good health, and her only medical problem is osteoporosis, treated with alendronate (Fosamax). She says she has no sinus or nasal symptoms. A physical examination is normal including an ear, nose, and throat examination. Which one of the following would be most appropriate at this point? (check one) A. Discontinuing the alendronate B. An anti-tissue transglutaminase antibody test C. A serum vitamin D level D. MRI of the brain

D. brain MRI - Rare tumors involving the olfactory region of the brain can affect smell, and are best detected by MRI. - bisphosphonates does not affect taste or smell - May be associated with deficiencies of vitamins A, B6, B12, and trace metals, but not with vitamin D deficiency. - Celiac disease is not known to cause a decreased ability to smell.

A 68-year-old white female with a several-year history of well-controlled essential hypertension and a history of acute myocardial infarction 2 years ago is brought to the emergency department complaining of sudden, painless, complete loss of vision in her left eye that began 1 hour ago. Her vital signs are stable, and her blood pressure is 148/90 mm Hg. Her corrected visual acuity is: left—absent, with no light perception; right—20/30. The external eye examination is entirely unremarkable. A retinal examination reveals the findings shown in fig 5 The most likely diagnosis is: (check one) A. acute narrow-angle glaucoma B. optic neuritis C. retinal hemorrhage D. central retinal artery occlusion E. central retinal vein occlusion

D. central retinal artery occlusion The painless, unilateral, sudden loss of vision over a period of seconds may be caused by thrombosis, embolism, or vasculitis. Acute narrow-angle glaucoma is an abrupt, painful, monocular loss of vision often associated with a red eye, which will lead to blindness if not treated.

A cement plant worker presents to your office with the recurrent acute skin eruption on his legs shown in Figure 7. Figure 7 It extends proximally from the dorsum of the feet to just below the knees. This is the third eruption in 2 years.This patient most likely has: (check one) A. tinea with a secondary id reaction B. rhus dermatitis C. methicillin-resistant Staphylococcus aureus (MRSA) cellulitis D. contact dermatitis related to his occupation

D. contact dermatitis related to occupation - recurrent and symmetric Rhus dermatitis - contact dermatitis - more acute and presents with bullae and vesicles that are more linear than those seen in this patient. MRSA - unilateral cellulitis - more commonly as inflammatory nodules or pustules Tinea - would have scaling & distinct border

A 47-year-old male with chronic kidney disease is being treated with epoetin alfa (Procrit). His hemoglobin level is 11.3 g/dL (N 13.0-18.0). Which one of the following would be most appropriate with regard to his epoetin alfa regimen? (check one) A. Increase the dosage until the hemoglobin level is >12.0 g/dL B. Increase the frequency of injections, using the same dose C. Decrease the frequency of injections, using the same dose D. Continue the current regimen

D. continue current regimen - patients with renal failure, the risk for death and serious cardiovascular events is increased with higher hemoglobin levels (≥13.5 g/dL), and it is therefore recommended that levels be maintained at 10-12 g/dL. - Studies have also demonstrated less mortality and morbidity when the dosage of epoetin alfa is set to achieve a *target hemoglobin of <12 g/dL*. Ref: Singh AK, Szczech L, Tang KL, et al: Correction of anemia with epoetin alfa in chronic kidney

You are treating a 53-year-old female for a deep-vein thrombosis in her left leg. The use of compression stockings for this problem has been shown to: (check one) A. increase the risk of pulmonary embolism B. increase the level of pain C. increase complications if used prior to completion of a course of anticoagulation therapy D. decrease the risk of post-thrombotic syndrome

D. decrease the risk of post-thrombotic syndrome - complication of acute deep-vein thrombosis (DVT) - chronic pain, swelling, and skin changes in the affected limb - Within 5 years of experiencing a DVT, one in three patients will develop PTS. - Compression stockings should be applied when anticoagulation therapy is started, not when it has been completed.

A previously healthy 73-year-old male is admitted to the intensive-care unit after an emergency appendectomy. He does well until the evening, when he suddenly appears confused. His speech is rambling and incoherent, and he is disoriented to person, place, and time. His wife says he was sleepy but otherwise acting normal 2 hours ago. On examination he has normal vital signs and no fever. Other than the cognitive changes and some mild peri-incisional tenderness the examination is normal. Serum electrolytes, a CBC, arterial blood gases, and a routine chemistry panel are normal. The most likely cause for his altered sensorium is (check one) A. sepsis B. acute psychosis C. dementia with Lewy bodies D. delirium E. ischemic stroke

D. delirium

An 18-year-old male comes to your office because of the recent onset of recurrent, unpredictable episodes of palpitations, sweating, dyspnea, gastrointestinal distress, dizziness, and paresthesias. His physical examination is unremarkable except for moderate obesity. Laboratory findings, including a CBC, blood chemistry profile, and thyroid-stimulating hormone (TSH) level, reveal no abnormalities. The most likely diagnosis is: (check one) A. mitral valve prolapse B. paroxysmal supraventricular tachycardia C. pheochromocytoma D. generalized anxiety disorder D. generalized anxiety disorder E. panic disorder

D. generalized anxiety disorder Pheochromocytoma is associated with headache and hypertension, and usually occurs in thin patients.

A 36-year-old female sees you for a 6-week postpartum visit. Her pregnancy was complicated by gestational diabetes mellitus. Her BMI at this visit is 33.0 kg/m2 and she has a family history of diabetes mellitus. This patient's greatest risk factor for developing type 2 diabetes mellitus is her: (check one) A. age B. obesity C. history of a completed pregnancy D. history of gestational diabetes E. family history of diabetes

D. history of gestational diabetes - Obesity and family history are risk factors for the development of diabetes, but having GDM leads to a fourfold greater risk of developing diabetes

Which one of the following is true concerning nausea and vomiting in pregnancy? (check one) A. Psychological factors play a causative role B. Pharmacologic therapy, in general, is no more effective than placebo in relieving symptoms and preventing hospitalization C. Metoclopramide (Reglan) is contraindicated in the first trimester but is safe and effective for nausea and vomiting in later trimesters D. If nausea and vomiting begin after 9 weeks' gestation, secondary causes are more likely to be present

D. if nausea and vomiting begin after 9 weeks gestation, secondary cause should be considered - Metoclopramide is more effective than placebo and has not been associated with an increased risk of adverse effects on the fetus.

Which one of the following is true regarding temporomandibular joint disorder? (check one) A. Dental splints are the treatment of choice B. Mandibular clicking is an essential diagnostic element C. Ultrasonic phonophoresis with cortisone is the treatment of choice D. The majority of cases resolve without treatment E. MRI is preferred over CT to confirm the diagnosis

D. majority resolve without treatment - while dental splints have been commonly recommended, not well studied - Physical therapy modalities such as iontophoresis or phonophoresis may benefit some patients, but there is no clearly preferred treatment. - imaging not needed

The most common identifiable cause of skin and soft-tissue infections presenting to metropolitan emergency departments is: (check one) A. Staphylococcus epidermidis B. Streptococcus pneumoniae C. Pseudomonas aeruginosa D. methicillin-resistant Staphylococcus aureus (MRSA) E. Bacillus cereus

D. methicillin-resistant Staphylococcus aureus (MRSA) - most common identifiable cause of skin and soft-tissue infections among patients presenting to emergency departments in 11 U.S. cities.

A 24-year-old gravida 1 para 1 who is 2 weeks post partum complains of double vision, shortness of breath, and almost dropping her baby while trying to hold her. She says her symptoms worsen as the day progresses. She has no family history of neurologic or muscular illness. A physical examination is normal except for unilateral ptosis and 4/5 proximal weakness of both arms. Breath sounds are generally decreased. Routine blood tests, including TSH and creatine kinase levels, are normal. A chest radiograph and an MRI of the brain and cervical spine are also normal. Of the following, this presentation is most consistent with (check one) A. fibromyalgia syndrome B. Sheehan's syndrome (postpartum hypopituitarism) C. polymyositis D. myasthenia gravis E. stroke

D. myasthenia gravis - Common neurologic disorders in young women include multiple sclerosis, Guillain-Barré syndrome, and myasthenia gravis. - Myasthenia gravis is part of the differential diagnosis for sudden neurologic weakness

In the United States the most common form of child abuse is (check one) A. physical abuse B. emotional abuse C. sexual abuse D. neglect

D. neglect

The parents of a 7-year-old male ask you to evaluate him because of increasing concerns about his temper tantrums over the past 9 months. He often becomes angry and hostile, argues with them constantly, and refuses to follow rules or directions. A major source of difficulty is his refusal to quit playing with his toys when he is asked to come to the dinner table. After the child ignored repeated attempts to get him to come to the table a few nights ago, the father became frustrated and told him he had lost his television privileges. In response, the child became aggressive and destructive, breaking his toys and sweeping his dinner plate and glass of milk onto the floor. The parents describe many similar scenarios at bedtime, bath time, and when he is getting dressed. They believe that their son is deliberately behaving this way to annoy them. This history is most consistent with: (check one) A. attention-deficit/hyperactivity disorder B. bipolar disorder C. conduct disorder D. oppositional defiant disorder E. normal childhood individualization

D. oppositional defiant disorder (no explanation for conduct disorder) - a pattern of negativistic, hostile, and defiant behavior lasting at least 6 months - child often lose his or her temper, argue with adults, actively defy or refuse to comply with adults' requests or rules, deliberately annoy people, blame others for his or her mistakes or misbehavior, be easily annoyed by others, appear angry and resentful, or be spiteful or vindictive. - At least four of these behaviors must be present to meet the criteria for diagnosis. - The disturbance in behavior must also cause clinically significant impairment in social, academic, or occupational functioning, and the behaviors must not occur exclusively during the course of a psychotic or mood disorder.

The most common cause of proteinuria in children is: (check one) A. Acute postinfectious glomerulonephritis B. Lupus glomerulonephritis C. Hydronephrosis D. Orthostatic proteinuria E. Reflux nephropathy

D. orthostatic proteinuria Orthostatic proteinuria accounts for up to 60% of all cases of asymptomatic proteinuria reported in children, with an even higher incidence in adolescents. Ref: Loghman-Adham M: Evaluating proteinuria in children.

You are evaluating a 5-month-old with fever, tachypnea, and mild respiratory distress in the emergency department. You hear mild basilar rales. The child does not appear toxic. Which one of the following tests would be the most appropriate as an initial study? (check one) A. A chest radiograph B. A CBC C. A C-reactive protein level D. Oxygen saturation by pulse oximetry

D. oxygen sat and pulse ox Pulse oximetry should be obtained on all pediatric patients with significant tachypnea, pallor, or respiratory distress. It has been found that CBCs, C-reactive protein levels, and erythrocyte sedimentation rates are not effective in differentiating between viral and bacterial pneumonia. Chest radiographs are also ineffective in distinguishing viral and bacterial pneumonia, and should be obtained in cases of ambiguous clinical findings, prolonged pneumonia, and pneumonia that is unresponsive to antibiotic therapy, as well as when there is the possibility of complications such as pleural effusions.

The most common initial symptom of Hodgkin lymphoma is: (check one) A. unexplained fever B. night sweats C. weight loss D. painless lymphadenopathy E. cough

D. painless lymphadenopathy - one-third of patients with Hodgkin lymphoma present with unexplained fever, night sweats, and recent weight loss, collectively known as "B symptoms."

A 19-year-old college student comes to your office with significant pain in his right great toe that is making it difficult for him to walk. He has never had this problem before. When you examine him you find increased swelling with marked erythema and seropurulent drainage and ulceration of the medial nail fold. The toe is very tender to touch, particularly when pressure is applied to the tip of the toe. The most appropriate initial management would be: (check one) A. oral antibiotics that cover common skin flora, for 5-7 days B. soaking the toe in warm, soapy water for 10-20 minutes twice daily, followed by application of a topical antibiotic, with a return visit in 3-5 days C. elevation of the nail with a wisp of cotton D. partial avulsion of the medial nail plate and phenolization of the matrix at this visit E. partial avulsion of both the medial and lateral nail plates at this visit

D. partial avulsion of the medial nail plate and phenolization of the matrix at this visit moderate severity - increased swelling, seropurulent drainage, infection, and ulceration of the nail fold. - antibiotics before or after phenolization of the matrix do not decrease healing time, postoperative morbidity, or recurrence rates (SOR B) Elevating the nail edge with a wisp of cotton or a gutter splint - only in patients with a mild to moderate ingrown toenail who do not have significant pain, substantial erythema, or purulent drainage. Immediate partial nail avulsion followed by phenolization or direct surgical excision of the nail matrix - effective for the treatment of ingrown nails (SOR B). Bilateral partial matricectomy - maintains the functional role of the nail plate (although it narrows the nail plate) - if patients with a severe ingrown toenail or to manage recurrences

A chest radiograph of the driver of an automobile involved in a head-on collision shows a widened mediastinum. This suggests: (check one) A. myocardial contusion B. spontaneous rupture of the esophagus C. rupture of a bronchus D. partial rupture of the thoracic aorta E. acute heart failure

D. partial rupture of the thoracic aorta Deceleration-type blows to the chest can produce partial or complete transection of the aorta

A 36-year-old white male complains of episodic pain in the rectum over the past several years. The pain occurs every 3-6 weeks and is sharThe induction or inhibition of the cytochrome P450 (CYP) enzyme is responsible for many adverse drug reactions. Which one of the following is an inducer of the cytochrome P450 enzyme? (check one) A. Ciprofloxacin (Cipro) B. Fluconazole (Diflucan) C. Phenytoin (Dilantin) D. Clarithromycin (Biaxin) E. Grapefruit juice p, cramp-like, and severe. It lasts from 1 to 15 minutes. He has no other gastrointestinal complaints. A physical examination, including a digital rectal examination and anoscopy, is normal. The most likely diagnosis is: (check one) A. fecal impaction B. coccygodynia C. anal fissure D. proctalgia fugax E. sacral nerve neuralgia

D. proctalgia fugax - Symptoms consistent with proctalgia fugax occur in 13%-19% of the general population - Episodic, sudden, sharp pains in the anorectal area lasting several seconds to minutes. - Diagnosis is from H&P All other - will have physical findings sacral nerve neuralgia - does not have physical finding but constant pain

A 32-year-old female presents with complaints of moderate irritability and anxiety during the week before nearly all of her menstrual periods. During this time she also has problems with weight gain and breast tenderness. She says she is her "usual happy self" at other times during the month. You diagnose premenstrual syndrome (PMS). Which one of the following complementary and alternative therapies has been shown to be helpful in reducing the symptoms of this problem? (check one) A. Saffron B. St. John's wort C. Soy D. Pyridoxine (vitamin B6) E. Vitamin E

D. pyridoxine (vitamin B6) 50-100 mg/day, also - chasteberry, 20 mg/day,

A 35-year-old female sees you because she has lost her voice. She has had no recent upper respiratory infection symptoms, cough, or heartburn, and she has not done anything that would strain her voice. Findings are normal on examination of the head and neck. A review of her chart shows this has happened before, but an ear, nose, and throat evaluation found no abnormalities. She also has been seen numerous times in the past few years for headaches, chest pains, abdominal pains, rectal pressure, and vaginal symptoms. Despite several workups and referrals, no definite cause has been found and the symptoms persist. Which one of the following would be the most reasonable plan of action? (check one) A. Test for food allergies B. Begin low-dose lorazepam (Ativan) C. Begin a 6-week trial of a proton pump inhibitor D. Schedule frequent office visits

D. schedule frequent office visits - Somatization disorders should be considered in patients who have a history of various complaints over a several-year period that involve multiple organ systems. - comorbid: anxiety disorder, personality disorder, and depression.

According to the guidelines developed by the JNC 8 panel, which one of the following should NOT be used as a first-line treatment for hypertension? (check one) A. ACE inhibitors B. Angiotensin receptor blockers C. Calcium channedl blockers D. β-Blockers E. Thiazide-type diuretics

D. β-Blockers

A 7-year-old male complains of left shoulder pain after a bicycle accident. The neurovascular evaluation is normal. A radiograph is shown in Figure 4. Figure 4 The best management for his condition is: (check one) A. chest radiography and frequent vital sign observation B. internal fixation under general anesthesia C. Steinmann's pin fixation under local anesthesia D. sending the patient home in a sling and swathe E. a modified shoulder spica cast

D. sending the patient home in a sling and swathe - Fractures of the medial third of the A 39-year-old female presents with a 4-month history of gradually worsening left elbow pain. She does not recall an injury but frequently lifts and holds her 10-month-old son in her left arm. She has tenderness over the lateral epicondyle. Her elbow range of motion is normal but she has pain with supination and pronation. The remainder of the examination is normal. For long-term pain relief, the best evidence supports which one of the following? (check one) A. Expectant/conservative management B. Physical therapy C. Oral anti-inflammatory agents D. A corticosteroid injection Correct. Lateral epicondylitis is a common condition characterized by degeneration of the extensor carpi radialis muscle tendon originating in the lateral epicondyle. It is a self-limited condition and A 39-year-old female presents with a 4-month history of gradually worsening left elbow pain. She does not recall an injury but frequently lifts and holds her 10-month-old son in her left arm. She has tenderness over the lateral epicondyle. Her elbow range of motion is normal but she has pain with supination and pronation. The remainder of the examination is normal. For long-term pain relief, the best evidence supports which one of the following? (check one) A. Expectant/conservative management B. Physical therapy C. Oral anti-inflammatory agents D. A corticosteroid injection Correct. Lateral epicondylitis is a common condition characterized by degeneration of the extensor carpi radialis muscle tendon originating in the lateral epicondyle. It is a self-limited condition and usually resolves within 12-18 months without treatment. It is not an inflammatory condition and anti-inflammatory agents have not been found to be beneficial. Corticosteroid injections have been found to be associated with poor long-term outcomes, as well as high recurrence rates. Neither physical therapy, bracing, nor splinting is proven to provide long-term pain relief. Approximately 90%-95% of all patients with lateral epicondylitis show improvement at 1 year despite the type of therapy utilized (SOR A).usually resolves within 12-18 months without treatment. It is not an inflammatory condition and anti-inflammatory agents have not been found to be beneficial. Corticosteroid injections have been found to be associated with poor long-term outcomes, as well as high recurrence rates. Neither physical therapy, bracing, nor splinting is proven to provide long-term pain relief. Approximately 90%-95% of all patients with lateral epicondylitis show improvement at 1 year despite the type of therapy utilized (SOR A). clavicle - common in peds - best treated by a figure-of-8 apparatus - Open surgical reduction with intramedullary fixation will minimize angular deformity at the fracture site but leaves a scar and may result in nonunion; only with rare exception of neurovascular injury accompanying the fracture,

A 42-year-old white female presents to your office as a new patient. She states that she has an 8-year history of abdominal cramps and diarrhea. Her symptoms have not responded to the usual treatments for irritable bowel syndrome. She has no rectal bleeding, anemia, weight loss, or fever, and no family history of colon cancer. Her medical history and a review of symptoms is otherwise negative, and a physical examination is normal. Which one of the following would be the most appropriate next step in evaluating this patient? (check one) A. A CBC B. A TSH level C. A complete metabolic panel D. Serologic testing for celiac sprue E. Stool testing for ova and parasites

D. serology for celiac sprue - IBS d/d: celiac sprue, microscopic and collagenous colitis, atypical Crohn's disease for patients with diarrhea-predominant IBS, and chronic constipation (without pain) for those with constipation-predominant IBS. - If there are no warning signs, laboratory testing is warranted only if indicated by the history.

Which one of the following statements is true about celiac disease (gluten-sensitive enteropathy) in adults? (check one) A. It is more common among African-Americans B. Symptoms are limited to gastrointestinal complaints C. Type 2 diabetics are at increased risk for the disease D. Serum antibody tests are sensitive and specific E. Colonoscopy with mucosal biopsy is required to make the diagnosis

D. serum antibody tests are sensitive and specific - Serum antibody testing, especially IgA antiendomysial antibody, is highly sensitive and specific and readily available at a cost of about $100 to $200.

The sensitivity of a test is defined as: (check one) A. the probability of disease before a test is performed B. the probability of disease after a test is performed C. the percentage of patients with a positive test result who are confirmed to have the disease D. the percentage of patients with the disease who have a positive test result E. the percentage of patients without the disease who have a negative test result

D. the percentage of patients with the disease who have a positive test result

Which one of the following is necessary to make a diagnosis of polymyalgia rheumatica? (check one) A. Joint swelling B. Early morning stiffness C. Reduction of symptoms with high-dose NSAID therapy D. An erythrocyte sedimentation rate ≥60 mm/hr E. Bilateral shoulder or hip stiffness and aching

E. Bilateral shoulder or hip stiffness and aching - must be bilateral shoulder or hip stiffness and aching for at least one month - does not respond to NSAIDs. - The erythrocyte sedimentation rate should be ≥40 mm/hr.

A 43-year-old female presents to your office for evaluation of a chronic cough that has been present for the past 6 months. She is not a smoker, and is not aware of any exposure to environmental irritants. She does not have any systemic complaints such as fever or weight loss, and does not have any symptoms of heartburn or regurgitation. She is not on any regular medications. Auscultation of the lungs and a chest radiograph show no evidence of acute disease. A trial of an inhaled bronchodilator and antihistamine therapy does not improve the patient's symptoms. Which one of the following would be the most appropriate next step? (check one) A. A methacholine inhalation challenge test B. Pulmonary function testing C. CT of the chest D. A trial of a proton pump inhibitor E. 24-hour pH monitoring

D. trial of a proton pump inhibitor - Gastroesophageal reflux disease (GERD) is one of the most common causes of chronic cough. - Aggressive acid reduction using a proton pump inhibitor twice daily before meals for 3-4 months is the best way to demonstrate a causal relationship between GERD and extra-esophageal symptoms (SOR B). Methacholine inhalation testing - not necessary since symptomatic asthma has been ruled out by the lack of response to bronchodilator therapy. Chest CT and pulmonary function tests - are not indicated given the lack of findings from the history, physical examination, and chest film to suggest underlying pulmonary disease

Which one of the following Mantoux tuberculin skin test results should be read as NEGATIVE for latent tuberculosis infection? (check one) A. 7 mm induration on an individual having recent household contact with a tuberculosis patient B. 8 mm induration on an HIV-positive individual who has no documented previous test result C. 10 mm induration on a nursing-home resident D. 12 mm induration on a homeless individual E. 9 mm induration on a hospital-based nurse who had a test with 2 mm induration 1 year ago

E. 9 mm induration on a hospital-based nurse who had a test with 2 mm induration 1 year ago 3 different cutoff levels defining a positive reaction 1. ≥5 mm is positive: for those who are at highest risk and/or immunocompromised, including HIV-positive patients, transplant patients, and household contacts of a tuberculosis patient, 2. ≥10 mm is positive: those who have an increased probability of exposure or risk 3. increased in 10mm or more w/in a 2-yr period: those who have repeated testing e.g. health-care workers 4. ≥15 mm is positive: for those at low risk of exposure (screening test not recommended)

A new home test is designed to detect a particular type of cancer. The gold standard test for this cancer is a biopsy, but a biopsy is more costly that a urine test, is invasive, and is associted with a number of adverse side effects. To test the effectiveness of the home urine test, 104 people took the test and then agreed to a biopsy. When the study was concluded, 77 people tested negative and 27 tested positive on the urine test. Biopsies were positive in 18 individuals, 8 of whom tested negative on the urine test. What is the negative predictive value of the home urine test, rounded to a whole number? (check one) A. 20% B. 37% C. 56% D. 80% E. 90%

E. 90% The negative predictive value is determined by dividing the true negatives (69) by the total number who tested negative (true plus false negatives = 77). The result is 89.6%, which rounded to a whole number is 90%.

Which one of the following patients is unlikely to benefit from vaccination against hepatitis A? (check one) A. A missionary traveling to Mexico B. A man who has sex with men C. A methamphetamine addict D. A patient with chronic hepatitis E. A 40-year old recent immigrant from India

E. A 40-year old recent immigrant from India Hepatitis A is so prevalent in developing countries such as India that virtually everyone is infected by the end of childhood, and therefore immune.

Which one of the following hospitalized patients is the most appropriate candidate for thromboembolism prophylaxis with enoxaparin (Lovenox)? (check one) A. An ambulatory 22-year-old obese male admitted for an appendectomy B. A 48-year-old male with atrial fibrillation on chronic therapeutic anticoagulation, admitted for cellulitis C. A 48-year-old male with end-stage liver disease and coagulopathy D. A 52-year-old female on chronic estrogen therapy, admitted with severe thrombocytopenia E. A 67-year-old female with hemiparesis, admitted for community-acquired pneumonia

E. A 67-year-old female with hemiparesis, admitted for community-acquired pneumonia - The patient on chronic anticoagulation, the patient with severe thrombocytopenia, and the patient with coagulopathy are at high risk for bleeding if given anticoagulants

A 30-year-old male presents with a 2-week history of swelling of the right posterior elbow. He recalls bumping his elbow against a door, but his pain quickly subsided. He began to notice the swelling over the next 2 days. On examination he has normal range of motion with a boggy, nontender mass over the olecranon. Which one of the following would be most appropriate at this point? (check one) A. A posterior splint B. Aspiration C. A corticosteroid injection D. A uric acid level and erythrocyte rate E. A compression dressing

E. A compression dressing - aseptic olecranon bursitis: preceded by minor trauma to the elbow followed by a nontender, boggy mass over the olecranon. Septic olecranon bursitis causes not just swelling, but also erythema, warmth, and pain. - Half of affected individuals will have a fever. - If septic bursitis is suspected, aspiration with bursal fluid analysis should be done and antibiotic therapy should be initiated. Aspiration is not recommended for the initial treatment of aseptic bursitis, as complications such as infection may occur. Management initially is with ice, compression dressings, and avoidance of activities that aggravate the problem. If conservative therapy is unsuccessful the problem can be managed by aspiration followed by compression dressings for 2 weeks. The bursa may be injected with a corticosteroid, but this could cause skin atrophy or infection. Surgical bursectomy can be offered for refractory cases lasting over 3 months.e6

A 55-year-old female presents with lateral hip pain over the outer thigh. She has no history of injury, although she has just begun a walking program to lose weight. She has increased pain when she lies on that side at night. Her examination is unremarkable except that she is overweight and has tenderness over the greater trochanter. There is no pain with internal and external rotation of the hip. A radiograph reveals minimal osteoarthritic changes. Which one of the following would be most appropriate at this point? (check one) A. Serum protein electrophoresis B. A bone scan C. A bone density study D. MRI E. A corticosteroid injection

E. A corticosteroid injection Trochanteric bursitis develops insidiously after repetitive use, and the patient may report morning stiffness and pain when lying on the affected side. Palpation of the greater trochanter elicits tenderness, and occasionally swelling may be noted as well. Early injection with a corticosteroid usually produces a satisfactory response.

Which one of the following, when confirmed with a repeat test, meets the diagnostic criteria for diabetes mellitus? (check one) A. A fasting blood glucose level of 120 mg/dL B. A 2-hour value of 180 mg/dL on an oral glucose tolerance test C. A random glucose level of 180 mg/dL in a patient with symptoms of diabetes mellitus D. A positive urine dipstick for glucose E. A hemoglobin A1c of 7.0%

E. A hemoglobin A1c of 7.0% - hemoglobin A1c level ≥6.5% be - fasting plasma glucose level ≥126 mg/dL - random glucose level ≥200 mg/dLw/ symptoms - 2-hour oral glucose tolerance test value ≥200 mg/dL - urine dipstick: not diagnostic

A 17-year-old female sees you for a preparticipation evaluation. She has run 5 miles a day for the last 6 months, and has lost 6 lb over the past 2 months. Her last menstrual period was 3 months ago. Other than the fact that she appears to be slightly underweight, her examination is normal. To fit the criteria for the female athlete triad, she must have which one of the following? (check one) A. A formal diagnosis of an eating disorder B. Amenorrhea for 1 year C. A Z-score on bone-density testing of -2.5 or less D. Withdrawal bleeding after progesterone administration E. A history of a stress fracture resulting from minimal trauma

E. A history of a stress fracture resulting from minimal trauma Energy availability - dietary energy intake minus exercise energy output, is now considered a risk factor for the triad Amenorrhea - Primary amenorrhea is defined as lack of menstruation by age 15 in females with secondary sex characteristics. - Secondary amenorrhea is the absence of three or more menstrual cycles in a young woman previously experiencing menses. - For those with secondary amenorrhea, a pregnancy test should be performed. If this is not conclusive, a progesterone challenge test may be performed. If there is withdrawal bleeding, the cause would be anovulation. - Those who do not experience withdrawal bleeding have hypothalamic amenorrhea, and fit one criterion for the triad. bone density test - Low bone mineral density for age is the term used to describe at-risk female athletes with a Z-score of -1 to -2 - Osteoporosis is defined as having clinical risk factors for experiencing a fracture, along with a Z-score <-2.

A 55-year-old female sees you because of a constant leakage of small amounts of urine. Her obstetric/gynecologic history includes two pregnancies, with vaginal deliveries. Her current medications include hydrochlorothiazide, metformin (Glucophage), and glyburide (DiaBeta). On examination she has mild diabetic retinopathy, decreased sensation to monofilament testing on her feet, and suprapubic fullness. The most appropriate initial treatment for this problem would be: (check one) A. tolterodine (Detrol LA) B. duloxetine (Cymbalta) C. estrogen replacement therapy D. bladder neck needle suspension E. a set schedule for urination

E. A set schedule for urination - Neurogenic bladder - can be caused by diabetes mellitus, multiple sclerosis, or spinal cord injury - usually initially treated with a strict voluntary urination schedule, which may be coupled with Crede's maneuver. - also adding bethanechol to the regimen - intermittent self-catheterization of the bladder. - Ultimately, the patient may require resection of the internal sphincter of the bladder neck. 4 types of urinary incontinence in women: 1. functional incontinence - the patient's inability to ambulate or transfer results in loss of urine 2. urinary stress incontinence - pelvic relaxation and is manifested as involuntary loss of urine with increases in abdominal pressure such as that which occurs with laughing, sneezing, or coughing 3. detrusor instability or overactive bladder - urge to urinate is quickly followed by loss of urine, usually a large volume 4. neurogenic bladder, which is marked by constant leakage of small amounts of urine.

A 7-year-old male with recurrent sinusitis has difficulty breathing through his nose. He has had chronic diarrhea and his weight is at the 5th percentile. Nasal polyps are noted on examination, in the form of grayish pale masses in both nares. No nasal purulence or odor is present. Which one of the following tests should you order? (check one) A. A serum angiotensin-converting enzyme level B. A serum alpha1-antitrypsin level C. A serum ceruloplasmin level D. An erythrocyte sedimentation rate E. A sweat chloride tes

E. A sweat chloride test - cystic fibrosis Any child 12 years or younger who presents with nasal polyps should be suspected of having cystic fibrosis until proven otherwise.

At a routine annual visit, a 31-year-old inner-city elementary school teacher asks you about a lesion on the nail of her ring finger, shown in Figure 8. On examination, you note that her other nails all have a slight linear depression or groove. Which one of the following is the most likely cause of this problem? (check one) A. A paronychial fungal infection B. Psoriasis C. Iron deficiency D. Lead exposure E. A traumatic/metabolic event

E. A traumatic/metabolic event - Beau's line - symmetric depression across the nail plate growing toward the distal edge - represents significant trauma to the body some weeks ago. T

A 25-year-old female presents with abdominal pain localized to the right lower quadrant. Which one of the following would be most helpful in diagnosing acute appendicitis? (check one) A. A CBC B. Urinalysis C. Plain abdominal films D. Abdominal/pelvic ultrasonography E. Abdominal/pelvic CT

E. Abd & pelvic CT - all others are not as specific or sensitive

A 45-year-old male presents with a 4-month history of low back pain that he says is not alleviated with either ibuprofen or acetaminophen. On examination he has no evidence of weakness or focal tenderness. Laboratory studies, including a CBC, erythrocyte sedimentation rate, C-reactive protein, and complete metabolic profile, are all normal. MRI of the lumbosacral region shows mild bulging of the L4-L5 disc without impingement on the thecal sac. Which one of the following has been shown to be beneficial in this situation? (check one) A. Traction B. Ultrasound C. Epidural corticosteroid injection D. A back brace E. Acupuncture

E. Acupuncture - Most chronic back pain (up to 70%) is nonspecific or idiopathic - Treatment options that have the best evidence for effectiveness include analgesics (acetaminophen, tramadol, NSAIDs), multidisciplinary rehabilitation, and acupuncture (all SOR A).

The mother of a 6-year-old male is concerned about his snoring, and she recently observed him stop breathing for a few seconds while he was sleeping. He has also been more sleepy during the day recently. His height and weight are normal. Polysomnography confirms obstructive sleep apnea. Which one of the following would be the most appropriate primary treatment? (check one) A. Methylphenidate (Ritalin) B. Lorazepam (Ativan) C. Fluoxetine (Prozac) on a daily basis D. A mouthguard E. Adenotonsillectomy

E. Adenotonsillectomy In children, obstructive sleep apnea (OSA) is most often due to enlarged tonsils and adenoids. OSA onset usu b/w 2-8, coinciding with peak tonsil growth. Adenotonsillectomy is the primary treatment for most non-obese children with OSA (SOR B). Not SSRIs - for nightmares because these medications can suppress rapid eye movement sleep. Not Benzodiazepines - for sleep terrors. Not Methylphenidate - stimulant used to treat attention-deficit/hyperactivity disorder and has no benefit for OSA. Not mouthguard - for temporomandibular joint syndrome to reduce excessive teeth grinding during sleep.

Which one of the following is true regarding hospice? (check one) A. Hospice benefits end if the patient lives beyond the estimated 6-month life expectancy B. A do-not-resuscitate (DNR) order is required for a patient receiving Medicare hospice benefits C. Patients in hospice cannot receive chemotherapy, blood transfusions, or radiation treatments D. Patients must be referred to hospice by their physician E. Any terminal patient with a life expectancy <6 months is eligible

E. Any patient with a life expectancy of less than 6 months who chooses a palliative care approach is an appropriate candidate for hospice. - Medicare does not require a DNR order to enroll in hospice, but it does require that patients seek only palliative, not curative, treatment. - Patients may receive chemotherapy, blood transfusions, or radiation if the goal of the treatment is to provide symptom relief. - Patients can be referred to hospice by anyone, including nurses, social workers, family members, or friends.

A 58-year-old male who works with heavy machinery at a local factory presents to your office for evaluation of hearing loss of several years' progression. He notes that the loss is mainly in the left ear and he also has mild tinnitus. He has had no trauma to his head, and he has no history of ear infections. Examination of the ears reveals normal tympanic membranes and a neurologic examination is negative. When a tuning fork is placed in the center of his forehead, he says the sound is much louder on the right side (Weber test). Comparing sound in front of the ear to the sound when the tuning fork is placed on the mastoid (the Rinne test) reveals that air conduction is better than bone conduction in the left ear. Which one of the following is true regarding further evaluation and management? (check one) A. No treatment or further diagnostic studies are indicated B. A hearing aid plus better hearing protection is all that is needed C. Carotid ultrasonography should be ordered D. A tympanogram is indicated E. Audiometry is the best initial screening test

E. Audiometry best initial test - Acoustic neuroma symptoms - cranial nerve involvement and progression of tumor size. - Hearing loss is present 95% of the time and tinnitus is very common. - The loss is usually chronic (over 3 years) and as many as one-third of patients are unaware it has occurred. - Vestibular nerve involvement most often causes mild unsteadiness and rarely has accompanying true vertigo. - Trigeminal involvement can cause pain, paresthesias, or numbness of the face. - Facial paresis occurs 6% of the time - diagnosis of acoustic neuroma is based on asymmetric sensorineural hearing loss or another cranial nerve deficit, with confirmation based on MRI with gadolinium contrast or a CT scan. - The best initial screening laboratory test is audiometry, as only 5% of patients with acoustic neuroma will have a normal test. - Sensorineural loss is usually in the higher frequencies. - Brainstem-evoked response audiometry may be used as a further screening measure when there are unexplained symmetrics and standard audiometric testing.

A 72-year-old female with longstanding diabetes mellitus presents to your office. During the review of systems, she complains of difficulty voiding and frequent "dribbling." A urinalysis is negative for infection and her post-void residual volume is 250 mL. Which one of the following is the most likely cause of this patient's urinary incontinence? (check one) A. Excess urine output due to hyperglycemia B. Atrophic vaginitis C. A grade II cystocele D. Asymptomatic bacteriuria E. Autonomic neuropathy

E. Autonomic neuropathy - Dribbling and increased post-void residual volume (>100 mL) are signs of overflow incontinence. - Overflow incontinence can be caused by outflow obstruction (e.g., prostate hypertrophy, urethral constriction, fecal impaction) - or by detrusor muscle denervation caused by diabetic or other neuropathies. not Excess urine output from hyperglycemia - urgency not Atrophic vaginitis and cystoceles - stress incontinence Not Asymptomatic bacteriuria - no infection

A 39-year-old male with a BMI of 41 kg/m2 is interested in weight loss. His medical history includes adequately controlled type 2 diabetes mellitus, well-controlled hypertension, hyperlipidemia, and obstructive sleep apnea. He has no history of coronary artery disease or COPD. Which one of the following is likely to be most effective for long-term weight loss in this patient? A. A very low calorie diet B. Increased physical activity C. Frequent, long-term weight-loss counseling D. Pharmacotherapy E. Bariatric surgery

E. Bariatric surgery Bariatric surgery - most effective weight reduction and long-term maintenance for a *BMI >40, or >35* with obesity-related comorbidities. - Gastric bypass a mean weight loss of 71.2% at 3 year - gastric banding the mean weight loss is 55.2% at 3 years. - In one study, 94% of gastric bypass patients maintained at least a 20% weight loss at 6 years. - resolution of diabetes mellitus in 31%-77% of lap band patients and 72%-100% of gastric bypass patients. The AAFP recommends screening for obesity and intensive counseling (more than 1 session per month for more than 3 months) with behavior modification for obese patients. Counseling is ineffective by itself and must be combined with lifestyle modification. Dietary modification, increased physical activity, and behavior modification are effective for maintaining modest weight loss for greater than 1 year (SOR B) (mostly 10% in 1 yr) Long-term pharmacotherapy can lead to weight loss, but regaining some weight is typical.

Which one of the following can help to minimize the pain of lidocaine (Xylocaine) injection? (check one) A. Slowly inserting the needle through the skin B. Avoiding injectionnn into the subcutaneous tissue C. Injection of the solution only after fully inserting the needle at the target site D. Cooling the solution to refrigerator temperature prior to injecting it E. Buffering the solution with sodium bicarbonate

E. Buffering the solution with sodium bicarbonate Lidocaine buffered with sodium bicarbonate decreases the pain associated with the injection. This effect is enhanced when the solution is warmed to room temperature (SOR B). Rapidly inserting the needle through the skin, injecting the solution slowly and steadily while withdrawing the needle, and injecting into the subcutaneous tissue also minimize the pain of injection.

Which one of the following organisms is NOT killed by alcohol-based hand disinfectants? (check one) A. Methicillin-resistant Staphylococcus aureus (MRSA) B. Methicillin-sensitive Staphylococcus aureus C. Pseudomonas aeruginosa D. Klebsiella pneumoniae E. Clostridium difficile

E. C. diff Sporulating organisms such as Clostridium difficile are not killed by alcohol products.

A 69-year-old female is unable to obtain adequate calcium from dietary sources. She is on long-term therapy with pantoprazole (Protonix) for peptic ulcer disease. Which one of the following would be the most appropriate calcium supplement for this patient? (check one) A. Oyster shell calcium B. Calcium carbonate C. Calcium lactate D. Calcium gluconate E. Calcium citrate

E. Calcium citrate - does not depend on stomach acid for absorption

Under current guidelines, hospice programs are most likely to serve patients dying from: (check one) A. heart failure B. COPD C. severe dementia D. multiple strokes E. cancer

E. Cancer - general requirement: a terminal illness and an estimated life expectancy of 6 months or less. - over 40% of hospice patients have a cancer diagnosis. - Cancer usually has a short period of obvious decline at the end and is predictable to a degree. - Diseases such as COPD, end-stage liver disease, and heart failure result in long-term disability with periodic exacerbations, any one of which could result in death, but far less predictably. - Those with severe dementia or frailty often experience a dwindling course that is also difficult to predict.

A 28-year-old male is seen for follow-up of acute low back pain. He has a past history of substance abuse. Ibuprofen and acetaminophen have helped some, but he is experiencing muscle spasms. It is best to avoid which one of the following when treating this patient's problem? (check one) A. Chlorzoxazone (Parafon Forte DSC) B. Metaxalone (Skelaxin) C. Cyclobenzaprine (Flexeril) D. Methocarbamol (Robaxin) E. Carisoprodol (Soma)

E. Carisoprodol (Soma) - muscle relaxants: low effectiveness, high toxicity - Carisoprodol is metabolized to meprobamate, which is a class III controlled substance. It has been shown to produce both physical and psychologic dependence.

A 30-year-old female at 36 weeks gestation has a positive culture for group B Streptococcus. Her past medical history is significant for the development of a nonurticarial rash in response to penicillin. Which one of the following is most appropriate for intrapartum antibiotic prophylaxis in this patient? (check one) A. Azithromycin (Zithromax) B. Clindamycin (Cleocin) C. Vancomycin (Vancocin) D. Ampicillin E. Cefazolin

E. Cefazolin Best for GBS

A 50-year-old female with significant findings of rheumatoid arthritis presents for a preoperative evaluation for planned replacement of the metacarpophalangeal joints of her right hand under general anesthesia. She generally enjoys good health and has had ongoing medical care for her illness. Of the following, which one would be most important for preoperative assessment of this patient's surgical risk? (check one) A. Resting pulse rate B. Resting oxygen saturation C. Erythrocyte sedimentation rate D. Rheumatoid factor titer E. Cervical spine imaging

E. Cervical spine imaging - detect atlantoaxial subluxation would be most important for preventing a catastrophic spinal cord injury during intubation. - cervical fusion must be performed before other elective procedures can be contemplated.

Which one of the following screening practices is recommended for the adolescent population by the U.S. Preventive Services Task Force? (check one) A. Lipid screening for 3 months B. Scoliosis screening C. Testicular examination D. Papanicolaou tests starting 3 years after first sexual intercourse E. Chlamydia screening in sexually active females

E. Chlamydia screening in sexually active females

Which one of the following would be the most effective monotherapy for alcohol withdrawal syndrome? (check one) A. Clonidine (Catapres) B. Phenytoin (Dilantin) C. Atenolol (Tenormin) D. Phenobarbital E. Chlordiazepoxide (Librium)

E. Chlordiazepoxide (Librium) - withdrawal mediated by the abrupt removal of alcohol-enhanced GABA inhibition of excitatory glutamate receptors in the central nervous system - three stages - seizures may occur - ideal pharmacologic agent should provide sedation and protection from seizures - *Long-acting benzodiazepines* such as chlordiazepoxide have been shown to be superior to the other choices in numerous studies.

A 50-year-old male with a history of methamphetamine abuse requests medication to treat this problem. According to evidence-based studies, which one of the following would be most likely to help this patient overcome methamphetamine dependence? (check one) A. Fluoxetine (Prozac) B. Amlodipine (Norvasc) C. Imipramine D. Bupropion (Wellbutrin) E. Cognitive therapy

E. Cognitive therapy The standard therapy for methamphetamine dependence is outpatient behavioral therapies, especially with case management included.

A 70-year-old female with type 2 diabetes mellitus is admitted to the hospital with a 4-week history of fever, anorexia, and weight loss. Two blood cultures are positive for Streptococcus bovis. In addition to being treated for the infection, she should be evaluated for which one of the following? (check one) A. B-cell lymphoma B. T-cell lymphoma C. Multiple myeloma D. Lung cancer E. Colorectal cancer

E. Colorectal cancer

A 45-year-old male with chronic nonmalignant back pain is on a chronic narcotic regimen. Which one of the following behaviors is LEAST likely to be associated with pseudoaddiction, as opposed to true addiction? (check one) A. Requesting a specific drug B. Aggressive complaining about needing more medication C. Hoarding drugs during periods of reduced symptoms D. Requesting medication exactly at prescribed times when hospitalized E. Concurrent abuse of alcohol or illicit drugs

E. Concurrent abuse of alcohol or illicit drugs Agreements for use of chronic opioids should include the expectation that alcohol and illicit drugs will not be used concurrently, and doing so suggests addiction rather than pseudoaddiction.

A 70-year-old male who recently moved to your area sees you for the first time. He has a previous history of myocardial infarction, has a pacemaker, and has hypertension that had been well controlled on hydrochlorothiazide and atenolol (Tenormin) for several years. About 6 months ago his previous physician had to add amlodipine (Norvasc) to his regimen. On examination he has mild arteriolar narrowing in his fundi and there is a systolic bruit just to the right of his umbilicus. He has a log of home blood pressure readings that average 138/88 mm Hg for the past 2 months. His serum creatinine level has gone from 1.2 mg/dL to 1.4 mg/dL (N=0.6-1.2) in the past 2 months. Which one of the following would be most appropriate at this time? (check one) A. Referral for stent placement B. Scheduling an arteriogram C. A captopril renal scan D. Adding losartan (Cozaar) to his regimen E. Continued monitoring of serum creatinine

E. Continued monitoring of serum creatinine - renal artery steosis - 5% of patients with hypertension: in CAD/PVD, HTN w/ 4-5 meds, abd bruit, low potassium, renal insufficiency (low Cr) - initiating therapy with an ACE inhibitor can all point toward renal artery stenosis as a diagnosis. - if BP control esp w/ ACE/ARB, no testing needed other than watching Cr - Catopril test not used anymore, screening tests: duplex ultrasonography, CT angiography, or MR cystography - stenting does not help

A 31-year-old female who is a successful professional photographer complains of hoarseness that started suddenly 3 weeks ago. She says she can remember exactly what day it was, because her divorce became final the next day. The day the problem began, she was only able to whisper from the time she woke up, and she is able to speak only in a weak whisper while relating her history. She does not appear to strain while speaking. She does not smoke, has had no symptoms of an upper respiratory infection, and has no pain, cough, or wheezing. She is on a proton pump inhibitor prescribed by an urgent care provider 2 weeks ago. This has not changed her symptoms. She takes no other medications and has no known allergies. A head and neck examination, including indirect laryngoscopy, is within normal limits. Which one of the following is the most likely diagnosis? (check one) A. Muscle tension aphonia B. Laryngopharyngeal reflux C. Spasmodic dysphonia D. Vocal abuse E. Conversion aphonia

E. Conversion aphonia

Patients with which rheumatologic condition have the highest relative risk of internal malignancy compared to the general population? (check one) A. Systemic scleroderma B. Systemic lupus erythematosus C. Sjögren's syndrome D. Rheumatoid arthritis E. Dermatomyositis

E. Dermatomyositis 32% of patients with dermatomyositis had cancer.

A 32-year-old primipara is ready to be discharged after a full-term vaginal delivery that was complicated by a prolonged second stage of labor. She required a second-degree posterior vaginal repair, but had no periurethral trauma. A transurethral catheter was removed a few hours after delivery, but 48 hours later she complained of abdominal pain and a persistent need to urinate. The catheter was replaced and yielded approximately 2000 cc of straw-colored urine. Urinary symptoms quickly resolved, but the patient continues to be unable to void on her own. A perineal examination is normal, as is a urinalysis. Which one of the following would be the most appropriate management at this time? (check one) A. Oxybutynin (Ditropan), 10 mg daily B. Prednisone, starting with 60 mg/day and tapering quickly over 7 days C. Urgent vaginal ultrasonography D. Urology consultation for cystoscopy E. Discharge with a catheter in place and close follow-up

E. Discharge with catheter in place and close follow-up - postpartum urinary retention (PUR). - post-void bladder residual of at least 150 cc that is present 6 hours or more after delivery - The use of intermittent self-catheterization or a transurethral catheter is recommended until the patient's ability to spontaneously micturate returns - more likely to occur in patients who are primiparous, have a prolonged first or second stage of labor, have instrumented vaginal deliveries, or require a cesarean section for failure to progress - The question of whether epidural anesthesia promotes the condition is still debated. - Most cases of PUR will resolve 2-6 days after delivery, but some can take up to several weeks. - Imaging studies and referrals to a specialist are rarely necessary, and no medication has been proven helpful.

A 24-year-old primigravida has nausea and vomiting associated with pregnancy. Which one of the following is recommended by the American Congress of Obstetricians and Gynecologists (ACOG) as first-line therapy? (check one) A. Droperidol (Inapsine) B. Ondansetron (Zofran) C. Prochlorperazine D. Metoclopramide (Reglan) E. Doxylamine (Unisom) and vitamin B

E. Doxylamine (Unisom) and vitamin B

A 50-year-old male is brought to the emergency department because of a syncopal episode.Prior to the episode, he felt bad for 30 minutes, then developed nausea followed by vomiting. During a second bout of vomiting he blacked out and fell to the floor. His wife did not observe any seizure activity, and he was unconscious only for a few seconds. His history is otherwise negative, his past medical history is unremarkable, and he currently takes no medications. A physical examination is normal. Which one of the following would be the most helpful next step? (check one) A. CT of the head B. Carotid ultrasonography C. A CBC and complete metabolic profile D. Echocardiography E. An EKG

E. EKG - Patients who have a prodrome of 5 seconds or less may have a cardiac arrhythmia - Patients with longer prodromes, nausea, or vomiting are likely to have vasovagal syncope, which is a benign process - Patients who pass out after standing for 2 minutes are likely to have orthostatic hypotension. - In most cases, the recommended test is an EKG. If the EKG is normal, dysrhythmias are not a likely cause of the syncopal episode. - Laboratory testing and advanced studies such as CT or echocardiography are not necessary unless there are specific findings in either the history or the physical examination.

A 70-year-old female becomes psychotic and risperidone (Risperdal) is prescribed. Which one of the following should be used to monitor the patient for adverse cardiac effects of this drug? (check one) A. Serum sodium levels B. Echocardiography C. Nuclear stress testing D. Lower-extremity venous duplex ultrasonography E. Electrocardiography

E. EKG Both typical and atypical antipsychotics can cause prolongation of the QTc interval, resulting in torsades de pointes, ventricular tachycardia, and sudden death.

The mother of a 2-year-old calls you for advice because her child has an acute cough that is keeping him awake at night. Which one of the following has been shown in a double-blind, randomized, placebo-controlled study to decrease nighttime cough and improve sleep in children with this problem? (check one) A. Sugar water B. Cinnamon C. Turmeric D. Ginger E. Honey

E. Honey but not for peds under 12 mos

A mother brings her 5-year-old daughter to see you because she found a mass in the child's neck. The mass appeared over the past week and was preceded by a sore throat. Her pharyngitis is now resolved but she still has a fever, although it is not as high. The mother is most concerned because the mass developed over a short span of time, and it is warm, red, and tender. When asked, she says that her daughter has had no recent exposure to cats. When you examine the child you note that her temperature is 38.0°C (100.4°F). You also find shotty adenopathy in both anterior cervical lymph node chains, and a 2.5-cm warm, firm, moderately tender lymph node in the right anterior cervical chain. The overlying skin is also erythematous. Which one of the following would be the most appropriate management at this time? (check one) A. Ultrasonography of the neck mass B. CT with intravenous contrast of the neck mass C. Ultrasound-guided fine-needle aspiration of the mass D. Immediate referral to a head and neck surgeon E. Empiric antibiotic therapy with observation for 4 weeks

E. Empiric antibiotic therapy with observation for 4 weeks - cervical lymphadenitis - systemic symptoms, unilateral lymphadenopathy, skin erythema, node tenderness, and a node that is 2-3 cm in size. - most common: Staph aureus & group A Strep - if symptoms do not resolve, or if the mass increases in size during antibiotic treatment, further evaluation is appropriate. Not U/S: not needed, though preferred Not CT w/ contrast: for malignancy or retropharyngeal or deep neck abscess Not surgeon: only if signs of cancer - >3.0 cm in size, hard, firm, immobile - with type B symptoms such as fever, malaise, weight loss, or night sweats

In older patients with aortic stenosis and a systolic murmur, which one of the following would be most concerning? (check one) A. Weight loss B. Frequent urination C. Jaundice D. Worsening headache E. Exertional dyspnea

E. Exertional dyspnea When symptoms begin to appear in a patient with aortic stenosis the prognosis worsens. It is therefore important to be aware of systolic murmurs in older patients presenting with exertional dyspnea, chest pain, or dizziness. This can be the first presentation of a downward spiral and the need for rapid valve replacement. .

A 25-year-old female at 31 weeks gestation presents to the labor wing with painful uterine contractions every 3 minutes. On examination her cervix is 3 cm dilated and 50% effaced. Her membranes are intact and fetal heart monitoring is reassuring. She is treated with tocolysis,betamethasone, antibiotics, and intravenous hydration, and cultured for group B Streptococcus. The neonatal intensive care unit is notified, but the contractions ease and eventually stop. After 2 days of observation, her cervix is unchanged and she is discharged home. One week later, the patient presents with contractions for the last 8 hours. Her cervical findings are unchanged. Her group B Streptococcus culture was negative. Which one of the following would be the most appropriate next step in the management of this patient? (check one) A. Repeat tocolysis, betamethasone, antibiotics, and intravenous hydration B. Betamethasone, antibiotics, and intravenous hydration only C. Antibiotics and intravenous hydration only D. Tocolysis only E. Expectant management

E. Expectant management - Preterm labor before 34 weeks gestation goal: allow time to administer corticosteroids. - Treatment does not substantially delay delivery beyond 1 week - monitor & GYN consult is neccessary Not Betamethasone - Repeated administration of corticosteroids does not confer more benefit than a single course Not Antibiotics: - Prophylaxis of group B Streptococcus - useful for delaying delivery if membranes are ruptured. - They do not add any benefit otherwise, even though subclinical amnionitis may be a causative factor Not Prolonged and repeated tocolysis - harmful. - not indicated in this patient because she has had no cervical change and is therefore having preterm contractions, not preterm labor

A full-term newborn, born 72 hours ago, is noted to be jaundiced. The pregnancy was uneventful and the delivery uncomplicated. The mother has type A-positive blood and the child has type O-positive. The child is breastfed and has lost 9 ounces from a birth weight of 8 lb. He is feeding for 20 minutes every 4 hours, and except for being icteric, has a normal examination. Laboratory evaluation reveals a total serum bilirubin level of 16 mg/dL (N 1.4-8.7), with a conjugated bilirubin level of 1.0 mg/dL. His hemoglobin level is 17.8 g/dL (N 13.4-19.8), his hematocrit is 55% (N 41-65), and his reticulocyte count is 3% (N 3-7). Appropriate management would include: (check one) A. Phototherapy B. Exchange transfusion C. Blood cultures and antibiotic therapy D. Dextrose and water supplementation E. A recommendation to increase feedings to 10 times a day

E. Feed 10 times a day - Phototherapy should be initiated when the bilirubin level is *above 15 mg/dL* for infants at age 29-48 hours old, at *18 mg/dL* for infants 49-72 hours old, and at *20 mg/dL* in infants older than 72 hours. - *rises by more than 5 mg/dL/day or is higher than 17 mg/dL*, or if the infant has signs or symptoms suggestive of a serious underlying illness such as sepsis. - breat-feeding jaundice (i.e. lack of) bilirubin rising above 12 mg/dL. - frequency of feeding should be increased to more than 10 times per day

An elderly male patient takes aspirin, 81 mg daily, for prevention of a heart attack. He also takes herbal supplements. Which one of the following supplements can have a negative interaction with aspirin? (check one) A. Kava B. Yohimbine C. Saw palmetto D. Echinacea E. Ginkgo biloba

E. Ginko biloba - associated with serious intracerebral bleeding. - esp. with concurrent anticoagulant drugs were being used. - inhibit platelet aggregation causing bleeding. Kava: gastrointestinal side effects and skin rashes. Yohimbine: hypertension Saw palmetto and echinacea: not associated with bleeding.

In a patient with symptoms of thyrotoxicosis and elevated free thyroxine (T4 ), the presence of thyroid TSH receptor site antibodies would indicate which one of the following as the cause of thyroid gland enlargement? (check one) A. Toxic multinodular goiter B. Toxic adenoma C. Hashimoto's (lymphadenoid) thyroiditis D. Subacute (giant cell) thyroiditis E. Graves' disease

E. Grave's disease - TSH receptor immunoglobulins causes goiter & thyrotoxicosis Hashimoto's disease - TSH receptor-blocking antibodies Both are autoimmune, both antibodies goe away with therapy

A 20-year-old female is seen for follow-up 6 weeks after delivery. Her pregnancy was complicated by preeclampsia. Her examination is unremarkable. This patient will be at increased risk for which one of the following in midlife? (check one) A. Breast cancer B. Diabetes mellitus C. Hypothyroidism D. Kidney disease E. Hypertension

E. Hypertension - Preeclampsia affects as many as 5% of first pregnancies - hypertension, proteinuria, edema, and rapid weight gain after 20 weeks gestation. - Very young mothers and those over age 35 have a higher risk. - Patients who have had preeclampsia have a fourfold increased risk of hypertension and a two fold increased risk of ischemic heart disease, stroke, and venous thromboembolism

A 14-year-old female is brought to your office by her parents because of concerns regarding her low food intake, excessive exercise, and weight loss. Her weight is less than 75% of ideal for her height. Which one of the following sets of additional findings would indicate that the patient suffers from severe anorexia nervosa? (check one) A. Hypertension, tachycardia, and hyperthermia B. Hypertension, tachycardia, and hypothermia C. Hypotension, tachycardia, and hypothermia D. Hypotension, bradycardia, and hyperthermia E. Hypotension, bradycardia, and hypothermia

E. Hypotension, bradycardia, and hypothermia - Characteristic vital signs in patients with severe anorexia nervosa include hypotension, bradycardia, and hypothermia. - Criteria for hospital admission include a heart rate <40 beats/min, blood pressure <80/50 mm Hg, and temperature <36°C (97°F). - Increased cardiac vagal hyperactivity is thought to cause the bradycardia.

An abandoned infant is brought to the hospital for evaluation. Based on the presence of a dried umbilical cord remnant and her overall appearance, you believe her to be no more than 5 days of age. A thorough examination is normal except for a finding of bilateral conjunctival erythema and exudate. A Gram stain of the exudate is remarkable for numerous WBCs, very few of which are noted to contain gram-negative diplococci. Which one of the following treatment options is most appropriate? (check one) A. Application of moist, warm saline eye compresses B. Irrigation of both eyes with povidone-iodine (Betadine) C. One-time application of ophthalmic erythromycin ointment into both eyes D. Instillation of silver nitrate solution into both eyes E. Intramuscular injection of ceftriaxone (Rocephin)

E. IM ceftriaxone (Rocephin) - infantile gonococcal - 2-5 days after birth - Topical antibiotics are appropriate for prophylaxis, but not for treatment - Silver was used for prophylaxis at one time, but is no longer available. - Povidone-iodine has not been studied for prevention.

A 45-year-old male presents with shortness of breath and a cough. On pulmonary function testing his FVC is <80% of predicted, his FEV1/FVC is 90% of predicted, and there is no improvement with bronchodilator use. The diffusing capacity of the lung for carbon monoxide (DLCO) is also low. Based on these results, which one of the following is most likely to be the cause of this patient's problem? (check one) A. Asthma B. Bronchiectasis C. COPD D. Cystic Fibrosis E. Idiopathic pulmonary fibrosis

E. Idiopathic pulmonary fibrosis Pulmonary fibrosis is compatible with this pattern. A patient with any of the other listed diagnoses would be expected to have an obstructive pattern on testing.

A 45-year-old female presents to your office with a 1-month history of pain and swelling posterior to the medial malleolus. She does not recall any injury, but reports that the pain is worse with weight bearing and with inversion of the foot. Plantar flexion against resistance elicits pain, and the patient is unable to perform a single-leg heel raise. Which one of the following is true regarding this problem? (check one) A. The patient most likely has a medial ankle sprain B. NSAIDs will improve the long-term outcome C. Injecting a corticosteroid into the tendon sheath of the involved tendon is recommended D. A lateral heel wedge should be prescribed E. Immobilization in a cast boot for 3 weeks is indicated

E. Immobilization in a cast boot for 3 weeks is indicated - posterior tibial tendinopathy - most common in women at 40 - posterior tibial tendon function: plantar flexion, inversion, stabilization medial longitudinal arch - An injury can, over time, elongate the midfoot and hindfoot ligaments, causing a painful flatfoot deformity or tendon rupture - The patient usually recalls no trauma, although the injury may occur from twisting the foot by stepping in a hole - usu. have pain and swelling of the tendon; and is misdiagnosed as a medial ankle sprain - With the patient standing on tiptoe, the heel should deviate in a varus alignment, but this does not occur on the involved side. - A single-leg toe raise should reproduce the pain, and if the process has progressed, this maneuver indicates progression of the problem.

A resting ankle-brachial index of 1.50 indicates which one of the following? (check one) A. Normal circulation to a lower extremity B. Borderline normal circulation which may not be problematic in an asymptomatic patient C. Mild peripheral artery disease in a lower extremity D. Severe peripheral artery disease in a lower extremity E. Incompressible vessels in a lower extremity

E. Incompressible vessels in a lower extremity ABI - over 1.40 is incompressible vessel & not reliable e.g. DM2, elderly (may use toe-brachial) normal 1.00-1.04 borderline 0.91-0.99 abnormal <0.90

A 68-year-old African-American male with a history of hypertension and heart failure continues to have shortness of breath and fatigue after walking only one block. He has normal breath sounds, no murmur, and no edema on examination. His current medications include furosemide (Lasix), 20 mg/day, and metoprolol extended-release (Toprol-XL), 50 mg/day. He previously took lisinopril (Prinivil, Zestril), but it was discontinued because of angioedema. A recent echocardiogram showed an ejection fraction of 35%. Which one of the following would be most likely to improve both symptoms and survival in this patient? (check one) A. Valsartan (Diovan) B. Metolazone (Zaroxolyn) C. Digoxin D. Verapamil (Calan, Isoptin) E. Isosorbide/hydralazine (BiDil)

E. Isosorbide/hydralazine - systolic heart failure, the usual management 1: ACE inhibitor and a ß-blocker - Since this patient had angioedema with an ACE inhibitor, an angiotensin receptor blocker may cause this side effect 2. combination of direct-acting vasodilators such as isorbide and hydralazine - For patients who cannot tolerate an ACE inhibitor, especially African-Americans, Metolazone (thiazide-like diuretic) - not necessary unless the patient has volume overload that does not respond to increased doses of furosemide. Digoxin (inhibiting the Na+/K+-ATPase, cardiac glycosides) - may improve symptoms, but has not been shown to increase survival Verapamil (CCB) - negatively ionotropic; bad

Treatment of rhabdomyolysis should routinely include which one of the following? (check one) A. Bicarbonate-containing fluids B. Loop diuretics C. Mannitol D. Parenteral corticosteroids E. Isotonic saline

E. Isotonic saline Sodium bicarbonate administration is unnecessary and is not better than normal saline diuresis and increasing urine pH.

A healthy 48-year-old female consults you about continuing the use of her estrogen/progestin oral contraceptives. She has regular menstrual periods, is not hypertensive or diabetic, and does not smoke. What advice would you give her? (check one) A. She should stop the oral contraceptives B. She should switch to a progestin-only pill C. She should discontinue the contraceptive for 1 month, and if FSH is then elevated to postmenopausal levels, the pills should be stopped D. She can safely continue to take the contraceptive if screening for thrombophilic conditions is negative E. It is safe to continue the oral contraceptives

E. It is safe to continue the oral contraceptives Healthy women may continue combination birth control pills into their fifties, and this patient has no contraindications. Screening for thrombophilic conditions is not indicated due to the low yield. FSH is not specific enough to stop OCP

A 32-year-old female who is an avid runner presents with knee pain. You suspect patellofemoral pain syndrome. Which one of the following signs or symptoms would prompt an evaluation for an alternative diagnosis? (check one) A. Peripatellar pain while running B. Knee stiffness with sitting C. A popping sensation in the knee D. Locking of the joint E. A positive J sign (lateral tracking of the patella when moved from flexion to full extension)

E. Joint locking - Patellofemoral pain syndrome - clinical diagnosis - most common cause of knee pain in the outpatient setting - anterior knee pain, particularly with activities that overload the joint, such as stair climbing, running, and squatting. - Patients complain of popping, catching, stiffness, and giving way. - On examination there will be a positive *J sign*, with the patella moving from a medial to a lateral location when the knee is fully extended from the 90° position. This is caused by an imbalance in the medial and lateral forces acting on the patella. Locking is not characteristic of patellofemoral pain syndrome, so a loose body or meniscal tear should be considered if this is reported.

A 13-year-old female is being evaluated for primary amenorrhea. On examination she has short stature, a webbed neck, and a low hairline. A physical examination reveals no signs of pubertal development. Which one of the following is most likely to provide a diagnosis? (check one) A. MRI of the pituitary B. FSH and LH levels C. A prolactin level D. Pelvic ultrasonography E. Karyotyping

E. Karyotyping Primary amenorrhea is defined as a history of no menses in a female 13 years of age or older with no pubertal development, or 5 years after initial breast development, or in a patient older than 15 years. Primary amenorrhea is typically due to chromosomal problems that lead to primary ovarian insufficiency or anatomic abnormalities. If the patient has dysmorphic features such as short stature, a low hairline, or a webbed neck, the suspicion for Turner's syndrome should be high. While FSH and LH levels may be elevated, the definitive diagnosis would be made from a karyotype.

Which one of the following is most appropriate in the management of frostbite? (check one) A. Complete surgical debridement of hemorrhagic blisters B. Rapid rewarming with dry heat and warm blankets C. Withholding analgesics to ensure accurate assessment D. Vigorously rubbing affected tissues to restore circulation E. Leaving simple, non-tense areas of clear blistering intact

E. Leaving simple, non-tense areas of clear blistering intact

Which one of the following drugs would be the most appropriate empiric therapy for nursing home-acquired pneumonia in a patient with no other underlying disease? (check one) A. Cefazolin B. Erythromycin C. Ampicillin D. Tobramycin (Nebcin) E. Levofloxacin (Levaquin)

E. Levofloxacin (Levaquin) The major concern with regard to pneumonia in the nursing-home setting is the increased frequency of oropharyngeal colonization by gram-negative organisms. In the absence of collectible or diagnostic sputum Gram's stains or cultures, empiric therapy must cover Streptococcus pneumoniae, Staphylococcus aureus, Haemophilus influenzae, and gram-negative bacteria. Levofloxacin is the best single agent for providing coverage against this spectrum of organisms

Which one of the following, especially in homeless people, is a vector for Bartonella quintana, which causes trench fever, an influenza-like syndrome with relapsing fever? (check one) A. Fleas B. Maggots C. Bedbugs D. Scabies E. Lice

E. Lice This disease got its name in World War I, when soldiers in the trenches were often infested with body lice. This is a serious disease that can be treated with antibiotics.

A 72-year-old male with a history of hypertension and a previous myocardial infarction is diagnosed with heart failure. Echocardiography reveals systolic dysfunction, and recent laboratory tests indicated normal renal function, with a serum creatinine level of 1.1 mg/dL (N <1.5), a sodium level of 139 mEq/L (N 136-145), and a potassium level of 3.5 mEq/L (N 3.5-5.0). He is currently asymptomatic. Which one of the following medications would be the best choice for initial management in this patient? (check one) A. Furosemide (Lasix) B. Isosorbide dinitrate (Isordil) C. Spironolactone (Aldactone) D. Digoxin E. Lisinopril (Prinivil, Zestril)

E. Lisinopril (Prinivil, Zestril) These agents have beeC. Repeat echocardiography in 3 yearsn shown to reduce mortality from heart failure. - Direct-acting vasodilators such as isosorbide dinitrate also could be used in this patient, but ACE inhibitors have been shown to be superior in randomized, controlled trials (SOR B). - β-Blockers are also recommended in heart failure patients with systolic dysfunction (SOR A), except those who have dyspnea at rest or who are hemodynamically unstable. These can be used in patients with a recent myocardial infarction who develop symptomatic systolic dysfunction and in those with diabetes mellitus (SOR B). - A diuretic such as furosemide may be indicated to relieve congestion in symptomatic patients. - Aldosterone antagonists such as spironolactone are also indicated in patients with symptomatic heart failure. - Digoxin currently is recommended for patients with heart failure and atrial fibrillation, and can be considered in patients who continue to have symptoms despite maximal therapy with other agent

A 26-year-old female presents with a 1-year history of recurring abdominal pain associated with intermittent diarrhea, 5-7 days per month. Her pain improves with defecation. There has been no blood in her stool and no weight loss. Laboratory findings are normal, including a CBC, chemistry profile, TSH level, and antibodies for celiac disease. Which one of the following would be most appropriate at this point? (check one) A. Colonoscopy B. An upper GI series with small-bowel follow-through C. Abdominal CT with contrast D. A gluten-free diet E. Loperamide (Imodium)

E. Loperamide Rome criteria - 3 days per month of abdominal pain for the past 3 months - a change in the frequency of stool - and improvement with defecation. According to current clinical guidelines IBS can be diagnosed by history, physical examination, and routine laboratory testing, as long as there are no warning signs. - Warning signs include rectal bleeding, anemia, weight loss, fever, a family history of colon cancer, onset of symptoms after age 50, and a major change in symptoms. Colonoscopy, CT, and GI contrast studies are not indicated. - A gluten-free diet would not be indicated since the antibody tests for celiac disease are negative

A 50-year-old male presents to your office with a 1-hour history of an intense retro-orbital headache. This started while he was jogging and eased somewhat when he stopped, but has persisted along with some pain in his neck. Other than a blood pressure of 165/100 mm Hg, his examination is unremarkable. Noncontrast CT of the head is also unremarkable. His pain has persisted after 2 hours in the emergency department. Which one of the following would be most appropriate at this time? (check one) A. MRI of the head B. Angiography C. Nifedipine (Procardia) sublingually D. Sumatriptan (Imitrex) subcutaneously E. A lumbar puncture

E. Lumbar puncture nontraumatic subarachnoid hemorrhage Risk factors: smoking, hypertension, heavy alcohol use, and a family history of aneurysm or hemorrhagic stroke. The initial evaluation should consist of noncontrast CT of the head (SOR C). If it is negative or equivocal the next step would be to perform a lumbar puncture to determine whether or not the cerebrospinal fluid is xanthochromic. The absence of xanthochromia rules out subarachnoid hemorrhage (SOR C).

A 58-year-old male delivery truck driver is diagnosed with type 2 diabetes mellitus and after several months of working on lifestyle modification his hemoglobin A1c is 8.0%. You suggest it is time to start a medication to help control his condition but he is very worried about having a "low sugar reaction" that would prevent him from driving. He is on no other medications at this time. His only other health problem is long-standing controlled hypertension. His BMI is 33.1 kg/m2 and his serum creatinine level is 1.2 mg/dL (N 0.6-1.5). Which one of the following medications would be least likely to cause hypoglycemia in this patient? (check one) A. Canagliflozin (Invokana) B. Glimepiride (Amaryl) C. Glipizide (Glucotrol) D. Insulin glargine (Lantus) E. Metformin (Glucophage)

E. Metformin (Glucophage) - inexpensive - increase the sensitivity of the liver and peripheral tissues to insulin - assists the patient with weight loss efforts - unlike insulin secretagogues, has been proven to reduce mortality with long-term use - can increase the risk for lactic acidosis in heart failure, hypovolemia, or renal impairment - don't use in pts with Cr >1.5, but newer evidence says can use even if GFR >30 mL/min (mild-moderate renal impairment)

Which one of the following NSAIDs is safest for patients with a previous history of myocardial infarction? (check one) A. Ibuprofen B. Celecoxib (Celebrex) C. Diclofenac (Zorvolex) D. Meloxicam (Mobic) E. Naproxen (Naprosyn)

E. Naproxen no explaination

A 45-year-old female presents with a rash on the central portion of her face. She states that she has intermittent flushing and intense erythema that feels as if her face is stinging. She has noticed that her symptoms can be worsened by sun exposure, emotional stress, alcohol, or eating spicy foods. She has been in good health and has taken conjugated estrogens (Premarin), 0.625 mg daily, since a hysterectomy for benign reasons. A general examination is normal except for erythema of the cheeks and chin. No pustules or comedone formation is noted around her eyes, but telangiectasias are present. Which one of the following would be appropriate in the management of this problem? (check one) A. Increasing her estrogen dosage B. Referral to a rheumatologist C. Low-potency non-fluorinated topical corticosteroids D. Oral prednisone E. Metronidazole gel (MetroGel)

E. Metronidazole gel (MetroGel) - Rosacea - most often in women b/w 30-60 - early: central face redness, telangiectasias - chronic: infiltrate with papules and sometimes sterile pustules - may have facial edema, rhinopyma from gland hypertrophy - Ocular problems occur in half of patients with rosacea e.g.intermittent inflammatory conjunctivitis with or without blepharitis. - Management: Oral metronidazole, doxycycline, or tetracycline also can be used, especially if there are ocular symptoms (topical in mild cases) - Tx does not help w/ flushing: low-dose clonidine or a nonselective β-blocker may be added.

A 35-year-old white female complains of unilateral frontotemporal headaches. During these episodes, which occur every 2-3 weeks, she becomes nauseated, sometimes to the point of vomiting. The headaches are throbbing in character and last for 1-3 hours, often causing her to leave work early. Relief is sometimes obtained with simple analgesics, but more often with sleep or the passage of time. On the basis of this history alone, the most likely diagnosis is (check one) A. sinusitis B. a brain tumor C. muscle tension headache D. cluster headache E. migraine headache

E. Migraine - patient is young and female - unilateral, infrequent, and throbbing - associated with nausea and vomiting; and sleep offers relief. NOT sinusitis - will have fever, facial pain, and a purulent nasal discharge. Not tumor - daily and becomes more frequent and severe with time. - prevalence of brain tumor is far less than that of migraine. Not muscle tension headache - pressure or band-like tightening, often in a circumferential or cap distribution. -also has a pattern of daily persistence Cluster headache - males - very severe, constant, agonizing orbital pain - usually beginning within 2 or 3 hours after falling asleep.

An obese, hypertensive 53-year-old physician suffers a cardiac arrest while making rounds. He is resuscitated after 15 minutes of CPR, but remains comatose. Which one of the following is associated with the lowest likelihood of neurologic recovery in this situation? (check one) A. Duration of CPR >10 minutes B. No pupillary light reflex at 30 minutes C. No corneal reflex at 2 hours D. No motor response to pain at 6 hours E. Myoclonic status epilepticus at 24 hour

E. Myoclonic status epilepticus at 24 hour - the absence of pupillary and corneal reflexes, as well as motor responses to pain, are not reliable predictors before 72 hours - Myoclonic status epilepticus at 24 hours suggests no possibility of a recovery.

Which one of the following is the most appropriate first-line therapy for primary dysmenorrhea? A. Combined monophasic oral contraceptives B. Combined multiphasic oral contraceptives C. Subdermal etonogestrel (Nexplanon) D. Intramuscular medroxyprogesterone (Depo-Provera) E. NSAIDs

E. NSAIDs - 1st line

A 60-year-old male with a long-standing history of hypertension seeks your advice about pain relief from his osteoarthritis. He has tried acetaminophen and topical capsaicin cream without much benefit. He is concerned about media reports of NSAIDs causing heart problems and is unsure which ones would be safest for him to use. Based on current evidence, which one of the following NSAIDs would you recommend as being LEAST likely to be associated with an increased risk of myocardial infarction? (check one) A. Celecoxib (Celebrex) B. Diclofenac (Zorvolex) C. Ibuprofen D. Meloxicam (Mobic) E. Naproxen (Naprosyn)

E. Naproxen (Naprosyn) - does not increase MI NSAIDs cause - increase BP due to their salt and water retention properties causing edema & CHF - trash the kidney

A local dentist contacts you for a prescription for the appropriate antibiotic dosage for one of your patients who has an appointment for dental cleaning to eliminate a significant plaque buildup. The patient is a 55-year-old male who has controlled hypertension and mitral valve prolapse with mitral regurgitation. He is allergic to sulfonamides. Which one of the following would be the most appropriate prophylaxis for this patient? (check one) A. Amoxicillin, 2 g orally 1 hour prior to the procedure B. Amoxicillin, 3 g orally 1 hour prior to the procedure and 1.5 g orally 6 hours after the procedure C. Ceftriaxone (Rocephin), 1 g intramuscularly 1 hour prior to the procedure D. Clindamycin (Cleocin), 600 mg orally 1 hour prior to the procedure E. No antibiotic prophylaxis

E. No antibiotic prophylaxis American Heart Association's 2007 guidelines, prophylaxis to prevent bacterial endocarditis associated with dental, gastrointestinal, or genitourinary procedures is now indicated only for: 1. high-risk patients with prosthetic valves 2. a previous history of endocarditis 3. unrepaired cyanotic congenital heart disease (CHD) 4. CHD repaired with prosthetic material 5. cardiac transplant recipients who develop valvular disease; also increase in adverse event

A 5-year-old white male is brought to your office with a chief complaint of chronic nocturnal limb pain. His mother states that his pain is often severe enough that it awakens him at night and she often gives him ibuprofen to help alleviate his calf pain, but she has never seen him limp or heard him complain of pain during the day. She also has not noticed any grossly swollen joints, fever, rash, or weight change. She is concerned because of a family history of juvenile rheumatoid arthritis in a distant cousin. The physical examination is within normal limits, as are a CBC and an erythrocyte sedimentation rate. Which one of the following would be most appropriate at this point? A. Bilateral plain radiographs of the lower extremities B. Testing for antinuclear antibody C. Testing for rheumatoid factor D. Referral to orthopedic surgery E. No further workup

E. No further workup benign nocturnal limb pains

A large wooden splinter went deep into the forearm of a 24-year-old male while he was working in a horse barn, and he has required local anesthesia and a small incision to remove it completely. After thorough wound cleansing, you inquire about his tetanus status. He is certain that he received all of his primary childhood vaccines and a "tetanus booster" at age 20, but does not know which vaccine he received. Which one of the following is the best choice for this patient regarding tetanus immunization at this time? (check one) A. TT (tetanus toxoid) B. Td (tetanus toxoid with reduced diphtheria) C. Tdap (tetanus toxoid with reduced diphtheria and acellular pertussis) D. TIG (tetanus immune globulin) E. No immunization

E. No immunization - the patient is certain of completing the primary vaccinations and received a tetanus booster within the previous 5 years. - Had the interval been longer than 5 years, then a single dose of Tdap would be appropriate unless his previous booster was Tdap.

A 65-year-old asymptomatic female is found to have extensive sigmoid diverticulosis on screening colonoscopy. She asks whether there are any dietary changes she should make. In addition to increasing fiber intake, which one of the following would you recommend? (check one) A. Limiting intake of dairy products B. Limiting intake of spicy foods C. Limiting intake of wheat flour D. Limiting intake of nuts E. No limitations on other intake

E. No limitations on other intake Patients with diverticulosis should increase dietary fiber intake or take fiber supplements to reduce progression of the diverticular disease. Avoidance of nuts, corn, popcorn, and small seeds has not been shown to prevent complications of diverticular disease

A 25-year-old male who came to your office for a pre-employment physical examination is found to have 2+ protein on a dipstick urine test. You repeat the examination three times within the next month and results are still positive. Results of a 24-hour urine collection show protein excretion of <2 g/day and normal creatinine clearance. As part of his further evaluation you obtain split urine collections with a 16-hour daytime specimen containing an increased concentration of protein, and an 8-hour overnight specimen that is normal. Additional appropriate evaluation for this man's problem at this time includes which one of the following? (check one) A. Serum and urine protein electrophoresis B. Antinuclear antibody C. Serum albumin and lipid levels D. Renal ultrasonography E. No specific additional testing

E. No more testing - Persons younger than 30 years of age who excrete less than 2 g of protein per day and who have a normal creatinine clearance should be tested for orthostatic proteinuria - This benign condition occurs in about 3%-5% of adolescents and young adults. - increased protein excretion in the upright position, but normal protein excretion when the patient is supine. - diagnosed using split urine collections as described in the question (daytime vs night) - a benign condition with normal renal function, no further evaluation is necessary.

A 72-year-old female sees you for preoperative evaluation prior to cataract surgery. Her history and physical examination are unremarkable, and she has no medical problems other than bilateral cataracts. Which one of the following is recommended prior to surgery in this patient? (check one) A. An EKG only B. An EKG and chest radiography C. A CBC only D. A CBC and serum electrolytes E. No testing

E. No testing The American Heart Association recommends against routine preoperative testing in asymptomatic patients undergoing low-risk procedures, since the cardiac risk associated with such procedures is less than 1%.

A 52-year-old healthy male presents with a 2½-week history of diarrhea, consisting of 4-6 watery stools daily. He is afebrile and his examination is normal. You recommend symptomatic care. Two days later the laboratory notifies you that Salmonella is growing in his stool culture. You call the patient and he remains free of fever but with ongoing diarrhea. Which one of the following would you recommend? (check one) A. Azithromycin (Zithromax) B. Ciprofloxacin (Cipro) C. Clindamycin (Cleocin) D. Doxycycline E. No treatment

E. No treatment The recommended management for patients who have non-severe Salmonella infection and are otherwise healthy is no treatment. Patients with high-risk conditions that predispose to bacteremia, and those with severe diarrhea, fever, and systemic toxicity or positive blood cultures should be treated with levofloxacin, 500 mg once daily for 7-10 days (or another fluoroquinolone in an equivalent dosage), or with a slow intravenous infusion of ceftriaxone, 1-2 g once daily for 7-10 days (14 days in patients with immunosuppression).

A 48-year-old female with type 2 diabetes has been hospitalized for 4 days with persistent fever. Her diabetes has been controlled with diet and glyburide (Micronase, DiaBeta). You saw her 2 weeks ago in the office with urinary frequency, urgency, and dysuria. At that time a urinalysis showed 25 WBCs/hpf, and a urine culture subsequently grew Escherichia coli sensitive to all antibiotics. She was placed on trimethoprim/sulfamethoxazole (Bactrim, Septra) empirically, and this was continued after the culture results were reported. She improved over the next week, but then developed flank pain, fever to 39.5°C (103.1°F), and nausea and vomiting. She was hospitalized and intravenous cefazolin (Kefzol) and gentamicin were started while blood and urine cultures were performed. This urine culture also grew E. coli sensitive to the current antibiotics. Her temperature has continued to spike to 39.5°C since admission, without any change in her symptoms. Which one of the following would be most appropriate at this time? (check one) A. Add vancomycin (Vancocin) to the regimen B. Order a radionuclide renal scan C. Order intravenous pyelography D. Order a urine culture for tuberculosis E. Order CT of the abdomen

E. Order CT of the abdomen - Perinephric abscess - collection of pus in the tissue surrounding the kidney, generally in the space enclosed by Gerota's fascia. - Mortality rates 50% have been reported, usually from failure to diagnose the problem in a timely fashion - The difficulty in making the diagnosis can be attributed to the variable constellation of symptoms and the sometimes indolent course of this disease. - The diagnosis should be considered when a patient has fever and persistence of flank pain. - Most perinephric infections occur as an extension of an ascending urinary tract infection, commonly in association with renal calculi or urinary tract obstruction. - Patients with anatomic urinary tract abnormalities or diabetes mellitus have an increased risk. - persistence of fever for more than 4 days after initiation of antibiotic therapy. - CT. This can detect perirenal fluid, enlargement of the psoas muscle (both are highly suggestive of the diagnosis), and perirenal gas (which is diagnostic). - Drainage, either percutaneously or surgically, along with appropriate antibiotic coverage reduces both morbidity and mortality from this condition.

A 59-year-old male who is morbidly obese suffers a cardiac arrest. Intravenous access cannot be obtained. Which one of the following is true regarding intraosseous drug administration in this patient? (check one) A. The patient's age and size are a contraindication to intraosseous administration B. The time needed to establish intraosseous access is too great C. Many drugs cannot be administered intraosseously D. Endotracheal administration is preferred E. There are no contraindications to intraosseous administration in this patient

E. There are no contraindications to intraosseous administration in this patient

An 86-year-old mildly demented male nursing-home resident rarely leaves the facility. He has frequent fecal incontinence that is disturbing to both him and his family. He has diet-controlled diabetes mellitus and hypertension, and a history of transurethral resection of the prostate (TURP) for benign prostatic hypertrophy. An examination is remarkable only for an empty rectum and no focal neurologic findings. Which one of the following is the most likely cause of this patient's fecal incontinence? (check one) A. Decreased rectal sensation secondary to diabetes mellitus B. Decreased rectal storage capacity C. Internal sphincter weakness D. Puborectalis weakness E. Overflow

E. Overflow incontinence is common in the institutionalized elderly, and is often due to constipating medications. Not Reduced storage capacity - is usually seen with inflammatory bowel disease Not diabetes mellitus - does not cause decreased rectal sensation Not puborectalis and internal sphincter weakness - uncommon in males, as they usually result from vaginal delivery.

A 44-year-old female is distressed because of incontinence. She reports frequent episodes of an immediate need to urinate, which cannot always be deferred. She admits to urinating more than 10 times a day, but denies any urine leakage with coughing, laughing, or straining. Which one of the following is the most appropriate initial treatment for this patient? (check one) A. Solifenacin (Vesicare) B. Oxybutynin (Ditropan XL) C. Tamsulosin (Flomax) D. Phenazopyridine (Pyridium) E. Pelvic floor muscle training and bladder training

E. Pelvic floor muscle and bladder training - effective in urge incontinence or overactive bladder, as well as in stress and mixed incontinence - In motivated patients, training may be more effective than medications such as oxybutynin and newer muscarinic receptor antagonists such as solifenacin. - Tamsulosin is used in benign prostatic hypertrophy and phenazopyridine is a urinary tract anesthetic that has not been recommended for treating overactive bladder.

A 30-year-old male complains of the gradual onset of anterior right knee pain on climbing the stairs. On examination there is no effusion, but there is tenderness over the medial retinaculum. There is good ligament strength, and range of motion is normal. When the knee is extended from 90° flexion to full extension, the patella deviates laterally. Which one of the following would be the best initial treatment for this condition? (check one) A. Bracing B. Taping C. NSAIDs D. Arthroscopic surgery E. Physical therapy

E. Physical Therapy - patellofemoral stress syndrome - runner's knee or anterior knee pain. - The patellofemoral joint comprises the patella and femoral trochlea. - The best initial treatment is physical therapy. Not Bracing, taping, and medications - not useful Not Arthroscopic surgery - not indicated.

A 65-year-old male present to the urgent care center with a foot ulcer. His past medical history is significant for hypertension, COPD, and diabetes mellitus. He has been hospitalized several times in the past year for COPD exacerbations and a hip fracture. He does not have any other current problems. On examination he has a temperature of 37.3°C (99.1°F), a pulse rate of 105 beats/min, a respiratory rate of 16/min, and a blood pressure of 142/83 mm Hg. His examination is unremarkable except for a 2-cm ulcer on the ball of his left foot that has 3 cm of surrounding erythema and some purulent drainage. His CBC is normal except for a WBC count of 14,300/mm3 (N 4300-10,800). Which one of the following would be the most appropriate choice for initial treatment? (check one) A. Amoxicillin/clavulanate (Augmentin) B. Linezolid (Zyvox) C. Ciprofloxacin (Cipro) D. Ceftriaxone (Rocephin) and levofloxacin (Levaquin) E. Piperacillin/tazobactam (Zosyn) and vancomycin (Vancocin)

E. Piperacillin/tazobactam (Zosyn) and vancomycin (Vancocin); severe diabetic foot ulcer - with a systemic inflammatory response. This is an indication for intravenous antibiotics. Piperacillin/tazobactam and vancomycin together will cover the most common pathogens in diabetic foot ulcers, as well as MRSA, which is present in 10%-32% of diabetic foot ulcers. This patient has recently been hospitalized and would thus be at high risk for a MRSA infection. Moderate to severe diabetic foot ulcers are often polymicrobial and can include gram-positive cocci, gram-negative bacilli, and anaerobic pathogens.

A 27-year-old female radiology technician developed an area of redness over the left interscapular region while visiting a friend in Paris last week. The rash has progressed to include the area shown in Figure 10 and the patient says it itches. She recalls feeling somewhat tired and achy once she arrived in Paris but attributed this to jet lag. She denies any other systemic symptoms. Your examination reveals no significant findings except for the rash. Of the following, which one is most consistent with this patient's history and examination? (check one) A. Guttate psoriasis B. Tinea versicolor C. Radiation dermatitis D. Cutaneous T-cell lymphoma E. Pityriasis rosea

E. Pityriasis rosea - prodrome - development of an ovoid salmon-colored, slightly raised herald patch, most commonly seen on the trunk. - followed by an outbreak of multiple smaller, similar lesions that trend along Langer's lines Guttate psoriasis - similiar to pityriasis rosea i.e. acute after strep infection (looks like prodrome) - no herald patch, thicker erythematous lesions Tinea versicolor - upper trunk, lightly red, scaling rash - onset is more gradual Cutaneous T-cell lymphoma - usu. nonspecific dermatitis, most commonly in men over the age of 50.

Pretibial myxedema is a cutaneous manifestation of? (check one) A. Subclinical diabetes mellitus B. Collagen vascular disease C. Hyperlipidemia, type III D. Ischemia E. Graves disease

E. Pretibial myxedema is a complication of Graves disease, - hypo-or hyperthyroidism - dermopathy that most often occurs in the lower legs and results from increased deposition of mucin due to the endocrine abnormalit

A 12-month-old male is brought to your office by his mother because of concerns about his eating. She states that he throws tantrums while sitting in his high chair, dumps food on the floor, and refuses to eat. She has resorted to feeding him cookies, crackers, and juice, which are "all he will eat." A complete physical examination, including a growth chart of weight, length, and head circumference, is normal. Which one of the following would be the most appropriate recommendation? (check one) A. Use disciplinary measures to force the child to eat a healthy breakfast, lunch, and dinner B. Leave the child in the high chair until he has eaten all of the healthy food provided C. Play feeding games to encourage consumption of healthy meals or snacks D. Skip the next meal if the child refuses to eat E. Provide healthy foods for all meals and snacks, and end the meal if the child refuses to eat

E. Provide healthy foods for all meals and snacks, and end the meal if the child refuses to eat - 3%-10% of infants and toddlers refuse to eat, according to their caregivers; this problem tends to persist without intervention. - parents should determine schedule, children determine amount; nothing b/w meals - Food should not be offered as a reward or present. - Parents can be reassured that a normal child will learn to eat enough to prevent starvation

A 21-year old female comes to her family physician's office with an unintended pregnancy and states that she wishes to have a medical abortion. Elective abortion is against the physician's personally held moral principles. According to the American Academy of Family Physicians, which one of the following would be the most appropriate course of action for the physician in this situation? (check one) A. Explaining the rationale for morally opposing medical abortions B. Providing no further assistance at this visit C. Offering to match the patient with prospective adoptive parents D. Advising the patient that it would be safer for her to continue the pregnancy E. Providing resources that explain how to access a safe and legal medical abortion

E. Providing resources that explain how to access a safe and legal medical abortion

A 63-year-old female with corticosteroid-dependent COPD has developed pneumonia. Which one of the following pathogens should the antibiotic regimen cover in this patient that would be unlikely in someone with pneumonia and otherwise healthy lungs? (check one) A. Streptococcus pneumoniae B. Mycoplasma pneumoniae C. Haemophilus influenzae D. Staphylococcus aureus E. Pseudomonas aeruginosa

E. Pseudomonas aeruginosa All of the pathogens listed can cause pneumonia in any patient. However, in patients with chronic lung disease who are taking corticosteroids, pseudomonas is more common than in those with otherwise healthy lungs. The antibiotics chosen empirically should cover this pathogen.

A 19-year-old female high-school student is brought to your office by a friend who is concerned about the patient having cut her wrists. The patient denies that she was trying to kill herself, and states that she did this because she "just got so angry" at her boyfriend when she caught him sending a text message to another woman. She denies having a depressed mood or anhedonia, and blames her fluctuating mood on everyone who "keeps abandoning her," making her feel like she's "nothing." She admits that she has difficulty controlling her anger. Her sleep quality and pattern appear normal, as does her appetite. She denies hallucinations or delusions. The wounds on her wrists appear superficial and there is evidence of previous cutting behavior on her forearms. Her vital signs are stable. Which one of the following would be most beneficial for this patient? (check one) A. Clonazepam (Klonopin) B. Fluoxetine (Prozac) C. Quetiapine (Seroquel) D. Inpatient psychiatric admission E. Psychotherapy

E. Psychotherapy - Borderline Personality Inpatient hospitalization may be an appropriate treatment option if the person is experiencing extreme difficulties in living and daily functioning, and pharmacotherapy may offer a mild degree of symptom relief.

Ultrasonography shows a complete placenta previa in a 23-year-old primigravida at 20 weeks gestation. She has not experienced any vaginal bleeding. Which one of the following would be the most appropriate management for this patient? (check one) A. Schedule a cesarean section at 38 weeks gestation B. Perform a digital examination to assess for cervical dilation C. Administer corticosteroids to promote fetal lung maturity D. Order MRI to rule out placenta accreta E. Repeat the ultrasonography at 28 weeks gestation

E. Repeat the ultrasonography at 28 weeks gestation Because many placenta previas resolve close to term, a decision regarding mode of delivery should not be made until after ultrasonography is performed at 36 weeks gestation.

In early February, you receive a call from your office nurse. Her 5-month-old daughter has been ill for several days. What started as a mild upper respiratory infection has progressed and she now has profuse rhinorrhea, a temperature of 100.2° F (37.9° C), and audible wheezing. In spite of an almost nonstop cough, she does not appear acutely ill. The organism responsible for this child's illness is most likely to be: (check one) A. Group B Streptococcus B. Mycoplasma pneumoniae C. Bordetella pertussis D. Parainfluenza virus 3 E. Respiratory syncytial virus

E. Respiratory syncytial virus - most common cause of pneumonia in children age 4 months to 4 years is respiratory syncytial virus. - peak incidence of respiratory syncytial virus is between 2 and 7 months of age - Wheezing and profuse rhinorrhea are characteristic and the disease typically occurs in mid-winter or early spring epidemics. Not Parainfluenza 3 - affects older infants and is not common in winter. Not Mycoplasma - affect older children and children with bacterial illnesses; those infected with this organism generally appear more acutely ill.

A 67-year-old female comes to your office because she noticed flashing lights in her left eye 2 hours ago, and since then has had decreased vision in the lateral aspect of that eye. On examination she has a blind spot in the lateral visual field of her left eye. Her fundus is difficult to examine because of an early cataract. Which one of the following is the most likely diagnosis? (check one) A. Posterior vitreous detachment B. Vitreous hemorrhage C. Macular degeneration D. Ocular migraine E. Retinal detachment

E. Retinal detachment - flashes of light and a visual field defect Vitreous detachment - asymptomatic, and it does not cause sudden visual field defects in the absence of a retinal detachment Vitreous hemorrhage - more blurring of vision in the entire field of vision Ocular migraine - binocular symptoms. )

A pet reptile is most likely to transmit which one of the following to human contacts? (check one) A. Hantavirus B. Psittacosis (Chlamydophila psittaci) C. Plague (Yersinia pestis) D. Pasteurella multocida E. Salmonella

E. Salmonella Pasteurella multocida: dog or cat bites. Yersinia pestis (plague): rodents or their fleas. Hantavirus: rodents, and psittacosis: bird species.

A 70-year-old male with hypertension, benign prostatic hyperplasia, depression, and well-controlled diabetes mellitus sees you because of increasing fatigue. His medical history also includes stent placement for coronary artery disease. A physical examination is unremarkable except for decreased peripheral pulses. A CBC, basic metabolic profile, hemoglobin A1c level, free T4 level, and TSH level are all normal, except for a serum sodium level of 125 mEq/L (N 135-145). His serum osmolality is 268 mOsm/kg (N 275-290). His urine sodium level is 50 mEq/L (N <20) and his urine osmolality is 300 mOsm/kg. Which one of the patient's medications is most likely to cause this problem? (check one) A. Losartan (Cozaar) B. Tamsulosin (Flomax) C. Metformin (Glucophage) D. Atorvastatin (Lipitor) E. Sertraline (Zoloft)

E. Sertraline (Zoloft) euvolemic hyponatremia: decreased serum osmolality, and elevated urine osmolality so syndrome of inappropriate secretion of antidiuretic hormone SIADH is likely Other causes to rule out include thyroid disorders, adrenal insufficiency, and diuretic use. Renal function has to be normal as well. Common drugs that cause SIADH include SSRIs (particularly in patients over 65),chlorpropamide, barbiturates, carbamazepine, opioids, tolbutamide, vincristine, diuretics, and NSAIDs. Treatment of the problem consists of discontinuing the offending drug. Temporary fluid restriction may also be required.

Which one of the following is a physiologic difference between males and females that can affect the pharmacokinetics of medications with a narrow therapeutic index? (check one) A. A consistently higher glomerular filtration rate in women B. The typically higher BMI in women C. Smaller fat stores in women D. Greater gastric acid secretion in women E. Slower gastrointestinal transit times in women

E. Slower GI transit in women - lower the absorption of medications e.g. metoprolol, theophylline, and verapamil. - women should wait longer after eating before taking medications that should be administered on an empty stomach e.g. ampicillin, captopril, levothyroxine, loratadine, and tetracycline. - Women secrete less gastric acid than men, need an acidic beverage to aid in absorption of medications that require an acidic environment e.g. ketoconazole. - Women usually have lower BMIs than men, and may need smaller loading or bolus dosages of medications to avoid unnecessary adverse reactions. - Women typically have higher fat stores than men, so lipophilic drugs e.g. benzodiazepines and neuromuscular blockers have a longer duration of action - Women also have lower GFR than men, resulting in slower clearance of medications that are eliminated renally, e.g. digoxin and methotrexate.

A 66-year-old male who was hospitalized 2 months ago for an episode of heart failure sees you for follow-up. He complains of pain in his chest and on examination you note tenderness and a slight fullness deep to his nipple bilaterally. Which one of the following drugs on his medication reconciliation list is most likely to cause this type of discomfort? (check one) A. Digoxin (Lanoxin) B. Enalapril (Vasotec) C. Eplerenone (Inspra) D. Hydralazine E. Spironolactone (Aldactone)

E. Spironolactone (Aldactone) Spironolactone, an aldosterone antagonist, can bind to androgen and progesterone receptors, in addition to the mineralocorticoid receptors, resulting in breast tenderness and gynecomastia. Eplerenone, another aldosterone antagonist, has greater specificity for the mineralocorticoid receptors and is therefore less likely to cause breast tenderness and gynecomastia than spironolactone.

Secondary causes of osteoporosis in males include which one of the following? (check one) A. Weekly consumption of 3-6 alcoholic drinks B. Male hormone supplementation C. Vitamin D excess D. Obesity E. Corticosteroid use

E. Steroid use; also - excessive Etoh - hypohonadism - vitamin D deficiency - low BMI

An 11-year-old female has been diagnosed with "functional abdominal pain" by a pediatric gastroenterologist. Her mother brings her to see you because of concerns that another diagnosis may have been overlooked despite a very thorough and completely normal evaluation for organic causes. Which one of the following would you recommend? (check one) A. A trial of inpatient hospital admission B. Increased testing and levels of referral until a true diagnosis is reached C. Removing the child from school and activities whenever symptoms occur D. Medications to eradicate symptoms E. Stress reduction and participation in usual activities as much as possible

E. Stress reduction and participation in usual activities as much as possible - common cause of recurrent abdominal pain in children 4-16 years of age. - 30%-50% of children with functional abdominal pain will have resolution of their symptoms within 2 weeks of diagnosis

A 55-year-old male presents for an evaluation of heel pain. He has a relatively sedentary office job, but exercises daily by jogging 3 miles. He has pain in the right heel at the medial aspect of the calcaneus and is tender on examination. The pain is worse with the first few steps of the morning. Which one of the following would be the most appropriate initial treatment for this patient? (check one) A. Corticosteroid injection B. Extracorporeal shockwave therapy C. Surgical referral for bone spur removal D. Non-weight bearing for 1 month E. Stretching exercises for the Achilles tendon

E. Stretching achilles tendon - Plantar fasciitis is an overuse injury due to microtrauma of the plantar fascia where it attaches at the medial calcaneal tubercle - The patient experiences heel or arch pain, which often is worse upon arising and taking the first few steps of the morning - Examination reveals tenderness at the site and pain with dorsiflexion of the toes. - Treatment: relative rest, ice, NSAIDs, and shoe inserts for arch support, as well as heel cord and plantar fascia stretching. Not Stress fractures - cause pain at rest that intensifies with weight bearing Not steroid injection - only if all other fails

A 22-year-old male with no previous history of shoulder problems is injured in a fall. He has immediate pain and is unable to abduct his arm. You examine him and order an MRI, which reveals an acute tear of the rotator cuff. Which one of the following is the best initial treatment for this injury? (check one) A. Observation without treatment for 1 month B. Immobilization for 1 month C. Physical therapy for 1 month D. Corticosteroid injection E. Surgical repair

E. Surgical repair - acute rupture of any major tendon should be repaired as soon as possible - Acute tears of the rotator cuff should be repaired within 6 weeks of the injury if possible (SOR C).

Patients being treated with amiodarone (Cordarone) should be monitored periodically with serum A. cortisol B. creatine phosphokinase C. creatine D. LDH E. TSH

E. TSH Patients on amiodarone can develop either hyperthyroidism or hypothyroidism. It is recommended that a patient on amiodarone have baseline thyroid function tests (free T4, TSH) with follow-up testing every 6 months to monitor for these conditions. Hyperadrenalism and hypoadrenalism are not associated with amiodarone treatment.

Pay-for-performance (P4P) programs provide financial incentives for meeting predetermined quality targets. Contracts with major payors often include these programs. When considering P4P programs in such contracts, physicians should negotiate for which one of the following? (check one) A. Guidelines developed by academic medicine researchers B. Guidelines based on consensus opinions C. Mandatory physician participation D. Reporting of negative performance results to licensure boards E. Taking patient compliance into account when performing the evaluation

E. Taking patient compliance into account when performing the evaluation The AAFP has seven main principles in its support for pay-for-performance programs: (1) the focus should be on improved quality of care; (2) physician-patient relationships should be supported; (3) evidence-based clinical guidelines should be utilized; (4) practicing physicians should be involved with the program design; (5) reliable, accurate, and scientifically valid data should be used; (6) physicians should be provided with positive incentives; and (7) physician participation should be voluntary. Ensuring that patient adherence is included helps prevent conflicts between patients and their physicians. - A pay-for-performance program should not result in a reduction of fees paid to the physician as a result of implementing a program. - Negative results should not penalize the physician with regard to health plan credentialing, verification, or licensure.

Which one of the following is most accurate regarding somatization disorder? (check one) A. Onset before age 40 is atypical B. It is a form of malingering C. Symptoms tend to resolve spontaneously within weeks of onset D. Symptoms are limited to one organ system or bodily function E. The incidence is increased among female first degree relatives of patients with the disorder

E. The incidence is increased among female first degree relatives of patients with the disorder - chronic presence of several unexplained symptoms beginning before the age of 30 years. - must include a minimum of two symptoms relating to the gastrointestinal system, one neurologic complaint, one sexual complaint, and four pain complaints - more common in women than in men - the incidence is increased as much as tenfold in female first degree relatives of affected patients.

Many of the changes that occur as part of aging affect pharmacokinetics. Which one of the following is INCREASED in geriatric patients? (check one) A. Drug absorption B. The glomerular filtration rate C. Lean body mass D. The volume of distribution of water-soluble compounds such as digoxin E. The percentage of body fat

E. The percentage of body fat - a relative decrease in lean body mass - increased distribution of fat-soluble drugs such as diazepam. - increases the elimination half-life of such medications. - volume of distribution of water-soluble compounds such as digoxin is decreased i.e. a smaller dose is required to reach a given target plasma concentration. - predictable reduction in glomerular filtration rate and tubular secretion - absorption of drugs changes little

Which one of the following is true concerning the use of short-acting inhaled β-agonists for asthma? (check one) A. They should be given before any inhaled corticosteroid to facilitate lung delivery B. They are ineffective in patients taking β-blockers C. They are less effective than oral β-agonists D. They are less effective than anticholinergic bronchodilators when given with inhaled corticosteroids E. Their effects begin within 5 minutes and last 4-6 hours

E. Their effects begin within 5 minutes and last 4-6 hours

A 17-year-old male high school football running back is hit on the lower leg by an opposing player's helmet when the other player dives for a fumble. The running back presents to the emergency department after the game with significant swelling and bruising of the lower leg. Symptoms include exceptionally severe pain that is worse with stretching the calf muscles. There is no weakness of the extremity and sensation is intact. You examine the leg and can palpate pulses. Plain radiographs do not show a fracture. Which one of the following should be ordered next? (check one) A. Noninvasive arterial ultrasonography of the leg B. Noninvasive venous ultrasonography of the leg C. CT of the calf region D. MRI of the calf region E. Tissue pressure studies

E. Tissue pressure studies - acute compartment syndrome - can be from minor to major injury - e.g. fractures, a badly bruised muscle, crush injuries, constricting bandages, and bites with swelling

A 60-year-old female receiving home hospice care was taking oral morphine, 15 mg every 2 hours, to control pain. When this was no longer effective, she was transferred to an inpatient facility for pain control. She required 105 mg of morphine in a 24-hour period, so she was started on intravenous morphine, 2 mg/hr with a bolus of 2 mg, and was well controlled for 5 days. However, her pain has worsened over the past 2 days. Which one of the following is the most likely cause of this patient's increased pain? (check one) A. An inadequate initial morphine dose B. Addiction to morphine C. Pseudoaddiction to morphine D. Physical dependence on morphine

E. Tolerance to morphine The intravenous dose should be a third of the oral dose, so the starting intravenous dose was adequate.

A mother calls for advice regarding her 2-year-old son. She found an open container of immediate-release diltiazem (Cardizem) on the floor, with some spilled and partially chewed tablets, and estimates that her son opened the container about 90 minutes ago. He does not appear to be in any distress. Which one of the following would you advise her to do? (check one) A. Administer syrup of ipecac at home and observe B. Transport the child to the emergency department for gastric lavage C. Transport the child to the emergency department for administration of activated charcoal D. Transport the child to the emergency department for administration of activated charcoal and a cathartic E. Transport the child to the hospital for admission to the pediatric intensive-care unit for observation

E. Transport the child to the hospital for admission to the pediatric intensive-care unit for observation The use of gastric emptying, cathartics, or adsorptive agents is unlikely to be helpful and should be considered only in patients presenting within 1 hour of ingestion, if then.

A 52-year-old female presents to the emergency department with a complaint of chest pain. The symptoms began 2 hours ago while she was shopping. She describes the pain as a tightness on the left side of her chest that radiates to her left shoulder. She has some shortness of breath with the pain, but no nausea or diaphoresis. Her past medical history is significant for panic disorder.Her vital signs and a physical examination are within normal limits.Which one of the following would be the most appropriate next step in the management of this patient? (check one) A. Admit to a monitored bed for further evaluation B. Obtain a CBC, a blood chemistry profile, liver function tests, and an EKG C. Administer a short-acting benzodiazepine and observe for 60 minutes D. Consult with a cardiologist for immediate heart catheterization E. Obtain a troponin I measurement and an EKG

E. Troponin & EKG - acute coronary syndrome, - chest pain with activity that radiates to the shoulder - diagnosis should be established prior to heart catheterization.

A 45-year-old male sees you for follow-up after a pre-employment physical examination reveals blood in his urine. He brings a copy of a urinalysis report that shows 3-5 RBCs/hpf. He has not seen any gross blood himself. He is asymptomatic, is on no medications, and does not smoke. You perform a physical examination, with normal findings. A repeat urinalysis confirms the presence of red blood cells but is otherwise normal. Which one of the following would be most appropriate at this point? (check one) A. Observation and reassurance B. A repeat urinalysis in 6 months C. Urine cytology only D. Ultrasonography of the kidneys and urine cytology only E. Ultrasonography of the kidneys, urine cytology, and cystoscopy

E. Ultrasound of kidneys, urine cytology, cystoscopy - significant microscopic hematuria = ≥3 RBCs/hpf. - may be associated with urologic malignancy in up to 10% of adults. - to exam upper urinary tract: no clear guidelines so intravenous urography, ultrasonography, or CT can be considered. - to exam lower urinary tract: urine cytology and cystoscopy - The AUA recommends that patients with microscopic hematuria have radiographic assessment of the upper urinary tract, followed by urine cytology studies. - The AUA also recommends that all patients older than 40 and those who are younger but have risk factors for bladder cancer undergo cystoscopy to complete the evaluation. - Cystoscopy is the only reliable method of detecting transitional cell carcinoma of the bladder and urethra.

A previously healthy 67-year-old male sees you for a routine health maintenance visit. During the physical examination you discover a harsh systolic murmur that is loudest over the second right intercostal space and radiates to the carotid arteries. The patient denies any symptoms of dyspnea, angina, syncope, or decreased exertional tolerance. An echocardiogram shows severe aortic stenosis, with an aortic valve area of <1 cm 2, a mean gradiant >40 mm Hg, and an ejection fraction of 60%. Which one of the following would be most appropriate at this point? (check one) A. Coronary angiography B. Exercise stress testing C. Treatment with prazosin (Minipress) D. Referral for aortic valve replacement E. Watchful waiting

E. Watchful waiting - asymptomatic aortic stenosis, including those with severe disease (SOR B). - surgical risk of aortic valve replacement outweighs the approximately 1% annual risk of sudden death in asymptomatic patients with aortic stenosis. - Peripheral α-blockers, such as prazosin, should be avoided because of the risk of hypotension or syncope. - Coronary angiography should be reserved for symptomatic patients who do not have evidence of severe aortic stenosis on echos, or for preoperative evaluation prior to aortic valve replacement. - Exercise stress testing is not safe with severe aortic stenosis because of the risk of death during the test.

A 52-year-old female with morbid obesity is incidentally noted to have mildly elevated AST (SGOT) levels. She does not consume alcohol and denies using recreational drugs. A workup for chronic viral hepatitis and hemochromatosis is negative. Which one of the following is most likely to improve her hepatic condition? (check one) A. Pentoxifylline B. Simvastatin (Zocor) C. L-carnitine D. Vitamin E E. Weight loss

E. Weight loss A healthy diet, weight loss, and exercise are first-line therapeutic measures to reduce insulin resistance in patients with nonalcoholic fatty liver disease.

A 67-year-old male sees you for knee pain from osteoarthritis. It has not responded to his usual treatment, and you treat him with an intra-articular corticosteroid injection. It is mid-November, and he tells you that he has not received the influenza vaccine this year. He has also never received pneumococcal vaccine. He has a history of allergic rhinitis, treated with intranasal corticosteroids. Which one of the following is true regarding pneumococcal vaccine and influenza vaccine for this patient? (check one) A. The immunizations should be administered at least 4 weeks apart B. Administration of both immunizations should be delayed 4 weeks because of immunosuppression C. Administration of influenza vaccine should be delayed for 4 weeks because it is a live attenuated vaccine D. Administration of pneumococcal vaccine should be delayed for 4 weeks because it is contraindicated with simultaneous intra-articular corticosteroids E. It is acceptable to administer both immunizations at this

E. acceptable to administer both vaccine - Low-dose topical, oral, nasal, and intra-articular corticosteroids are not immunosuppressive and do not contraindicate administration of any vaccine. - Influenza vaccine and pneumococcal vaccine can be given together. - live vaccine is also ok

A 25-year-old white female with heavy menstrual periods is noted to have a hemoglobin level of 9.8 g/dL (N 12.0-16.0). The red cell distribution width is 16.0% (N 11.5-14.5) and the mean corpuscular volume is 75 µm3 (N 78-102). The appropriate treatment for this condition can be enhanced by the use of: (check one) A. Antacids B. Soy milk C. Iced tea D. Bran E. Ascorbic acid

E. ascobic acid - iron deficiency anemia. - There are several substances that decrease the absorption of iron, including antacids, soy protein, calcium, tannin (which is in tea), and phytate (which is found in bran). - Since an acidic environment increases iron absorption, ascorbic acid (vitamin C) can enhance absorption of an iron supplement.

Which one of the following is the most accurate imaging study for assessing early osteomyelitis? (check one) A. Plain radiography B. Ultrasonography C. CT D. MRI E. A bone scan

E. bone scan Osteomyelitis - diabetic foot infections and is present in up to 20% of mild to moderate infections and in 50%-60% of severe infections. Plain radiography - may show bony destruction but not sensitive - may take weeks for these infections to become apparent on plain radiographs. Bone scans - not specific

A 65-year-old male comes to your office with symptoms consistent with intermittent claudication in both lower extremities. These symptoms are making it difficult for him to walk any significant distance and to manage his daily activities. He has smoked one pack of cigarettes per day for the past 40 years and has moderate hypertension, an elevated LDL-cholesterol level, and a low HDL-cholesterol level. On examination you note that the skin on the patient's lower legs is cool and shiny, with sparse hair. Distal pulses are not palpable and capillary refill is prolonged. His ankle-brachial index is 0.85 (N 1.0-1.4). His cardiac examination is normal, with no evidence of heart failure. Which one of the following pharmacologic options for improving this patient's claudication symptoms is supported by the best available evidence? (check one) A. Aspirin B. Warfarin (Coumadin) C. Clopidogrel (Plavix) D. Pentoxifylline E. Cilostazol (Pletal)

E. cilostazol - initial treatment for peripheral arterial disease (PAD) that limits the lifestyle of patients without heart failure (SOR A). - increase walking distance and improve health-related quality of life (level of evidence 2). - neither aspirin nor clopidogrel improves claudication symptoms, antiplatelet therapy is recommended to reduce the risk of myocardial infarction, stroke, or vascular death in patients with symptomatic PAD. Not Warfarin - does not improve cardiovascular outcomes in patients with PAD, increase bleeding without Not pentoxifylline - not studied

Actinic keratosis is a precursor lesion to: (check one) A. keratoacanthoma B. nodular melanoma C. superficial spreading melanoma D. basal cell carcinoma E. cutaneous squamous cell carcinoma

E. cutaneous squamous cell carcinoma Thicker lesions, cutaneous horns, and lesions that show ulceration have a higher malignant potential

A 28-year-old white female consults you with a complaint of irregular heavy menstrual periods. A general physical examination, pelvic examination, and Papanicolaou test are normal and a pregnancy test is negative. A CBC and chemistry profile are also normal. The next step in her workup should be: (check one) A. endometrial aspiration B. dilatation and curettage C. LH and FSH assays D. administration of estrogen E. cyclic administration of progesterone for 3 months

E. cyclic administration of progesterone for 3 months Once pregnancy, hematologic disease, and renal impairment are excluded, administration of intramuscular or oral progesterone will usually produce definitive flow and control the bleeding. No further evaluation should be necessary unless the bleeding recurs.

A 24-year-old female presents to the emergency department because she thinks she is having an allergic reaction to her medication for depression. About 3 hours after taking her first dose of citalopram (Celexa) she noted extreme anxiety, agitation, palpitations, and a dry mouth. On examination she has a blood pressure of 180/110 mm Hg, a pulse rate of 120 beats/min, a respiratory rate of 24/min, and a temperature of 37.2°C (99.0°F). Her pupils are dilated and she has slow, continuous horizontal eye movements. Marked hyperreflexia is noted in the lower extremities. In addition to supportive care, the patient should be given intravenous (check one) A. propranolol B. diphenhydramine C. haloperidol lactate (Haldol Lactate) D. flumazenil (Romazicon) E. diazepam

E. diazepam; if ineffectual, cyproheptadine Drug of choice is an IV benzodiazepine such as lorazepam or diazepam. If treatment for tachycardia or hypertension is needed, propranolol should not be used due to its longer activity. Haloperidol should be avoided, as it may actually increase anticholinergic activity. Flumazenil is rarely used, although it has been used for tricyclic antidepressant overdosage, and it carries a significant risk of inducing seizures.

An 11-year-old male with dark brown urine and hand and foot edema, which one of the following would be most suggestive of glomerulonephritis? (check one) A. WBC casts in the urine B. RBC casts in the urine C. Eosinophils in the urine D. Positive serum antinuclear antibody levels E. Elevated C3 and C4 complement levels

E. elevated C3 and C4; Acute glomerulonephritis (AGN) in children manifests as brown or cola-colored urine - which may be painless or associated with mild flank or abdominal pain. T - IgA nephropathy (which may directly follow an acute upper respiratory tract infection) and acute poststreptococcal glomerulonephritis following a streptococcal throat or skin infection (usually 7-21 days later). - RBC casts are the classic finding on urinalysis in a patient with AGN - WBC casts are seen in acute pyelonephritis, often manifested by high fever, and costovertebral angle or flank pain and tenderness. Patients may also appear septic. Positive serum antinuclear antibodies are associated with lupus nephritis. Urine eosinophils are seen in drug-induced tubulointerstitial nephritis. Serum complement levels are reduced, not elevated, in various forms of acute glomerulopathies, including poststreptococcal AGN.

A 40-year-old female sees you for a health maintenance visit. She has no complaints and other than being overweight she has an unremarkable examination. Laboratory results are also unremarkable except for her lipid profile. She has a total cholesterol level of 251 mg/dL, an HDL-cholesterol level of 31 mg/dL, and a triglyceride level of 1250 mg/dL. The LDL-cholesterol level could not be calculated and measured 145 mg/dL. In addition to lifestyle changes, this patient would most likely benefit from (check one) A. niacin B. omega-3 fatty acid supplementation C. atorvastatin (Lipitor) D. ezetimibe (Zetia) E. fenofibrate (Tricor)

E. fenofibrate - *hypertriglyceridemia* 150-199 (mild): 200-999 (moderate): - statins may be helpful - but not used alone 1000-1999 (severe) >2000 (very severe): - *pancreatitis*; need *fibrates* Not omega-3 - decrease triglycerides and VLDL - increase LDL-cholesterol levels - does not decrease total mortality or cardiovascular events, and therefore is not recommended. Not niacin - seem to have the advantage of raising HDL cholesterol and lowering LDL cholesterol - has increases in adverse events.

A 69-year-old male sees you for a routine evaluation. He has been in good health and takes no medication other than tamsulosin (Flomax) for symptoms of benign prostatic hyperplasia. He has never smoked. His blood pressure is 121/78 mm Hg, pulse rate 72 beats/min, and respiratory rate 18/min. His general physical examination is unremarkable, including cardiac and abdominal examinations. A digital rectal examination reveals mild enlargement of the prostate, without nodules. According to the U.S. Preventive Services Task Force, this patient should be screened for (check one) A. elder abuse B. aortic aneurysm C. multifactorial fall risk D. dementia E. hepatitis C

E. hepatitis C one-time screening for hepatitis C for individuals born between the years 1945 and 1965 (USPSTF B recommendation). i.e. 52-72 y/o

The most common cause of acute interstitial nephritis is: (check one) A. hypertension B. pyelonephritis C. collagen vascular disease D. dehydration E. hypersensitivity to medications

E. hypersensitivity to medications - 85% of cases - other cases are due to mechanisms such as an immunologic response to infection or an idiopathic immune syndrome - Hypertension and dehydration do not cause interstitial nephritis.

A 90-year-old female nursing-home patient has a 1.5×2.0-cm lesion on her face (shown in Figure 8). She states that the "spot" has been present for years and that it doesn't bother her. Closer examination reveals a flat maculopapular lesion with varying colors and an irregular border. Figure 8 Which one of the following is the most likely diagnosis? (check one) A. Actinic keratosis B. Metastatic breast carcinoma C. Seborrheic keratosis D. Lentigo maligna melanoma E. Basal cell carcinoma

E. malignant melanoma aka lentigo maligna melanoma. - appear during the seventh or eighth decade of life, - most often located on the face

A 42-year-old male seeks your advice regarding smoking cessation. You recommend a smoking cessation class, as well as varenicline (Chantix). You caution him that the most common side effect is: (check one) A. dermatitis B. diarrhea C. edema D. hirsutism E. nausea

E. nausea The most common adverse event attributed to varenicline at a dosage of 1 mg twice a day is nausea, occurring in approximately 30%-50% of patients. Taking the drug with food lessens the nausea.

A 50-year-old female presents to your office for evaluation of a 2-month history of dyspnea on exertion and a nonproductive cough. She has a previous history of hypertension, overactive bladder, gastroesophageal reflux disease, and recurrent urinary tract infections. Vital signs are unremarkable and she has an oxygen saturation of 94%. She has inspiratory crackles in the posterior lung bases that do not clear with coughing. Office spirometry shows that the FVC is only 80% of normal, but the FEV1/FVC ratio is 0.85. Which one of the patient's current medications is most likely to be the cause of her problem? (check one) A. Lisinopril (Prinivil, Zestril) B. Conjugated estrogens (Premarin) C. Omeprazole (Prilosec) D. Solifenacin (Vesicare) E. Nitrofurantoin (Macrodantin)

E. nitrofurantoin - chronic dyspnea d/t interstitial lung disease; w/ chronic non-productive cough Spirometry: - Obstructive: ratio <0.7; FVC is normal or decreased and the FEV1 is decreased - restrictive: ratio >0.7 Diffuse parenchymal lung disease - caused by environmental or occupational exposures. - Many collagen vascular diseases and medications used to treat them can induce interstitial lung disease. - Common offenders for interstitial lung disease: amiodarone and nitrofurantoin which can induce a pneumonitis.

A 70-year-old female had a lumbar vertebral fracture 3 years ago. At that time she had a dual-energy x-ray absorptiometry (DEXA) scan, with a T score of -2.6, and was placed on alendronate (Fosamax), calcium, and vitamin D. She recently quit smoking. Her BMI is 21. A DEXA scan today shows her bone mineral density to be -2.1. Which one of the following would be most appropriate in the management of this patient? (check one) A. Replace alendronate with raloxifene (Evista) B. Stop alendronate, but continue calcium and vitamin D C. Add raloxifene to her regimen D. Add teriparatide (Forteo) to her regimen E. Make no change to her regimen

E. no change to her regimen Even though the patient's DEXA has improved and she is technically osteopenic, she still has risk factors for osteoporosis, including recent smoking, low BMI, and a prior fragility fracture. She should continue her current regimen

A 3-year-old female is brought to your office with a 3-hour history of skin lesions that are prominent, warm, papular, and serpiginous (see Figure 3). Which one of the following is the most likely cause of these lesions? Figure 3 (check one) A. Heredity B. Physical abuse C. Infection D. A topical agent E. An oral medication

E. oral medication - acute uticaria - commonly from parenteral drugs, food, and, less frequently, infections. - Topical agents and physical abuse are unlikely to present in this manner, and hereditary angioedema is more a systemic illness than a skin disorder.

A 78-year-old female presents with a red eye. She reports drainage and pain in her left eye since she woke up today, but no photophobia. Examination of the eye shows conjunctival erythema and a mucopurulent discharge. The pupil is normal in size and reactive to light. Which one of the following should prompt immediate referral to an ophthalmologist? (check one) A. Bilateral eye redness B. A corneal abrasion noted on fluorescein staining C. Copious mucopurulent drainage from the eye D. Bright red blood noted under the conjunctiva E. Reduction of visual acuity

E. reduction of visual acuity - acute angle-closure glaucoma Copious mucopurulent drainage = infectious conjunctivitis, most likely bacterial Bilateral eye redness = allergic conjunctivitis. Allergic or infectious conjunctivitis Small corneal abrasions = only needs family physician Bright red blood under the conjunctiva = subconjunctival hemorrhage, can resolve on its own

Complications of hypoparathyroidism include (check one) A. somnolence B. low vitamin D C. muscle flaccidity D. hyperkalemia E. refractory heart failure

E. refractory heart failure - causes low calcium - refractory heart failure: cardiac myocytes - tetany - seizures - altered mental status - stridor Not Low vitamin D - leads to low calcium hypocalcemia but is not caused by it

Effective treatments for obsessive-compulsive disorder include (check one) A. Freudian analysis B. benzodiazepines C. amphetamine salts D. atypical antipsychotics E. repetitive exposure to fearful stimuli

E. repetitive exposure to fearful stimuli

A 56-year-old male with a history of nephrolithiasis presents with a complaint of right flank pain. Further evaluation reveals a right ureteral calculus 4 mm in diameter. Laboratory tests reveal a serum calcium level of 12.1 mg/dL (N 8.5-10.5), a normal albumin level, and normal kidney and liver function tests. The patient takes no chronic medications. Which one of the following is most likely to reveal the cause of this patient's elevated calcium? (check one) A. A 24-hour urine calcium level B. A repeat serum calcium level in 4-6 weeks C. A serum 25-hydroxyvitamin D level D. A serum calcitonin level E. A serum intact parathyroid hormone level

E. serum PTH level - Primary hyperparathyroidism and malignancy are the most common causes of hypercalcemia, accounting for about 90% of cases. - An intact parathyroid hormone (PTH) level should be obtained initially, as the results will indicate what kind of additional evaluation is needed. Not vitamin D and urine calcium studies - useful in evaluating hypercalcemia - but a PTH level should be obtained first. Not be appropriate to wait 4-6 weeks - because this patient has nephrolithiasis and a calcium level 12 mg/dL i.e. need surgery Not Calcitonin levels - not needed

A previously healthy 18-month-old male is brought to your office with a 2-day history of cough and fever. On examination the child has a temperature of 38.3°C (100.9°F), a respiratory rate of 30/min, and mild retractions and mild wheezes bilaterally. Oxygen saturation is 90%. The most appropriate initial management would be: (check one) A. azithromycin (Zithromax) B. a short course of corticosteroids C. aerosolized racemic epinephrine every 4 hours D. postural drainage E. a single treatment with aerosolized albuterol, continued only if there is a positive response

E. single albuterol aerolized treatment, continue if positive response - bronchiolitis, - Treat: a trial of an inhaled bronchodilator, albuterol, or epinephrine - continued only if the initial dose proves beneficial - no evidence to support the use of antibiotics unless another associated infection is present (e.g., otitis media). - Neither corticosteroids nor postural drainage has been found to be helpful.

A 13-year-old African-American male is brought to your office by his mother for a limp that she has noticed for about 1 week. The patient admits to vague right-sided hip and knee pain present only with activity. He says the pain has never awakened him from sleep and is never present at rest. Neither the mother nor the patient has noticed any systemic symptoms such as fever, night sweats, weight loss, or appetite changes. There is no recent history of trauma. On examination the patient's weight is in the 90th percentile and his height is in the 50th percentile. He has an antalgic gait. Examination of the left hip is normal. Examination of the right hip is significant for decreased internal rotation. The right hip externally rotates involuntarily with passive flexion. There is no external deformity and no skin changes are noted. He has a negative FABER test. Palpation of the bursa and bone does not elicit pain. Examination of both knees is normal. Plain radiographs confirm your clinical impression. Which one of the following is the most appropriate next step in the management of this patient? (check one) A. Reassurance with close follow-up B. Physical therapy C. Injection of the sacroiliac joint D. Hospital admission for tests E. Surgery

E. surgery - stable slipped capital femoral epiphysis (SCFE) - external rotation deformity and limited internal rotation - Once the diagnosis of SCFE is made, the patient should not bear weight and should be referred promptly for surgery to prevent complications. - more frequent in males than in females, and is more common in African-Americans and Pacific Islanders - average age of onset is 13.5 years for males and 12 years for females. Obesity is strongly associated with SCFE. - No symptoms so not osteomyelitis, abscess, or a septic joint much less likely - No night pain so not Malignancy - Negative FABER so not Sacroiliitis - Not the right age for Legg-Calvé-Perthes disease less likely.

A 56-year-old African-American male has pain and tingling in the medial aspect of his ankle and the plantar aspect of his foot. He jogs 3 miles daily and has no history of any injury. The symptoms are aggravated by activity, and sometimes keep him awake at night. The only findings on examination are paresthesias when a reflex hammer is used to tap just inferior to the medial malleolus. This patient probably has (check one) A. a stress fracture B. a herniated nucleus pulposus at L5 or S1 C. plantar fasciitis D. diabetic neuropathy E. tarsal tunnel syndrome

E. tarsal tunnel syndrome - Entrapment of the posterior tibial nerve or its branches as the nerve courses behind the medial malleolus results in a neuritis known as tarsal tunnel syndrome. - Causes varices of the posterior tibial vein, tenosynovitis of the flexor tendon, structural alteration of the tunnel secondary to trauma, and direct compression of the nerve - Pronation of the foot causes pain and paresthesias in the medial aspect of the ankle and heel, and sometimes the plantar surface of the foot. Not stress fracture - usu site is the shaft of the second, third, or fourth metatarsals. Not herniated nucleus pulposus - would produce reflex and sensory changes. Not Plantar fasciitis - common cause of heel pain in runners and often presents with pain at the beginning of the workout -pain decreases during running only to recur afterward. Not Diabetic neuropathy - usually bilateral and often produces paresthesias and burning at night - absent or decreased deep tendon reflexes.

A 21-year-old primigravida at 28 weeks gestation complains of the recent onset of itching. On examination she has no obvious rash. The pruritus started on her palms and soles and spread to the rest of her body. Laboratory evaluation reveals elevated serum bile acids and mildly elevated bilirubin and liver enzymes. The most effective treatment for this condition is: (check one) A. triamcinolone (Kenalog) cream B. cholestyramine (Questran) C. diphenhydramine (Benadryl) D. doxylamine succinate E. ursodiol (Actigall)

E. ursodiol (Actigall) - intrahepatic cholestasis

A 19-year-old college wrestler presents with cellulitis of his left arm extending from a small pustule on his hand to the axilla. He appears acutely ill and has a temperature of 38.9°C (102.0°F). His WBC count is 22,000/mm3 (N 4300-10,800). He is admitted to the hospital. The initial drug of choice for this patient would be (check one) A. ciprofloxacin (Cipro) B. clindamycin (Cleocin) C. doxycycline D. trimethoprim/sulfamethoxazole E. vancomycin

E. vancomycin Methicillin-resistant Staphylococcus aureus (MRSA) - community-acquired strains have been susceptible to many antibiotics, clindamycin is associated with Clostridium difficile enterocolitis, trimethoprim/sulfamethoxazole is usually used orally only for outpatient treatment, and doxycycline and minocycline are often effective clinically but seldom used for serious infections. Resistance to quinolones is increasing and may emerge during treatment. Vancomycin given parenterally is generally still the drug of choice for hospitalized patients.

A 55-year-old hospitalized white male with a history of rheumatic aortic and mitral valve disease has a 3-day history of fever, back pain, and myalgias. No definite focus of infection is found on your initial examination. His WBC count is 24,000/mm3(N 4300-10,800) with 40% polymorphonuclear leukocytes and 40% band forms. The following day, two blood cultures have grown gram-positive cocci in clusters. Until the specific organism sensitivity is known, the most appropriate antibiotic treatment would be: (check one) A. ciprofloxacin (Cipro) B. nafcillin C. streptomycin and penicillin D. ceftriaxone (Rocephin) E. vancomycin and gentamicin

E. vancomycin and gentamicin - endocarditis caused by a gram-positive coccus. - Until sensitivities of the organism are known, treatment should include intravenous antibiotic coverage for Enterococcus, Strep, and methicillin-sensitive and methicillin-resistant Staphylococcus. - A patient who does not have a prosthetic valve should be started on vancomycin and gentamicin, with monitoring of serum levels. - Enterococcus and methicillin-resistant Staphylococcus are often resistant to cephalosporins. - If the organism proves to be Staphylococcus sensitive to nafcillin, the patient can be switched to a regimen of nafcillin and gentamicin.

The advance directive specifications contained in an individual's living will become effective: (check one) A. at the time it is signed and witnessed B. when it is confirmed by the individual's health care surrogate C. at the time of admission to a health care facility such as a hospital D. when the patient develops a terminal illness E. when the individual becomes unable to communicate health care wishes

E. when the pt is unable to communicate health care wishes The living will, a written advance directive, allows a competent person to indicate his or her health care references while cognitively and physically healthy. A living will may list medical interventions the patient would prefer to have withheld or withdrawn when he or she becomes unable to communicate.

You evaluate a healthy 4-year-old female about an hour after she had 1 minute of generalized tonic-clonic jerking. She has no previous history of similar symptoms. Additional history reveals that last night she developed a runny nose, a cough, body aches, and a fever. Her past medical, social, and family histories are unremarkable. She takes no medications. A thorough physical examination is notable only for a temperature of 39.2°C (102.6°F) and a clear nasal discharge. An influenza A test is positive. Which one of the following would be most appropriate for the seizures? (check one) A. Reassurance only B. A loading dose of valproic acid (Depakene) C. A neurology consultation and an EEG D. CT of the brain E. A lumbar puncture and cerebrospinal fluid analysis

Febrile seizures occur in 2%-5% of children ≤5 years of age. Simple febrile seizures are characterized as being general, tonic-clonic, and less than 15 minutes in duration, and occur in patients with no prior history of neurologic disease. Prospective cohort studies support reassurance after a simple febrile seizure. Retrospective cohort studies fail to show a benefit from neuroimaging or electroencephalography. Two large case review studies showed no risk or a very low risk for meningitis in the absence of altered mental status or meningeal signs, so a lumbar puncture is not indicated. Randomized, controlled trials failed to show benefit and did show a potential side-effect risk from starting antiepileptic medication.

A 57-year-old female on dialysis for end-stage renal disease develops chronic, severe generalized pain. Which one of the following opioids is preferred for management of her pain? (check one) A. Codeine B. Fentanyl C. Hydrocodone D. Morphine

Fentanyl and methadone are the preferred opioids for use in patients with end-stage renal disease (SOR C). Fentanyl is metabolized in the liver and has no active metabolites.

A 54-year-old male presents with hearing loss associated with tinnitus. Which one of the following additional characteristics would be an indication for MRI of the brain to assess for an intracranial tumor? (check one) A. A rapid onset of symptoms B. Unilateral symptoms C. Association with pain and otorrhea in the affected ear D. Exposure to loud noise shortly before the symptoms began

Hearing loss and tinnitus are both common and typically benign complaints in primary care. If both are present in only one ear, the diagnosis of acoustic neuroma, also known as vestibular schwannoma, should be considered. Acoustic neuroma: -Slow-growing benign tumor of the Schwann cells surrounding the vestibular cochlear (8th cranial) nerve -Hearing loss associated with acoustic neuroma is typically slow in onset. -The presence of vertigo on the affected side is another symptom of abnormal function of the vestibular cochlear nerve and should further raise suspicion of acoustic neuroma or another process affecting that nerve. -MRI is the preferred imaging study for diagnosing acoustic neuroma (SOR A).

A 40-year-old white male was seen 4 weeks ago for a sudden onset of cough and shortness of breath. At that visit his O2 saturation was 92%, but his examination and a chest radiograph were normal. You prescribed azithromycin (Zithromax) and an albuterol inhaler (Proventil, Ventolin). Ten days later he was feeling well and his oxygen saturation was 97%. Today he returns to the office with a dry cough and shortness of breath. On examination he has rare inspiratory rales that clear with deep breaths, and he has an O2 saturation of 86%. A chest film and a D-dimer test are normal. Pulmonary function tests show significant restriction that improves only minimally with albuterol. He has not been exposed to anyone with a similar illness, has no history of asthma, and has no smoking history or occupational exposure. However, he reports that 2 months ago his home was flooded after a heavy rain, and he has been tearing out carpeting that was ruined by the flood. Which one of the following is the most likely diagnosis? (check one) A. Persistent asthma with acute exacerbations B. Legionnaires' disease C. Pulmonary embolism D. Hypersensitivity pneumonitis

Hypersensitivity pneumonitis can present in acute, subacute, or chronic forms. The case described includes two episodes of the acute form. The patient was exposed to mold antigens in his flooded home. Within 4-8 hours of exposure, chills, cough, and shortness of breath will be noted, and at times will be dramatic. A chest film can be normal, even with significant hypoxia. Pulmonary function tests will show restrictive changes, as compared to the reversible obstructive changes of acute asthma. Blood tests often show an elevated erythrocyte sedimentation rate. Serum IgG tests for the probable antigen confirm the diagnosis. Symptoms of acute hypersensitivity pneumonitis resolve over several days, but will suddenly and violently recur with repeated exposure to the offending antigen. The subacute form begins gradually over weeks or months, causing a cough and increasing shortness of breath. The chronic form develops over years of exposure, causing fibrotic changes to the lungs that will be evident on radiographs, as well as chronic crackles on auscultation. Asthma would be an unlikely diagnosis in this case, with the pulmonary function tests showing restrictive changes rather than obstructive changes, and little improvement with albuterol. Also, the lack of a previous history of asthma makes it less likely. Legionnaires' disease is always possible, but is unlikely in this case given the sudden onset, quick recovery over several days, and sudden recurrence. Pulmonary embolism is ruled out by the negative D-dimer test.

Which one of the following has been shown to have a beneficial effect for symptoms of the common cold in an adult? (check one) A. Diphenhydramine (Benadryl) B. Ipratropium (Atrovent) nasal spray C. Intranasal zinc D. Intranasal corticosteroids E. Systemic corticosteroids

Ipratropium - nasally inhaled anticholinergic recommended for a cough caused by the common cold. - nasal formulation decreases rhinorrhea and sneezing, - Antihistamine monotherapy (either sedating or nonsedating) such as diphenhydramine was no more effective than placebo (SOR A). - Intranasal zinc should not be used because it may result in the permanent loss of smell.

A 64-year-old male presents with increasing dyspnea on exertion. He feels well otherwise and has no chronic medical problems. A physical examination is normal. Pulmonary function testing reveals normal spirometry, with no evidence of an obstructive or restrictive pattern. However, his lung carbon monoxide diffusing capacity (DLCO) is low. Based on these results, which one of the following is the most likely diagnosis? (check one) A. Asthma B. Bronchiectasis C. Chronic pulmonary emboli D. COPD E. Pulmonary fibrosis

Low diffusing capacity of the lungs for carbon monoxide (DLCO) with normal spirometry indicates a disease process that disrupts gas transfer in the lungs without causing lung restriction or airflow obstruction. Common causes include chronic pulmonary emboli, heart failure, connective tissue disease with pulmonary involvement, and primary pulmonary hypertension. Asthma, bronchiectasis, COPD, and pulmonary fibrosis are associated with abnormalities on spirometry.

A 68-year-old female presents with recent poor oral intake, fatigue, and confusion. Osmotic demyelination syndrome (central pontine myelinolysis) and permanent neurologic deficits could result from overly rapid correction of which one of the following abnormalities? (check one) A. Hyperglycemia B. Hyperkalemia C. Hypokalemia D. Hypernatremia E. Hyponatremia

Overly rapid correction of hyponatremia may cause osmotic demyelination syndrome, or central pontine myelinolysis, sometimes resulting in permanent neurologic deficits after a brief improvement in neurologic status. Signs and symptoms may include dysarthria, dysphagia, paresis, coma, and seizures. It is believed that brain volume shrinks because it cannot assimilate the new electrolytes fast enough and water is lost from the cells. Rapid correction of hypernatremia that has been present for a short time is relatively safe. Hyperkalemia is a life-threatening condition that should be corrected promptly. Rapid correction of hypoglycemia is not an issue. Overly rapid correction of hyperglycemia and subsequent cerebral edema is unusual and is primarily seen in children.

A 46-year-old male with a prosthetic heart valve requests your advice regarding antibiotic prophylaxis for an upcoming dental procedure. The patient is allergic to penicillin. Which one of the following would be most appropriate? (check one) A. Ciprofloxacin (Cipro) B. Clindamycin (Cleocin) C. Doxycycline D. Rifampin (Rifadin) E. Trimethoprim/sulfamethoxazole (Bactrim)

Patients with an artificial heart valve should be given antibiotic prophylaxis prior to dental procedures to prevent infectious endocarditis. The organisms that most frequently cause infectious endocarditis include Staphylococcus aureus (31%), Streptococcus viridans (17%), coagulase-negative staphylococci (11%), Enterococcus (11%), Streptococcus bovis (7%), and other streptococci (5%). Amoxicillin is the preferred medication for prophylaxis. Clindamycin or azithromycin can be used in patients with a penicillin allergy. If the penicillin allergy is not associated with anaphylaxis, angioedema, or urticaria, then cephalexin would be an appropriate antibiotic choice.

An 80-year-old female is admitted to your service at a skilled nursing facility 5 days after repair of a hip fracture. When you review her records you note that she has not received any previous treatment for osteoporosis. You are considering ordering zoledronic acid (Reclast) to reduce her risk of another fracture. Which one of the following should be evaluated before administering zoledronic acid to this patient? (check one) A. Vitamin D levels B. Liver enzyme levels C. Estimated glomerular filtration rate D. A CBC

Secondary prevention of fractures is an important component of care following a hip fracture. Options to consider include bisphosphonates, calcium supplementation, and vitamin D supplementation. Bisphosphonates, including zoledronic acid, can reduce rates of clinical fractures among patients who have had a hip fracture (SOR A). While long-term use of bisphosphonates may increase the risk of jaw osteonecrosis and anemia, a CBC is not necessary before initiating therapy. Contraindications to zoledronic acid include hypocalcemia and a creatinine clearance <35 mL/min or other evidence of acute renal impairment.

A 24-year-old male complains of feeling on edge all of the time. For the past 2 years he has had difficulty controlling his worrying about work, school, and relationships. He has had more difficulty concentrating at work and school, is more irritable, and has difficulty staying asleep all night. He drinks alcohol moderately and does not use drugs. You recommend regular exercise and refer him to a therapist for cognitive-behavioral therapy to help manage his symptoms. Which one of the following would be first-line medical therapy for this patient? (check one) A. Bupropion (Wellbutrin) B. Fluoxetine (Prozac) C. Lorazepam (Ativan) D. Methylphenidate (Ritalin, Concerta) E. Quetiapine (Seroquel)

Though symptoms of generalized anxiety disorder (GAD) overlap with other psychiatric and medical conditions, the case presented is most consistent with GAD. SSRIs are first-line therapy for GAD (SOR B). Benzodiazepines such as lorazepam can improve anxiety-related symptoms, but due to the side effects and addiction potential they are recommended for short-term use (SOR B). Bupropion is approved for the treatment of depression but is not used to treat GAD. Quetiapine may be considered as second-line therapy for GAD (SOR B). Methylphenidate is first-line therapy for attention-deficit/hyperactivity disorder but is not indicated to treat GAD. Psychotherapy, especially cognitive-behavioral therapy, is also first-line treatment for GAD (SOR A), and exercise can also improve symptoms (SOR B).

When should you get imaging on tinnitus?

Tinnitus that is bilateral and not bothersome can be treated conservatively with cognitive-behavioral therapy, sound therapy, and, if appropriate, hearing aids. Antidepressants are not recommended. Pulsatile tinnitus, unilateral tinnitus, or tinnitus associated with asymmetric hearing loss is more likely to be associated with a pathologic cause.


Related study sets

Acct 209 Statement of Retained Earnings

View Set

Chapter 4: Costing and Pricing for Transportation

View Set

Real Estate License Exam - Level 6: Law of Agency

View Set

Real Estate Finance: Module 3 (Chapter 5: Government Financing)

View Set

Chapter 41: Assessment of the Nervous System

View Set